Sie sind auf Seite 1von 407

Download From : www.EasyEngineering.

net

ww
w.E
a syE
ngi
nee
rin
g.n
et

**Note: Other Websites/Blogs Owners Please do not Copy (or) Republish


this Materials, Students & Graduates if You Find the Same Materials with
EasyEngineering.net Watermarks or Logo, Kindly report us to
easyengineeringnet@gmail.com Download From : www.EasyEngineering.net
Download From : www.EasyEngineering.net

Unit Number Page Number

Unit 1 03
ww 69
w.E Unit 2

Unit 3
asy 163
Unit 4 En 249
Unit 5
g324
ine
e rin
By www.EasyEngineering.net
g.n
et

Download From : www.EasyEngineering.net


Download From : www.EasyEngineering.net

1 Differential calculus

1.1 Representation of a function.

wwDefinition. A function f is a rule that assigns to each element x in a set D exactly

w.E
one element, called f (x), in a set E.
The set D is called the domain of the function. The number f (x) is the value of

asy
f at x. The range of f is the set of all possible values of f (x) as x varies throughout
the domain. A symbol that represents an arbitrary number in the domain of a

En
function f is called an independent variable. A symbol that represents a number

gin
in the range of f is called a dependent variable.
Example. The area A of a circle depends on the radius r of the circle. The rule

ee
that connects r and A is given by the equation A = πr2 . With each positive number

rin
r, there is associated one value of A and we say that A is a function of r. Here, r is
the independent variable and A is the dependent variable.

g.n
We can think of a function as a machine. If x is in the domain of the function

et
f , then when x enters the machine, it is accepted as an input and the machine
produces an output f (x) according to the rule of the function. Thus, we can think
of the domain as the set of all possible inputs and the range as the set of all possible
outputs.
input x → f → f (x) output

Another way to represent a function is by an Arrow diagram. Each arrow


connects an element of D to an element of E. The arrow indicates that f (x) is

Download From : www.EasyEngineering.net


Download From : www.EasyEngineering.net

2 Engineering Mathematics - I

associated with x, f (a) is associated with a and so on.

x f (x)
a f (a)
y f (y)

The most common method for visualizing a function is its graph. If f is a

wwfunction with domain D, then its graph is the set of ordered pairs {(x, f (x))/x ∈ D}.
In other words, the graph of f consists of all points (x, y) in the coordinate plane

w.E
such that y = f (x), and x is in the domain of f .

Y
Y

asy
(x, f (x))

En
f (x)
range y = f (x)

0 1
f (1)

2
f (2)

x gin X 0 Domain
X

ee rin
The graph of a function f gives us a useful picture of the behavior of a function.

g.n
Since the y−coordinate of any point (x, y) on the graph is y = f (x), we can read the
value of f (x) from the graph as being the height of the graph above the point x. The
graph of f also allows us to picture the domain of f on the x−axis and its range on
the y−axis. et
Example 1.1. Sketch the graph of f (x) = 2x + 1 and find the domain and range.
Solution.

Y
−1
x 0
2 1
y 1 0
X
−1/2

y = 2x + 1

Download From : www.EasyEngineering.net


Download From : www.EasyEngineering.net

Differential calculus 3

The equation of the graph is y = 2x + 1. This represents a line with slope 2 and
y−intercept 1. With these informations, we can sketch the portion of the graph of
f . The expression 2x + 1 is defined for all real numbers, and hence the domain of
f is the set of all real numbers, which we denote by R. The graph shows that the
range is also R.

Example 1.2. Sketch the graph of g(x) = x2 and find the domain and range.
Solution. The equation of the graph is y = x2 .

ww Y

w.E x
y
−2
4
−1
1
0
0
1
1
2
4
8
6

asy 4
2
y = x2

En −2 −1 0 1 2
X

gin
From the tabular column we can plot the points (−2, 4), (−1, 1), (0, 0), (1, 1) and (2, 4)

ee
and join them to produce the graph, which represents a parabola. The domain of

rin
g is R. The range of g consists of all values of g(x), that is, all numbers of the form
x2 . But x2 ≥ 0 for all numbers x and any positive number y is a square. Hence, the
range of g is {y : y ≥ 0} = [0, ∞].
g.n
the xy−plane represent the graph of some function. This can be answered by the
vertical line test.
et
The graph of a function is a curve in the xy−plane. But whether all curves in

The vertical line test. A curve in the xy−plane is the graph of a function of
x if and only if no vertical line intersects the curve more than once.
Consider the following graphs.

Download From : www.EasyEngineering.net


Download From : www.EasyEngineering.net

4 Engineering Mathematics - I

Y Y
(a, c)
x=a x=a

(a, b) (a, b)

X X
a 0 a
0

ww In the first graph, each vertical line x = a intersects the curve only once, at

w.E
(a, b), then exactly one function value is defined by f (a) = b. But in the second
graph, the line x = a intersects the curve twice at (a, b) and (a, c), that is, the

asy
function assigns two different values to a. Hence, the first curve represents the
graph of a function while the second graph is not.

En
As an illustration consider the parabola x = y2 − 4. The curve that represents
this equation is give below.
gin
Y
ee This curve does not represent the

rin
graph of a function of x because, we can
see that every vertical line intersects
x = y2 − 4

g.n
the parabola twice. This parabola is the
union of the graphs of two functions of
(−4, 0)

0 X

we get y = ± x + 2.
et
x. Rewriting the equation as y2 = x + 4

Hence, the upper and lower halves of the parabola are the graphs of the
√ √
functions f (x) = x + 4 and g(x) = − x + 4. The separate graphs are given below.

Download From : www.EasyEngineering.net


Download From : www.EasyEngineering.net

Differential calculus 5


Y
Y y= x+4

(−4, 0)
(−4, 0) 0 X
0 X

y=− x+4

ww If we reverse the roles of x and y, then we get the equation x = h(y) = y2 − 4


definitely define x as a function of y with y as the independent variables and x as

w.E
the dependent variable and in this case, the parabola represent the graph of the
function h.

Piecewise difined functions.


asy
En

1 − x , if x ≤ −1



Example 1.3. Consider the function f defined by f (x) =  .

gin

 x2

 , if x > −1

For all x ≤ −1.


x -3 -2 -1
ee Y

6
rin
y=1−x 4 3 2
5
4 g.n
For all x > −1.
x
y = x2
-1
1
0
0
1
1
2
4
3
9
3
2
1
et
The combined graph of the function f is X
−4 −3 −2 −1 0 1 2 3 4
as follows

Notice that the point (−1, 1) in y = x2 is not included and there is a discontinuity
between the graph for all x ≤ 1 (i.e.y = 1 − x) and the graph for all x > 1 (i.e. y = x2 ).

Download From : www.EasyEngineering.net


Download From : www.EasyEngineering.net

6 Engineering Mathematics - I

Example 1.4. Consider the absolute value function f (x) = |x|, which is defined as

x , if x ≥ 0



follows f (x) =  . The graph of f (x) consists of two branches namely


−x , if x < 0


y = x for all x ≥ 0 and y = −x for all x < 0.

The combined graph of f is given below.


y = x (x ≥ 0)
Y

ww x
y
0
0
1
1
2
2
3
3
3 y = |x|

w.E x
y = −x (x < 0)
-3 -2 -1 0
2
1
X
y 3 2
asy
1 0
−4 −3 −2 −1 0 1 2 3 4

En
Notice that the point (0, 0) is not included in the graph of y = −x (x < 0).

gin







x , if 0 ≤ x ≤ 1

ee


Example 1.5. Consider the function f (x) =  , if 1 < x ≤ 2 .


2−x

rin



0 , if x > 2

y = x (0 ≤ x ≤ 1)
y = 2 − x (1 ≤ x ≤ 2)
x 1 2 x g.n
y = 0 (x > 2)
2 3 4 5
x
y
0
0
1
1
y 1 0 y 0
et 0 0 0

The graph of f is given below.

1
X
0 1 2 3 4 5

Download From : www.EasyEngineering.net


Download From : www.EasyEngineering.net

Differential calculus 7

Example 1.6. Consider the greatest integer function defined by f (x) = [x].
By definition [x]= Largest integer that is less than or equal to x
= max{m ∈ Z; m ≤ x}.
Example. [2.7] = 2, [−1.3] = −2, [4] = 4
The graph of the function is as follows

The open dots are not included at the


Y

ww 5
corresponding points.
The Examples 1.3 to 1.6 indicate that

w.E 4
3
2
there is jump from one value to the
next. Hence, the above functions are
1

−2 −1 0 1 2 3 4 5
asy X
called piecewise functions. The function
discussed in Example 1.6 is called step
−1
−2 En function.

gin
symmetry

ee rin
Even function. If a function f satisfies f (−x) = f (x) for every number x in its
domain, then f is called an even function.

Example. f (x) = x2 is even,


g.n
since f (−x) = (−x)2 = x2 = f (x).
Geometrically, tha graph of an even
Y

et
function is symmetric about the y−axis.
f (−x) f (x)
Consider the curve of y = x2 − 4.

0 X
−x x

Download From : www.EasyEngineering.net


Download From : www.EasyEngineering.net

8 Engineering Mathematics - I

Odd function. If f statifies f (−x) = − f (x) for every number x in the domain, then
f is called an odd function.
Example. Consider f (x) = x3
f (−x) = (−x)3 = −x3 = − f (x).
∴ f (x) = x3 is an odd function.
Y

f (x)
−x
X

ww − f (x)
0 x

w.E
The graph of the odd function is symmetric about the origin.

(i) f (x) = x5 − x. asy


Example 1.7. Determine the nature of the following functions.
(ii) g(x) = 1 + x4 . (iii) h(x) = x + x2 .
Solution.
(i) f (x) = x5 − x. En
gin
f (−x) = (−x)5 − (−x) = −x5 + x = −(x5 − x) = − f (x).
∴ f (x) is odd.
(ii) g(x) = 1 + x4 . ee
g(−x) = 1 + (−x)4 = 1 + x4 = g(x).
rin
∴ g(x) is even.
(iii) h(x) = x + x2 . g.n
h(−x) = (−x) + (−x)2 = −x + x2 .
h(−x) , h(x) and h(−x) , −h(x)
et
∴ h(x) is neither even nor odd.

Increasing and decreasing functions. A function f is called increasing on an


interval I, if f (x1 ) < f (x2 ) whenever x1 < x2 in I.
It is called decreasing on I, if f (x1 ) > f (x2 ) whenever x1 < x2 in I.

Download From : www.EasyEngineering.net


Download From : www.EasyEngineering.net

Differential calculus 9

1.2 Limit of a function

Definition. Intuitive definition of a limit. Suppose f (x) is defined. When x is


near the number a, then we write lim f (x) = L. This means that the limit of f (x),
x→a
as x approaches a equals L.
lim f (x) = L can also be represented as f (x) → L as x → a.
x→a
One sided limits. The left hand limit of f (x) as x approaches a is written as
lim f (x) = L which means that the limit of f (x) as x approaches a from the left

ww x→a−
is equal to L if we can make the values of f (x) arbitrarily close to L by taking x
sufficiently close to a with x less than a.

w.E Similarly, if we require that x greater than a, we get the right hand limit of
f (x) as x approaches a is equal to L and we write lim f (x) = L.

x→a asy x→a−


x→a+

Result. lim f (x) = L if and only if lim f (x) = L and lim f (x) = L.
x→a+

Infinite limits.
En
gin
(i) Let f ba a function defined on both sides of a, except possibly at a itself. Then
lim f (x) = ∞ means that the value of f (x) can be made arbitrarily large by taking x
x→a

ee
sufficiently close to a, but not equal to a.

rin
(ii) Let f be a function defined on both sides of a, expect possibly at a itself. Then

x→a
by taking x sufficiently close to a, but not equal to a. g.n
lim f (x) = −∞ means that the values of f (x) can be made arbitrarily large negative

1
Example. lim 2 = ∞.
x→0 x
1
!
lim − 2 = −∞.
x→0 x
et
Asymptote. The vertical line x = a is called a vertical asymptote of the curve
y = f (x) if atleast one of the following statements is true:
lim f (x) = ∞, lim f (x) = ∞, lim f (x) = ∞.
x→a x→a− x→a+
lim f (x) = −∞, lim f (x) = −∞, lim f (x) = −∞.
x→a x→a− x→a+
2x
Example. (i) For the function f (x) = , x = 3 is a vertical asymptote.
x−3

Download From : www.EasyEngineering.net


Download From : www.EasyEngineering.net

10 Engineering Mathematics - I

π
(ii) For the curve f (x) = tan x, x =
is a vertical asymptote.
2
Laws of limits. Suppose that c is a constant and the limits lim f (x) and lim g(x)
x→a x→a
exist, then the following laws are true.

1. lim f (x) + g(x) = lim f (x)+ lim g(x).


 
x→a x→a x→a
 
2. lim f (x) − g(x) = lim f (x) − lim g(x).
x→a x→a x→a
 
3. lim c f (x) = c lim f (x).
x→a x→a

ww 4. lim f (x)g(x) = lim f (x) × lim g(x).


 
x→a x→a x→a
" # lim f (x)
f (x)

w.E 5. lim
x→a g(x)
= x→a

 
lim g(x)
x→a
. if lim g(x) , 0.

n
x→a

6. lim f (x) n = lim f (x) , where n is a positive integer.

asy

x→a x→a

7. lim c = c.

En
x→a

8. lim x = a.

gin
x→a

9. lim xn = an , where n is a positive integer


x→a

10. lim
x→a

11. lim
√n

p
n
x=
√n

f (x) =
q
n lim f (x).
ee
a, where n is a positive integer and a > 0.

rin
g.n
x→a x→a

Direct substitution property. If f is a polynomial or a rational function and a


is in the domain of f , then lim f (x) = f (a).
x→a
Theorem. If f (x) ≤ g(x), when x is near a and the limits of f and g both exist as x
approaches a, then lim f (x) ≤ lim g(x).
x→a x→a
et
The Squeeze Theorem. If f (x) ≤ g(x) ≤ h(x), when x is near a and
lim f (x) = lim h(x) = L, then lim g(x) = L.
x→a x→a x→a
The above theorem is sometimes called Sandwich Theorem or the Pinching
Theorem.
✎ ☞
Worked Examples
✍ ✌

Download From : www.EasyEngineering.net


Download From : www.EasyEngineering.net

Differential calculus 11

Example 1.8. Find lim (2x2 − 3x + 4).


x→5

Solution. lim (2x2 − 3x + 4) = lim 2x2 − lim 3x + lim 4


x→5 x→5 x→5 x→5

= 2 lim x2 − 3 lim x + 4
x→5 x→5
2
= 2 × 5 − 3 × 5 + 4 = 50 − 15 + 4 = 39.

x3 + 2x2 − 1
Example 1.9. Find lim .
x→−2 5 − 3x

wwSolution. lim
x→−2
x3 + 2x2 − 1
5 − 3x
=
lim x3 + 2x2 − 1
x→−2
lim 5 − 3x
x→−2

w.E =
lim x3 + lim 2x2
x→−2 x→−2
lim 5 − lim 3x
− lim 1
x→−2

asy
x→−2 x→−2
lim x3 + 2 lim x2 − 1
x→−2 x→−2
=

En
lim 5 − 3 lim x
x→−2 x→−2
−8 + 2(−2)2
−1
=

=gin 5 − 3(−2)
−8 + 8 − 1
=− .
1

Example 1.10. Find lim


x→1
x2 − 1
x−1
.
ee
5+6 11

rin
Solution. lim
x2 − 1
x→1 x − 1
= lim
x→1
(x + 1)(x − 1)
x−1
= lim (x + 1) = 1 + 1 = 2.
x→1 g.n
Example 1.11. Find lim f (x) where f (x) = 
x→1

 x + 1 , ifx , 1






π
 , ifx = 1
. et
Solution. f is defined at x = 1 and g(1) = π.
But the value of a limit as x approahces 1 does not depend on the value
of the function at 1.
Since g(x) = x + 1 for x , 1, we have
lim g(x) = lim (x + 1) = 2.
x→1 x→1

Download From : www.EasyEngineering.net


Download From : www.EasyEngineering.net

12 Engineering Mathematics - I

(3 + x)2 − 9
Example 1.12. Evaluate lim .
x→0 x

(3 + x)2 − 9 (3 + x − 3)(3 + x + 3)
Solution. lim = lim
x→0 x x→0 x
x(6 + x)
= lim
x→0 x
= lim (6 + x) = 6.
x→0

x2 + 9 − 3
Example 1.13. Find lim .

ww x→0 x2
Solution. On rationalizing the numerator we get
√ √ √

w.E lim
x→0
x2 + 9 − 3
x2
= lim
x→0
x2 + 9 − 3

2
x2
x +9−9
× √
x2 + 9 + 3
x2 + 9 + 3

asy
= lim √
x→0 x2 x2 + 9 + 3

x2

En
= lim √
x→0 x2 x2 + 9 + 3
1 1 1
= lim √
x→0
gin =
x2 + 9 + 3 3 + 3 6
= .

Example 1.14. Show that lim |x| = 0.


x→0

Solution. We know that |x| = 



x





ee
, ifx ≥ 0
−x , ifx < 0
.
rin
g.n


Since |x| = −x for x < 0, we have lim |x| = lim −x = 0.
x→0− x→0−
Also |x| = x for x > 0, we have lim |x| = lim x = 0.
∴ lim |x| = lim |x| = 0.
x→0− x→0+
Hence, lim |x| = 0.
x→0
x→0+ x→0+

et
|x|
Example 1.15. Prove that lim does not exist.
x→0 x

x , ifx ≥ 0




Solution. We know that |x| = 
 .
−x , ifx < 0

Download From : www.EasyEngineering.net


Download From : www.EasyEngineering.net

Differential calculus 13

|x| x
Now, lim = lim = 1.
x→0+ x x→0+ x
|x| −x
lim = lim = −1.
x→0− x x→0− x
|x| |x| |x|
Since lim , lim , lim does not exist.
x→0+ x x→0− x x→0 x
√
 x − 4 , ifx > 4



Example 1.16. If f (x) =  , determine whether lim f (x) exists.

 x→4
8 − 2x , ifx < 4



Solution. We have f (x) = x − 4 for x > 4.

ww

∴ lim f (x) = lim x − 4 = 0.
x→4+ x→4+
Also, f (x) = 8 − 2x for x < 4.

w.E ∴ lim f (x) = lim (8 − 2x) = 0.


x→4− x→4−
Since lim f (x) = lim f (x) = 0, we have lim f (x) exists and lim f (x) = 0.

asy
x→4+ x→4− x→4 x→4

Example 1.17. Show that for the greatest integer function [x], lim [x] does not
x→3
exist.
En
Solution. By definition [x] = 3 for 3 ≤ x < 4.
∴ lim [x] = lim 3 = 3.
x→3+ x→3+ gin
Also [x] = 2 for 2 ≤ x < 3.
∴ lim [x] = lim 2 = 2.
x→3− x→3− ee
Since lim [x] , lim [x], lim [x] does not exist.
rin
g.n
x→3+ x→3− x→3

1.3 Continuity

Definition. A function f is continuous at a number a, if lim f (x) = f (a).


x→a
et
f is discontinuous at a if it is not continuous at a.
Note. For proving a function f to be continuous, we have to prove the following.

1. f (a) must be defined.

2. lim f (x) exists.


x→a

Download From : www.EasyEngineering.net


Download From : www.EasyEngineering.net

14 Engineering Mathematics - I

3. lim f (x) = f (a).


x→a

x2 − x − 2
Example. Consider f (x) = .
x−2
Here, f (x) is not defined at x = 2.
∴ f is not continuous at x = 2.

x2 −x−2
, ifx , 2



 x−2

If we define f (x) as f (x) = 
 .
, ifx = 2

1

Then

ww(i) f (2) is defined and f (2) = 1.


x2 − x − 2 (x − 2)(x + 1)

w.E
(ii) lim f (x) = lim
x→2 x→2 x−2
= lim
x→2 x−2
= lim (x + 1) = 3
x→2

(iii) Since f (2) = 1 , lim f (x), f (x) is not continuous at x = 2.


x→2

asy
Definition. A function f is said to be continuous from the right at a number a if

En
lim f (x) = f (a), and f is said to be continuous from the left at a if lim f (x) = f (a).
x→a+ x→a−
Example. Consider the greatest integer function f (x) = [x]. We notice that

x→n+ x→n+
gin
lim f (x) = lim [x] = n = f (n)
lim f (x) = lim [x] = n − 1 , f (n).

left.
x→n− x→n−

ee
∴ f (x) = [x] is continuous from the right but not continuous from the

rin
Definition. A function f is continuous on an interval if it is continuous at every
number in the interval. g.n
Theorem. If f and g are continuous at a and c is a constant, then the following
functions are also continuous at a.
1. f + g 2. f − g 3. c f 4. f g 5.
f
g(a) , 0.
et
g
Theorem.

(a) Any polynomial is continuous everywhere. i.e., it iscontinuous on R = (−∞, ∞).

(b) Any rational function is continuous wherever it is defined, i.e., it is continuous


on its domain.

Download From : www.EasyEngineering.net


Download From : www.EasyEngineering.net

Differential calculus 15

Theorem. The following types of functions are continuous at every number in


their domains.

1. Polynomials.

2. Rational functions.

3. Root functions.

4. Trigonometric functions.

ww 5. Inverse trigonometric functions.

w.E6. Exponential functions.

7. Logarithmic functions.

asy
Theorem. If f is continuous at b, and lim g(x) = b, then lim f (g(x)) = f (b).

En  x→a
 x→a
i.e., lim f (g(x)) = f lim g(x) .
x→a x→a

gin
Theorem. If g is continuous at a and f is continuous at g(a), then the composite
function f og given by ( f og)(x) = f (g(x)) is continuous at a.

ee
The intermediate value theorem. Suppose that f is continuous on the closed

rin
interval [a, b] and let N be any number between f (a) and f (b), where f (a) , f (b).

g.n
Then there exists a number c in (a, b) such that f (c) = N.
Result. The intermediate value theorem is useful in the location of the roots
of the given equation.

Example 1.18. Find the domain where the fnction f is continuous. Also find the
et
numbers  at which the function f is discontinous where
1 + x2





 , if x ≤ 0



f (x) =  , if 0 < x ≤ 2 . [A.U. Dec. 2015]


2−x



(x − 2)2 x

, if x > 2


Solution. The function f changes its value at x = 0, and x = 2.

Download From : www.EasyEngineering.net


Download From : www.EasyEngineering.net

16 Engineering Mathematics - I

lim f (x) = lim (1 + x2 ) = 1.


x→0− x→0−
lim f (x) = lim (2 − x) = 2.
x→0+ x→0+
Since, lim f (x) = lim f (x), f is not continuous at x = 0.
x→0− x→0+
lim f (x) = lim (2 − x) = 0.
x→2− x→2−
lim f (x) = lim (x − 2)2 = 0.
x→2+ x→2+
Since, lim f (x) = lim f (x) = 0 = f (2), f (x) is continuous at x = 2.
x→2− x→2+
∴ The only number at which the function is discontinuous is at x = 0.

ww The domain of continuity of f is {(−∞, 0) ∪ (0, ∞).


The graph of f is given below

w.E
For all x ≤ 0.
The combined graph of the function f is

y = f (x) = 1 + x2 .
asy as follows
Y
x
y
-3
10
-2
5
-1
2 En
0
1
10
9
For all 0 < x ≤ 2.
gin 1 + x2
8
7
y = f (x) = 2 − x.

x
y
0
2
1
1
2
0
ee 6
5
4 rin
For all x > 2. 3
g.n
y = f (x) = (x − 2)2 .

x
y
2
0
3
1
4
4
−4 −3 −2 −1 0
2
1
2−x

1 2
et3
(x − 2)2

4
X

Notice that the point (−1, 1) in y = x2 is not included and there is a discontinuity
between the graph for all x ≤ 1 (i.e.y = 1 − x) and the graph for all x > 1 (i.e. y = x2 ).

Example 1.19. Show that there is a root of the equation 4x3 − 6x2 + 3x − 2 between

Download From : www.EasyEngineering.net


Download From : www.EasyEngineering.net

Differential calculus 17

1 and 2.

Solution.Let f (x) = 4x3 − 6x2 + 3x − 2.

f (1) = 4(1)3 − 6(1)2 + 3(1) − 2 = 4 − 6 + 3 − 2 = −1 < 0.

f (2) = 4(2)3 − 6(2)2 + 3(2) − 2

= 4 × 8 − 6 × 4 + 3 × 2 − 2 = 32 − 24 + 6 − 2 = 12 > 0.

wwThe function changes its sign between 1 and 2.

w.E
Since f (x) is a polynomial, which is
continuous in its domain the graph of
Y

asy
y = f (x) must cross the x−axis atleast
at one point say x = c between x = 1
f (1)

En
and x = 2 such that f (c) = 0, which is c 2
X

gin
the root of the equation f (x) = 0. This
proves that the given equation f (x) = 0
1
f (2)

has a root between 1 and 2.


ee rin
Example 1.20. Prove that the equation x3 − 15x + 1 = 0 has atmost one real root
in the interval [−2, 2].
g.n
[A.U.Nov.2016]
Solution. Let f (x) = x3 − 15x + 1

f (−2) = −8 + 30 + 1 = 23 = +ve.
et
f (−1) = −1 + 15 + 1 = 15 = +ve. + + +
f (0) = 1 = +ve.
f (1) = 1 − 15 + 1 = −13 = −ve. −2 −1 0 1 2
f (2) = 8 − 30 + 1 = −21 = −ve. − −

Download From : www.EasyEngineering.net


Download From : www.EasyEngineering.net

18 Engineering Mathematics - I

Hence, f (0) > 0 > f (1).


f changes sign between 0 and 1.
∴ By intermediate value theorem, there is a number c between 1 and 2
such that f (c) = 0.
∴ The given equation has atleast one root c in the interval (0, 1).
Since f (x) changes sign in the interval [−2, 2] only once between 0 and
1, y = f (x) crosses the x axis only once.
i.e., f (x) attains 0 only once in [−2, 2]

ww Hence, there is atmost one real root in the interval [−2, 2].

w.E
1.4 Derivatives and differentiation rules

asy
Definition. The tangent line to the curve y = f (x) at the point P(a, f (a)) is the line
f (x) − f (a)
through P with slope m = lim
x→a
En x−a
.

gin
Definition. The derivative of a function f at a number a, denoted by f ′ (a) is
f (a + h) − f (a)
defined as f ′ (a) = lim

is f ′ (a) = lim
h→0

f (x) − f (a)
.
h
ee , if the limit exists.

rin
Note. Let x = a+h. As h → 0, x → a. The equivalent definition for the derivative

x→a x−a

Worked Examples

g.n
✍ ✌

et
Example 1.21. Find an equation to the tangent line to the parabola y = x2 at the
point P(1, 1).
Solution. Given a = 1, f (a) = 1, f (x) = x2 .

f (x) − f (1)
Slope m = lim
x→1 x−1
x2 − 1 (x − 1)(x + 1)
= lim = lim = lim (x + 1) = 1 + 1 = 2.
x→1 x − 1 x→1 x−1 x→1

Download From : www.EasyEngineering.net


Download From : www.EasyEngineering.net

Differential calculus 19

Equation of the tangent line through (1, 1) is

y − y1 = m(x − x1 ).

y − 1 = 2(x − 1)

y = 2x − 2 + 1

y = 2x − 1.

wwExample 1.22. Find an equation to the tangent line to the hyperbola y =


point (3, 1).
x
at the

w.E
Solution. Given a = 3, f (a) = f (3) = 1.

We have m = lim
f (a + h) − f (a)

asy
h→0

= lim
h
f (3 + h) − f (3)

En
h→0 h
3
−1
= lim 3+h
h→0

= lim gin
h
3−3−h
h→0 h(3 + h)

= lim
−h
ee
h→0 h(3 + h)
= lim
−1
h→0 (3 + h)
=
−1
3
.

rin
∴ The equation of the tangent line is

g.n
y − y1 = m(x − x1 ).

y−1=
−1
3
(x − 3)
et
3y − 3 = −x + 3

x + 3y − 3 − 3 = 0

x + 3y − 6 = 0.

Example 1.23. Find the derivative of the function f (x) = x2 − 8x + 9 at a number

Download From : www.EasyEngineering.net


Download From : www.EasyEngineering.net

20 Engineering Mathematics - I

a. Find also the equation the tangent line at the point (3, −6).
f (a + h) − f (a)
Solution. f ′ (a) = lim
h→0 h
(a + h) − 8(a + h) + 9 − (a2 − 8a + 9)
2
= lim
h→0 h
a + h + 2ah − 8a − 8h + 9 − a2 + 8a − 9
2 2
= lim
h→0 h
h2 + 2ah − 8h
= lim
h→0 h

ww = lim (h + 2a − 8) = 2a − 8.
h→0

We have m = f ′ (a) = f ′ (3) = 2 × 3 − 8 = −2.

w.E ∴ The equation of the tangent line is

asy
y − y1 = m(x − x1 ).

y + 6 = −2(x − 3)

En = −2x + 6

gin
2x + y = 0.

ee
Derivative of a function. Let f (x) be a given function. The derivative of f (x) at
f (x + h) − f (x)
any variable point x is defined by f ′ (x) = lim .
h→0 h
Example 1.24. If f (x) = x3 − x, find a formula for f ′ (x). rin
Solution.Given f (x) = x3 − x. g.n
f ′ (x) = lim
h→0

= lim
f (x + h) − f (x)
h
(x + h)3 − (x + h) − (x3 − x)
et
h→0 h
x + 3x h + 3xh2 + h3 − x − h − x3 + x
3 2
= lim
h→0 h
2 2
3x h + 3xh + h − h3
= lim
h→0 h
= lim 3x2 + 3xh + h2 − 1 = 3x2 − 1.
h→0

Download From : www.EasyEngineering.net


Download From : www.EasyEngineering.net

Differential calculus 21


Example 1.25. If f (x) = x, find the derivative of f . State the domain of f ′ .


Solution.Given f (x) = x.
f (x + h) − f (x)
f ′ (x) = lim
h→0 h
√ √
x+h− x
= lim
h→0 h
√ √ √ √
x+h− x x+h+ x
= lim × √ √
h→0 h x+h+ x

ww = lim  √
h→0 h
x+h−x
x+h+ x
√ 

w.E = lim  √
h→0 h
h
√ 
x+h+ x
1 1 1

asy
= lim  √
h→0
√ = √
x+h+ x
√ = √ .
x+ x 2 x

En
From this we notice that f ′ (x) exists if x > 0.
∴ The domain of f ′ is (0, ∞).

gin
ee
Example 1.26. Where is the function f (x) = |x| differentiable. [A.U. Nov. 2015]

x if x ≥ 0

rin



Solution. We have that |x| =  .
−x if x < 0


Consider x > 0. Then |x| = x. We choose h small enough such that x + h > 0.
∴ |x + h| = x + h. g.n
∴ When x > 0,

f ′ (x) = lim
f (x + h) − f (x)
et
h→0 h
|x + h| − |x|
= lim
h→0 h
x+h−x h
= lim = lim = lim 1 = 1.
h→0 h h→0 h h→0

∴ f is differentiable for x > 0.

Download From : www.EasyEngineering.net


Download From : www.EasyEngineering.net

22 Engineering Mathematics - I

Consider x = 0.

f (0 + h) − f (x)
f ′ (0) = lim
h→0 h
|0 + h| − |x|
= lim
h→0 h
|h|
= lim [if it exists]
h→0 h

Let us find the left and right limits.


|h| h
lim = lim = lim 1 = 1.

ww
h→0+ h h→0+ h h→0+
|h| −h
lim = lim = lim −1 = −1.
h→0− h h→0− h h→0
Since the two limits are different, f ′ (0) does not exist.

w.E ∴ f is not differentiable at x = 0.


Consider x < 0.

asy
In this case |x| = −x.

En
Choose h small so that x + h < 0.
∴ |x + h| = −(x + h)
∴ For x < 0,
gin
f ′ (x) = lim
h→0

= lim
h→0
h
−(x + h) − (−x)
h
ee
f (x + h) − f (x)

rin
= lim
h→0
−x − h + x
h
= lim
−h
h→0 h
= lim −1 = −1.
h→0
g.n
∴ f is differentiable for x < 0.
et
Combining all the three cases we obtain that f is differentiable at all x except 0.

ifx > 0
1





∴ f (x) = 
 .
−1 ifx < 0

Theorem. If f is differentiable at a, then f is continuous at a.


Proof. Given that f is differentiable at a.
f (x) − f (a)
f ′ (x) = lim exits. (1)
x→a x−a

Download From : www.EasyEngineering.net


Download From : www.EasyEngineering.net

Differential calculus 23

f (x) − f (a)
Now, f (x) − f (a) = (x − a)
x−a
Taking limits as x → a both sides we get
f (x) − f (a)
lim f (x) − f (a) = lim (x − a)
x→a x→a x−a
f (x) − f (a)
= lim × lim (x − a)
x→a x−a x→a

= f ′ (a) × 0 [by (1)]

lim f (x) − f (a) = 0 (2)


x→a

ww Also lim f (x) = lim f (a) + ( f (x) − f (a))


 
x→a x→a

= lim + lim ( f (x) − f (a))

w.E x→a

= f (a) + 0
x→a

[by (1)]

x→a

asy
lim f (x) = f (a)

∴ f (x) is continuous at a.

En
Result. The converse of the above theorem is false.

gin
i.e., If a function is continuous at a need not be differentiable at a.

For example consider the function f (x) = |x|.


lim f (x) = lim |x| = 0 = f (0).
x→a x→a
ee
But by example 1.26, f (x) = |x| is not differentiable at 0.
rin
g.n
Higher derivatives. If a function f is differentiable, then its derivative f ′ is also
a function. Hence, f ′ may have a derivative of its own denoted by ( f ′ )′ = f ′′ . f ′′ is
called !the second derivative of f . The second derivative of y = f (x) is denoted by
d dy
dx dx
d2 y
= 2 . In a similar way we can have the third derivative of f denoted by
dx
d d2 y
!
d3 y
et
( f ′′ )′ = f ′′′ . If y = f (x) then we have y′′′ = f ′′′ (x) = = . If this process is
dx dx2 dx3
continued we can get in general, the nth derivative of f denoted by f (n) is obtained
dn y
by differentiating f n times. Generally if y = f (x) then we write y(n) = f (n) (x) = n .
dx
Rules on differentiation

1. The sum rule. If f and g are both differentiable, then

Download From : www.EasyEngineering.net


Download From : www.EasyEngineering.net

24 Engineering Mathematics - I

d   d   d 
g(x) .

f (x) + g(x) = f (x) +
dx dx dx
Proof. Let F(x) = f (x) + g(x). Then,

F(x + h) − F(x)
F ′ (x) = lim
h→0 h
f (x + h) + g(x + h) − [ f (x) + g(x)]
= lim
h→0 h
[ f (x + h) − f (x)] + [g(x + h) − g(x)]
= lim
h→0 h
[ f (x + h) − f (x)]
= lim + lim [g(x + h) − g(x)]h

ww h→0 h
= f ′ (x) + g′ (x).
h→0

w.E 2. Constant multiple rule. If c is a constant and f is a differentiable function,


then
d  
c f (x) = c
d  
f (x) .
dx
asy
dx
Proof. Let F(x) = c f (x). Then,

En F ′ (x) = lim
h→0
F(x + h) − F(x)
h

gin = lim
h→0
c f (x + h) − c f (x)
h

ee
c[ f (x + h) − f (x)]
= lim
h→0 h
= c lim
h→0
[ f (x + h) − f (x)]
h
rin
= c f ′ (x) = c
d
dx
 
f (x) .
g.n
3. The difference rule.
d 
dx

f (x) − g(x) =
d 
dx
f (x) −
If f and g are both differentiable, then
 d 
dx

g(x) .
et
Proof. We have f (x) − g(x) = f (x) + (−)g(x).

d   d  
f (x) − g(x) = f (x) + (−1)g(x)
dx dx
d   d  
= f (x) + (−1)g(x) [by sum rule]
dx dx
d   d  
= f (x) + (−1) g(x) [by constant multiple rule]
dx dx

Download From : www.EasyEngineering.net


Download From : www.EasyEngineering.net

Differential calculus 25

d   d  
= f (x) − g(x) .
dx dx

4. The product rule. If f and g are both differentiable, then


d   d   d  
f (x) × g(x) = f (x) g(x) + g(x) f (x) .
dx dx dx

f (x + h)g(x + h) − f (x)g(x)
Proof. ( f (x)g(x))′ = lim
h→0 h
f (x + h)g(x + h) − f (x + h)g(x) + f (x + h)g(x) − f (x)g(x)
= lim
h→0 h

ww = lim
h→0
f (x + h){g(x + h) − g(x)} + g(x){ f (x + h) − f (x)}
(
h
g(x + h) − g(x) f (x + h) − f (x)
)

w.E = lim f (x + h)
h→0

= lim f (x + h) lim
h
+ g(x)
g(x + h) − g(x)
h
+ lim g(x) lim
f (x + h) − f (x)

asy
h→0 h→0

( f (x)g(x))′ = f (x)g′ (x) + g(x) f ′ (x).


h h→0 h→0 h

5. Quotient rule. En
 If f d and g are both differentiable, then
"
d f (x)
dx g(x)
#
=
d
g(x) dx
[g(x)] 2 gin
f (x) − f (x) dx g(x)
.

Proof.
f (x)
g(x)
!′ 


= lim 
h→0 


ee
 f (x+h) f (x) 
 g(x+h) − g(x) 
h







rin
= lim
h→0
(
f (x + h)g(x) − g(x + h) f (x)
hg(x + h)g(x)
)

g.n
= lim
h→0

= lim
(

(
hg(x + h)g(x)
g(x)[ f (x + h) − f (x)] − f (x)[g(x + h) − g(x)]
et
f (x + h)g(x) − f (x)g(x) + f (x)g(x) − g(x + h) f (x)

)
)

h→0 hg(x + h)g(x)


f (x+h)− f (x)
− f (x) g(x+h)−g(x)
 
 g(x)

 h h


= lim 

g(x + h)g(x)

h→0 
 

f (x + h) − f (x) g(x + h) − g(x)
lim g(x) − lim f (x)
h→0 h h→0 h
=
lim g(x + h)g(x)
h→0

Download From : www.EasyEngineering.net


Download From : www.EasyEngineering.net

26 Engineering Mathematics - I

f (x + h) − f (x) g(x + h) − g(x)


lim g(x) lim − lim f (x) lim
h→0 h→0 h h→0 h→0 h
=
g(x)g(x)
g(x) f ′ (x) − f (x)g′ (x)
= .
[g(x)]2

Derivatives of simple functions


d
1. If c is a constant, prove that (c) = 0.
dx
Proof. Let f (x) = c.

ww f (x + h) − f (x)
f ′ (x) = lim
h→0 h

w.E = lim
c−c
h→0 h
0
= lim = 0.
h→0 h

asy
2. If n is a positive integer, then prove that
Proof. Let f (x) = xn .
d n
dx
(x ) = nxn−1 .

En
f (x + h) − f (x)
f ′ (x) = lim
h→0

= lim
h
(x + h)n − xn
gin
ee
h→0 h
x + C1 xn−1 h + nC2 xn−2 h2 + nC3 xn−3 h3 + · · · + hn − xn
n n

rin
= lim
h→0 h
n−1 n(n−1) n−2 2 n(n−1)(n−2) n−3 3
nx h + 2 x h + x h + · · · + hn

g.n
6
= lim
h→0 h
n−1 n(n − 1) n−2 1 n(n − 1)(n − 2) n−3 2
= lim nx + x h + x h + · · · + hn−1
h→0

= lim nxn−1 + lim


h→0 h→0
2
n(n − 1) n−2 1
2
x h + lim
h→0
6
n(n − 1)(n − 2) n−3 2
6
et
x h + · · · + lim hn−1
h→0

= nxn−1 + 0 + 0 + · · · + 0 = nxn−1 .

d n
General rule. If n ia any real number, then (x ) = nxn−1 .
dx

Download From : www.EasyEngineering.net


Download From : www.EasyEngineering.net

Differential calculus 27

d x
3. Derivative of e x . Prove that (e ) = e x .
dx
eh − 1
Definition of the number e. e is a number such that lim = 1.
h→0 h
Proof. Let f (x) = e x .
f (x + h) − f (x)
f ′ (x) = lim
h→0 h
e x+h − e x
= lim
h→0 h
e e − ex
x h

ww = lim
h→0

= lim e
h
h
x e −1
!

w.E h→0

x
= lim e × lim
h
eh − 1
h
!
= ex × 1 = ex .

asy
h→0 h→0

Two important limits

1. Prove that lim


sin θ
θ→0 θ
= 1.
En
gin
Proof. Consider a circle with center
D

O and radius 1 unit.


subtends an angle θ at the centre
π
ee
Let arc(AB) B

rin E
and assume that 0 < θ < . Draw
2
BC perpendicular to OA. Let the
θ
0 C
g.n A

tangents at A and B meet at E.


Produce AE and OB to intersect at D. et
Now we have arcAB = θ.
BC BC
From ∆OAB, sin θ = OB = 1 .
∴ |BC| = sin θ.
From the figure we have
|BC| < |AB| < arcAB.

Download From : www.EasyEngineering.net


Download From : www.EasyEngineering.net

28 Engineering Mathematics - I

∴ sin θ < θ.
sin θ
⇒ < 1. (1)
θ
Again form the figure,
arcAB < |AE| + |EB|
∴ θ = arcAB < |AE| + |EB| < |AB| + |ED| = |AD| = |OA| tan θ = tan θ.

∴ θ < tan θ.
sin θ
i.e., θ <

ww cos θ
θ cos θ < sin θ
sin θ

w.E cos θ <


θ

Taking limit as θ → 0 we get


<1 [by(1)]

lim cos θ < lim


θ→0
sin θ
θ→0 θ asy
< lim 1
θ→0

1 < lim
θ→0 θ
sin θ
<1
En
sin θ
By squeeze theorem lim
θ→0 θ
gin= 1.

2. Prove that lim


θ→0
cos θ − 1
θ
= 0. ee rin
Proof. lim
θ→0
cos θ − 1
θ
= lim
θ→0
cos θ − 1 cos θ + 1
θ
2
×
cos θ + 1
− sin2 θ
g.n
et
cos θ − 1
= lim = lim
θ→0 θ(cos θ + 1) θ→0 θ(cos θ + 1)
sin θ sin θ 0
= − lim × lim = −1 × = 0.
θ→0 θ θ→0 (cos θ + 1) 1+1

cos θ − 1
∴ lim = 0.
θ→0 θ

Derivatives of trigonometric functions

d
1. Prove that (sin x) = cos x.
dx

Download From : www.EasyEngineering.net


Download From : www.EasyEngineering.net

Differential calculus 29

Proof. Let f (x) = sin(x)


f (x + h) − f (x)
f ′ (x) = lim
h→0 h
sin(x + h) − sin(x)
= lim
h→0 h
sin(x) cos(h) + cos(x) sin(h) − sin(x)
= lim
h→0 h
sin(x)(cos(h) − 1) + cos(x) sin(h)
= lim
h→0 h
!
sin(x)(cos(h) − 1) cos(x) sin(h)
= lim +

ww h→0

= lim sin(x)
h
(cos(h) − 1)
h
h

+ lim cos(x)
sin(h)
h
!

w.E
h→0 h→0
!
(cos(h) − 1) sin(h)
= sin(x) lim + cos(x) lim
h→0 h h→0 h

asy
= sin(x) × 0 + cos(x) × 1 = 0 + cos(x) = cos(x).

2. Prove that
d
dx
En
(cos x) = − sin x.

gin
Proof. Let f (x) = cos(x)
f (x + h) − f (x)
f ′ (x) = lim
h→0

= lim
h→0

= lim
ee h
cos(x + h) − cos(x)
h
cos(x) cos(h) − sin(x) sin(h) − cos(x)
rin
h→0

= lim
h
cos(x)(cos(h) − 1) − sin(x) sin(h)
g.n
h→0

= lim
h→0 h
h
cos(x)(cos(h) − 1) sin(x) sin(h)

(cos(h) − 1)

h
!

sin(h)
!
et
= lim cos(x) − lim sin(x)
h→0 h h→0 h
= cos(x) × 0 − sin(x) × 1 = − sin(x).

f ′ (x) = − sin x.

d
3. Prove that (tan x) = sec2 x.
dx

Download From : www.EasyEngineering.net


Download From : www.EasyEngineering.net

30 Engineering Mathematics - I

Proof. Let f (x) = tan(x)


d d
cos x dx (sin x) − sin x dx (cos x)
!
d d sin x
(tan x) = = 2
dx dx cos x cos x
cos x × cos x − sin x × (− sin x)
=
cos2 x
2
cos2 x + sin x 1
= = = sec2 x.
cos2 x cos2 x
d
4. Prove that (cosecx) = −cosecx cot x.
dx

ww Proof.
d
dx
(cosecx) =
d 1
dx sin x
!
=
d
sin x dx
sin2 x
d
(1) − 1 × dx (sin x)

w.E =
sin x × 0 − 1 × (cos x)

− cos x
sin2 x
−1 cos x
=

asy 2
sin x
= ×
sin x sin x
= −cosecx cot x.

En
d
5. Prove that (sec x) = sec x tan x.
dx

gin d
cos x dx d
(1) − 1 × dx (cos x)
!
d d 1
Proof. (sec x) = = 2
dx dx cos x cos x
=

=
sin x
2
=
ee
cos x × 0 − 1 × (− sin x)
cos2 x
sin x
cos x cos x cos x
×
1
= sec x tan x.
rin
6. Prove that
d
(cot x) = −cosec2 x. g.n
et
dx
d d
d d  cos x  sin x dx (cos x) − cos x × dx (sin x)
Proof. (cot x) = =
dx dx sin x sin2 x
sin x × (− sin x) − cos x × (cos x)
=
sin2 x
− sin2 x − cos2 x −(sin2 x + cos2 x) 1
= 2
= 2
= − 2 = −cosec2 x.
sin x sin x sin x

The chain rule. If g is differentiable at x and f is differentiable at g(x), then the


composite function F = f og defined by F(x) = f (g(x)) is differentiable at x and F ′ is

Download From : www.EasyEngineering.net


Download From : www.EasyEngineering.net

Differential calculus 31

given by the product F ′ (x) = f ′ (g(x)) × g′ (x).


Proof. Let y = F(x) = f (g(x))
Let u = g(x) ⇒ y = f (u)
Let ∆x be a small change in x, then ∆u is the corresponding change in u.
∆u = g(x + ∆x) − g(x).
Then the corresponding change in y is given by
∆y = f (u + ∆u) − f (u)

dy ∆y

ww Now, = lim
dx ∆x→0 ∆x
= lim
∆y ∆u
×

w.E ∆x→0 ∆u

= lim
∆y
∆x→0 ∆u
∆x
× lim
∆u
∆x→0 ∆x

= lim
asy
∆y
∆x→0 ∆u
dy du
× lim
∆u
∆x→0 ∆x
[∆u → 0 as ∆x → 0]

= ×
du dx
d En d

d
=
du
gin
( f (u)) ×
dx
(g(x)) = f ′ (u) × g′ (x)

[F(x)] = f ′ (g(x)) × g′ (x)


dx


ee
F ′ (x) = f ′ (g(x)) × g′ (x)

rin

Worked Examples

1 + cos 2x

g.n
Example 1.27. Evaluate limπ

Solution. limπ
1 + cos 2x
=
x→ 2

lim
(π − 2x)2

2 cos2 x
.

et
[A.U.Jan. 2015]

x→ 2 (π − 2x)2 x→ π2 (π − 2x)2
π
Let −x=y
2
π
As x → , y → 0
2
1 + cos 2x 2 cos2 x
∴ limπ = limπ 
x→ 2 (π − 2x)2 x→ 2
2
4 π−x 2

Download From : www.EasyEngineering.net


Download From : www.EasyEngineering.net

32 Engineering Mathematics - I

 
1 cos2 π2 − y
= lim
2 y→0 y2
1 sin2 y
= lim 2
2 y→0 y
!2
1 sin y
= lim
2 y→0 y
!2
1 sin y 1 1
= lim = × 12 = .
2 y→0 y 2 2

wwExample 1.28. If y = x8 + 12x5 − 4x4 + 10x3 − 6x + 5, find


dy
dx
.

w.E
Solution. Given, y = x8 + 12x5 − 4x4 + 10x3 − 6x + 5
dy d  8

asy

= x + 12x5 − 4x4 + 10x3 − 6x + 5
dx dx
d 8 d d d d d
=
dx
En
(x ) + (12x5 ) − (4x4 ) + (10x3 ) − (6x) + (5)
dx
d
dx
d
dx
d
dx
d
= 8x8−1 + 12 (x5 ) − 4 (x4 ) + 10 (x3 ) − 6 (x) + 0
dx

dx
gin dx dx dx
= 8x7 + 12 × 5(x5−1 ) − 4 × 4(x4−1 ) + 10 × 3(x3−1 ) − 6 × 1

ee
= 8x7 + 60x4 − 16x3 + 30x2 − 6.

rin
ex
Example 1.29. If y = − x, find
dy
dx
d2 y
and 2 .
dx g.n
Solution. Given y = e x − x.

dy
dx
= ex − 1
et
d2 y
= ex .
dx2

dy
Example 1.30. If y = x2 sin x, find .
dx
Solution. Given y = x2 sin x.

Download From : www.EasyEngineering.net


Download From : www.EasyEngineering.net

Differential calculus 33

Applying the product rule we obtain

dy d  2 
= x sin x
dx dx
d d
= x2 (sin x) + sin x (x2 )
dx dx
= x2 × (cos x) + sin x × (2x)

= x2 cos x + 2x sin x.

sec x dy
Example 1.31. If f (x) = , find . For what values of x, does the graph of

ww 1 + tan x
f , has a horizontal tangent.
Solution. Given f (x) =
sec x
.
dx

w.E 1 + tan x
By the quotient rule, we have

asy
d d
(1 + tan x) dx (sec x) − sec x dx (1 + tan x)
f ′ (x) =
(1 + tan x)2

En
 
(1 + tan x)(sec x tan x) − sec x 0 + sec2 x
=
(1 + tan x)2

=
gin
sec x tan x + sec x tan2 x − sec3 x
(1 + tan x)2

=
ee
sec x(tan x + tan2 x − sec2 x)
(1 + tan x)2
sec x(tan x − (sec2 x − tan2 x))
rin
=
(1 + tan x)2
sec x(tan x − 1) g.n
f ′ (x) =
(1 + tan x)2
sec x(tan x − 1)
(1 + tan x)2
. et
The points where the graph of f has a horizontal tangent are given by f ′ (x) = 0

sec x(tan x − 1)
i.e., =0
(1 + tan x)2
⇒ sec x(tan x − 1) = 0

But sec x , 0

Download From : www.EasyEngineering.net


Download From : www.EasyEngineering.net

34 Engineering Mathematics - I

∴ tan x − 1 = 0
π
i.e., tan x = 1 ⇒ x = nπ + , n is an integer.
4

Example 1.32. Find the points on the curve y = x4 − 6x2 + 4 where the tangent line
is horizontal.
dy
Solution. Horizontal tangent occur at points where = 0.
dx

i.e.,4x3 − 6 × 2x = 0

ww 4x3 − 12x = 0

4x(x2 − 3) = 0

w.E √
i.e.,x = 0 or x2 − 3 = 0 ⇒ x2 = 3 ⇒ x = ± 3.

asy
When x = 0, y = 4.
The point is (0, 4).

When x = 3, y =
En
 √ 4  √ 2
3 − 6 3 + 4 = 9 − 18 + 4 = −5.

gin

The point is ( 3, −5).
√  √ 4  √ 2
When x = − 3, y = − 3 − 6 − 3 + 4 = 9 − 18 + 4 = −5.

The point is (− 3, −5).
ee √

rin√
The given curve has horizontal tangents at the points (0, 4), ( 3, −5) and (− 3, −5).

g.n
Example 1.33. At what point on the cure y = e x , is the tangent line parallel to the
line y = 2x.
Solution. Given y = e x ⇒ y′ = e x .
slope of the tangent at x = e x .
et
Since the tangent line is parallel to the line y = 2x,
Slope of the tangent = 2.
i.e., e x = 2 ⇒ x = log 2.
When x = log 2, y = elog 2 = 2.
∴ The required point is (log 2, 2).

Download From : www.EasyEngineering.net


Download From : www.EasyEngineering.net

Differential calculus 35

Example 1.34. If f (x) = xe x find f (n) (x).


Solution. Given f (x) = xe x .

d
f ′ (x) = xe x

dx
d x d
=x e + e x (x) = xe x + e x .1 = e x (x + 1).
dx dx
d
f ′′ (x) = (x + 1)e x

dx
d x d
= (x + 1) e + e x (x + 1)
dx dx

ww = (x + 1)e x + e x .1 = e x (x + 1 + 1) = e x (x + 2).
d

w.E f ′′′ (x) = (x + 2)e x



dx
d x d
= (x + 2) e + e x (x + 2)
dx dx

asy
= (x + 2)e x + e x .1 = e x (x + 2 + 1) = e x (x + 3).

En
Applying this process successively n times we get

gin f (n) (x) = e x (x + n).

Example 1.35. If y =
x2 + x − 2
3
x +6

x2 + x − 2
ee
, find
dy
dx
.

rin
Solution. Given y =

dy
x3 + 6
d
(x3 + 6) dx
.
d
(x2 + x − 2) − (x2 + x − 2) dx (x3 + 6) g.n
dx

=
=
(x3 + 6)2
(x3 + 6)(2x + 1) − (x2 + x − 2)(3x2 )
(x3 + 6)2
et
2x + x + 12x + 6 − 3x4 − 3x3 + 6x2
4 3
=
(x3 + 6)2
−x4 − 2x3 + 6x2 + 12x + 6
= .
(x3 + 6)2

ex
Example 1.36. Find the equation of the tangent line to the curve y = at the
1 + x2

Download From : www.EasyEngineering.net


Download From : www.EasyEngineering.net

36 Engineering Mathematics - I

e
point (1, ).
2
ex
Solution. Given y =
1 + x2
ex
!
dy d
=
dx dx 1 + x2
d d
(1 + x2 ) dx (e x ) − e x dx (1 + x2 )
=
(1 + x2 )2
(1 + x2 )e x − e x 2x
=
(1 + x2 )2

ww !
=
e x (1 + x2 − 2x) e x (1 − x)2
(1 + x2 )2
=
(1 + x2 )2

w.E
dy e(1 − 1)
e ! = = 0.
dx 1, (1 + 1)2
2

asy
∴ Slope of the tangent m = 0.
Equation of the tangent line is

y − y1 = m(x − x1 )
e En
y − = 0(x − 1)
2
e
y − = 0. gin
2



ee
Example 1.37. Find F ′ (x) if F(x) = x2 + 1.

Solution. Let y = x2 + 1, let u = x2 + 1 and y = u.
rin
NowF ′ (x) =
dy du
× .
du dx g.n
Example 1.38. Find
1 −1
2
dy
1
= u 2 × 2x = √ x = √
u
x
2
x +1
.

if (i) y = sin(x2 ) (ii) y = sin2 x.


et
dx
Solution. (i) Given y = sin(x2 ).
Let u = x2 . Then y = sin u.
dy dy du
Now = × .
dx du dx
= cos u × 2x = cos(x2 )2x = 2x cos(x2 ).

Download From : www.EasyEngineering.net


Download From : www.EasyEngineering.net

Differential calculus 37

(ii) Given y = sin2 x


Let u = sin x. Then y = u2 .

dy dy du
Now = × .
dx du dx
= 2u cos x = 2 sin x cos x = sin 2x.

Example 1.39. Differentiate the following functions

ww 3
(i) y = (x − 1) 100
.
(iv) y = esin x .

w.E
(ii) y = √
3
1
x2 + x + 1
.
(v) y = sin(cos(tan x)).

(vi) y = esec 3x .

(iii) y = (2x + 1)5 (x3 − x + 1)4 . asy (vii) y =


x−2
2x + 1
!9
.

En
Solution. (i) Given y = (x3 − 1)100

gin
Let u = x3 − 1. Then y = u100 .

Now
dy dy du
= × .
dx du dx ee
= 100u99 × 3x2 = 300x2 (x3 − 1)99 . rin
(ii) Given y = 1 g.n
et

3
x2 +x+1
−1
Let u = x2 + x + 1. Then y = u 3 .

dy dy du
Now = × .
dx du dx
−1 −1 −1
= u 3 × (2x + 1)
3
−1 −4 −1 −(2x + 1)
= u 3 × (2x + 1) = 4 × (2x + 1) = 4
.
3 3u 3 3(x2 + x + 1) 3

(iii) Given y = (2x + 1)5 (x3 − x + 1)4

Download From : www.EasyEngineering.net


Download From : www.EasyEngineering.net

38 Engineering Mathematics - I

−1
Let u = x2 + x + 1. Then y = u 3 .

dy d  
Now = (2x + 1)5 (x3 − x + 1)4
dx dx
d  3  d  
= (2x + 1)5 (x − x + 1)4 + (x3 − x + 1)4 (2x + 1)5
dx dx
   
= (2x + 1) 4(x − x + 1) (3x − 1) + (x − x + 1)4 5(2x + 1)4 × 2
5 3 3 2 3

= 4(2x + 1)5 (x3 − x + 1)3 (3x2 − 1) + 10(x3 − x + 1)4 (2x + 1)4

= 2(2x + 1)4 (x3 − x + 1)3 {2(2x + 1)(3x2 − 1) + 5(x3 − x + 1)}

ww = 2(2x + 1)4 (x3 − x + 1)3 {12x3 − 4x + 6x2 − 2 + 5x3 − 5x + 5)}

w.E
(iv) Given y = esin x
= 2(2x + 1)4 (x3 − x + 1)3 {17x3 + 6x2 − 9x + 3)}.

dy
dx asy
= esin x cos x.
(v) Given y = sin (cos(tan x)).

En
dy
dx gin
= cos (cos(tan x)) (− sin(tan x)) sec2 x

(vi) Given y = esec 3x .


ee
= − cos (cos(tan x)) sin(tan x) sec2 x.

rin
dy
= esec 3x × sec 3x × tan 3x × 3 = 3 sec 3x tan 3xesec 3x .
g.n
dx

(vii) Given y =
x−2
!9
.
et
2x + 1

!8 !
dy x−2 (2x + 1) × 1 − (x − 2) × 2
=9
dx 2x + 1 (2x + 1)2
!8
(x − 2)8
!
x−2 2x + 1 − 2x + 4
=9 = 45 .
2x + 1 (2x + 1)2 (2x + 1)10

Download From : www.EasyEngineering.net


Download From : www.EasyEngineering.net

Differential calculus 39

1.5 Implicit differentiation, logarithmic differentiation and


Inverse trigonometric function.

Whenever an equation containing two variables x and y, it is not always


dy
possible to express y interns of x and it is not easy to find directly. In such
dx
case, we differentiate both sides with respect to x and then solve the resultant
dy
equation for .
✎ ☞ dx
Worked Examples
✍ ✌

wwExample 1.40. If x2 + y2 = 25, find


dy
dx
. Find also the equation of the tangent to the

w.E
circle at the point (3, 4).
Solution. Given x2 + y2 = 25.

asy
Differentiating both sides w.r.to x we get
dy

En
2x + 2y =0
dx
dy
x+y =0

y
dx
dy
= −x ⇒ gin
dy −x
= .

ee
dx dx y
dy −3
At (3, 4)
dx
=
4
.
−3 rin
Slope of the tangent at (3, 4) = m =
Equation of the tangent is
4
.

g.n
y−4=
−3
4
(x − 3)

4y − 16 = −3x + 9
et
3x + 4y = 16 + 9

3x + 4y = 25.

dy
Example 1.41. If x3 +y3 = 6xy, find . Find the tangent to the Folium of Descartes
dx
x3 + y3 = 6xy at the point (3, 3). At what point in the first quadrant is the tangent

Download From : www.EasyEngineering.net


Download From : www.EasyEngineering.net

40 Engineering Mathematics - I

line horizontal.
Solution. Given x3 + y3 = 6xy.
Differentiating both sides w.r.to x we get
" #
2 2 dy dy
3x + 3y =6 x +y×1
dx dx
" #
2 2 dy dy dy
x +y = 2 x + y = 2x + 2y
dx dx dx
dy dy
y2 − 2x = 2y − x2
dx dx

ww dy 2
dx
(y − 2x) = 2y − x2
dy 2y − x2

w.E At(3, 3)
=
dx y2 − 2x
dy 2 × 3 − 32
=
.

= −1.

asy
dx 32 − 2 × 3

Slope of the tangent at (3, 3) = m = −1 .

En
The equation of the tangent is y − 3 = −1(x − 3) = −x + 3 ⇒ x + y = 6.
dy

2y − x
i.e., 2
2
=0gin
The points at which the tangent line is horizontal are given by
dx
= 0.

y − 2x
i.e., 2y − x2 = 0
i.e., y = .
x2
2
ee rin
Substituting y =
x2
2
g.n
in the equation of the curve we obtain

3
x +

x3 +
x2
2
x6
!3

!
= 6x ×

= 3x3
x2
2 et
8
x6
− 2x3 = 0
8
3
!
3 x
x −2 =0
8
 
x3 x3 − 16 = 0

Download From : www.EasyEngineering.net


Download From : www.EasyEngineering.net

Differential calculus 41

x3 = 0 or x3 − 16 = 0

x3 = 0 or x3 = 16

But x , 0
4
∴ x3 = 16 = 24 ⇒ x = 2 3 .
4  4 2 8 8 5
when x = 2 3 , y = 12 2 3 = 12 2 3 = 2 3 −1 = 2 3 .
4 5
∴ In the first quadrant, the tangent line is horizontal at (2 3 , 2 3 ).

dy

wwExample 1.42. Find


dx
if sin(x + y) = y2 cos x.
Solution. Given sin(x + y) = y2 cos x.

w.E Differentiating both sides w.r.to x we get

cos(x + y) 1 +
dy
!
= y2 (− sin x) + cos x × 2y
dy

asy dx
cos(x + y) + cos(x + y)
dy
= −y2 sin x + 2y cos x
dy
dx

dy
En
cos(x + y) − 2y cos x
dx
dy 2
= −y sin x − cos(x + y)
dx

gin
dx dx
dy  
(cos(x + y) − 2y cos x) = − y2 sin x + cos(x + y)
dx

ee
 
2
dy − y sin x + cos(x + y)
=
dx (cos(x + y) − 2y cos x)
y2 sin x + cos(x + y)
rin
g.n
= .
2y cos x − cos(x + y)

et
dy
Example 1.43. If sin y = x sin(a + y) find .
dx
Solution. Given sin y = x sin(a + y)
sin y
∴x=
sin(a + y)
Differentiating with respect to y we get

dx sin(a + y) cos y − sin y cos(a + y) sin(a + y − y) sin(a)


= 2
= 2
=
dy sin (a + y) sin (a + y) sin2 (a + y)
dy sin2 (a + y)
∴ = .
dx sin(a)

Download From : www.EasyEngineering.net


Download From : www.EasyEngineering.net

42 Engineering Mathematics - I

p √ dy 1
Example 1.44. If x 1 + y + y 1 + x = 0, prove that =− .
p √ p dx √ (1 + x)2
Solution. Given x 1 + y + y 1 + x = 0 ⇒ x 1 + y = −y 1 + x
squaring both sides we get

x2 (1 + y) = y2 (1 + x)

x2 + x2 y = y2 + y2 x

x2 + x2 y − y2 − y2 x = 0

x2 − y2 + x2 y − y2 x = 0

ww (x − y)(x + y) + xy(x − y) = 0

w.E (x − y)(x + y + xy) = 0

∴ x − y = 0 or x + y + xy = 0

asy
i.e.,x = y or x + y + xy = 0

But x , y.
∴ x + y + xy = 0 En (1)

gin
Differentiating both sides w.r.to x we get

ee
!
dy dy
1+ + x +y =0
dx dx
1+
dy
dx
dy
+x +y=0
dx rin
dy
dx
(1 + x) = −y − 1
g.n
dy −(y + 1)
dx
=
(1 + x)
.

But form (1) we have y(1 + x) = −x ⇒ y =


−x
.
et
x
1+x
dy 1 − 1+x 1+x−x 1
∴ =− =− 2
=− .
dx 1+x (1 + x) (1 + x)2
r q √ dy
Example 1.45. If y = x + x + x + · · · ∞, find .
r dx
q √
Solution. Given y = x + x + x + · · · ∞.

Download From : www.EasyEngineering.net


Download From : www.EasyEngineering.net

Differential calculus 43


y= x+y
y2 = x + y.
Differentiating w.r.to x we get
dy dy
2y =1+
dx dx
dy
(2y − 1) = 1
dx
dy 1
= .
dx (2y − 1)
r q √ dy

wwExample 1.46. If y = sin x + sin x + sin x + · · · ∞, find


r q √
Solution. Given y = sin x + sin x + sin x + · · · ∞.
dx
.

w.E p
y = sin x + y.
Squaring on both sides we get
y2 = sin x + y.
asy
En
Differentiating w.r.to x we get
dy dy
2y = cos x +
dx dx

dy
dy dy
2y −
dx dx
= cos x
gin
dx
dy
(2y − 1) = cos x

=
cos x
dx (2y − 1)
. ee rin
Example 1.47. Find y′′ if x4 + y4 = 16.
Solution. Given x4 + y4 = 16 g.n
Differentiating w.r.to x we get

4x3 + 4y3
dy
=0
et
dx
dy
x3 + y3 =0
dx
3 dy
y = −x3
dx
dy x3
=− .
dx y3

Download From : www.EasyEngineering.net


Download From : www.EasyEngineering.net

44 Engineering Mathematics - I

Again differentiating w.r.to x


dy
d2 y y3 × 3x2 − x3 × 3y2
=− dx
dx2 y6
! !
2 2 dy 2 2 dy
−3x y y − x −3x y y − x
dx dx
= =
y6 y6
−x3
!!
2
−3x y − x 3  
y −3x2 y4 + x4 −3x2 × 16 −48x2
= = = = .
y4 y7 y7 y7

wwDerivative of Inverse Trigonometric functions

w.E 1. Find
dy
dx
if y = sin−1 x.
Solution. Given y = sin−1 x.
sin y = x.
asy
Differentiating w.r.to x
cos y
dy
dx
=1
En
gin
dy 1 1 1 1
= = p = q = √ .
dx cos y cos2 y 2 1 − x 2
1 − sin y

dy
2. If y = cos−1 x, find
dx
.
Solution. Given y = cos−1 x.
ee rin
cos y = x.
Differentiating w.r.to x g.n
− sin y
dy
=
dx sin y
dy
dx
−1
=1

= q
−1
2
= p
1
−1
− cos 2y
= √
1
−1
− x 2
.
et
sin y

dy
3. If y = tan−1 x, find
.
dx
Solution. Given y = tan−1 x.
tan y = x.
Differentiating w.r.to x

Download From : www.EasyEngineering.net


Download From : www.EasyEngineering.net

Differential calculus 45

dy
sec2 y =1
dx
dy 1 1 1
= = = .
dx sec y 1 + tan y 1 + x2
2 2

dy
4. If y = cot−1 x, find
.
dx
Solution. Given y = cot−1 x.
cot y = x.
Differentiating w.r.to x
dy
−cosec2 y

ww
=1
dx
dy −1 −1 −1
= = = .
dx cosec y 1 + cot y 1 + x2
2 2

w.E dy
5. If y = sec−1 x, find
dx
.
Solution. Given y = sec−1 x.
sec y = x.
asy
sec y tan y
dy
En
Differentiating w.r.to x
=1
dy
=
dx
1
gin
= p
1
= p
1
= √
1
dx sec y tan y x tan2 y x sec2 y − 1 x x2 − 1
.

dy
6. If y = cosec−1 x, find
dx
.
Solution. Given y = cosec−1 x.
ee rin
cosecy = x.
g.n
Differentiating w.r.to x
−cosecy cot y
dy
=
−1
dy
dx
=1

= p
−1
= p
−1
= √
−1
dx cosecy cot y x cot y x cosec y − 1 x x2 − 1
2 2
.
et
✎ ☞
Worked Examples
✍ ✌
1 √ 
Example 1.48. Find the derivatives of (i) −1 (ii) x tan−1 x.
sin x
1 
−1 −1

Solution. (i) Let y = = sin x .
sin−1 x

Download From : www.EasyEngineering.net


Download From : www.EasyEngineering.net

46 Engineering Mathematics - I

dy  −2 1 1
= (−1) sin−1 x √ = 2 √ .
dx 1 − x2 sin−1 x

1 − x2
√ 
(ii) Let y = x tan−1 x .

dy 1 −1 √ x .1 = x −1 √ x .
   
1
= (x) √ 2 × 2√ x
+ tan 2(1+x) + tan
dx 1 + ( x)
√ 
Example 1.49. Differentiate w.r.to x (i) sin−1 x (ii) (1 + x2 ) tan−1 x
 cos x 
(iii) sec−1 (x4 ) (iv) tan−1 .
√1 + sin x
Solution. (i) Let y = sin−1 ( x).
dy 1 1 1 1

ww dx
= q
1− x
 √ 2 × 2 √ x = 2 √ x √1 − x = 2 √ x(1 − x) .

(ii) Let (1 + x2 ) tan−1 x.

w.E dy
dx
= (1 + x2 ) ×
1
1 + x2
+ tan−1 x × 2x = 1 + 2x tan−1 x.

1 asy
(iii) Let sec−1 (x4 ).
dy
× 4x3 = √
4
.

En
= q
dx 4 4
2 x x 8−1
x x −1
 cos x 
(iv) Let y = tan−1
1 + singin
x 
.

ee

 sin π2 − x 
 
−1 
= tan    .

rin

π
1 + cos 2 − x
  
 sin 2 π4 − 2x 

= tan−1  
π
1 + cos 2 4 − 2 x 
  .
g.n
et
   
 2 sin π4 − 2x cos π4 − 2x 

= tan−1     .
2 π x
2 cos 4 − 2

  π x 
= tan−1 tan − .
π x 4 2
y= −
4 2
dy −1
= .
dx 2
!
−1 2x dy
Example 1.50. If y = tan 2
find .
1−x dx

Download From : www.EasyEngineering.net


Download From : www.EasyEngineering.net

Differential calculus 47

!
−1 2x
Solution. Given y = tan .
1 − x2
Let x = tan θ !
−1 2 tan θ
∴y = tan = tan−1 (tan 2θ) = 2θ
1 − tan2 θ
y = 2 tan−1 x.
dy 2
= .
dx 1 + x2
  dy
Example 1.51. If sin−1 3x − 4x3 find .
  dx
−1 3
Solution. Let y = sin 3x − 4x .

ww Let x = sin θ.
y = sin−1 (3 sin θ − 4 sin3 θ) = sin−1 (sin 3θ) = 3θ = 3 sin−1 x.

w.E dy
dx
= √
3
1 − x2
.

Example 1.52. If y = tan−1


asy
 a−x 

 a − x  1 + ax
find
dy
dx
.

Solution. y = tan−1
1 + ax
En
Let a = tan α, x = tan θ.
∴ y = tan−1
gin
 tan α − tan θ 
1 + tan α tan θ
= tan−1 (tan(α − θ)) = α − θ = α − tan−1 (x).

ee
dy −1
= .
dx 1 + x2

Logarithmic Differentiation
dy rin
Result. If y = loge x, find
Solution. Given y = loge x
dx
.
g.n
∴ x = ey .
Differentiating w.r.to y
dx
= ey = x
et
dy
dy 1
= .
dx x
dy
Note. (i) If y = b x find .
dx
Solution. Given y = b = e x log b
x

Differentiating w.r.to x

Download From : www.EasyEngineering.net


Download From : www.EasyEngineering.net

48 Engineering Mathematics - I

dy
= e x log b log b = b x log b.
dx
dy
(ii) If y = logb x find .
dx
Solution. Given y = logb x
∴ by = x.
Differentiating w.r.to x
dy
by log b = 1.
dx
dy
= y1 = 1 .
dx b log b x log b ✎ ☞

ww Worked Examples
✍ ✌

w.E
Example 1.53.! Differntiate (i) log(x3 + 1) (ii) log(sin x) (iii) log(2 + sin x)
x+1 p

asy
(iv) log √ (v) log |x| (vi) log(x).
x+2

Solution. (i) Let y = log(x3 + 1).


En
dy
= 3
1
dx x + 1
3x2 = 3
gin
3x2
x +1
.

(ii) Let y = log(sin x).


dy
=
1
dx sin x
ee
cos x = cot x.
rin
(iii) Let y = log10 (2 + sin x).
dy 1 cos x g.n
=
dx (2 + sin x) log 10

(iv) Let y = log √


x+1
x+2
!
.
cos x =

√ 
(2 + sin x) log 10
.

et
= log (x + 1) − log x + 2 .

= log (x + 1) − log (x + 2)1/2 .


1
= log (x + 1) −
log (x + 2) .
2
dy 1 1
= − .
dx (x + 1) 2(x + 2)

Download From : www.EasyEngineering.net


Download From : www.EasyEngineering.net

Differential calculus 49


log x , if x > 0



(v) Let y = log |x| =  .


log(−x) , if x < 0


 
1 1
, if x > 0  x , if x > 0
 
dy 
 

x
 
= = .
dx 


 1 (−1) , if x < 0 

  1 , if x < 0

−x x
dy 1
∴ = for allx , 0.
dx x
(vi) Let y = log x = log x 1/2 .
p 

ww dy 1
dx 2
= log x −1/2 =
 1
p
1
x 2x log x
.

w.E
Example 1.54. If y =
3
3 √
x 4 x2 + 1
(3x
√ + 2) 5
, find
dy
dx
.

Solution. Given y =
asy
x 4 x2 + 1
(3x + 2)5
Taking log on both sides we get
 
En
p
log y = log x3/4 + log x2 + 1 − log(3x + 2)5

=
3
4
1
gin
log x + log(x2 + 1) − 5 log(3x + 2).
2

1 dy 3 1 1
= × + × 2
y dx 4 x 2 x + 1
1 ee
Differentiating w.r.to x we get
2x − 5
1
3x + 2√
×3=
3
+ 2
x

15
4x x + 1 3x + 2
rin
g.n
" # 3 " #
dy 3 x 15 2
x x +1 3
4 x 15
∴ =y + 2 − = + − .
dx 4x x + 1 3x + 2 (3x + 2)5 4x x2 + 1 3x + 2

Example 1.55. If y = x
Solution. Given y = x x .



x
find
dy
dx
.
et
log y = x log x.
Differentiating w.r.t x
1 dy √ 1 1
= x + log x √
y dx x 2 x
1 1
= √ + √ log x
x 2 x

Download From : www.EasyEngineering.net


Download From : www.EasyEngineering.net

50 Engineering Mathematics - I

!
1 1
= √ 1 + log x
x 2
1 
= √ 2 + log x
2 x

dy 1 x x
=y √ = √ .
dx 2 x 2 + log x 2 x 2 + log x
d
Example 1.56. Find (sin x)cos x .

[A.U. Jan. 2015]
dx
Solution. Let y = (sin x)cos x .

ww Taking logarithms on both sides we get


log y = log (sin x)cos x = cos x log (sin x).

w.E Differentiating w.r.to x.

1 dy 1

asy
= cos x cos x + log(sin x)(− sin x)
y dx sin x
cos2 x
= − sin x log(sin x)
dy
=y
sin
" x2
cos x
En #
− sin x log(sin x) = (sin x)
" 2
cos x cos x
#
− sin x log(sin x) .
dx sin x
gin dy
sin x

Example 1.57. If y = (sin x) x , find


Solution. Let y = (sin x) x .
Taking log both sides we get
ee dx
.

rin
log y = log (sin x) x = x log sin x.
Differentiating w.r.t. x. g.n
1 dy
y dx
=x
1
sin x
cos x + log sin x.1 et
= x cot x + log sin x.
dy
= y x cot x + log sin x = (sin x) x x cot x + log sin x .
   
dx

x··· dy
xx
Example 1.58. If y = x , find .
dx
···∞
xx
Solution. Let y = x x ⇒ y = xy .

Download From : www.EasyEngineering.net


Download From : www.EasyEngineering.net

Differential calculus 51

Taking log both sides we get


log y = log xy = y log x.
Differentiating w.r.t. x.
1 dy 1 dy
= y + log x .
y dx x dx
1 dy dy y
− log x = .
y dx dx
! x
dy 1 y
− log x = .
dx y ! x
dy 1 − y log x y
= .

ww dx
dy
=
y
y2

dx x(1 − y log x)
x
.

w.E
Hyperbolic functions

e x − e−x
We define sinh x =
2
asy
e x + e−x
cosh x =
2
sinh x En
gin
tanh x =
cosh x
1
cosechx =

sechx =

coth x =
sinh x
1
cosh x
cosh x
ee rin
Hyperbolic identities
sinh x
g.n
1. sinh(−x) = − sinh x. et
2. cosh(−x) = cosh x.

3. cosh2 x − sinh2 x = 1.

4. 1 − tanh2 x = sech2 x.

5. sinh(x + y) = sinh x cosh y + cosh x sinh y.

Download From : www.EasyEngineering.net


Download From : www.EasyEngineering.net

52 Engineering Mathematics - I

6. cosh(x + y) = cosh x cosh y + sinh x sinh y.

Derivatives of hyperbolic functions.


d
1. Prove that (sinh x) = cosh x.
dx
d e x − e−x e x + e−x
! !
d 1 x −x
Proof. (sinh x) = = (e − e (−1)) = = cosh x.
dx dx 2 2 2
In the same way we can establish the following.
d
2. (cosh x) = sinh x.
dx

ww 3.
d
dx
(tanh x) = sech2 x.

w.E 4.
d
dx
d
(cosechx) = −cosechx coth x.

asy
5. (sechx) = sechx tanh x.
dx
d
6. (coth x) = −cosech2 x.
dx
Inverse hyperbolic functions. En
−1
 p
2

1. Prove that sinh x = log x + x + 1 . gin
Proof. Let y = sinh−1 x.
∴ x = sinh y =
i.e., ey − e−y = 2x.
2
ee
ey − e−y
.
rin
1
ey − y − 2x = 0.
e g.n
e2y − 2xey − 1 = 0.
This is a quadratic
ey =

2
=
in ey .√Solving for ey we get
2x ± 4x2 + 4 x ± x2 + 1
1
p
= (x ± x2 + 1).
et
We have ey > 0 always.

But x − x2 − 1 < 0.

∴ ey = x − x2 + 1 is not possible.
√ √
∴ ey = x + x2 + 1 y = log(x + x2 + 1).
In a similar way we can prove the following.

Download From : www.EasyEngineering.net


Download From : www.EasyEngineering.net

Differential calculus 53

 p 
2. cosh−1 x) = log x + x2 − 1 .
!
−1 1 1+x
3. tanh x) = log .
2 1−x
Differentiation of Inverse hyperbolic functions.
d 1
1. Prove that (sinh−1 x) = √ .
dx x2 + 1
Proof. y = sinh−1 x.
∴ sinh y = x.

ww Differentiating w.r.t. x we get


cosh y
dy
= 1.

w.E dy
=
dx

dx cosh y
1
= q
1
cosh2 y
= q
1
1 + sinh2 y
= √
1
1 + x2
.

d 1 asy
In a similar way we can prove the following.

2.
dx
(cosh−1 x) = √
x2 − 1
.

En
3.
d
dx
(tanh−1 x) =
1
1 − x2
.
gin
4.

5.
d
dx

d
(cosech−1 x) = √

(sech−1 x) = √
−1
|x| 1 + x2
−1
.
.
ee rin
dx
d
(coth−1 x) =
x 1 − x2
1 g.n
6.
dx
Exercise 1 (E)
1 − x2
.

et
1. Find the derivatives of the following functions.

(i) x2 − 4xy + y2 = 4 (iv) y cos x = x2 + y2



(ii) x4 + x2 y2 + y3 = 5 (v) x + y = x4 + y4
x
x2
(iii) x+y = y2 + 1 (vi) e y = x − y

Download From : www.EasyEngineering.net


Download From : www.EasyEngineering.net

54 Engineering Mathematics - I

(ix) cos(xy) = 1 + sin y


(vii) tan−1 (x2 y) = x + xy2

(viii) sin(xy) = cos(x + y) (x) x sin y + y sin x = 1

dy
2. If (sin x)cos y = (sin y)cos x , find .
dx

sin x··· dy
3. If y = (sin x)sin x , find .
dx

ww 4. If x1+y + y1+x = a, where a is a constant, find


dy
dx
.

w.E 5. Find the derivatives of the following using logarithmic differentiation.

(iv) xsin x
(i) (x2 + 2)2 (x4 + 4)4

(ii)
q
(x−1) asy (v) (cos x) x

En
(x4 +1)
1
(iii) x x (vi) (tan x) x

gin
6. Find the equation of the tangent line to the curve y sin 2x = x cos 2y at ( π2 , π4 ).

point (2, 1).


ee
7. Find the equation of the tangent line to the hyperbola x2 − xy − y2 = 1 at the

rin
8. Find the equation to the tangent line to the Cardioid x2 + y2 = (2x2 + 2y2 − x)2
  g.n
at 0, 21 .

9. Find the derivative of the following functions.



et
√ (v) cosh−1 ( x).
(i) tanh x.   √
(vi) x sinh−1 x
3 − 9 + x2 .
(ii) sinh(log(x)).

(iii) ecosh 3x . (vii) coth−1 (sec x).



(iv) x coth 1 + x2 . (viii) tanh−1 (sin x).

Download From : www.EasyEngineering.net


Download From : www.EasyEngineering.net

Differential calculus 55

10. At what point of the curve y = cosh x does the tangent have slope 1.

Definition. Let c be a number in the domain D of a function f . Then f (c) is the


absolute maximum value of f on D if f (c) ≥ f (x) for all x in D. f (c) is the absolute
minimum value of f on D if f (c) ≤ x for all x in D.
Maximum and minimum values of f are called extreme values of f .
Definition. The number f (c) is a local maximum value of f if f (c) ≥ f (x) when x
is near c. f (c) is the local minimum value of f if f (c) ≤ f (x) when x is near c.

wwIncreasing and decreasing test

(i) If f ′ (x) > 0, on an interval, then f is increasing on that interval.

w.E
(ii) If f ′ (x) < 0, on an interval, then f is decreasing on that interval.

asy
Fermat’s theorem. If f has a local maximum or minimum at c, and f ′ (c) exits,
then f ′ (c) = 0.

En
Critical number. A critical number of a function f is a number c in the domain

gin
of f such that either f ′ (c) = 0 or f ′ (c) does not exist.
Result. If f has a local maximum or minimum at c, then c is a Critical number
of f .

First derivative test.


ee rin
Suppose that c is a critical number of a continuous
function f .
g.n
et
(i) If f ′ changes from positive to negative at c, then f has a local maximum at c.

(ii) If f ′ changes from negative to positive at c, then f has a local minimum at c.

(iii) If f ′ is positive to the left and right of c or negative to the left and right of c,
then f has no local maximum or minimum at c.

Second derivative test. Suppose f ′′ is continuous near c.

(i) If f ′ (c) = 0 and f ′′ (c) > 0, then f has a local minimum at c.

Download From : www.EasyEngineering.net


Download From : www.EasyEngineering.net

56 Engineering Mathematics - I

(ii) If f ′ (c) = 0 and f ′′ (c) < 0, then f has a local maximum at c.

Concavity. If the graph of f lies above all of its tangents on an interval I, then it
is called concave upward on I. If the graph of f lies below all of its tangents on I,
it is called concave downward on I.
Concavity test

(i) If f ′′ (x) > 0, for all x in I, then the graph of f is concave upward on I.

ww(ii) If f ′′ (x) < 0, for all x in I, then the graph of f is concave downward on I.

w.E
Point of inflexion. A point P on a curve y = f (x) is called a point of inflexion

asy
if f is continuous there and the curve changes from concave upward to concave
downward or from concave downward to concave upward at P.

En
Note. Points of inflection occurs at points where f ′′ (x) = 0.
✎ ☞

gin Worked Examples


✍ ✌

increasing or decreasing.
downwards.
ee
Example 1.59. Find the intervals on which the function f (x) = sin x + cos x is

rin
Also find the curve is concave upwards or concave
[A.U.Jan.2015]
Solution. f (x) = sin x + cos x.
g.n
f ′ (x) = cos x − sin x.
f ′′ (x) = − sin x − cos x.
At critical points f ′ (x) = 0
et
i.e., cos x − sin x = 0.
i.e., sin x = cos x.
i.e., tan x = 1.
π 5π
∴x= , .
4 4
Let us evaluate the intervals at which f decrease or increases.

Download From : www.EasyEngineering.net


Download From : www.EasyEngineering.net

Differential calculus 57

Interval f ′ (x) f
π π
0<x< + increases in (0, )
4 4
π 5π π 5π
<x< - decreases in ( , )
4 4 4 4
5π 5π
< x < 2π + increases in ( , 2π)
4 4
π
Since f ′ changes from positive to negative at x = ,
4
π
f has a maximum at x = .
4
π π π 1 1 √
Maximum value of f = f ( ) = sin + cos = √ + √ = 2.
4 4 4

ww
2 2
′ 5π
Also f changes from negative to positive at x = .
4
5π 5π 5π 1 1 √

w.E Minimum value of f = f ( ) = sin


4 4
+ cos
4
= − √ − √ = − 2.
2 2
Let us evaluate the intervals at which the curve is concave upwards or
concave downwards.

asy
Making f ′′ = 0 we get

En
− sin x − cos x = 0.
i.e., sin x + cos x = 0.

gin
π !
 3π
cos x = − sin x = cos +x (or) cos x = cos −x
 π2  2
Taking cos x = cos
π
2
i.e., = 0 which is absurd.
2

ee !
π
+ x implies x = + x
2

3π rin
Taking cos x = cos

i.e., 2x =

⇒x=
2
− x implies x =

2
−x
g.n
2 4
The interval to be considered are 0,

4
!
,

4
, 2π
!
et
Interval f ′ (x) f

0<x< - concave downwards
4

< x < 2π + concave upwards
4
3π 3π
Since f ′′ changes sign from negative to positive at x = ,x= is a
4 4
point of inflexion.

Download From : www.EasyEngineering.net


Download From : www.EasyEngineering.net

58 Engineering Mathematics - I

Example 1.60. Find the intervals on which the function f (x) = x4 − 2x2 + 3 is
increasing or decreasing. Also find the local maximum and minimum values of f
by using first derivative test. [A.U.Nov.2016]
Solution. f (x) = x4 − 2x2 + 3.
f ′ (x) = 4x3 − 4x = 4x(x2 − 1) = 4x(x − 1)(x + 1).
Critical numbers occur at points where f ′ (x) = 0
i.e., 4x(x − 1)(x + 1) = 0.
x = −1, x = 0, x = 1.

ww The critical numbers are x = −1, x = 0, x = 1.


Let us evaluate the intervals at which f decreases or increases.

w.E Interval 4x
-
x−1
-
x+1
-
f ′ (x)
-
f
decreases in (−∞, −1)

asy
x < −1
−1 < x < 0 - - + + increases in (−1, 0)

En
0<x<1 + - + - decreases in (0, 1)
x>1 + + + + increases in (1, ∞)

gin
Since f ′ changes from negative to positive at x = −1,

ee
f has a local minimum at x = −1.
Minimum value of f = f (−1) = 1 − 2 + 3 = 1.
Also f ′ changes from positive to negative at x = 0.
rin
∴ f has a maximum at x = 0.
Maximum value of f = f (0) = 3. g.n
Again f ′ changes from negative to positive at x = 1.
∴ f has a local minimum at x = 1.
et
Minimum value of f = 1 − 2 + 3 = 2.

Example 1.61. Find the intervals on which the function f (x) = 3x4 − 4x3 − 12x2 + 5
is increasing and decreasing. Also find the local maximum and minimum values
of f by using first derivative test. [A.U.Nov.2016]
Solution. f (x) = 3x4 − 4x3 − 12x2 + 5.

Download From : www.EasyEngineering.net


Download From : www.EasyEngineering.net

Differential calculus 59

f ′ (x) = 12x3 − 12x2 − 24x = 12x(x2 − x − 2) = 12x(x − 2)(x + 1).


Critical numbers occur at points where f ′ (x) = 0
i.e., 12x(x − 2)(x + 1) = 0.
x = −1, x = 0, x = 2.
The critical numbers are x = −1, x = 0, x = 2.
Let us evaluate the intervals at which f decreases or increases.

Interval 12x x−2 x+1 f ′ (x) f


- - - - decreases in (−∞, −1)

ww
x < −1
−1 < x < 0 - - + + increases in (−1, 0)
0<x<2 + - + - decreases in (0, 2)

w.E x>2 + + + + increases in (2, ∞)

Since f ′ changes from negative to positive at x = −1,

asy
f has a local minimum at x = −1.

En
Minimum value of f = f (−1) = 3 + 4 − 12 + 5 = 20.
Also f ′ changes from positive to negative at x = 0.

gin
∴ f has a local maximum at x = 0.

ee
Maximum value of f = f (0) = 5.
Again f ′ changes from negative to positive at x = 2.
∴ f has a minimum at x = 2.
rin
g.n
Minimum value of f = 3 × 16 − 4 × 8 − 12 × 4 + 5 = 48 − 32 − 48 + 5 = −27.

Example 1.62. Find the local maximum and minimum values of the function
f (x) = x + 2 sin x, 0 ≤ x ≤ 2π.
Solution. f (x) = x + 2 sin x.
et
f ′ (x) = 1 + 2 cos x.
f ′′ (x) = −2 sin x.
Local maximum and minimum occur at points where f ′ (x) = 0.
i.e., 1 + 2 cos x = 0.
2 cos x = −1.

Download From : www.EasyEngineering.net


Download From : www.EasyEngineering.net

60 Engineering Mathematics - I

−1
cos x = 2 .
cos x is negative in the II and III quadrants.
2π 4π
x= 3 and x = 3 .
First derivative test
2π 2π 4π 4π
The intervals to be considered are 0 < x < , <x< , < x < 2π.
3 3 3 3
Let us evaluate the intervals at which f decreases and increases.

Interval f ′ (x) = 1 + 2 cos x f


ww
0<x< + increasing
3
2π 4π
<x< - decreasing
3 3

w.E

< x < 2π + increasing
3

Since f ′ changes from positive to negative at 3 ,

∴ Maximum value of f =
asy
f has a local maximum at x =


3 .
+ 2 sin 2π
3 =

+2

3
= 2π
3 +

3.

En 3
Since f ′ changes from negative to positive at x =
3 2

3 ,
f has a minimum at x =
∴ Minimum value of f = gin 4π
3 .

3 + 2 sin 4π
3 =

+2
 √ 
− 3
= 4π


3.

ee
3 2 3
Second derivative test

rin
√ √
2π 3
When x = 3 , f ′′ (x) = −2 sin 2π
3 = −2 2 = − 3 < 0.

g.n

∴ f has a maximum at x = 3 .


Maximum value of f = 3 + 3.

et
√ √
4π − 3
When x = f ′′ (x) = −2 sin 4π
3 , 3 = −2 2 = 3 > 0.

∴ f has a minimum at x = .
3
4π √
∴ Minimum value of f = − 3.
3
Example 1.63. Find the local maxima and minima and the points of inflexion for
the function f (x) = x4 − 4x3 .
Solution. f (x) = x4 − 4x3 .
f ′ (x) = 4x3 − 12x2 .

Download From : www.EasyEngineering.net


Download From : www.EasyEngineering.net

Differential calculus 61

f ′′ (x) = 12x2 − 24x.


Local maximum and minimum occur at points where f ′ (x) = 0
i.e., 4x3 − 12x2 = 0.
4x2 (x − 3) = 0.
x = 0, x = 3.
When x = 0, f ′′ (x) = 0.
∴ f (x) does not have either mxaimum or minimum.
When x = 3, f ′′ (x) = 12 × 9 − 24 × 3 = 108 − 72 = 36 > 0

ww ∴ f has a local minimum at x = 3.


Minimum value of f = 81 − 4 × 27 = 81 − 108 = −27.

w.E At the points of inflexion, f ′′ (x) = 0


12x2 − 24x = 0 ⇒ 12x(x − 2) = 0 ⇒ x = 0, x = 2.

asy
When x = 0, f (0) = 0.
∴ (0, 0) is a point of inflexion.

En
When x = 2, f (2) = 24 − 4 × 23 = 16 − 32 = −16.

gin
Another point of inflexion is (2, −16).

1
ee
Example 1.64. Find the maxima and minima of the function f (x) = x2/3 (6 − x)1/3 .
Solution. f (x) = x2/3 (6 − x)1/3 .
2 rin
f ′ (x) = x2/3 (6 − x)−2/3 (−1) + (6 − x)1/3 x−1/3 .

=
1
"
3
−x 2/3
+
2(6 − x) 1/3
#
.
3
g.n
=
3 (6 − x)2/3
"
1 −x + 2(6 − x)
3 x1/3 (6 − x)2/3
"
#

#
x1/3
et
1 −x + 12 − 2x
=
3 x1/3 (6 − x)2/3
" # " #
1 12 − 3x 4−x
= = 1/3 .
3 x1/3 (6 − x)2/3 x (6 − x)2/3
Now, f ′ (x) = 0 when x = 4 and f ′ (x) does not exist when x = 0 and x = 6.
∴ The critical numbers are 0, 4, 6.

Download From : www.EasyEngineering.net


Download From : www.EasyEngineering.net

62 Engineering Mathematics - I

Let us evaluate the intervals at which f increases or decreases.

Interval 4−x x1/3 (6 − x)2/3 f ′ (x) f


−∞ < x < 0 + - + - decreases on (−∞, 0)
0<x<4 + + + + increases on (0, 4)
4<x<6 - + + - decreases on (4, 6)
x>6 - + + - decreases on (6, ∞)

Since f ′ changes from negative to positive at x = 0, f has a local minimum at x = 0.


∴ Minimum value of f = f (0) = 0.

ww Also f ′ changes from positive to negative at x = 4.


∴ f has a local maximum at x = 0.

w.E
∴ Maximum value of f = f (4) = 42/3 (6 − 4)1/3 = (22 )2/3 (21/3 ) = 24/3 21/3 = 25/3 .
Again f ′ does not changes sign at x = 6.

asy
∴ f has no maximum or minimum at x = 6.

En
Example 1.65. Using first derivative test, examine for maximum and minimum
of the function f (x) = x3 − 3x + 3, x ∈ R.
Solution. f (x) = x3 − 3x + 3.
gin
ee
f ′ (x) = 3x2 − 3 = 3(x2 − 1) = 3(x − 1)(x + 1).
Critical numbers occur at points where f ′ (x) = 0.
i.e., 3(x − 1)(x + 1) = 0. rin
i.e., x = −1, x = 1.
g.n
et
The critical numbers are x = −1, 1.
Let us evaluate the intervals at which f decreases or increases.

Interval x−1 x+1 f ′ (x) f


x < −1 - - + increases in (−∞, −1)
−1 < x < 1 - + - decreases in (−1, 1)
x>1 + + - increases in (1, ∞)

Since f ′ changes from positive to negative at x = −1.


f has a local maximum at x = −1.

Download From : www.EasyEngineering.net


Download From : www.EasyEngineering.net

Differential calculus 63

∴ Maximum value of f = f (−1) = (−1)3 − 3(−1) + 3 = −1 + 3 + 3 = 5.


Also f ′ changes from negative to positive at x = 1.
∴ f has a local minimum at x = 1.
∴ Minimum value of f = f (1) = 13 − 1 × 3 + 3 = 1 − 3 + 3 = 1.

Example 1.66. Using first derivative test examine the maximum and minimum
of f (x) = sin2 x, 0 < x < π.
Solution. f (x) = sin2 x.
f ′ (x) = 2 sin x cos x = sin 2x.

ww Critical numbers occur at points where f ′ (x) = 0.

w.E i.e., sin 2x = 0.


∴ 2x = 0, or 2x = π.
∴ x = 0, or x = π2 .

asy
The critical numbers are x = 0, x = π2 .

En
Let us evaluate the intervals at which f decreases or increases.

f ′ (x)

gin
Interval sin 2x f
π
0<x< 2 + + increases in (0, π2 )

ee
π
x> 2 - - decreases in ( π2 , π)

Since f ′ changes from positive to negative at x = π2 .


∴ f has a maximum at x = π2 .
rin
g.n
 
∴ Maximum value of f = f ( π2 ) = sin2 π2 = 1.

Example 1.67. Find


f (x) = 2x3 − 3x2 − 36x + 10.
the

Solution. f (x) = 2x3 − 3x2 − 36x + 10.


maximum and minimum of
et
the function

f ′ (x) = 6x2 − 6x − 36.


f ′′ (x) = 12x − 6.
Critical pints occur at f ′ (x) = 0.
i.e., 6x2 − 6x − 36 = 0.
∴ x2 − x − 6 = 0.

Download From : www.EasyEngineering.net


Download From : www.EasyEngineering.net

64 Engineering Mathematics - I

∴ (x − 3)(x + 2) = 0.
∴ x = −2, x = 3.
The critical points are at x = −2, x = 3.
At x = −2, f ′′ (x) = 12 × (−2) − 6 = −24 − 6 = −30 = −ve
∴ f is maximum at x = −2.
Maximum value of
f = 2(−2)3 − 3(−2)2 − 36(−2) + 10 = −16 − 12 + 72 + 10 = 54.
At x = 3, f ′′ (x) = 12 × 3 − 6 = 36 − 6 = 30 = +ve.

ww ∴ f has minimum at x = 3.
∴ Minimum value of

w.E f = f (3) = 2 × 33 − 3 × 32 − 36 × 3 + 10 = 54 − 27 − 108 + 10 = −71.

asy
Example 1.68. Find the maxima and minima of the function f (x) = sin x(1 + cos x),
0 ≤ x ≤ 2π.

En
Solution. Given f (x) = sin x(1 + cos x)
f ′ (x) = sin x(0−sin x)+(1+cos x)(cos x) = − sin2 x+cos x+cos2 x = cos x+cos 2x.

gin
f ′′ (x) = − sin x − 2 sin 2x.

ee
At critical points f ′ (x) = 0.

cos x + cos 2x = 0

cos x + 2 cos2 x − 1 = 0 rin


i.e., 2 cos2 x + cos x − 1 = 0 g.n
cos x =

cos x =

2×2
−1 + 3 1
= or cos x =
=

−1 ± 1 − 4 × 2 × (−1) −1 ± 1 + 8 −1 ± 3
4
−1 − 3
= −1.
=
4 et
4 2 4
1 π 5π
cos x = gives x = and .
2 3 3
cos x = −1 gives x = π.
π √ √ √
When x = , f ′′ (x) = − sin π3 − 2 sin 2π
3 = − 23 − 2 23 = − 3 2 3 < 0.
3
π
∴ f has a maximum at x = .
3

Download From : www.EasyEngineering.net


Download From : www.EasyEngineering.net

Differential calculus 65

√ √
      3
Maximum value of f = f π π π
= sin 3 1 + cos 3 =
3 (1 + 21 ) = 3 4 3 .
5π ′′  √   √  √2 √ √
When x = , f (x) = − sin 3 − 2 sin 3 = − 2 − 2 − 23 = 23 + 2 23 = 3 2 3 > 0.
5π 10π − 3
3

∴ f has a minimum at x = .
3 √
      − 3 √ √
5π 5π
∴ Minimum value of f = f 3 = sin 3 1 + cos 5 3 = π
(1 + 21 ) = − 2 3 × 23 = −34 3 .
2
When x = π, f ′′ (x) = − sin π − 2 sin 2π = 0.
∴ f has neiher maximum nor minimum at x = π.

3 3
∴ Maximum value = 4√ .

ww ∴ Minimum value = −3 3
4 .

w.E
asy
En
gin
ee rin
g.n
et

Download From : www.EasyEngineering.net


Download From : www.EasyEngineering.net

ww
w.E
asy
En
gin
ee rin
g.n
et

Download From : www.EasyEngineering.net


Download From : www.EasyEngineering.net

2 Function of Several Variables

2.1 Limits and Continuity

wwLimit. Let f (x, y) be a function of two independent variables x and y. f (x, y) is said

w.E
to have the limit ℓ as (x, y) tends to (x0 , y0 ), if corresponding to any positive number
ǫ, however small, there is a positive number η such that

asy| f (x, y) − ℓ| < ǫ where 0 < d < η,

En
where d is the distance between (x, y) and (x0 , y0 ) given by d2 = (x − x0 )2 + (y − y0 )2 .

gin
Note. If the limit exists, then it can be written as lim f (x, y) = ℓ.
x→x0
y→y0

ee
Continuity. f (x, y) is said to be continuous at (x0 , y0 ) if, to any positive number ǫ,
however small, there corresponds a positive number η such that
rin
| f (x, y) − f (x0 , y0 )| < ǫ where 0 < d < η,
g.n
where d is the distance between (x, y) and (x0 , y0 ) given above.

Result 1. The above definition is equivalent to lim f (x, y) = f (x0 , y0 ).


x→x0
y→y0
et
Result 2. Usually the limit is the same irrespective of the path along which
the point (x, y) approaches (x0 , y0 ).
( )  
i.e., lim lim f (x, y) = lim lim f (x, y) .
x→x0 y→y0 y→y0 x→x0

But this is not true always.

Download From : www.EasyEngineering.net


Download From : www.EasyEngineering.net

2 Function of Several Variables

2.1 Limits and Continuity

wwLimit. Let f (x, y) be a function of two independent variables x and y. f (x, y) is said

w.E
to have the limit ℓ as (x, y) tends to (x0 , y0 ), if corresponding to any positive number
ǫ, however small, there is a positive number η such that

asy| f (x, y) − ℓ| < ǫ where 0 < d < η,

En
where d is the distance between (x, y) and (x0 , y0 ) given by d2 = (x − x0 )2 + (y − y0 )2 .

gin
Note. If the limit exists, then it can be written as lim f (x, y) = ℓ.
x→x0
y→y0

ee
Continuity. f (x, y) is said to be continuous at (x0 , y0 ) if, to any positive number ǫ,
however small, there corresponds a positive number η such that
rin
| f (x, y) − f (x0 , y0 )| < ǫ where 0 < d < η,
g.n
where d is the distance between (x, y) and (x0 , y0 ) given above.

Result 1. The above definition is equivalent to lim f (x, y) = f (x0 , y0 ).


x→x0
y→y0
et
Result 2. Usually the limit is the same irrespective of the path along which
the point (x, y) approaches (x0 , y0 ).
( )  
i.e., lim lim f (x, y) = lim lim f (x, y) .
x→x0 y→y0 y→y0 x→x0

But this is not true always.

Download From : www.EasyEngineering.net


Download From : www.EasyEngineering.net

68 Engineering Mathematics - I

Consider the following example.


x − 2y
Let f (x, y) = .
x + 2y
As (x, y) → (0, 0) along the line y = mx, we have

x − 2y
lim f (x, y) = lim
(x,y)→(0,0) (x,y)→(0,0) x + 2y
x − 2mx
= lim
x→0 x + 2mx
x(1 − 2m)
= lim
x→0 x(1 + 2m)

ww =
1 − 2m
1 + 2m
,

w.E
which is (different for
Also lim lim
x − 2y
x→0 y→0 x + 2y
) lineswith
= lim
) x→0 x !
x
different slopes.
=1

asy
(
x − 2y −2y
and lim lim = lim = −1.
y→0 x→0 x + 2y y→0 2y
Hence, as (x, y) approaches (0, 0) along different paths, f (x, y) approaches different
limits.
En
Hence, the two repeated limits are not equal and hence f (x, y) is
discontinuous at the origin.
gin
ee
Note. If a function is continuous at every point of the domain a ≤ x ≤ a′ ,
b ≤ y ≤ b′ , it is said to be continuous in that domain.
✎ ☞
rin
2x2 y

Worked Examples

g.n
Example 2.1. Find lim

Solution.
x→1
y→2
x2 + y2 + 1

lim
.

2x2 y
=
2×1×2 4 2
= = .
et
x→1 x2 + y2 + 1 1 + 4 + 1 6 3
y→2

xy + 1
Example 2.2. Find lim .
x→∞
y→2
x2 + 2y2
x y + 1x y + 1x

xy + 1 2
Solution. lim 2 2
= lim  y 2
 = lim  y 2
 = = 0.
x→∞ x + 2y
y→2
x→∞ 2
y→2 x 1 + 2 x
 x→∞
y→2 x 1 + 2 x
 ∞

Download From : www.EasyEngineering.net


Download From : www.EasyEngineering.net

Function of Several Variables 69

( )
x−y  
Example 2.3. If f (x, y) = , show that lim lim f (x, y) , lim lim f (x, y) .
2x + y x→0 y→0 y→0 x→0
Solution.
( ) ( )
x−y
lim lim f (x, y) = lim lim
x→0 y→0 x→0 y→0 2x + y
 x 1
= lim = .
x→0 2x 2
( )
  x−y
lim lim f (x, y) = lim lim
y→0 x→0 y→0 x→0 2x + y
!
−y

ww∴ The two limits are not equal.


= lim
y→0 y
= −1.

w.E
Example 2.4. If f (x, y) =
x3 y2 + x2 y3 − 3
2 − xy
, show that lim f (x, y) = lim f (x, y).
x→0
y→0
y→0

asy
! x→0

Solution. lim f (x, y) = lim lim f (x, y)


x→0 x→0 y→0
y→0

= lim lim
x→0 y→0 En
x3 y2 + x2 y3 − 3
2 − xy
!

= lim
−3
x→0 2
!
=
gin
−3
2
.

ee

lim f (x, y) = lim lim f (x, y)
y→0 y→0 x→0
x→0

= lim lim
x3 y2 + x2 y3 − 3
!
rin
y→0 x→0

= lim
−3
!
=
−3
2 − xy

. g.n
y→0 2

∴ lim f (x, y) = lim f (x, y) =


x→0
y→0
y→0
x→0
−3
2
.
2
et
2.2 Partial Derivatives

We know that, if y is a continuous function of the independent variable x, and


∆y
∆y the increment in y corresponding to an increment ∆x in x, then lim if
∆x→0 ∆x
exists, is called the differential coefficient or the derivative of y with respect to x.

Download From : www.EasyEngineering.net


Download From : www.EasyEngineering.net

70 Engineering Mathematics - I

Now let us consider a function of several independent variables.


Let u be a function of two independent variables x and y and let ∆u be the
increment in u corresponding to an increment in x, y remaining constant. If u is a
∆u
continuous function of x, the limit lim if exists is called the partial derivative
∆x→0 ∆x
∂u
of u with respect to x, and is represented by or u x .
∂x
∂u u(x + ∆x, y) − u(x, y)
Hence, = lim .
∂x ∆x→0 ∆x
In the same way, if u is a continuous function of y, the increment ∆u in u,

wwcorresponding to an increment ∆y in y, x remaining constant, then the limit


lim
∆y→0
u(x, y + ∆y) − u(x, y)
∆y
if exists is called the partial derivative of u with respect to

w.E
y and is represented by
∂u
∂y
or uy .
It can be easily seen that the change in the value of u corresponding to a

asy
change in the value of x will not in general be the same as the change in the value
of u corresponding to an equal change in the value of y. For example, the area A of

En
a rectangular box is a function of the length x and the breadth y, being given by

gin
the relation A = xy. When x alone changes,

ee
∆A = (x + ∆x)y − xy = y∆x.

When y alone changes, ∆A = x(y + ∆y) − xy = x∆y. rin


g.n
Though ∆x = ∆y, the values of ∆A will not be equal unless x = y. i.e., unless the
∂A ∂A

et
box is in the form of a square. Hence, we can see that, in general , .
∂x ∂y
Likewise, if u is a continuous function of several independent variables
x, y, z, . . . , we can define the partial derivatives of u with respect to each of these
independent variables. As an example, let us consider a function with three
independent variables x, y and z.
∂u ∂u ∂u
Let u = f (x, y, z). The first partial derivative of u denoted by , and
∂x ∂y ∂z
treating x, y and z respectively alone as variables can be obtained. We can also
∂2 u ∂2 u ∂2 u ∂2 u ∂2 u ∂2 u
find the higher order derivatives 2 , 2 , 2 , , , , . . ..
∂x ∂y ∂z ∂x∂y ∂y∂x ∂x∂z

Download From : www.EasyEngineering.net


Download From : www.EasyEngineering.net

Function of Several Variables 71

They are evaluated as follows.

∂2 u ∂  ∂u  ∂2 u ∂  ∂u  ∂2 u ∂  ∂u 
= , = and = .
∂x2 ∂x ∂x ∂y2 ∂y ∂y ∂z2 ∂z ∂z

∂2 u ∂  ∂u 
Also, =
∂x∂y ∂x ∂y

∂2 u ∂  ∂u 
and = .
∂y∂x ∂y ∂x

wwGenerally,
∂2 u
=
∂2 u
∂x∂y ∂y∂x
.

w.E
The third and higher orders of the partial derivatives can be obtained similarly.

Worked Examples


asy
Example 2.5. If V = πr2 h where r and h are independent variables, find
∂V
∂r
and
∂V
∂h
.
En
Solution. Given: V = πr2 h.
gin
Differentiating partially w.r.t. r,
∂V
∂r
= πh × 2r = 2πrh.
ee
Differentiating partially w.r.t. h,
∂V
= πr2 × 1 = πr2 . rin
∂h

∂u ∂u g.n
Example 2.6. If u = log(e x + ey ), show that
Solution. Given: u = log(e x + ey ).
∂u
= x
ex
.
+
∂x ∂y
= 1.
et
∂x e +y ey
∂u e
= .
∂y e x + ey
∂u ∂u e + eyx
+ = = 1.
∂x ∂y e x + ey

 x ∂z ∂z
Example 2.7. If z = tan−1 , prove that x + y = 0.
y ∂x ∂y

Download From : www.EasyEngineering.net


Download From : www.EasyEngineering.net

72 Engineering Mathematics - I

 x
Solution. Given: z = tan−1 .
y

∂z 1 1 y2 1 y
= = = 2 .
∂x 1 + x2 y x + y y x + y2
2 2 2
y
∂z xy
x= 2 .
∂x x + y2
∂z 1  x xy2 x
= 2
− 2
= − 2 2 2
=− 2
∂y x y y (x + y ) x + y2
1+ 2
y

ww ∂z
∂z
y =− 2
∂y
∂z
xy
x + y2
xy xy

w.E x +y = 2
∂x ∂y x + y 2
− 2
x + y2
= 0.

∂u ∂u ∂u

asy
Example 2.8. If u = (x − y)4 + (y − z)4 + (z − x)4 , find the value of
Solution. u = (x − y)4 + (y − z)4 + (z − x)4 .
+ + .
∂x ∂y ∂z

∂u
∂x En
= 4(x − y)3 + 4(z − x)3 (−1) = 4(x − y)3 − 4(z − x)3 .

∂u
gin
= 4(y − z)3 − 4(x − y)3 .

ee
∂y
∂u
∂z
= 4(z − x)3 − 4(y − z)3 .

∂u ∂u ∂u rin
+
∂x ∂y ∂z
+ = 0.
g.n
Example 2.9. If u = log(x3 + y3 + z3 − 3xyz), prove that
Solution. u = log(x3 + y3 + z3 − 3xyz).
∂u ∂u ∂u
+ +
∂x ∂y ∂z
=
3

et
x+y+z
.

∂u 3x2 − 3yz
= 3 .
∂x x + y3 + z3 − 3xyz

∂u 3y2 − 3xz
= 3 .
∂y x + y3 + z3 − 3xyz
∂u 3z2 − 3yx
= 3 .
∂z x + y3 + z3 − 3xyz

Download From : www.EasyEngineering.net


Download From : www.EasyEngineering.net

Function of Several Variables 73

∂u ∂u ∂u 3x2 + 3y2 + 3z2 − 3xy − 3yz − 3zx


+ + =
∂x ∂y ∂z x3 + y3 + z3 − 3xyz

3(x2 + y2 + z2 − xy − yz − zx) 3
= 2 2 2
= .
(x + y + z)(x + y + z − xy − yz − zx) x + y + z

∂u ∂u ∂u
Example 2.10. If u = (x − y)(y − z)(z − x), show that + + = 0.
∂x ∂y ∂z
Solution. Given, u = (x − y)(y − z)(z − x).
∂u
= (y − z){(x − y)(−1) + (z − x) · 1}
∂x

ww = (y − z)(−x + y + z − x)

= (y − z)(y + z − 2x) = y2 − z2 − 2x(y − z).

w.E ∂u
∂y
= (z − x){(x − y) · 1 + (y − z)(−1)}

asy
= (z − x){x − y − y + z}

= (z − x)(z + x − 2y)

En
= z2 − x2 − 2y(z − x).
∂u
∂z
gin
= (x − y){(y − z) · 1 + (z − x) · (−1)}

= (x − y)(y − z − z + x)

= (x − y)(x + y − 2z)

= x2 − y2 − 2z(x − y).
ee rin
∂u ∂u ∂u
+ +
∂x ∂y ∂z g.n
= y2 − z2 − 2x(y − z) + z2 − x2 − 2y(z − x) + x2 − y2 − 2z(x − y)

= −2{xy − xz + yz − xy + zx − yz} = −2 × 0 = 0.

p ∂2 u ∂2 u
et
Example 2.11. If u = log x2 + y2 , show that 2 + 2 = 0.
p ∂x ∂y
2 2 1 2 2
Solution. u = log x + y = 2 log(x + y ).
Differentiating partially w.r.t. x we get

∂u 1 1 x
= · 2 2
· 2x = 2 .
∂x 2 x + y x + y2

Download From : www.EasyEngineering.net


Download From : www.EasyEngineering.net

74 Engineering Mathematics - I

Again differentiating partially w.r.t. x we get

∂2 u (x2 + y2 ) · 1 − x · 2x y2 − x2
= = .
∂x2 (x2 + y2 )2 (x2 + y2 )2

Differentiating u partially w.r.t. y we get

∂u 1 1 y
= · 2 2
· 2y = 2 .
∂y 2 x + y x + y2

ww Again differentiating partially w.r.t. y we get

w.E ∂2 u (x2 + y2 ) · 1 − y · 2y
= =
x2 − y2
.

asy
∂y2 (x2 + y2 )2 (x2 + y2 )2

En
∂2 u ∂2 u y2 − x2 + x2 − y2
+

gin
∂x2 ∂y2
=
(x2 + y2 )2
= 0.

Example 2.12. If u = log(x2 ee


+ y2
Solution. Given, u = log(x2 + y2 + z2 )
+ z2 ), 2 2 2
prove that (x + y + z )
∂2 u ∂2 u ∂2 u

rin
+ +
∂x2 ∂y2 ∂z2
!
= 2.

∂u 2x g.n
= 2
∂x x + y2 + z2
.

∂2 u (x2 + y2 + z2 ) · 2 − 2x · 2x
∂x2
=
(x2 + y2 + z2 )2
et
2x + 2y2 + 2z2 − 4x2
2
=
(x2 + y2 + z2 )2
2y2 + 2z2 − 2x2
= 2
(x + y2 + z2 )2
2(y2 + z2 − x2 )
= 2 .
(x + y2 + z2 )2

Download From : www.EasyEngineering.net


Download From : www.EasyEngineering.net

Function of Several Variables 75

Similarly, we have

∂2 u 2(x2 + z2 − y2 )
= 2
∂y2 (x + y2 + z2 )2
2
∂ u 2(x2 + y2 − z2 )
and 2 = 2 .
∂z (x + y2 + z2 )2
∂2 u ∂2 u ∂2 u 2[y2 + z2 − x2 + x2 + z2 − y2 + x2 + y2 − z2 ]
Now, + + =
∂x2 ∂y2 ∂z2 (x2 + y2 + z2 )2
2(x2 + y2 + z2 )
= 2
(x + y2 + z2 )2

ww 2 ∂ u
2 ∂2 u ∂2 u
!
= 2
2
x + y2 + z2
.

w.E 2 2
∴ (x + y + z ) +
∂x2 ∂y2 ∂z2
+ =2

∂ 2u ∂ 2u 1  ∂u 2
 !
∂u
!2 

asy

Example 2.13. If u = e xy , show that 2 + 2 =  + . [Jan 2013]
∂x ∂y u ∂x ∂y 
Solution. u = e xy
∂u
∂x
= e xy · y = uy.
En
∂2 u
∂x
∂u
2
=y·
∂u
∂x
= yuy = uy2 .
gin
∂y
∂2 u
∂y 2
= e xy · x = ux.

= x·
∂u
∂y
= x · ux = ux2 .
ee rin
g.n
!2 !2
∂u ∂u
+ = u2 y2 + u2 x2
∂x ∂y
= u[uy2 + ux2 ]

=u
∂ u ∂2 u
" 2
+
∂x2  ∂y2
# et
!2 !2 
1  ∂u ∂u  ∂2 u ∂2 u
+ = + .
∂y  ∂x2 ∂y2

u  ∂x
!
x y ∂u ∂u
Example 2.14. If u = y f +g find the value of x + y . [Dec 2012]
! y x ∂x ∂y
x y
Solution. u = y f +g
y x

Download From : www.EasyEngineering.net


Download From : www.EasyEngineering.net

76 Engineering Mathematics - I

!
∂u ′ x 1  y   −y 
= yf · + g′
∂x y y x x2

!
∂u x y y
x = xf′ − g′ . (1)
∂x y x x

! ! !
∂u ′ x −x x ′ y
  1
= yf · 2 +f ·1+g · .
∂y y y y x x

ww y
∂u
∂y
= −x f ′ x
y
!
+ yf
x
y
!
y y
+ g′
x x
(2)

w.E ∂u
(1) + (2) ⇒ x + y
∂x
∂u
∂y
= yf
x
y
!
.

asy
Example 2.15. If r2 = x2 + y2 + z2 , prove that
∂2 r ∂2 r ∂2 r 2
+ + = .

En
Solution. Given: r2 = x2 + y2 + z2 .
∂x2 ∂y2 ∂z2 r

gin
Differentiating w.r.t. x partially we get,

∂r
2r
∂x
2
∂ r
ee
= 2x ⇒

r − x
∂r
∂x r
∂r
= .
x

x 2
∂x = r − x r = r − x .
2
rin
∂x2
2
=
∂ r r −y
Similarly, 2 =
2
r2
2
r2 r3
g.n
∂y
∂ r r − z2
2
and 2 =
∂z
2
r3

r3
. et
∂2 r ∂2 r ∂2 r 3r2 − (x2 + y2 + z2 )
+ + =
∂x2 ∂y2 ∂z2 r3
2
3r − r 2 2r2 2
= = = .
r3 r3 r

y  x ∂2 u x2 − y2
Example 2.16. If u = x2 tan−1 − y2 tan−1 , prove that = 2 .
x y ∂x∂y x + y2

Download From : www.EasyEngineering.net


Download From : www.EasyEngineering.net

Function of Several Variables 77

Solution. Differentiating u partially w.r.t. y, we get

∂u 1 1 h 2 1  −x  y i
= x2 − y + tan−1 2y
∂y y2 x x2 y2 x
1+ 1+
x2 y2
x3 xy2 −1 y
 
= + − 2y tan
x2 + y2 x2 + y2 x
2 2
x(x + y ) −1 x −1 x
   
= 2 − 2y tan = x − 2y tan
x + y2 y y

wwDifferentiating this partially w.r.t. x, we get

∂2 u
= 1 − 2y
11
= 1 −
2y2

w.E ∂x∂y
1+ 2

x + y − 2y2
2 2
x2 y
y
x2 + y2

x2 − y2

asy =
x2 + y2

Example 2.17. If u = xy , show that u xxy = u xyx .


=
x2 + y2
.

Solution. Given: u = ey log x


En [Jan 2012, Jun 2008]

gin
uy = ey log x log x = xy log x

ee
u xy = yxy−1 log x + xy
1
x
= yxy−1 log x + xy−1 = xy−1 (y log x + 1)
rin
u xxy
y
= xy−1 + (y log x + 1)(y − 1)xy−2
x
g.n
= xy−2 y + xy−2 (y log x + 1)(y − 1)

= xy−2 (y − y log x − 1 + y2 log x + y)

= xy−2 (2y − 1 + y log x(y − 1))


et
= xy−2 (y(y − 1) log x + 2y − 1) (1)

u = xy

u x = yxy−1 = ye(y−1) log x

uyx = ye(y−1) log x log x + e(y−1) log x .1

Download From : www.EasyEngineering.net


Download From : www.EasyEngineering.net

78 Engineering Mathematics - I

= yxy−1 log x + xy−1 = xy−1 (y log x + 1)


y
u xyx = xy−1 + (y log x + 1)(y − 1)xy−2
x
= xy−2 y + xy−2 (y − 1)(y log x + 1)

= xy−2 (y + y2 log x + y − y log x − 1)

= xy−2 (2y − 1 + y log x(y − 1)) (2)

From (1) and (2) we have u xxy = u xyx .

ww y ∂2 u ∂2 u ∂2 u

w.E
Example 2.18. If u = (x − y) f

y
x
, find the value of x2 2 + 2xy
∂x ∂x∂y
+ y2 2 .
∂y
[May 2001]
Solution. u = (x − y) f
x
asy
∂u
∂x
= (x − y) f ′
En
 y   −y 
x x2
+ f
y
x
.
∂2 u
∂x2
= −(x − y)
y
gin
x2 
· f ′′ y
   −y 
x x2   
− (x − y) f ′ y
 
x
· y(−2)x−3
y
− 2 · f′
x
y2 ′′  y 
= (x − y) 4 f
x
ee
y
x

x
· 1 + f′
y
y −y
x x2
+ 2 3 (x − y) f
x
′ y
.
  y y y y
x
− 2 f′
x rin
x
− 2 f′
x x
2
∂ u y 2
x2 2 = 2 (x − y) f ′′
∂x x
 y
x
 2y
+ (x − y) f ′
x
 y
x

− 2y f ′
 y
x

.
g.n (1)
∂u
∂y
= (x − y) f ′

= 1−
y ′ y
x
f
 y 1

x x

x
· +f

−f
y
x
 y
x
.

(−1)
et
∂2 u  y  ′′  y  1 ′ y
  −1 !   1
′ y
= 1 − f · + f − f · .
∂y2 x x x x x x x
(x − y) ′′  y  2 ′  y 
= f − f
x2 x x x
2 2 2
2∂ u y ′′ y 2y ′  y 
 
y = 2 (x − y) f − f . (2)
∂y2 x x x x

Download From : www.EasyEngineering.net


Download From : www.EasyEngineering.net

Function of Several Variables 79

∂u
Differentiating w.r.t. x partially we get
∂y
∂2 u
!
 y  ′′  y   −y  ′ y
  −1 ′ y
   −y 
= 1− f + f (−y) − f
∂x∂y x x x2 x x2 x x2
(x − y)y ′′ y   y  y  y  y 
=− 3
f + 2 f′ + 2 f′
x x x x x x
(x − y) · y ′′  y  2y ′  y 
=− f + 2f .
x3 x x x
∂2 u −2y2   4y2  y 
′′ y
2xy = 2 (x − y) f + f′ . (3)
∂x∂y x x x x

ww ∂2 u
(1) + (2) + (3) ⇒ x2 2 + 2xy
∂x
∂2 u
∂x∂y
∂2 u
+ y2 2 = 0.
∂y

w.E
Homogeneous function. A function f (x, y) is said to be a homogeneous function
in x and y of degree n if f (tx, ty) = tn f (x, y) for any positive t.
Example

asy
(i) f (x, y) = x2 + y2 + 2xy is a homogeneous function of degree 3 in x and y.

En
 
(ii) If tan−1 yx = u then tan u is a homogeneous function of degree 0.
Note. If u = f (x, y) is a homogeneous function of degree n in x and y, then u can
be written as u = xn F .
y
x gin
ee
3
x +y 3
Example. Let u = .
x+y

u(tx, ty) =
t3 x3 + t3 y3 t3 (x3 + y3 )
= = t2
x3 + y3
= t2 u. rin
tx + ty
u is a homogeneous function of degree 2.
t(x + y) x+y

g.n
Now u = 

x3 1 + 3

x 1+
y3 
x
y
x
et
 y 3
1 + x 
2
=x y
1+
x
 y 3
y y 1+
= x2 F where F x
= y .
x x 1+
x

Download From : www.EasyEngineering.net


Download From : www.EasyEngineering.net

80 Engineering Mathematics - I

∴ u is a homogeneous function of degree 2 in x and y.

2.3 Euler’s Theorem

Statement. If f (x, y) is a homogeneous function of degree n in x and y having


∂f ∂f
continuous partial derivatives, then x +y = nf.
∂x ∂y
Proof.
Given that f (x, y) is a homogeneous function of degree n in x and y.

wwHence f (x, y) can be written as

w.E
y
f (x, y) = xn F .
x
∂f y  y  −y 
= nxn−1 F + xn F ′

asy∂x
= nxn−1 F
x
y
x
x x2
− xn−2 yF ′ .
y
x
∂f
∂y En n ′ y 1
=x F
  
x x
=x Fn−1 ′ y
 
x
.

x
∂f
∂x
+y
∂f
∂y gin
= nxn F
y
x
− xn−1 yF ′
y
x
+ xn−1 yF ′
y
x

x
∂f
∂x
+y
∂f
∂y
ee
= nxn F
y

= n f (x, y).
x

rin
Extension.
g.n
If f (x1 , x2 , . . . , xn ) is a homogeneous function of degree n in

et
∂f ∂f ∂f
x1 , x2 , . . . , xn , then x1 + x2 + · · · + xn = nf.
∂x1 ∂x2 ∂xn
Euler’s theorem on higher partial derivatives
Statement. If f (x, y) is a homogeneous function of degree n in x and y then,
∂2 f ∂2 f ∂2 f
x2 2 + 2xy + y2 2 = n(n − 1) f .
∂x ∂x∂y ∂y
Proof. Given, f (x, y) is a homogeneous function of degree n in x and y.
∴ By Euler’s theorem we have

∂f ∂f
x +y = n f. (1)
∂x ∂y

Download From : www.EasyEngineering.net


Download From : www.EasyEngineering.net

Function of Several Variables 81

Differentiating this partially w.r.t. x we obtain

∂2 f ∂ f ∂2 f ∂f
x + + y =n .
∂x2 ∂x ∂x∂y ∂x

∂2 f ∂2 f ∂f ∂f ∂f
=⇒ x + y =n − = (n − 1) .
∂x2 ∂x∂y ∂x ∂x ∂x
Multiplying both sides by x we get

∂2 f ∂2 f ∂f
x2 + xy = (n − 1)x . (2)

ww ∂x 2

Differentiating (1) partially w.r.t. y we get


∂x∂y ∂x

w.E x
∂2 f
∂y∂x
∂2 f ∂ f
+y 2 +
∂y ∂y
=n
∂f
∂y

x
asy∂2 f ∂2 f
+y 2 =n
∂f ∂f

∂f
= (n − 1) .

En
∂y∂x ∂y ∂y ∂y ∂y

∂2 f ∂2 f

gin
Since the partial derivatives of f are continuous we have =
∂x∂y ∂y∂x
.

Multiplying both sides by y we get


∴x
ee
∂2 f
∂x∂y
∂2 f
∂y
∂f
+ y 2 = (n − 1) .
∂y

rin
∂2 f ∂2 f ∂f
+ y2 2 = (n − 1)y .
g.n
et
xy (2)
∂x∂y ∂y ∂y

∂2 f ∂2 f 2
2∂ f
 ∂f ∂f 
(1) + (2) =⇒ x2 + 2xy + y = (n − 1) x + y
∂x2 ∂x∂y ∂y2 ∂x ∂y
= (n − 1)n f [By Euler’s theorem]

= n(n − 1) f.
✎ ☞
Worked Examples
✍ ✌

Download From : www.EasyEngineering.net


Download From : www.EasyEngineering.net

82 Engineering Mathematics - I

∂u ∂u ∂u
Example 2.19. If u = (x − y)(y − z)(z − x) prove that + + = 0 and
∂x ∂y ∂z
∂u ∂u ∂u
x +y +z = 3u.
∂x ∂y ∂z
Solution. u = (x − y)(y − z)(z − x).

∂u 
= (y − z) (x − y)(−1) + z − x
∂x
= (y − z)(−x + y + z − x)

= (y − z)(y + z − 2x).

ww ∂u
Similarly,
∂y
= (z − x)(z + x − 2y) and
∂u
∂z
= (x − y)(x + y − 2z).

w.E ∂u ∂u ∂u
+ +
∂x ∂y ∂z
= y2 − z2 − 2x(y − z) + z2 − x2 − 2y(z − x) + x2 − y2 − 2z(x − y).

asy
= −2xy + 2xz − 2yz + 2xy − 2xz + 2yz = 0

Also, u(tx, ty, tz) = (tx − ty)(ty − tz)(tz − tx)

En
= t (x − y)(y − z)(z − x) = t3 u.
3

gin
∴ u is a homogeneous function of degree 3 in x, y, z.

ee
∂u ∂u ∂u
∴ By Euler’s theorem, x + y + z = 3u.
∂x ∂y ∂z

 x3 + y3  ∂u ∂u rin
Example 2.20. If u = log
x+y
 x3 + y3 
, prove that x + y
x3 + y3
∂x ∂y
= 2.
g.n
Solution. Given: u = log
x+y
⇒ eu =

eu(tx,ty) =
x+y

t3 x3 + t3 y3 t3 (x3 + y3 )
=
et
tx + ty t(x + y)
3
x +y 3
= t2 = t2 eu .
x+y

∴ eu is a homogeneous function of degree 2.


Here n = 2.

Download From : www.EasyEngineering.net


Download From : www.EasyEngineering.net

Function of Several Variables 83

∴ By Euler’s theorem,
∂ u ∂
(e ) + y (eu ) = 2eu .
x
∂x ∂y
∂u ∂u
=⇒ xeu + yeu = 2eu
∂x ∂y
 ∂u ∂u 
eu x + y = 2eu .
∂x ∂y
∂u ∂u
x +y = 2.
∂x ∂y
y z ∂u ∂u ∂u

wwExample 2.21. If u = + , find the value of x + y + z .


x x
y z
Solution. Given: u = + .
x x
∂x ∂y ∂z

w.E u(tx, ty, tz) =


ty tz
+
tx tx
y z
= t0 +
x x
= t0 u.

asy
=⇒ u is a homogeneous function in x, y, z of degree 0.
By Euler’s theorem,
∂u ∂u ∂u
∂x
x y z
x
En
+y +z
∂y ∂z
= nu = 0.u = 0.

∂u ∂u ∂u
y z x
x y z
Solution. u(x, y, z) = + + . gin
Example 2.22. If u = + + , then show that x + y + z
∂x ∂y ∂z
= 0. [Jun 2012]

y z x
ee
u(tx, ty, tz) =
tx ty tz
+ +
ty tz tx
tx y z rin
= + +
t y z x
= t0 u(x, y, z). g.n
u is a homogeneous function of degree 0 in x, y, z.
∂u ∂u
∴ By Euler’s theorem, x + y + z
∂x ∂y
∂u
∂z
= 0.u = 0.
et
x2 y2 ∂u ∂u
Example 2.23. If sin u = , show that x + y = 3 tan u.
x+y ∂x ∂y
x2 y2
Solution. sin u(x, y) = .
x+y
t2 x2 t2 y2 t4 (x2 y2 )
sin u(tx, ty) = = = t3 sin u.
tx + ty t(x + y)

Download From : www.EasyEngineering.net


Download From : www.EasyEngineering.net

84 Engineering Mathematics - I

sin u is a homogeneous function of degree 3 in x and y.


By Euler’s theorem,
∂ ∂
x(sin u) + y (sin u) = 3 sin u.
∂x ∂y
∂u ∂u
x cos u + y cos u = 3 sin u
∂x ∂y
 ∂u ∂u 
cos u x + y = 3 sin u
∂x ∂y
∂u ∂u 3 sin u 3 sin u
x +y = = = 3 tan u.

ww
∂x ∂y cos u cos u
x y z ∂u ∂u ∂u
Example 2.24. If u = f , , , prove that x + y + z = 0.
y z x ∂x ∂y ∂z

w.E
Solution.

u(x, y, z) = f
x y z
, ,

asy
u(tx, ty, tz) = f
y z x
 tx ty tz 
, ,
ty tz tx

En x y z
= f , ,
y z x

gin = u(x, y, z) = t0 u(x, y, z)

∂u ∂u
∴ By Euler’s theorem, x + y + z
∂x ∂y
∂u
∂z
ee
u is a homogeneous function of degree 0 in x, y, z.
= 0.u = 0.
rin
Example 2.25. If u = tan−1
x−y
 x3 + y3  ∂u
show that x + y
∂x
∂u
∂y
= sin 2u.
g.n
Solution. tan u =
x3 + y3
x−y
.
tan u is a homogeneous function of degree 2 in x and y.
By Euler’s theorem
et
∂ ∂
x (tan u) + y (tan u) = 2 tan u
∂x ∂y
∂u ∂u
sec2 ux + sec2 uy = 2 tan u
∂x ∂y
∂u ∂u 2 tan u 2 sin u
x +y = = cos2 u
∂x ∂y sec2 u cos u

Download From : www.EasyEngineering.net


Download From : www.EasyEngineering.net

Function of Several Variables 85

= sin 2u.
 x+y  ∂u ∂u 1
Example 2.26. If u = sin−1 √ √ prove that x + y = tan u. [Jun 2009]
x+ y ∂x ∂y 2
 x+y  x+y
Solution. u = sin−1 √ √ ⇒ sin u = √ √ .
x+ y x+ y
sin u is a homogeneous function of degree 21 in x and y.
By Euler’s theorem,
∂ ∂ 1
x (sin u) + y (sin u) = sin u.

ww
∂x ∂y 2
∂u ∂u 1
cos ux + cos uy = sin u
∂x ∂y 2

w.E ∂u
x +y
∂x
∂u 1
∂y 2
= tan u.

asy
 
 x + y 
−1  ∂u ∂u −1
Example 2.27. If u = cos  √ √ , prove that x + y = cot u. [Dec 2011]
∂x ∂y 2
 x+ y

En
 x + y 
Solution. u = cos−1  √ √ .
x + y

gin

 x + y 
cos u =  √ √ .
x+ y
cos u is a homogeneous function of degree 21 in x and y.
∴ By Euler’s theorem

x

ee∂
(cos u) + y (cos u) =
1
· cos u. rin
∂x
∂u
∂y
x(− sin u) + y(− sin u)
∂u
=
2
1
cos u. g.n
∂x
∂u
− sin u x + y
∂x
∂u
∂y

∂y

!
=
2
1
2
cos u.

−1 cos u −1
et
∴ x +y = = cot u.
∂x ∂y 2 sin u 2
 x y
Example 2.28. Verify Euler’s theorem for the function u = sin−1 + tan−1 .
 x y y x
Solution. Given: u = sin−1 + tan−1 .
y x
 tx   ty 
u(tx, ty) = sin−1 + tan−1
ty tx

Download From : www.EasyEngineering.net


Download From : www.EasyEngineering.net

86 Engineering Mathematics - I

  x  y 
= t0 sin−1 + tan−1
y x
= t0 u(x, y).
=⇒ u is a homogeneous function of degree 0 in x and y.
∂u ∂u
∴ By Euler’s theorem, x + y = 0.
∂x ∂y
Verification.

∂u 1 1 1  −y 
= s +
∂x
x2
y y2 x2
1 +

ww 1− 2

= p
y
y 1
x2

− 2 2
x2 y

w.E = p
y2 − x2 y x (x + y )
1
− 2
y
x + y2
.
2

asy
2
y −x 2

∂u x xy
x = p − 2 (1)
∂x 2
y −x 2 x + y2

En ∂u
= s
1  −x 
2
+
1 1
y2 x

gin
∂y
x2 y 1 +
1− 2 x2
y

ee= p

=− p
−xy
y2 y2 − x2 x + y
x
+ 2

+ 2
x
x

+
x
2

y2 rin
g.n
2
y y −x 2

∂u x xy
y =−p + 2 (2)
∂y x + y2

et
2
y −x 2

∂u ∂u
(1) + (2) =⇒ = x +y = 0.
∂x ∂y

Hence, Euler’s theorem is verified.

1 1
Example 2.29. Verify Euler’s theorem for the function u = x 2 + y 2 (xn + yn ).
 
Solution. It is easy to verify that u is a homogeneous function of degree n + 21 in
x and y.

Download From : www.EasyEngineering.net


Download From : www.EasyEngineering.net

Function of Several Variables 87

Hence, by Euler’s theorem we have


∂u ∂u  1
x +y = n + u. (1)
∂x ∂y 2
Verification
∂u 1 1 1 1 
= x 2 + y 2 nxn−1 + (xn + yn ) x− 2
∂x 2
∂u 1 1 1 1
x = n x 2 + y 2 xn + (xn + yn )x 2 .
∂x 2

wwSince the function is symmetric in x and y we have


∂u 1 1 1 1
= n x 2 + y 2 yn + (xn + yn )y 2 .
y

w.E ∂u
x +y
∂x
∂y
∂u
∂y
1 1
2
1
2
1 1
= n x 2 + y 2 (xn + yn ) + (xn + yn ) x 2 + y 2

= n+
asy1 1
2
1
x 2 + y 2 (xn + yn )

En
1
= n + u,
2
which verifies Euler’s theorem.
ygin  x
Example 2.30. If u = x2 tan−1
∂2 u
x2
∂x2
+ 2xy
∂2 u
∂x∂y
+ y
2
2∂ u
∂y2
.
x
ee
− y2 tan−1
y
, find the value of

rin
g.n
Solution. u is a homogeneous function of degree 2 in x and y.
∂2 u ∂2 u ∂2 u
By Euler’s theorem, x2 2 + 2xy + y2 2 = 2(2 − 1)u = 2u [n = 2].
∂x ∂x∂y ∂y

Example 2.31. If z = x f

Solution. Let u = x f
y
y
x
y
x
y
and v = g .
∂2 z
+ g , show that x2 2 + 2xy
∂x
∂2 z
∂x∂y
∂2 z
+ y2 2 = 0.
∂y
et
x x
∴ z = u + v.
u is a homogeneous function of degree 1 in x and y.
∴ By Euler’s theorem,
∂2 u ∂2 u 2
2∂ u
x2 + 2xy + y = n(n − 1)u = 1(1 − 1)u = 0. (1)
∂x2 ∂x∂y ∂y2

Download From : www.EasyEngineering.net


Download From : www.EasyEngineering.net

88 Engineering Mathematics - I

v is a homogeneous function of degree 0 in x and y.


By Euler’s theorem,
∂2 v ∂2 v 2
2∂ v
x2 + 2xy + y = 0(0 − 1)v = 0. (2)
∂x2 ∂x∂y ∂y2
∂2 (u + v) ∂2 (u + v) 2
2 ∂ (u + v)
(1) + (2) ⇒ x2 + 2xy + y =0
∂x2 ∂x∂y ∂y2
∂2 z ∂2 z 2
2∂ z
x2 + 2xy + y = 0.
∂x2 ∂x∂y ∂y2

ww
3 3
!
−1 x + y
Example 2.32. If u = tan , prove that
x−y
∂2 u ∂2 u ∂2 u
x2 2 + 2xy + y2 2 = 2 sin u cos 3u.

w.E ∂x
Solution. u = tan−1
∂x∂y
x 3 + y3

x−y
!∂y
.

tan u =
x3 + y3
x−y
!

asy
En
tan u is an homogeneous function of degree 2 in x and y.
By Euler’s theorem,
∂ ∂

gin
x (tan u) + y (tan u) = 2 tan u.
∂x ∂y

ee
∂u ∂u
x · sec2 u + y · sec2 u = 2 tan u.
∂x ∂y

rin
!
2 ∂u ∂u
sec u x + y = 2 tan u.
∂x ∂y
∂u
x +y
∂x
∂u 2 tan u
∂y
= 2
sec u
=2·
sin u
cos u
· cos2 u
g.n
x
∂u
∂x
+y
∂u
∂y
= 2 sin u cos u

= sin 2u.
et (1)

Differentiating (1) partially w.r.t. x we get


∂2 u ∂u ∂2 u ∂u
x· 2 + +y = 2 cos 2u .
∂x ∂x ∂x∂y ∂x
Multiplying by x we get
∂2 u ∂u ∂2 u ∂u
x2 2
+ x + xy = 2 cos 2u · x . (2)
∂x ∂x ∂x∂y ∂x

Download From : www.EasyEngineering.net


Download From : www.EasyEngineering.net

Function of Several Variables 89

Differentiating (1) partially w.r.t. y we get


∂2 u ∂2 u ∂u ∂u
x +y 2 + = 2 cos 2u .
∂y∂x ∂y ∂y ∂y
Multiplying w.r.t. y we get
∂2 u ∂2 u ∂u ∂u
xy + y2 2 + y = 2 cos 2u · y . (3)
∂y∂x ∂y ∂y ∂y
(2) + (3) ⇒
2 ∂2 u 2
!
2∂ u 2∂ u ∂u ∂u ∂u ∂u
x + 2xy +y +x +y = 2 cos 2u x + y
∂x2 ∂x∂y ∂y2 ∂x ∂y ∂x ∂y

ww 2
∂ u
x2 2 + 2xy
∂x
2
∂ u
∂x∂y
2
∂ u
∂y
∂u
+ y2 2 = (2 cos 2u − 1) x + y
∂x
∂u
∂y
!

w.E = (2 cos 2u − 1) sin 2u

= sin 2u{2(2 cos2 u − 1) − 1}

asy = sin 2u{4 cos2 u − 3}

En = 2 sin u cos u(4 cos2 u − 3)

= 2 sin u(4 cos3 u − 3 cos u)

gin = 2 sin u · cos 3u.

2.4 Total Derivative - Implicit functions


ee rin
We know that if y = f (u) and u = φ(x), then
dy dy du
=
dx du dx g.n
. We shall now extend this
result to function of several variables.
Let z = f (x, y) possess continuous partial derivatives and let x, y be differentiable
functions of an independent variable t. Let ∆x, ∆y and ∆z be increments in x, y and
et
z respectively, corresponding to an increment ∆t in t. Then the total increment in z
is given by

∆z = f (x + ∆x, y + ∆y) − f (x, y)

= f (x + ∆x, y + ∆y) − f (x, y + ∆y) + f (x, y + ∆y) − f (x, y).

Download From : www.EasyEngineering.net


Download From : www.EasyEngineering.net

90 Engineering Mathematics - I

By mean value theorem, f (x + ∆x, y + ∆y) − f (x, y + ∆y) = f x (x + θ1 x, y + ∆y)∆x and


f (x, y + ∆y) − f (x, y) = fy (x, y + θ2 ∆y)∆y, 0 < θ1 , θ2 < 1.

∆z = f x (x + θ1 x, y + ∆y)∆x + fy (x, y + θ2 ∆y)∆y (1)

where θ1 and θ2 are positive proper fractions.

∆z ∆x ∆y
∴ = f x (x + θ1 x, y + ∆y) + fy (x, y + θ2 ∆y) .
∆t ∆t ∆t

Taking limits as ∆t → 0, we get the total derivative

ww dz
dt
dx dy
= f x (x, y) + fy (x, y) .
dt dt

w.E i.e.,
dz ∂z dx ∂z dy
= +
dt ∂x dt ∂y dt
. (2)

asy
In the same way if u = f (x1 , x2 , . . . , xn ) possesses continuous partial derivatives
with respect to each of the independent variables and if x1 , x2 , . . . , xn are

En
differentiable functions of an independent variable t, then the total derivative of u
with respect to t is given by

du gin
∂u dx1 ∂u dx2 ∂u dxn
dt
=
∂x1 dt
+

ee
∂x2 dt
+ ··· +
∂xn dt
.

Note. (2) expresses the rate of change of z w.r.t. t along the curve rin
(3)

x = x(t), y = y(t).
g.n
Composite functions. Let z = f (u, v) and u and v are themselves functions of the
independent variables x, y so that u = φ(x, y) and v = ψ(x, y).
To find
∂z
, we consider y as a constant so that u and v may be supposed to be
et
∂x
functions of x only. Hence, by the above result we have
∂z ∂z ∂u ∂z ∂v ∂z ∂z ∂u ∂z ∂v
= + . Similarly = + .
∂x ∂u ∂x ∂v ∂x ∂y ∂u ∂y ∂v ∂y
Now, in (2), if we replace t by x then y is a function of x we get

dz ∂z ∂z dy
= + .
dx ∂x ∂y dx

Download From : www.EasyEngineering.net


Download From : www.EasyEngineering.net

Function of Several Variables 91

dz ∂z dx ∂z
Similarly = + .
dy ∂x dy ∂y

Implicit Functions. Let f (x, y) = c, where c is a constant, define y as an implicit


function of x.
If u = f (x, y), then we have
du ∂ f ∂ f dy
= + .
dx ∂x ∂y dx
du
But = 0, since u = f (x, y) = c
dx

ww i.e.,
∂ f ∂ f dy
+
∂x ∂y dx
=0

w.E ∴
dy
dx
∂f
fx
= − ∂x = − .
∂f fy

Similarly
dx fy
=− .
asy ∂y

dy fx
En
Total differential. The total differential dz of z is defined as the principal part of
the increment ∆z which is given by
gin ∂z ∂z

ee

dz =
∂x
dx + dy.

Worked Examples

∂y


dz rin
Example 2.33. If z = xn ym where x = cos at, y = sin bt, find .

Solution. We have
dz ∂z dx ∂z dy
= + .
dt
g.n
Now
dx
∂z
∂x
= nxn−1 ym ,

= −a sin at,
dy
∂z
∂y
dt ∂x dt ∂y dt
= mxn ym−1

= b cos bt.
et
dt dt
dz
∴ = −anxn−1 ym sin at + bmxn ym−1 cos bt
dt
 an bm 
= −xn ym sin at − cos bt
x y
= − cosn at sinm bt(an tan at − bm cot bt).

Download From : www.EasyEngineering.net


Download From : www.EasyEngineering.net

92 Engineering Mathematics - I

du
Example 2.34. If u = x2 y3 , x = log t, y = et , find . [Jan 2000]
dt
du ∂u dx ∂u dy
Solution. We have = + .
dt ∂x dt ∂y dt
∂u
= y3 2x = 2xy3
∂x
∂u
= x2 3y2 = 3x2 y2
∂y
dx 1 dy
= = et
dt t dt

ww du 1
= 2xy3 + 3x2 y2 et

w.E
dt t
2 log te3t
= + 3(log t)2 e2t et
t

asy
2 
= e3t log t + 3 log t
t
e3t log t

En =
t
(2 + 3t log t).

Example 2.35. Find


Solution.
du
dt
gin
when u = x2 y, x = t2 , y = et .

ee
∂u
∂x
dx
= 2xy,

= 2t
∂u
∂y
dy
= x2

= et rin
dt

du ∂u dx ∂u dy
dt

g.n
we have
dt
= +
∂x dt ∂y dt
= 2xy.2t + x2 et

= 4t2 et t + t4 et = 4t3 et + t4 et = t3 et (4 + t).


et
1 du
Example 2.36. If u = xy + yz + zx where x = , y = et and z = e−t find . [Jun 2013]
t dt
Solution.
∂u ∂u ∂u
= y + z, = x + z, = x+y
∂x ∂y ∂z
dx −1 dy dz
= 2, = et , = −e−t
dt t dt dt

Download From : www.EasyEngineering.net


Download From : www.EasyEngineering.net

Function of Several Variables 93

du ∂u dx ∂u dy ∂u dz
we have = + +
dt ∂x dt ∂y dt ∂z dt
 −1 
= (y + z) 2 + (x + z)et + (x + y)(−e−t )
t
et + e−t 1 1 
=− 2
+ (e−t + )et − + et e−t
t t t
−2 et e−t
= 2 cos ht + 1 + − −1
t t t
−2 2 sin ht
= 2 cos ht +
t t
2

ww = 2 (2t sin ht − cos ht).


t

w.E
Example 2.37. Find
Solution.
du
dt
 
when u = sin yx , x = et , y = t2 .

asy
∂u
∂x
= cos
 x1
y y
,
∂u
∂y
x
= − 2 cos
y
 x
y

En
gindx
dt
= et
dy
dt
= 2t

du ∂u dx ∂u dy 1
dt
= +
∂x dt ∂y dt y
ee
= cos
 x
y
et −
x
y2
cos
 x
y
2t
rin
= cos
 x  et 2tet !
y t 2
− 4 = cos
t
 x
y
et
1 2t

t2 t4
!
=
et (t − 2)
t3
cos
g.n
 et 
t2
.

Example 2.38. If u = sin−1 (x − y) where x = 3t, y = 4t3 , show that


du
dt
et
= √
3
1 − t2
.
Solution.
∂u 1 ∂u −1
= p , = p
∂x 1 − (x − y)2 ∂y 1 − (x − y)2

dx dy
=3 = 12t2
dt dt

Download From : www.EasyEngineering.net


Download From : www.EasyEngineering.net

94 Engineering Mathematics - I

du ∂u dx ∂u dy
= +
dt ∂x dt ∂y dt
1 1
= p 3− p 12t2
1 − (x − y) 2 1 − (x − y) 2

3 − 12t 2 3 − 12t2
= p = p
1 − (3t − 4t3 )2 1 − (9t2 + 16t6 − 24t4 )
3 − 12t 2 3 − 12t2
= √ = p
1 − 9t2 − 16t6 + 24t4 −(16t6 − 24t4 + 9t2 − 1)
3(1 − 4t2 ) 3(1 − 4t2 )
= p = p

ww =
−(t2 − 1)(16t4 − 8t2 + 1)
3(1 − 4t2 )
p = √
3
.
−(t2 − 1)(1 − 4t2 )2

w.E du
(1 − 4t2 ) (1 − t2 ) 1 − t2

Example 2.39. Find


dx
Solution. We have du =
∂u
∂x
asy
if u = cos(x2 + y2 ) and a2 x2 + b2 y2 = c2 .
∂u
dx + dy
∂y
[Jan 2001]

En
du ∂u ∂u dy
= +
dx ∂x ∂y dx
gin
= − sin(x2 + y2 )2x − sin(x2 + y2 )2y
dy
dx

Given that a2 x2 + b2 y2 = c2 .
Diff. w.r.t. x we get
ee rin
dy
2a2 x + 2b2 y
dx
= 0 ⇒ b2 y
dy
dx
= −a2 x ⇒
dy
dx
=− 2
a2 x
b y g.n

du
dx

= −2 sin(x2 + y2 ) x + y
dy 
dx

= −2 sin(x2 + y2 ) x + y 2
 a2 − b2  2(a2 − b2 )x
et
 −a2 x 
b y

= 2 sin(x2 + y2 ) xy = sin(x2 + y2 ).
b2 y b2

dz
q
Example 2.40. If z = x2 + y2 and x3 + y3 + 3axy = 5a2 , find the value of when
dx
x = y = a.
p
Solution. Given z = x2 + y2 .

Download From : www.EasyEngineering.net


Download From : www.EasyEngineering.net

Function of Several Variables 95

We have

∂z ∂z
dz =dx + dy
∂x ∂y
∂z 2x x
Now = p = p
∂x 2 x2 + y2 x2 + y2
∂z y
= p .
∂y x2 + y2

Also x3 + y3 + 3axy = 5a2

wwDiff.w.r.t. x we get
3x2 + 3y2
dy
dx
h dy i
+ 3a x + y = 0
dx

w.E x2 + y2
dy
dx
dy
+ ax + ay = 0
dx

asy dy 2
dx
(y + ax) = −ay − x2

En dy −(x2 + ay)
dx
= 2
y + ax
.

dz
= + gin
∂z ∂z dy
= p
x
+ p
y −(x2 + ay)
!

ee
∴ 2
dx ∂x ∂y dx x2 + y2 x2 + y2 y + ax
a  −2a2 

rin
 dz  a a a
= √ + √ = √ − √ = 0.
dx x=y=a a 2 a 2 2a2 a 2 a 2

∂z ∂z ∂z ∂z
−v

g.n
Example 2.41. If z is a function of x and y and x = e + e , y = e−u − ev , show that
u


∂u ∂v
= x −y .
∂x ∂y
Solution. z is a composite function of u and v.
We have
et
[Jun 2001]

∂z ∂z ∂x ∂z ∂y ∂z u ∂z ∂z u ∂z −u
= + = e + (−e−u ) = e − (e ).
∂u ∂x ∂u ∂y ∂u ∂x ∂y ∂x ∂y
∂z ∂z ∂x ∂z ∂y ∂z ∂z ∂z ∂z
= + = (−e−v ) + (−ev ) = − e−v − (ev ).
∂v ∂x ∂v ∂y ∂v ∂x ∂y ∂x ∂y
∂z ∂z ∂z u ∂z ∂z ∂z
− = (e + e−v ) − (e−u − ev ) = x − y .
∂u ∂v ∂x ∂y ∂x ∂y

Download From : www.EasyEngineering.net


Download From : www.EasyEngineering.net

96 Engineering Mathematics - I

Example 2.42. If z = f (x, y) where x = eu cos v, y = eu sin v, show that


∂z ∂z ∂z
y + x = e2u . [Jan 2000]
∂u ∂v ∂y
Solution. z is a composite function of u and v.
∂z ∂z ∂x ∂z ∂y ∂z u ∂z
= + = e cos v + eu sin v.
∂u ∂x ∂u ∂y ∂u ∂x ∂y
∂z ∂z ∂x ∂z ∂y ∂z ∂z
= + = (−eu sin v) + eu cos v.
∂v ∂x ∂v ∂y ∂v ∂x ∂y
∂z ∂z u ∂z u ∂z ∂z
y = y e cos v + y e sin v = eu sin veu cos v + eu sin veu sin v
∂u ∂x ∂y ∂x ∂y

ww ∂z
∂z
= e2u sin v cos v + e2u sin2 v .
∂x
∂z
∂z
∂y
∂z

w.E x = −eu sin veu cos v + eu cos veu cos v


∂v
∂z
∂x
= −e2u sin v cos v + e2u cos2 v
∂z
∂y

Adding we get yasy∂z


∂x
∂z
+ x = e2u .
∂z
∂y

En
∂u
y − x z − x
∂v ∂y
∂u ∂u ∂u
Example 2.43. If u = f

Solution. Given: u = f
xy
,gin
,
zx
y − x z − x
prove that x2 + y2 + z2
∂x ∂y ∂z
= 0.

Let r =
y−x
xy
∴ u = f (r, s).
,s=
z−x
zx
xy
1 1
ee
zx
,r = − , s = − .
x y
1 1
x z
rin
∴ u is a function of r and s and r and s are functions of x, y and z.
Now g.n
∴ x2
∂u
∂x
∂u
=
∂u ∂r ∂u ∂s ∂u  −1  ∂u  −1 
+ =
∂r ∂x ∂s ∂x ∂r x2
∂u ∂u
=− − .
+
∂s x2 et (1)
∂x ∂r ∂s
∂u ∂u ∂r ∂u ∂s ∂u  −1  ∂u
= + = + (0)
∂y ∂r ∂y ∂s ∂y ∂r y2 ∂s
∂u ∂u
y2 = . (2)
∂y ∂r
∂u ∂u ∂r ∂u ∂s ∂u ∂u  1 
= + = (0) +
∂z ∂r ∂z ∂s ∂z ∂r ∂s z2

Download From : www.EasyEngineering.net


Download From : www.EasyEngineering.net

Function of Several Variables 97

∂u ∂u
z2 = . (3)
∂z ∂s

∂u ∂u ∂u
(1) + (2) + (3) =⇒ x2 + y2 + z2 = 0.
∂x ∂y ∂z

∂u ∂v
Example 2.44. If u2 + 2v2 = 1 − x2 + y2 and u2 + v2 = x2 + y2 − 2, find and .
∂x ∂x
Solution. Consider u and v as functions of x and y.

u2 + 2v2 = 1 − x2 + y2 .

wwDifferentiating partially w.r.t. x

w.E 2u
∂u
∂x
+ 4v
∂v
∂x
= −2x

asy
∂u ∂v
u + 2v = −x. (1)
∂x ∂x
u2 + v2 = x2 + y2 − 2

En
Differentiating partially w.r.t. x

gin ∂u ∂v

ee 2u
∂x
∂u
u +v
∂x
+ 2v
∂x
∂v
∂x
∂v
= 2x

= x.
rin (2)

(1) − (2) ⇒ v
∂x
∂v
= −2x
−2x g.n
et
⇒ =
∂x v
∂u (−2x)
(1) ⇒ u + 2v = −x.
∂x v
∂u
u = 3x
∂x
∂u 3x
⇒ = .
∂x u

Example 2.45. If z is a function of x and y and


∂2 z ∂2 z ∂2 z ∂2 u
x = u cos α − v sin α, y = u sin α + v cos α, show that + = + . [Jun 2000]
∂x2 ∂y2 ∂u2 ∂v2

Download From : www.EasyEngineering.net


Download From : www.EasyEngineering.net

98 Engineering Mathematics - I

Solution.
∂z ∂z ∂x ∂z ∂y ∂z ∂z
= + = cos α + sin α.
∂u ∂x ∂u ∂y ∂u ∂x ∂y
∂2 z ∂  ∂z  ∂x ∂  ∂z  ∂y
= +
∂u2 ∂x ∂u ∂u ∂y ∂u ∂u
∂  ∂z ∂z  ∂  ∂z ∂z 
= cos α + sin α cos α + cos α + sin α sin α
∂x ∂x ∂y ∂y ∂x ∂y
2
∂ z 2
∂ z 2
∂ z ∂2 z
= 2 cos2 α + sin α cos α + cos α sin α + 2 sin2 α. (1)
∂x ∂x∂y ∂y∂x ∂y

ww
∂z ∂z ∂x ∂z ∂y ∂z ∂z
= + = (− sin α) + cos α.
∂v ∂x ∂v ∂y ∂v ∂x ∂y
∂2 z ∂  ∂z  ∂x ∂  ∂z  ∂y

w.E =

=
∂x

∂x
+
∂v2 ∂x ∂v ∂v ∂y ∂v ∂v
∂  ∂z
sin α +
∂z
∂y

cos α (− sin α) +
∂  ∂z
∂y

∂x
sin α +
∂z
∂y

cos α cos α.
2
∂ z
= 2 sin2 α −
∂x asy
2
∂ z
∂x∂y
sin α cos α −
2
∂ z
∂y∂x
2
∂ z
cos α sin α + 2 cos2 α.
∂y
(2)

(1) + (2) ⇒ + En
∂2 z ∂2 z ∂2 z
= (sin2
α + cos 2
α) +
∂2 z
(sin2 α + cos2 α)

gin
∂u2 ∂v2 ∂x2

=
∂2 z ∂2 z
+ .
∂y2

Example 2.46. If F is
2
∂ F ∂ F
that 2 + 2 = e−2u
2 ∂ F ∂ F
+ 2
ee
" 2a function2
∂x2 ∂y2
u

rin
u
# of x and y and if x = e sin v, y = e cos v, prove
[Jan 2013]

g.n
∂x ∂y ∂u2 ∂v
∂F ∂F ∂x ∂F ∂y
Solution. = · + ·
∂u ∂x ∂u ∂y ∂u

∂2 F
∂u 2
=
∂ ∂F ∂x
=
!
∂F u
∂x
· e sin v +

+
∂x ∂u ∂u ∂y ∂u ∂u
∂ ∂F ∂y
!
∂F u
∂y
· e cos v.
et
" # " #
∂ ∂F u ∂F u ∂x ∂ ∂F u ∂F u ∂y
= · e sin v + · e cos v + · e sin v + · e cos v
∂x ∂x ∂y ∂u ∂y ∂x ∂y ∂u
" 2
∂2 F u
" 2
∂2 F u
# #
∂ F u u ∂ F u
= · e sin v + · e cos v · e sin v + · e cos v + · e sin v · eu cos v
∂x2 ∂x∂y ∂y2 ∂y∂x
∂2 F ∂2 F 2u ∂2 F 2u ∂2 F
= 2 · e2u sin2 v + · e sin v cos v + · e sin v cos v + 2 · e2u cos2 v (1)
∂x ∂x∂y ∂y∂x ∂y

Download From : www.EasyEngineering.net


Download From : www.EasyEngineering.net

Function of Several Variables 99

∂F ∂F ∂x ∂F ∂y
= · + · .
∂v ∂x ∂v ∂y ∂v
∂F u ∂F u
= · e cos v − · e sin v.
∂x ∂y
∂2 F
! !
∂ ∂F ∂x ∂ ∂F ∂y
= +
∂v2 ∂x ∂v ∂v ∂y ∂v ∂v
" # " #
∂ ∂F u ∂F u u ∂ ∂F u ∂F u
= · e cos v − · e sin v e cos v + · e cos v − · e sin v (−eu sin v)
∂x ∂x ∂y ∂y ∂x ∂y
∂2 F ∂2 F 2u ∂2 F 2u ∂2 F
= 2 · e2u cos2 v − · e sin v cos v − · e sin v cos v + 2 · e2u sin2 v. (2)
∂x ∂x∂y ∂y∂x ∂y

ww ∂2 F ∂2 F ∂2 F 2u ∂2 F 2u
(1) + (2) ⇒ 2 + 2 = 2 e + 2 · e
∂u ∂v ∂x
" 2
∂y
∂2 F
#

w.E e−2u
" 2
∂ F ∂ F 2
#
=e

+ 2 = 2 + 2.
2u ∂ F

2
∂x2
∂ F ∂ F 2
+ 2
∂y

∂u 2 ∂v
asy ∂x ∂y

En
Example 2.47. Transform the equation z xx + 2z xy + zyy = 0 by changing the

gin
independent variables using u = x − y and v = x + y.
Solution. Consider z as a function of u and v and u and v are functions of x and y.
[Jun 2012]

∂z
=
∂z ∂u ∂z ∂v
·
∂x ∂u ∂x ∂v ∂x
=
∂z
·1+
+
∂z
·1
·ee rin
=
∂u
∂z ∂z
+ .
∂v
g.n
∂2 z
=
∂u ∂v!
∂ ∂z ∂u
·
∂x2 ∂u ∂x ∂x ∂v ∂x ∂x
∂ ∂z ∂z
!
+
!
∂ ∂z ∂v
·

∂ ∂z ∂z
!
et
= + ·1+ + ·1
∂u ∂u ∂v ∂v ∂u ∂v
∂2 z ∂2 z ∂2 z ∂2 z
= 2+ + + 2
∂u ∂u∂v ∂v∂u ∂v
i.e.,z xx = zuu + zuv + zvu + zvv (1)
∂z ∂z ∂u ∂z ∂v
= · + ·
∂y ∂u ∂y ∂v ∂y

Download From : www.EasyEngineering.net


Download From : www.EasyEngineering.net

100 Engineering Mathematics - I

∂z ∂z
= (−1) + ·1
∂u ∂v
∂z ∂z
=− + .
∂u !∂v
∂2 z
!
∂ ∂z ∂u ∂ ∂z ∂v
= +
∂y2 ∂u ∂y ∂y ∂v ∂y ∂y
! !
∂ ∂z ∂z ∂ ∂z ∂z
= − + (−1) + − + ·1
∂u ∂u ∂v ∂v ∂u ∂v
∂2 z ∂2 z ∂2 z ∂2 z
= 2− − + 2.
∂u ∂u∂v ∂v∂u ∂v

ww i.e., zyy = zuu − zuv − zvu + zvv


∂2 z
=
∂ ∂z
!
(2)

w.E ∂x∂y ∂x ∂y

=
∂ ∂z ∂u
!
+
∂u ∂y ∂x ∂v ∂y ∂x
∂ ∂z ∂v
!

=

asy
∂z ∂z
− +
∂u ∂u ∂v
!
·1+

− +
∂z ∂z
∂v ∂u ∂v
!
·1

∂2 z
=− 2 +
∂u En∂2 z

∂2 z
∂u∂v ∂v∂u ∂v
∂2 z
+ 2.

gin
i.e., z xy = −zuu + zuv − zvu + zvv (3)

(1) + (2) + 2 × (3) ⇒

ee
z xx + 2z xy + zyy = 2zuv − 2zvu + 4zvv

0 = 2zuv − 2zvu + 4zvv . rin


∴ zuv − zvu + 2zvv = 0
g.n
which is the required equation.

Example 2.48. If z = f (x, y)! where x = u2 − v2 and y = 2uv, prove that


et
∂2 z ∂2 z 2
2
2 ∂ z ∂2 z
+ = 4(u + v ) + . [Jan 2012, Jan 2010]
∂u2 ∂v2 ∂x2 ∂y2
Solution. z is a function of x and y and x and y are functions of u and v.
∴ z is a composite function of u and v.

∂z ∂z ∂x ∂z ∂y
= · + ·
∂u ∂x ∂u ∂y ∂u

Download From : www.EasyEngineering.net


Download From : www.EasyEngineering.net

Function of Several Variables 101

∂z ∂z
= · 2u + · 2v.
∂x ∂y
∂2 z
!
∂ ∂z
=
∂u2 ∂u ∂u
! !
∂ ∂z ∂x ∂ ∂z ∂y
= · + · .
∂x ∂u ∂u ∂y ∂u ∂u
" # " #
∂ ∂z ∂z ∂ ∂z ∂z
= · 2u + · 2v 2u + · 2u + · 2v 2v
∂x ∂x ∂y ∂y ∂x ∂y
∂2 z ∂2 z ∂2 z ∂2 z
= 2 · 4u2 + 4uv + 4uv + 2 · 4v2 .
∂x ∂x∂y ∂y∂x ∂y

ww ∂z
∂v
=
∂z ∂x ∂z ∂y
·
∂x ∂v ∂y ∂v
+ ·

w.E =
∂z
∂x
· (−2v) +
∂z
= −2v + 2u .
∂z
∂y
∂z
· 2u

∂2 z
= asy
∂ ∂z
∂x
!
∂y

∂v2

=
∂v ∂v

En
∂ ∂z ∂x
!
+
∂ ∂z ∂y
!
.

=

"
gin
∂x ∂v ∂v ∂y ∂v ∂v
∂z
−2v + 2u
∂z
#
(−2v) +

"
−2v ·
∂z
+ 2u ·
∂z
#
2u

∂2 z ∂2 z
∂x
∂2 z
= 4v2 2 − 4uv
2
∂x
ee
∂x

∂x∂y
∂y
∂2 z
− 4uv
2
∂2 z
∂y∂x
∂y

+ 4v2 2
∂y
∂x
∂2 z

rin
∂y

g.n
∂ z ∂ z
+ = 2 (4u2 + 4v2 ) + 2 (4u2 + 4v2 )
∂u2 ∂v2 ∂x ∂y
" 2
∂2 z
#
∂ z

Example 2.49. If g(x, y) = χ(u,


2
∂ g ∂ g2 2 2
∂ χ ∂ χ
2
= 4(u + v ) 2
+
∂x2 ∂y2
2 2
.

! v) where u = x − y , v = 2xy, prove that


et
2
+ 2 = 4(x2 + y2 ) + .
∂x ∂y ∂u2 ∂v2
Solution.
Similar to the previous problem.

Download From : www.EasyEngineering.net


Download From : www.EasyEngineering.net

102 Engineering Mathematics - I

du
Example 2.50. If u = x2 + y2 + z2 and x = e2t , y = e2t cos 3t, z = e2t sin 3t, find .
dt
[Dec 2011]
Solution.

du ∂u dx ∂u dy ∂u dz
= · + · + ·
dt ∂x dt ∂y dt ∂z dt
h i
= 2x · 2e2t + 2y −3e2t sin 3t + 2e2t cos 3t
h i
+ 2z 3e2t cos 3t + 2e2t sin 3t

ww
h i
= 2e2t · 2e2t + 2e2t cos 3t −3e2t sin 3t + 2e2t cos 3t
h i
+ 2e2t sin 3t 3e2t cos 3t + 2e2t sin 3t

w.E = 4e4t − 6e4t cos 3t sin 3t + 4e4t cos2 3t + 6e4t sin 3t cos 3t + 4e4t sin2 3t

= 4e4t + 4e4t (cos2 3t + sin2 3t) = 4e4t + 4e4t = 8e4t .

Example 2.51. Find


dy
asy 2 2 2
if x and y are connected by the relation x 3 + y 3 = a 3 .

En
dx 2 2 2
Solution. Let f (x, y) = x 3 + y 3 − a 3

gin ∂f
fx =
∂x 3
2 1
= x− 3 .

Now
fy =
dy
dx
ee
∂f
∂y 3
fx
=− =−
fy
2 1
= y− 3 .
 x − 13
y
=−
 y  31
x
.
rin
Example 2.52. Find
dy
if f (x, y) = log(x2 + y2 ) + tan−1
y
. g.n
et
dx x
Solution.

∂f 2x 1  y 2x y 2x − y
fx = = 2 + − = 2 − 2 = 2 .
∂x x + y2 y2 x 2 x +y 2 x +y 2 x + y2
1+
x2
∂f 2y 1 1 2y x 2y + x
fy = = 2 + = + 2 = 2 .
∂y x + y2 y2 x x2 +y 2 x +y 2 x + y2
1+
x2
dy fx 2x − y y − 2x
Now =− =− = .
dx fy 2y + x x + 2y

Download From : www.EasyEngineering.net


Download From : www.EasyEngineering.net

Function of Several Variables 103

2.5 Jacobian and its Properties

Definition. If u and v are continuous functions of two independent variables x


and y having first order partial derivatives, then the Jacobian determinant or
the Jacobian of u and v is defined by

∂(u, v)  u, v  ∂u ∂u
or J or J = ∂x ∂y
.
∂(x, y) x, y ∂v ∂v
∂x ∂y

wwIf u, v, w are continuous functions of three independent variables x, y, z having first


order partial derivatives, then the Jacobian of u, v, w w.r.t. x, y, z is defined as

w.E
∂u
∂(u, v, w) ∂v
∂x
∂u
∂y
∂v
∂u
∂z

∂v .

asy
=
∂(x, y, z) ∂x ∂y ∂z

∂w ∂w ∂w
∂x ∂y ∂z

Properties of Jacobians
En
gin
Property I. If u and v are functions of r and s where r and s are functions of x, y
then

ee
∂(u, v) ∂(u, v) ∂(r, s)
=
∂(x, y) ∂(r, s) ∂(x, y)
.

rin
g.n
Proof. Since u and v are functions of x and y, we have

∂u ∂u ∂r ∂u ∂s
ux =

uy =
= +
∂x ∂r ∂x ∂s ∂x

∂u ∂u ∂r ∂u ∂s
= +
= ur r x + u s s x .

= ur ry + u s sy .
et
∂y ∂r ∂y ∂s ∂y

∂v ∂v ∂r ∂v ∂s
vx = = + = vr r x + v s s x .
∂x ∂r ∂x ∂s ∂x

∂v ∂v ∂r ∂v ∂s
vy = = + = vr ry + v s sy .
∂y ∂r ∂y ∂s ∂y

Download From : www.EasyEngineering.net


Download From : www.EasyEngineering.net

104 Engineering Mathematics - I


∂(u, v) ∂(r, s) ur u s r x ry
Now =
∂(r, s) ∂(x, y) vr v s s x

sy

ur u s r x s x
=
v v r s
r s y y

ur r x + u s s x ur ry + u s sy
=
v r + v s v r + v s
r x s x r y s y

u x uy
=

ww

v v
x y
∂(u, v)
= .

w.E ∂(x, y)

Property II. If J1 is the Jacobian of u, v with respect to x, y and J2 is the Jacobian

asy
of x, y w.r.t. u, v then J1 J2 = 1. i.e.,
∂(u, v) ∂(x, y)

∂(x, y) ∂(u, v)
= 1.

En
Proof. u is a function of x and y.
Differentiating partially with respect to u and v we get

gin
∂u ∂x ∂u ∂y
1=

ee +
∂x ∂u ∂y ∂u
= u x xu + uy yu .

rin
0=
∂u ∂x ∂u ∂y
+
∂x ∂v ∂y ∂v
= u x xv + uy yv .
g.n
v is a function of x and y.
Differentiating with respect to u and v partially we get
et
∂v ∂x ∂v ∂y
0= + = v x xu + vy yv .
∂x ∂u ∂y ∂v

∂v ∂x ∂v ∂y
1= + = v x xv + vy yv .
∂x ∂v ∂y ∂v

Download From : www.EasyEngineering.net


Download From : www.EasyEngineering.net

Function of Several Variables 105


∂(u, v) ∂(x, y) u x uy xu xv
Now =
∂(x, y) ∂(u, v) v x vy yu

yv

u x uy xu yu
=
v v x y
x y v v

u x xu + uy yv u x xv + uy yv
=
v x + v y v x + v y
x u y u x v y v

1 0
=

ww

0 1

J1 J2 = 1.

w.E
Property III. If the functions u, v, w of three independent variables x, y, z are not

asy
independent, then the Jacobian of u, v, w with respect to x, y, z vanishes.
Solution. Given: u, v and w are not independent variables.

En
=⇒ There exists a relation F(u, v, w) = 0.

gin
Differentiating this w.r.t x, y and z we get

∂F ∂u ∂F ∂v ∂F ∂w

ee
+ + =0
∂u ∂x ∂v ∂x ∂w ∂x

∂F ∂u ∂F ∂v ∂F ∂w
+ +
∂u ∂y ∂v ∂y ∂w ∂y
=0
rin
∂F ∂u ∂F ∂v ∂F ∂w
+ + = 0. g.n
et
∂u ∂z ∂v ∂z ∂w ∂z
∂F ∂F ∂F
Eliminating , and from the above three equations we get
∂u ∂v ∂w

∂u ∂v ∂w
∂x ∂x ∂x
∂u ∂v ∂w = 0.
∂y ∂y ∂y
∂u ∂v ∂w
∂z ∂z ∂z

∂(u, v, w)
=⇒ = 0.
∂(x, y, z)

Download From : www.EasyEngineering.net


Download From : www.EasyEngineering.net

106 Engineering Mathematics - I

Functional relationship. Let u1 , u2 , . . . , un be functions of x1 , x2 , x3 , . . . , xn . Then,


a necessary and sufficient condition that u1 , u2 , . . . , un are not independent is that
∂(u1 , u2 , . . . , un )
the Jacobian = 0.
∂(x1 , x2 , . . . , xn ) ✎ ☞
Worked Examples
✍ ✌

Example 2.53. If x = r cos θ, y = r sin θ, find the Jacobian of x and y w.r.t r and θ.
[Jan 2001]
Solution.

ww x = r cos θ =⇒
∂x
∂r
∂u
= cos θ,
∂x
∂θ
∂u
= −r sin θ.

w.E
y = r sin θ =⇒ = sin θ, = r cos θ.
∂r ∂θ
∂(x, y) ∂x
∂r
∂x
∂θ
cos θ −r sin θ
= r cos2 θ + r sin2 θ = r.
Now, = =

asy
∂(r, θ) ∂y ∂y sin θ r cos θ
∂r ∂θ

En
Example 2.54. The transformation equations in spherical polar coordinates is
x = r sin θ cos ϕ, y = r sin θ sin ϕ, z = r cos θ. Compute the Jacobian of x, y, z w.r.t. r, θ, ϕ.

Solution. gin [May 2011, Jan 2009]


∂x
∂(x, y, z) ∂y
= ∂r
∂r
∂x
∂θ
∂y

∂x
∂ϕ
∂y
ee

= sin θ sin ϕ r sin ϕ cos θ r sin θ cos ϕ

rin
sin θ cos ϕ r cos θ cos ϕ −r sin θ sin ϕ

g.n
∂(r, θ, ϕ) ∂θ ∂ϕ
∂z ∂z ∂z
cos θ
∂r ∂θ ∂ϕ −r sin θ 0

et

sin θ cos ϕ cos θ cos ϕ − sin ϕ

= r2 sin θ sin θ sin ϕ sin ϕ cos θ cos ϕ

cos θ − sin θ 0

= r2 sin θ sin θ cos ϕ(0 + sin θ cos ϕ) − cos θ cos ϕ(0 − cos θ cos ϕ)

− sin ϕ(− sin2 θ sin ϕ − cos2 θ sin ϕ)




= r2 sin θ sin2 θ cos2 ϕ + cos2 θ cos2 ϕ + sin2 θ sin2 ϕ + cos2 θ sin2 ϕ




= r2 sin θ(cos2 ϕ + sin2 ϕ) = r2 sin θ.

Download From : www.EasyEngineering.net


Download From : www.EasyEngineering.net

Function of Several Variables 107

Example 2.55. In cylindrical polar coordinates, x = ρ cos φ, y = ρ sin φ, z = z. Show


∂(x, y, z)
that = ρ.
∂(ρ, φ, z)
Solution.

∂x ∂x ∂x cos φ −ρ sin φ 0
∂ρ ∂φ ∂z
∂(x, y, z) ∂y ∂y ∂y
= ∂ρ ∂φ ∂z = sin φ ρ cos φ 0
∂(ρ, φ, z)
∂z ∂z ∂z 0 0 1
∂ρ ∂φ ∂z

= 1(ρ cos2 φ + ρ sin2 φ) = ρ.

wwExample 2.56. If u = 2xy, v = x2 − y2 , x = r cos θ, y = r sin θ, evaluate


∂(u, v)
∂(r, θ)
without

w.E
actual substitution.
Solution. Given that u and v are functions of x and y. x and y are functions of r
[Jan 2009]

asy
and θ.
∴ By property (1) of the Jacobians we have

En
∂(u, v) ∂(u, v) ∂(x, y)
=
∂(r, θ) ∂(x, y) ∂(r, θ)
.

gin

∂u

∂u
∂(u, v) ∂x ∂y
=

=
2y 2x


ee
∂(x, y) ∂v ∂v 2x −2y

∂x ∂y

= −4y2 − 4x2 = −4(x2 + y2 ) = −4r2 .



∂x

∂x

∂(x, y) ∂r ∂θ cos θ −r sin θ rin
g.n

= =
∂(r, θ) ∂y ∂y sin θ r cos θ

∂r ∂θ

∂(u, v)
∂(r, θ)
= r cos2 θ + r sin2 θ = r

= −4(r2 )r = −4r3 .
et
u+v ∂(u, v)
Example 2.57. If x = uv, y = , find .
u−v ∂(x, y)
∂(u, v) ∂(x, y) ∂(u, v) 1
Solution. We know that =1⇒ = .
∂(x, y) ∂(u, v) ∂(x, y) ∂(x, y)
∂(u, v)
x = uv.

Download From : www.EasyEngineering.net


Download From : www.EasyEngineering.net

108 Engineering Mathematics - I

∂x ∂x
= v, = u.
∂u ∂v
u+v
y= .
u−v
∂y u − v − (u + v) u − v − u − v −2v
= 2
= 2
= .
∂u (u − v) (u − v) (u − v)2
∂y (u − v).1 − (u + v)(−1) 2u
= 2
= .
∂v (u − v) (u − v)2

ww

∂x ∂x v u
∂(x, y) ∂u ∂v
= = −2v 2u

∂(u, v) ∂y ∂y
∂u ∂v (u − v)2
(u − v)2

w.E =
2uv
(u − v)
∂(u, v) (u − v)2
2
+
2uv
(u − v) 2
=
4uv
(u − v)2

asy
∂(x, y)
=
4uv
.

En
Example 2.58. If x = u(1 − v), y = uv then compute J1 and J2 and prove that
J1 J2 = 1.
Solution. We have
gin ∂(x, y) ∂(u, v)

ee
J1 = , J2 =
∂(u, v) ∂(x, y)


∂x
∂x

x = u − uv, y = uv.
rin
g.n
1 − v −u
J1 = ∂u
∂v
= = u − uv + uv = u.
∂y
∂y v u
∂u ∂v
We shall express u and v interms of x and y.

x = u − uv = u − y ⇒ x + y = u.
et
y y
y = uv ⇒ v = = .
u x+y
1
we get J2 = .
u
1
Now J1 J2 = u = 1.
u

Download From : www.EasyEngineering.net


Download From : www.EasyEngineering.net

Function of Several Variables 109

x2 x3 x3 x1 x1 x2 ∂(y1 , y2 , y3 )
Example 2.59. If y1 = , y2 = , y3 = , prove that = 4.
x1 x2 x3 ∂(x1 , x2 , x3 )
[Jan 2014]
∂y1 ∂y1 ∂y1
∂x1 ∂x2 ∂x3
∂(y1 , y2 , y3 ) ∂y2 ∂y2 ∂y2
Solution. = ∂x ∂x ∂x
∂(x1 , x2 , x3 ) 1 2 3
∂y3 ∂y3 ∂y3
∂x1 ∂x2
∂x3
2 −x2 x3 x3 x2
x1 x1 x1

= xx32 −x3 x1 x1

x22 x2


x2 x1 −x1 x2

ww =
x3
 2
x3

−x2 x3  x1 x2 x3
 2 2 −
2
x3

x12  x3  x1 x2 x3 x1 x2 



 −

− −

 + x2

+ 2

 x1 x3 x1 x2 x3 

w.E
 
x12 x2 x3 x2 x3  x1  x2 x32 x2 x3  x1 x2 x3 x2 x3
= −1 + 1 + 1 + 1 + 1 + 1 = 4.

asy
Example 2.60. If u = x + y + z, uv = y + z, uvw = z, find
∂(x, y, z)
∂(u, v, w)
.

En
Solution. Given u = x + y + z ⇒ u = x + uv ⇒ x = u − uv = u(1 − v).
[Jan 2012, Jan 2010]

gin
uv = y + z ⇒ uv = y + uvw ⇒ y = uv − uvw = uv(1 − w), uvw = z.

∂x
∂(x, y, z) ∂y
= ∂u
∂(u, v, w)
∂u
ee∂x
∂v
∂y
∂v

∂x
∂w
∂y

1 − v

−u 0
= v(1 − w) u(1 − w) −uv
∂w



rin
g.n
∂z ∂z ∂z
vw
∂u ∂v ∂w uw uv

1 − v −1 0

et

= u(uv) v(1 − w) 1 − w −1

vw w 1

= u2 v {(1 − v)(1 − w + w) + 1(v(1 − w) + vw)}

= u2 v {(1 − v) + (v − vw + vw)}

= u2 v {1 − v + v} = u2 v.

∂(x, y, z)
Example 2.61. If u = xyz, v = x2 + y2 + z2 , w = x + y + z, find .
∂(u, v, w)

Download From : www.EasyEngineering.net


Download From : www.EasyEngineering.net

110 Engineering Mathematics - I

∂(x, y, z) ∂(u, v, w) ∂(x, y, z) 1


Solution. We have =1⇒ = .
∂(u, v, w) ∂(x, y, z) ∂(u, v, w) ∂(u, v, w)
∂(x, y, z)

∂u ∂u ∂u yz xz xy
∂x ∂y ∂z
∂(u, v, w) ∂v ∂v

∂v =

= ∂x ∂y ∂z 2x 2y 2z
∂(x, y, z)
∂w ∂w ∂w
1 1 1
∂x ∂y ∂z

= yz(2y − 2z) − xz(2x − 2z) + xy(2x − 2y)

= 2(yz(y − z) − xz(x − z) + xy(x − y))

ww = 2(y2 z − yz2 − x2 z + xz2 + xy(x − y))

w.E = 2(z2 (x − y) + xy(x − y) − z(x2 − y2 ))

= 2(z2 (x − y) + xy(x − y) − z(x − y)(x + y))

asy
= 2(x − y)(z2 + xy − xz − yz)

= 2(x − y)(z(z − x) − y(z − x))

En
= 2(x − y)(z − x)(z − y)

gin
= −2(x − y)(y − z)(z − x).

Now, ⇒
∂(x, y, z)
∂(u, v, w)
=
1
∂(u, v, w)
∂(x, y, z)
=
ee 1
−2(x − y)(y − z)(z − x)
.

rin
g.n
Example 2.62. If u = x + 2y + z, v = x − 2y + 3z and w = 2xy − xz + 4yz − 2z2 , show that
they are not independent. Find the relation between u, v and w.
Solution. Given: u = x + 2y + z.

∂u
= 1,
∂u
= 2,
∂u
= 1.
et
∂x ∂y ∂z

v = x − 2y + 3z.
∂v ∂v ∂v
= 1, = −2, = 3.
∂x ∂y ∂z

w = 2xy − xz + 4yz − 2z2 .

Download From : www.EasyEngineering.net


Download From : www.EasyEngineering.net

Function of Several Variables 111

∂w ∂w ∂w
= 2y − z, = 2x + 4z, = −x + 4y − 4z.
∂x ∂y ∂z


∂u ∂u ∂u 1 2 1
∂x ∂y ∂z
∂(u, v, w) ∂v ∂v ∂v

= ∂x ∂y ∂z
= 1 −2 3
∂(x, y, z)
∂w ∂w ∂w 2y − z 2x + 4z −x + 4y − 4z
∂x ∂y ∂z

= 1(−2(−x + 4y − 4z) − 3(2x + 4z)) − 2(−x + 4y − 4z − 3(2y − z))

ww + 1(2x + 4z + 2(2y − z))

= 2x − 8y + 8z − 6x − 12z + 2x − 8y + 8z + 12y − 6z + 2x + 4z + 4y − 2z = 0.

w.E
asy
Hence, u, v, w are not independent.
Now u + v = 2x + 4z, u − v = 4y − 2z.

En
(u + v)(u − v) = 2(x + 2z).2(2y − z)

gin u2 − v2 = 4(2xy − xz + 4yz − 2z2 )

ee
u2 − v2 = 4w.

rin
2.6 Taylor’s expansion for functions of two variables g.n
is
We know that for a function f (x) of one single variable x, the Taylor’s expansion
et
h2 ′′ h3 ′′′
f (x + h) = f (x) + h f ′ (x) + f (x) + f (x) + · · ·
2! 3!

Now let f (x, y) be a function of two independent variables x, y defined in a region


R of the xy−plane and let (a, b) be a point in R. Suppose f (x, y) has all its partial

Download From : www.EasyEngineering.net


Download From : www.EasyEngineering.net

112 Engineering Mathematics - I

derivatives in a neighbourhood of (a, b) then

 ∂ ∂
f (a + h, b + k) = f (a, b) + h + k f (a, b)
∂x ∂y
1 ∂ ∂ 2 1 ∂ ∂ 3
+ h +k f (a, b) + h +k f (a, b) + · · · +
2! ∂x ∂y 3! ∂x ∂y
 
= f (a, b) + h f x (a, b) + k fy (a, b)
1 2 
+ h f xx (a, b) + 2hk f xy (a, b) + k2 fyy (a, b)
2!
1 3
h f xxx (a, b) + 3h2 k f xxy (a, b) + 3hk2 f xyy (a, b)

ww
+
3!

+ k3 fyyy (a, b) + · · · .

w.E
Put x = a + h, y = b + k then h = x − a, k = y − b.

asy
∴ The Taylor’s series can be written as

 
f (x, y) = f (a, b) + (x − a) f x (a, b) + (y − b) fy (a, b)

+
1
En
(x − a)2 f xx (a, b) + 2(x − a)(y − b) f xy (a, b) + (y − b)2 fyy (a, b)


+
2!
1
3! gin
(x − a)3 f xxx (a, b) + 3(x − a)2 (y − b) f xxy (a, b)

ee 
+ 3(x − a)(y − b)2 f xyy (a, b) + (y − b)3 fyyy (a, b) + · · · .

This is known as the Taylor’s expansion of f (x, y) in the neighbourhood of (a, b) or rin
about the point (a, b).
g.n
Put a = 0, b = 0. we get


f (x, y) = f (0, 0) + x f x (0, 0) + y fy (0, 0)
 et
1 2 
+ x f xx (0, 0) + 2xy f xy (0, 0) + y2 fyy (0, 0)
2!
1 3 
+ x f xxx (0, 0) + 3x2 y f xxy (0, 0) + 3xy2 f xyy (0, 0) + y3 fyyy (0, 0) + · · · .
3!

This is called Maclaurin’s series for f (x, y) in powers of x and y.


✎ ☞
Worked Examples
✍ ✌

Download From : www.EasyEngineering.net


Download From : www.EasyEngineering.net

Function of Several Variables 113

Example 2.63. Expand e x sin y in powers of x and y as far as the terms of third
degree. [Jun 2013]
Solution. f (x, y) = e x sin y f (0, 0) = 0

f x (x, y) = e x sin y f x (0, 0) = 0

fy (x, y) = e x cos y fy (0, 0) = 1

f xx (x, y) = e x sin y f xx (0, 0) = 0

ww f xy (x, y) = e x cos y

fyy (x, y) = −e x sin y


f xy (0, 0) = 1

fyy (0, 0) = 0

w.E f xxx (x, y) = e x sin y f xxx (0, 0) = 0

asy
f xxy (x, y) = e x cos y

f xyy (x, y) = −e x sin y


f xxy (0, 0) = 1

f xyy (0, 0) = 0

fyyy (x, y) = −e x cos y En fyyy (0, 0) = −1.

gin
ee rin
Now f (x, y) = f (0, 0) + x f x (0, 0) + y fy (0, 0) +
1 2
2! g.n
x f xx (0, 0) + 2xy f xy (0, 0) + y2 fyy (0, 0)


1 3

et

+ x f xxx (0, 0) + 3x2 y f xxy (0, 0) + 3xy2 f xyy (0, 0) + y3 fyyy (0, 0) + · · ·
3!
1 
= 0 + x.0 + y.1 + x2 .0 + 2xy.1 + y2 .0
2
1 3 
x .0 + 3x y.1 + 3xy2 .0 + y3 (−1) + · · ·
2
6
x2 y y3
= y + xy + − + ··· .
2 6

Example 2.64. Expand e x loge (1 + y) in powers of x and y upto terms of third


degree. [Jan 2014, Dec 2011, Jan 2003]
Solution.

Download From : www.EasyEngineering.net


Download From : www.EasyEngineering.net

114 Engineering Mathematics - I

2e x
fyyy =
(1 + y)3

f (x, y) = e x loge (1 + y)
f (0, 0) = 0
f x (x, y) = e x loge (1 + y)
ex f x (0, 0) = 0
fy (x, y) =
1+y fy (0, 0) = 1

ww
x
f xx = e log(1 + y)
f xx (0, 0) = 0
ex
f xy = f xy (0, 0) = 1

w.E
1+y
−e x fyy (0, 0) = −1
fyy =
(1 + y)2

asy
f xxx (0, 0) = 0
f xxx = e x log(1 + y)
ex f xxy (0, 0) = 1

En
f xxy =
1+y f xyy (0, 0) = −1
−e x

gin
f xyy = fyyy (0, 0) = 2
(1 + y)2
By Maclaurin’s series we have,

+
2!
ee
f (x, y) = f (0, 0) + x f x (0, 0) + y fy (0, 0)
1 2
rin
x f xx (0, 0) + 2xy f xy (0, 0) + y2 fyy (0, 0)


+
1 3
3!
x f xxx (0, 0) + 3x2 y f xxy (0, 0)
g.n
et

+ 3xy2 f xyy (0, 0) + y3 fyyy (0, 0) + · · · .
1
e x loge (1 + y) = 0 + x.0 + y.1 + (x2 .0 + 2xy.1 + y2 (−1))+
2
1 3
[x · 0 + 3x2 y · 1 + 3xy2 (−1) + y3 (2)] + . . . .
6
y2 x2 y xy2 y3
= y + xy − + − + − ···
2 2 2 3

Example 2.65. Expand x2 y + 3y − 2 in powers of x − 1 and y + 2 upto third degree


terms. [Jun 2012]

Download From : www.EasyEngineering.net


Download From : www.EasyEngineering.net

Function of Several Variables 115

Solution. f (x, y) = x2 y + 3y − 2. f (1, −2) = 1 × (−2) + 3(−2) − 2

= −2 − 6 − 2 = −10.
f x (x, y) = 2xy.
f x (1, −2) = −4.
2
fy (x, y) = x + 3.
fy (1, −2) = 4.
f xx (x, y) = 2y.
f xx (1, −2) = −4.
f xy (x, y) = 2x.
f xy (1, −2) = 2.

ww fyy (x, y) = 0.

f xxx (x, y) = 0.
fyy (1, −2) = 0.

w.E
f xxx (1, −2) = 0.
f xxy (x, y) = 2.
f xxy (1, −2) = 2.
f xyy (x, y) = 0.

fyyy (x, y) = 0.asy f xyy (1, −2) = 0.

fyyy (1, −2) = 0.

En
gin
By Taylor’s theorem we have,
ee
f (x, y) = f (a, b) + (x − a) f x (a, b) + (y − b) fy (a, b) rin
+
1 h
2! g.n
(x − a)2 f xx (a, b) + 2(x − a)(y − b) f xy (a, b) + (y − b)2 fyy (a, b)
i

+
1 h
3!
(x − a)3 f xxx (a, b) + 3(x − a)2 (y − b) f xxy (a, b)

+3(x − a)(y − b)2 f xyy (a, b) + (y − b)3 fyyy (a, b) + · · ·


i et
x2 y + 3y − 2 = f (1, −2) + (x − 1) f x (1, −2) + (y + 2) fy (1, −2)
1 h i
+ (x − 1)2 f xx (1, −2) + 2(x − 1)(y + 2) f xy (1, −2) + (y + 2)2 fyy (1, −2)
2!
1 h
+ (x − 1)3 f xxx (1, −2) + 3(x − 1)2 (y + 2) f xxy (1, −2)
3!
i
+3(x − 1)(y + 2)2 f xyy (1, −2) + (y + 2)3 fyyy (1, −2) + · · ·

Download From : www.EasyEngineering.net


Download From : www.EasyEngineering.net

116 Engineering Mathematics - I

1h i
= −10 − 4(x − 1) + 4(y + 2) + −4(x − 1)2 + 2(x − 1)(y + 2)
2
1h i
+ 6(x − 1)2 (y + 2) + · · ·
6
= −10 − 4(x − 1) + 4(y + 2) − 2(x − 1)2 + (x − 1)(y + 2) + (x − 1)2 (y + 2) + · · ·

Example 2.66. Find the Taylor’s series expansion of x2 y2 + 2x2 y + 3y2 in powers of
(x + 2) and y − 1 upto third degree terms. [Jan 2012, Jun 2010, Jan 2010]
Solution.

ww f (x, y) = x2 y2 + 2x2 y + 3y2 .

f x (x, y) = 2xy2 + 4xy.


f (−2, 1) = 4 + 8 + 3 = 15.

f x (−2, 1) = −4 − 8 = −12.

w.E fy (x, y) = 2x2 y + 2x2 + 6y. fy (−2, 1) = 8 + 8 + 6 = 22.

asy
f xx (x, y) = 2y2 + 4y.

f xy (x, y) = 4xy + 4x.


f xx (−2, 1) = 2 + 4 = 6.

f xy (−2, 1) = −8 − 8 = −16.

fyy (x, y) = 2x2 + 6.


En fyy (−2, 1) = 8 + 6 = 14.

f xxx (x, y) = 0.

f xxy (x, y) = 4y + 4. gin f xxx (−2, 1) = 0.

f xxy (−2, 1) = 4 + 4 = 8.

f xyy (x, y) = 4x.

fyyy (x, y) = 0.
ee f xyy (−2, 1) = −8.

fyyy (−2, 1) = 0.
rin
g.n
By Taylor’s theorem we have,
et
f (x, y) = f (a, b) + (x − a) f x (a, b) + (y − b) fy (a, b)
1 h i
+ (x − a)2 f xx (a, b) + 2(x − a)(y − b) f xy (a, b) + (y − b)2 fyy (a, b)
2!
1 h
+ (x − a)3 f xxx (a, b) + 3(x − a)2 (y − b) f xxy (a, b)
3!
i
+3(x − a)(y − b)2 f xyy (a, b) + (y − b)3 fyyy (a, b) + · · ·

Download From : www.EasyEngineering.net


Download From : www.EasyEngineering.net

Function of Several Variables 117

x2 y2 + 2x2 y + 3y2 = 15 − 12(x + 2) + 22(y − 1)


1h i
+ 6(x + 2)2 − 2 × 16(x + 2)(y − 1) + 14(y − 1)2
2
1h i
+ 24(x + 2)2 (y − 1) − 24(x + 2)(y − 1)2 + · · ·
6
= 15 − 12(x + 2) + 22(y − 1) + 3(x + 2)2 − 16(x + 2)(y − 1) + 7(y − 1)2

+ 4(x + 2)2 (y − 1) − 4(x + 2)(y − 1)2 + · · ·

Example 2.67. Use Taylor’s formula to expand the function defined by f (x, y) =

ww x3 + y3 + xy2 in powers of (x − 1) and (y − 2).


Solution.
[May 2011]

w.E f (x, y) = x3 + y3 + xy2 . f (1, 2) = 1 + 8 + 4 = 13.

asy
f x (x, y) = 3x2 + y2 .

fy (x, y) = 3y2 + 2xy.


f x (1, 2) = 3 + 4 = 7.

fy (1, 2) = 12 + 4 = 16.

f xx (x, y) = 6x.
En f xx (1, 2) = 6.

f xy (x, y) = 2y.
gin f xy (1, 2) = 4.

ee
fyy (x, y) = 6y + 2x. fyy (1, 2) = 12 + 2 = 14.

rin
f xxx (x, y) = 6. f xxx (1, 2) = 6.

f xxy (x, y) = 0. f xxy (1, 2) = 0.

f xyy (x, y) = 2. f xyy (1, 2) = 2.


g.n
fyyy (x, y) = 6. fyyy (1, 2) = 6.
et
By Taylor’s theorem we have,

f (x, y) = f (a, b) + (x − a) f x (a, b) + (y − b) fy (a, b)


1 h i
+ (x − a)2 f xx (a, b) + 2(x − a)(y − b) f xy (a, b) + (y − b)2 fyy (a, b)
2!

Download From : www.EasyEngineering.net


Download From : www.EasyEngineering.net

118 Engineering Mathematics - I

1 h
+ (x − a)3 f xxx (a, b) + 3(x − a)2 (y − b) f xxy (a, b)
3!
i
+3(x − a)(y − b)2 f xyy (a, b) + (y − b)3 fyyy (a, b) + · · ·
1h i
x3 + y3 + xy2 = 13 + 7(x − 1) + 16(y − 2) + 6(x − 1)2 + 8(x − 1)(y − 2) + 14(y − 2)2
2
1h i
+ 6(x − 1)3 + 6(x − 1)(y − 2)2 + 6(y − 2)3 + · · ·
6
= 13 + 7(x − 1) + 16(y − 2) + 3(x − 1)2 + 4(x − 1)(y − 2) + 7(y − 2)2

+ (x − 1)3 + (x − 1)(y − 2)2 + (y − 2)3 + · · ·

wwExample 2.68. Expand e−x log y as a Taylor’s series in powers of x and y − 1 upto

w.E
third degree terms. [Jun 2011]
Solution.

f (x, y) = e−x log y.


asy f (0, 1) = 0.

f x (x, y) = −e−x log y.


e−x En f x (0, 1) = 0.

fy (x, y) =
y
.

f xx (x, y) = e−x log y. gin


fy (0, 1) = 1.

f xx (0, 1) = 0.

f xy (x, y) = −
e−x
y
. ee
f xy (0, 1) = −1.

rin
fyy (x, y) = −
e−x
y2
. fyy (0, 1) = −1.
g.n
f xxx (x, y) = −e−x log y.

f xxy (x, y) =
e−x
y
.
f xxx (0, 1) = 0.

f xxy (0, 1) = 1. et
e−x
f xyy (x, y) = . f xyy (0, 1) = 1.
y2
2e−x
fyyy (x, y) = . fyyy (0, 1) = 2.
y3

Download From : www.EasyEngineering.net


Download From : www.EasyEngineering.net

Function of Several Variables 119

By Taylor’s theorem we have,

f (x, y) = f (a, b) + (x − a) f x (a, b) + (y − b) fy (a, b)


1 h i
+ (x − a)2 f xx (a, b) + 2(x − a)(y − b) f xy (a, b) + (y − b)2 fyy (a, b)
2!
1 h
+ (x − a)3 f xxx (a, b) + 3(x − a)2 (y − b) f xxy (a, b)
3!
i
+3(x − a)(y − b)2 f xyy (a, b) + (y − b)3 fyyy (a, b) + · · ·
1 1
e−x log y = y − 1 + [−2x(y − 1) − (y − 1)2 ] + [3x2 (y − 1) + 3x(y − 1)2 + 2(y − 1)3 ] + · · ·
2! 3!

wwExample 2.69. Expand f (x, y) = tan−1


Hence compute f (1.1, 0.9) approximately.
y
x
about (1, 1) upto the second degree terms.
[Jan 2005]

w.E
Solution. Given, f (x, y) = tan−1
(a, b) = (1, 1)
y
x
a = 1, b = 1
f (1, 1) = tan−1
asy
1 π
1
=
4

fx =
1  −y 
y
1 + x2
2
x 2
= −
En
(x
x2 y
2 + y2 )x2
= 2
x
−y
+ y2
f x (1, 1) =
−1
2

fy =
1 1
2
= 2 gin
x2
2 )x
= 2
x
2
fy (1, 1) =
1

ee
y x (x + y x + y 2
1 + x2
2xy 2 1
f xx = −y(−1)(x2 + y2 )−2 2x = 2
2 2 2
(x + y )
2
2 2

rin
f xx (1, 1) = =
4 2

g.n
x + y − x2x y −x
f xy = 2 2 2
= 2 f xy (1, 1) = 0
(x + y ) (x + y2 )2
−2xy −1
fyy = x(−1)(x2 + y2 )−2 2y = 2
(x + y ) 2 2
fyy (1, 1) =
2
et
By Taylor’s theorem we have

f (x, y) = f (a, b) + (x − a) f x (a, b) + (y − b) fy (a, b)


1 
+ (x − a)2 f xx (a, b) + 2(x − a)(y − b) f xy (a, b) + (y − b)2 fyy (a, b) + · · ·
2!

Download From : www.EasyEngineering.net


Download From : www.EasyEngineering.net

120 Engineering Mathematics - I

y
tan−1 = f (1, 1) + (x − 1) f x (1, 1) + (y − 1) fy (1, 1)
x
1 
+ (x − 1)2 f xx (1, 1) + 2(x − 1)(y − 1) f xy (1, 1) + (y − 1)2 fyy (1, 1)
2!
π  −1  1 1 1
= + (x − 1) + (y − 1) + (x − 1)2 + 2(x − 1)(y − 1)0
4 2 2 2 2
2 (−1)

+ (y − 1) + ···
2
π 1 11 1 
= − (x − 1 − y + 1) + (x − 1)2 − (y − 1)2 + · · ·
4 2 2 2 2
π 1 1 2 2
= − (x − y) + (x − y − 2x + 2y) + · · ·

ww tan−1
x
4 2
y π 1
4
1 1 1
= − (x − 1) + (y − 1) + (x − 1)2 − (y − 1)2 + · · · .
4 2 2 4 4

w.E π 1 1 1 1
f (1.1, 0.9) = − (0.1) + (−0.1) + (0.1)2 − (−0.1)2 approximately
π
4 2 2 4 4

asy
= − 0.1 = 0.685 approximately.
4
 π
Example 2.70. Find the Taylor’s series expansion of e x sin y at the point − 1,
upto third degree terms.
En 4
[Jan 2009]
Solution.
gin
ee
π 1
f (x, y) = e x sin y f − 1, = √
4 e 2
f x (x, y) = e x sin y f x − 1,
π
4
= √
1
e 2 rin
fy (x, y) = e x cos y fy − 1,
π
4
= √
1
e 2 g.n
f xx = e x sin y

f xy = e x cos y
f xx − 1,

f xy − 1,
π
4
π
= √

= √
1
e 2
1
et
4 e 2
π 1
fyx = e x cos y fyx − 1, = √
4 e 2
π −1
fyy = −e x sin y fyy − 1, = √
4 e 2
π 1
f xxx = e x sin y f xxx − 1, = √
4 e 2

Download From : www.EasyEngineering.net


Download From : www.EasyEngineering.net

Function of Several Variables 121

π 1
f xxy = e x cos y f xxy − 1, = √
4 e 2
π −1
f xyy = −e x sin y f xyy − 1, = √
4 e 2
π −1
fyyx = −e x sin y fyyx − 1, = √
4 e 2
π −1
fyyy = −e x sin y

fyyy − 1, = √
4 e 2

ww
w.E
By Taylor’s theorem we have

asy
f (x, y) = f (a, b) + (x − a) f x (a, b) + (y − b) fy (a, b)
1

En

+ (x − a)2 f xx (a, b) + 2(x − a)(y − b) f xy (a, b) + (y − b)2 fyy (a, b)
2!
1
+
3!
gin
(x − a)3 f xxx (a, b) + 3(x − a)2 (y − b) f xxy (a, b)

+ 3(x − a)(y − b)2 f xyy (a, b) + (y − b)3 fyyy (a, b) + · · · .


+
1
π

2
ee π
π
e x sin y = f − 1, + (x + 1) f x − 1, + (y − ) fy − 1,
4 4
π
4
(x + 1) f xx − 1, + 2(x + 1)(y − ) f xy − 1,
π
π
4

rin
π
2!
π 2
+ (y − ) fyy − 1,
4
π 
4
4 4 4

g.n
1
6
3

π 2
π
+ (x + 1) f xxx − 1, + 3(x + 1)2 (y − ) f xxy − 1,
4
π
+ 3(x + 1)(y − ) f xyy − 1, + (y − ) fyyy − 1,
4 4
π 3
4
π
4
π 
4
π
4 et
+ ··· .
1 1 1 π 1
= √ + √ (x + 1) + √ y − + √ (x + 1)2
e 2 e 2 e 2 4 2 2e
1 π 1 π 1
+ √ (x + 1) y − − √ y − 2 + √ (x + 1)3
2e 4 2 2e 4 6 2e
√ √
2 π 2 π 1 π
+ (x + 1)2 y − − (x + 1) y − 2 − √ y − 3 + · · · .
e 4 e 4 6 2e 4

Download From : www.EasyEngineering.net


Download From : www.EasyEngineering.net

122 Engineering Mathematics - I

 π
Example 2.71. Expand e x cos y near the point 1, by Taylor’s series as far as
4
quadratic terms. [Jan 1996]
Solution.

π e
f (x, y) = e x cos y f 1, = √
4 2
π π e
f x (x, y) = e x cos y f x 1, = e cos = √
4 4 2
π e
fy (x, y) = −e x sin y fy 1, =−√
4 2

ww f xx = e x cos y f xx 1,
π
4
= √
π  −e
e
2

w.E f xy = −e x sin y

fyy = −e x cos y
f xy 1,

fyy 1,
4
= √
π  −e
= √
2

asy 4 2

En
By Taylor’s theorem we have gin
+
1
ee
f (x, y) = f (a, b) + (x − a) f x (a, b) + (y − b) fy (a, b)

rin
(x − a)2 f xx (a, b) + 2(x − a)(y − b) f xy (a, b) + (y − b)2 fyy (a, b)


x
2!
π π
e cos y = f 1, + (x − 1) f x 1, + (y − ) fy 1,
4 4
π
4
π
4 g.n
+
1
2!
π 2
+ (y − ) fyy 1,
4
π 
4
π
4
+ ···
π
(x − 1)2 f xx 1, + 2(x − 1)(y − ) f xy 1,

4
π
4 et
e e π  (−e)
= √ + (x − 1) √ + y − √
2 2 4 2
1 e π  (−e) π 2 (−e) 
+ (x − 1)2 √ + 2(x − 1) y − √ + y− √ + ···
2 2 4 2 4 2
!
e π 1 π 1 π
= √ 1 + (x − 1) − y − + (x − 1)2 − y − (x − 1) − y − 2 + · · · .
2 4 2 4 2 4

Download From : www.EasyEngineering.net


Download From : www.EasyEngineering.net

Function of Several Variables 123

2.7 Maxima and Minima for functions of two variables


Definition. Let f (x, y) be a continuous function defined in a closed and bounded
domain D of the xy plane and let (a, b) be an interior point of D.
(i) f (a, b) is said to be a local maximum value of f (x, y) at the point (a, b) if there
exists a neighborhood N of (a, b) such that f (x, y) < f (a, b) for all points (x, y) in
N.
(ii) f (a, b) is said to be a local minimum if f (x, y) > f (a, b) for all points (x, y) in N

ww
other than (a, b).

f (x, y) f (x, y)

w.E f (a, b)
f (a, b)

asy (x, y) (a, b) (x, y)


X
(x, y) (a, b) (x, y)
X

Y En Y

gin
Local maximum or local minimum values are called extreme values.
Stationary point of f (x, y)
ee
A point (a, b) satisfying f x = 0 and fy = 0 is called a stationary point of f (x, y).
Necessary conditions for Maximum or minimum rin
If f (a, b) is an extreme value of f (x, y) at (a, b), then (a, b) is a stationary point of
g.n
f (x, y) if f x and fy exist at (a, b) and f x (a, b) = 0, fy (a, b) = 0.
Sufficient conditions for extreme values of f (x, y)
Let (a, b) be a stationary point of the differentiable function f (x, y).
et
i.e., f x (a, b) = 0, fy = (a, b) = 0.
Let us define f xx (a, b) = r, f xy (a, b) = s, fyy (a, b) = t.
(i) If rt − s2 > 0 and r < 0, then f (a, b) is a maximum value.
(ii) If rt − s2 > 0 and r > 0 then f (a, b) is a minimum value.
(iii) If rt − s2 < 0, then f (a, b) is not an extreme value but (a, b) is a saddle point of

Download From : www.EasyEngineering.net


Download From : www.EasyEngineering.net

124 Engineering Mathematics - I

f (x, y).
(iv) If rt − s2 = 0, then no conclusion is possible and further investigation is
required.
Working rule to find maxima and minima of f (x, y)
∂f ∂f
step (1). Find f x = and fy = and solve for f x = 0 and fy = 0 as simultaneous
∂x ∂y
equations in x and y.
Let (a, b), (a1 , b1 ), . . . be the solutions which are stationary points of f (x, y).
∂2 f ∂2 f ∂2 f
step (2). Find r = 2 , s = ,t = 2 .

ww ∂x ∂x∂y ∂y
step (3). Evaluate r, s, t at each stationary point.
At the point (a, b) if

w.E
(i) rt − s2 > 0 and r < 0 then f (a, b) is a maximum value of f (x, y).
(ii) rt − s2 > 0 and r > 0 then f (a, b) is a minimum value of f (x, y).

asy
(iii) rt − s2 < 0 then (a, b) is called a saddle point.
(iv) rt − s2 = 0, no conclusion can be made, further investigation is required.
Critical Point
En
gin
A point (a, b) is a critical point of f (x, y) if f x = 0 and fy = 0 at (a, b) or f x and fy do
not exist at (a, b).

✎ ee
Maxima or Minima occur at a critical point.
Worked Examples



rin
3 3
Example 2.72. Examine f (x, y) = x + y − 12x − 3y + 20 for its extreme values.
g.n
Solution. Given f (x, y) = x3 + y3 − 12x − 3y + 20.

f x = 3x2 − 12 fy = 3y2 − 3
et
[ Jun 2013, Jan 2012, May 2011, Jun 2010]

r = f xx = 6x s = f xy = 0 t = fyy = 6y

For stationary points, solve f x = 0 and fy = 0.


f x = 0 ⇒ 3x2 − 12 = 0 ⇒ x2 − 4 = 0 ⇒ x2 = 4 ⇒ x = ±2.
fy = 0 ⇒ 3y2 − 3 = 0 ⇒ y2 = 1 ⇒ y = ±1.

Download From : www.EasyEngineering.net


Download From : www.EasyEngineering.net

Function of Several Variables 125

Stationary points are (2, 1), (2, −1), (−2, 1) and (−2, −1).
rt − s2 = 6x6y − 0 = 36xy.
At (2, 1), rt − s2 = 36 × 2 × 1 = 72 > 0.
At (2, 1), r = 6(2) = 12 > 0.
∴ (2, 1) is a minimum point.
Minimum value is f (2, 1) = 8 + 1 − 24 − 3 + 20 = 29 − 27 = 2.
At (2, −1), rt − s2 = 36 × 2 × −1 = −72 < 0.
∴ (2, −1) is a saddle point.

wwAt (−2, 1), rt − s2 = 36 × (−2) × 1 = −72 < 0.


∴ (−2, 1) is a saddle point.

w.E
At (−2, −1), rt − s2 = 36 × −2 × −1 = 72 > 0.
r = 6(−2) = −12 < 0.

asy
∴ (−2, −1) is a maximum point.
Maximum value f (−2, −1) = −8 − 1 + 24 + 3 + 20 = 47 − 9 = 38.

En
Example 2.73. Examine f (x, y) = x3 + y3 − 3axy for maximum and minimum
values.
gin [Jan 1999]
Solution.

ee
Given f (x, y) = x3 + y3 − 3axy.

f x = 3x2 − 3ay
rin
fy = 3y2 − 3ax

r = f xx = 6x g.n
s = f xy = −3a t = fyy = 6y.

For stationary points, solve f x = 0 and fy = 0.


et
x2
f x = 0 ⇒ 3x2 − 3ay = 0 ⇒ ay = x2 ⇒ y = .
a
2 x4
fy = 0 ⇒ 3y − 3ax = 0 ⇒ 2 − ax = 0.
a
 x3 
3 3
x 2 − a = 0 ⇒ x(x − a ) = 0 ⇒ x = 0 or x = a.
a
x=0⇒y=0

Download From : www.EasyEngineering.net


Download From : www.EasyEngineering.net

126 Engineering Mathematics - I

a2
x=a⇒y= a = a.
Stationary points are (0, 0) and (a, a).
At (0, 0), rt − s2 = 6x6y − 9a2 = 36xy − 9a2 = −9a2 < 0.
r = 6x = 0.
∴ No maximum or minimum at (0, 0).
∴ (0, 0) is a saddle point.
At (a, a), rt − s2 = 36a2 − 9a2 = 27a2 > 0 if a , 0.
r = 6x = 6a.

wwIf a < 0, r < 0.


∴ (a, a) is a maximum point if a < 0.

w.E
If a > 0, r > 0.
∴ (a, a) is a minimum point if a > 0.

asy
Maximum value = a3 + a3 − 3a3 = −a3 if a < 0
Minimum value = −a3 if a > 0.

En
Example 2.74. Discuss the maxima and minima of f (x, y) = x3 y2 (1 − x − y).

gin [Jan 2014]

ee
Solution. Given: f (x, y) = x3 y2 (1 − x − y) = x3 y2 − x4 y2 − x3 y3 .

rin
f x = y2 [x3 (−1) + (1 − x − y)3x2 ] = x2 y2 [−x + 3 − 3x − 3y]
2 2
= x y [−4x − 3y + 3].
g.n
fy = 2x3 y − 2x4 y − 3x3 y2 .

r = f xx = y2 [−12x2 − 6xy + 6x].

s = f xy = 6x2 y − 8x3 y − 9x2 y2 .


et
t = fyy = 2x3 − 2x4 − 6x3 y.

For stationary points, solve f x = 0 and fy = 0.


f x = 0 ⇒ x2 y2 [−4x − 3y + 3] = 0.
⇒ x = 0, y = 0, 4x + 3y = 3.

Download From : www.EasyEngineering.net


Download From : www.EasyEngineering.net

Function of Several Variables 127

fy = 0 ⇒ x3 y(2 − 2x − 3y) = 0.
x = 0, y = 0, 2x + 3y = 2.

Solving 4x + 3y = 3 (1)

2x + 3y = 2 (2)

1
we get 2x = 1 ⇒ x = .
2
1 1
When x = , (1) ⇒ 2 + 3y = 3 ⇒ 3y = 1 ⇒ y = .
2 3

ww
1 1
∴ The stationary points are (0, 0), , .
2 3
At (0, 0), rt − s2 = 0.0 − 0 = 0.

w.E
We can not say maximum or minimum. Further investigation is required.
1 1
At , ,
2 3

r=
1
9 asy 1 1 1
− 12 × − 6 × × + 6 ×
4 2 3
1
2
=
1
9 En  1
− 3 − 1 + 3 = (−1) − .
9
1
9
1
t =2 −2 −6
8
1
16 gin
11 1 1 1
= − − =− .
83 4 8 4
1
8

ee
 11 11 1 1 2  1 1 1 2
2
s = 6 −8 −9 = − −
43 83 49 2 3 4
=
 1 1 2  1 2

4 3
= −
12
=
1
144
.
rin
 1  1 
rt − s2 = −
1
9

1
8

1
144
2−1 g.n
1 1
∴ ,
2 3
= −
72 144

is a maximum point.
=
144
> 0 and r < 0.
et
1 1 1 1 1 1 1 6 − 3 − 2 1 1
Maximum value is f , = 1− − = = = .
2 3 89 2 3 72 6 72 × 6 432
Example 2.75. Find the extreme values of the function f (x, y) = x4 + y2 + x2 y.
Solution. Given: f (x, y) = x4 + y2 + x2 y.

f x = 4x3 + 2xy. r = f xx = 12x2 + 2y.

Download From : www.EasyEngineering.net


Download From : www.EasyEngineering.net

128 Engineering Mathematics - I

fy = 2y + x2 . s = f xy = 2x. t = fyy = 2.

For stationary points, solve f x = 0 and fy = 0.


f x = 0 ⇒ 4x3 + 2xy = 0 ⇒ 2x(y + 2x2 ) = 0 ⇒ x = 0, y + 2x2 = 0 ⇒ y = −2x2 .
fy = 0 ⇒ 2y + x2 = 0 ⇒ −4x2 + x2 = 0 ⇒ −3x2 = 0 ⇒ x = 0.
When x = 0, y = 0.
∴ The only stationary point is (0, 0).

ww rt − s2 = (12x2 + 2y)2 − 4x2 .

w.E
At (0, 0), rt − s2 = 0.
We can not say maximum or minimum.

asy
We shall investigate the nature of the function in a neighbourhood of (0, 0).
We have f (0, 0) = 0. In a neighbourhood of (0, 0) on the x−axis, take the point (h, 0),
f (h, 0) = h4 > 0.
En
gin
On the y−axis take the point (0, k), f (0, k) = k2 > 0.
On y = mx, for any m, take the point (h, mh).

ee
f (h, mh) = h4 + m2 h2 + mh3 = h2 [h2 + m2 + mh].
rin
For the quadratic in m, m2 + mh + h2
g.n
discriminant = B2 − 4AC = h2 − 4.1.h2 = −3h2 < 0 if m , 0.
∴ f (h, mh) > 0 for all m , 0.
∴ In a neigbourhood of (0, 0) for all points (x, y), f (x, y) > 0.
et
∴ f (0, 0) is minimum and the minimum value = 0.

Example 2.76. Find the maximum and minimum values of x2 − xy + y2 − 2x + y.


[ Jun 2012, Jun 2010]
Solution. f (x, y) = x2 − xy + y2 − 2x + y.

Download From : www.EasyEngineering.net


Download From : www.EasyEngineering.net

Function of Several Variables 129

f x = 2x − y − 2.

fy = −x + 2y + 1.

r = f xx = 2.

s = f xy = −1.

t = fyy = 2.
For stationary points, solve f x = 0, fy = 0

2x − y − 2 = 0. (1)

ww −x + 2y + 1 = 0. (2)

w.E (1) ⇒ y = 2x − 2.

asy (2) ⇒ −x + 2(2x − 2) + 1 = 0

−x + 4x − 4 + 1 = 0

En 3x − 3 = 0

gin 3x = 3

ee x = 1.

∴ y = 2 − 2 = 0. rin
The stationary point is (1, 0). g.n
rt − s2 = 4 + 1 = 5 > 0 and r = 2 > 0. et
∴ (1, 0) is a minimum point.
Minimum value of f = 1 − 2 = −1.

Example 2.77. Find the extreme values of the function f (x, y) = x3 +y3 −3x−12y+20.
[Jan 2012]
Solution. f (x, y) = x3 + y3 − 3x − 12y + 20.

Download From : www.EasyEngineering.net


Download From : www.EasyEngineering.net

130 Engineering Mathematics - I

f x = 3x2 − 3.

fy = 3y2 − 12.

r = f xx = 6x.

s = f xy = 0.

t = fyy = 6y.
For stationary points, solve f x = 0, fy = 0.

3x2 − 3 = 0. 3y2 − 12 = 0.

ww 3x2 = 3. 3y2 = 12.

w.E x2 = 1. y2 = 4.

asy x = ±1. y = ±2.

En
The stationary points are (1, 2), (−1, 2), (1, −2), (−1, −2).

gin
At (1, 2), rt − s2 = 36xy = 36 × 1 × 2 = 72 > 0.
r = 6 > 0.
∴ (1, 2) is a minimum point.
ee
Minimum value of f = 1 + 8 − 3 − 24 + 20 = 2.
At (−1, 2), rt − s2 = 36xy = 36 × (−1) × 2 = −72 < 0. rin
∴ (−1, 2) is a saddle point.
g.n
At (1, −2), rt − s2 = 36xy = 36 × 1 × (−2) = −72 < 0.
∴ (1, −2) is a saddle point.
At (−1, −2), rt − s2 = 36xy = 72 > 0.
et
r = 6 × (−1) = −6 < 0.
∴ (−1, −1) is a maximum point.
Maximum value of f = −1 − 8 + 3 + 24 + 20 = 38.
Maxima = 38.
Minima = 2.

Download From : www.EasyEngineering.net


Download From : www.EasyEngineering.net

Function of Several Variables 131

Example 2.78. Test for maxima and minima of the function f (x, y) = x3 y2 (6 − x − y).
[Jan 2013]
Solution. f (x, y) = x3 y2 (6 − x − y)
= 6x3 y2 − x4 y2 − x3 y3 .
f x = 18x2 y2 − 4x3 y2 − 3x2 y3 .

fy = 12x3 y − 2x4 y − 3x3 y2 .

r = f xx = 36xy2 − 12x2 y2 − 6xy3 .

s = f xy = 36x2 y − 8x3 y − 9x2 y2 .

ww t = fyy = 12x3 − 2x4 − 6x3 y.

w.E
For stationary points, f x = 0, fy = 0.

f x = 0 ⇒ 18x2 y2 − 4x3 y2 − 3x2 y3 = 0

asy x2 y2 (18 − 4x − 3y) = 0

En
gin i.e., 4x + 3y = 18. (1)

ee
fy = 0 ⇒ 12x3 y − 2x4 y − 3x3 y2 = 0

x3 y(12 − 2x − 3y) = 0
rin
2x + 3y = 12. g.n (2)

(1) − (2) ⇒ 2x = 6
x = 3.
et
(1) ⇒ 12 + 3y = 18
3y = 6
y = 2.
The stationary point is (3, 2)
At (3, 2),

Download From : www.EasyEngineering.net


Download From : www.EasyEngineering.net

132 Engineering Mathematics - I

r = 36 × 3 × 4 − 12 × 9 × 4 − 6 × 3 × 8

= 432 − 432 − 144

= −144 < 0.

t = 12 × 9 − 2 × 81 − 6 × 27 × 2

= 108 − 162 − 324

= −378.

s = 34 × 9 × 2 − 8 × 27 × 2 − 9 × 9 × 4

ww = 612 − 432 − 324

w.E = −144.

rt − s2 = (−144)(−378) − (−144)2

= 54432 − 20736

= 33696 > 0 asy


Since rt − s2
En
> 0 and r < 0, (3, 2) is a maximum point.

gin
∴ Maximum value of f = 27 × 4(6 − 3 − 2) = 108.

ee
Example 2.79. Examine for minimum and maximum values sin x + sin y + sin(x + y).
Solution. We have f (x, y) = sin x + sin y + sin(x + y).

rin
f x = cos x + cos(x + y) fy = cos y + cos(x + y).

r = f xx = − sin x − sin(x + y) g.n


s = f xy = − sin(x + y)

t = fyy = − sin y − sin(x + y).


et
For stationary points, solve f x = 0 and fy = 0.

i.e., cos x + cos(x + y) = 0. (1)

cos y + cos(x + y) = 0. (2)

Download From : www.EasyEngineering.net


Download From : www.EasyEngineering.net

Function of Several Variables 133

(1) − (2) =⇒ cos x − cos y = 0.

i.e., cos x = cos y

=⇒x = y

Now (1) =⇒ cos x + cos 2x = 0

i.e., cos 2x = − cos x.

cos 2x = cos(π − x)

=⇒2x = π − x.

ww i.e., 3x = π
π

w.E π
When x = , y = .
π
x= .
3

π π 3
∴ ,
3 3 
π π
3
is a stationary point.
asy
At ,
3 3
En √ √
π
gin
r = − sin − sin
3
2π − 3
√3
= −
2 √ 2√
3 √
= − 3 < 0.

ee
2π 3 3 3 √
s = − sin =− ,t = − − = − 3.
3 2 2 2
2 3 9
rt − s = 3 − = > 0.
4 4 rin
Since rt − s2 > 0 and r < 0, f (x, y) has a maximum value at , .

π π
3√ 3 √ √ g.n
∴ Maximum value = f ,

Example 2.80.
π π
3 3
Find
π
3
the
π
= sin + sin + sin
3
maximum

3
=
2
3
+
2
and
3
+
2
3 3 3
=
2
minimum
.
et values of
sin x sin y sin(x + y), 0 < x, y < π. [Jan 1997]
Solution. Given f (x, y) = sin x sin y sin(x + y).

f x = sin y[sin x cos(x + y) + sin(x + y) cos x] = sin y sin(2x + y).

r = f xx = 2 sin y cos(2x + y).

Download From : www.EasyEngineering.net


Download From : www.EasyEngineering.net

134 Engineering Mathematics - I

fy = sin x[sin y cos(x + y) + sin(x + y) cos y] = sin x sin(x + 2y).

s = f xy = sin x cos(x + 2y) + sin(x + 2y) cos x = sin(2x + 2y).

t = fyy = 2 sin x cos(x + 2y).

For stationary points, solve f x = 0 and fy = 0.


f x = 0 ⇒ sin y sin(2x + y) = 0.
fy = 0 ⇒ sin x sin(x + 2y) = 0.
Since, x, y , 0& , π ⇒ sin x , 0, sin y , 0.

ww∴ sin(2x + y) = 0 and sin(x + 2y) = 0.


Since, 0 < x < π, 0 < 2x < 2π

w.E
0 < y < π ⇒ 0 < 2x + y < 3π.
Similarly 0 < x + 2y < 3π. Since, sin(2x + y) = 0 ⇒ 2x + y = π or 2π.
Similarly x + 2y = π or 2π.
asy
En If 2x + y = π (1)

gin and x + 2y = π (2)

ee
then x − y = 0 ⇒ x = y.
π
∴ (1) ⇒ 3x = π ⇒ x = .
∴y= .
π
3
3
rin
g.n
π π
∴ one stationary point is , .
3 3
If 2x + y = π and x + 2y = 2π then, x − y = −π ⇒ x = y − π.
∴ 2(y − π) + y = π ⇒ 3y − 2π = π ⇒ 3y = 3π ⇒ y = π,
which is not admissible since y , π.
et
Similarly, 2x + y = 2π and x + 2y = π is also not possible.
Now take 2x + y = 2π and x + 2y = 2π
⇒ x−y=0⇒ x=y

⇒ 3x = 2π ⇒ x =
3

∴y= .
3

Download From : www.EasyEngineering.net


Download From : www.EasyEngineering.net

Function of Several Variables 135

 2π 2π 
∴ Another stationary point is , .
π π 3 3
At , .
3 3 √
π 3(−1)
r = 2 sin cos π = 2 < 0.
3 2

4π  π π 3
s = sin = sin π + = − sin = − .
3 3 3 2

π 2 3(−1) √
t = 2 sin cos π = = − 3.
3 2

ww √ √  − 3 2 3 9
rt − s2 = (− 3)(− 3) − = 3 − = > 0.
2 4 4
π π

w.E
∴ ,
3 3
is a maximum point.

Maximum value = f ,
π π
√3 √
π π
= sin . sin sin
3 √ 3 √ 3

3
=

2
3

2
3 
sin π −
π
3

 2π 2π 
=
asy
3 3 3 3 3
2 2 2
=
8
.

At
3 3
,
2π 6π

3 En √
r = 2 sin

t = 2 sin
3

cos

cos
3
6π √
=2

= 3.
2
gin
.1 = 3 > 0.

s = sin

3
rt − s2 = 3 3 −
3

√ √

3
= sin 3π −

π
3
 3 2
=

2
3
ee
.
3 9
= 3 − = > 0. rin

 2π 2π 
3 3
,
2
is a minimum point.
4 4

√ √ √ g.n

Minimum value = f
 2π 2π 
,
3 3
= sin

3
sin

3
sin

3
=
3 3
2 2

2
Example 2.81. In a plane triangle, find the maximum value of cos A cos B cos C.
3  −3 3
=
8 et
.

[Jan 2000]
Solution. The angles of the ∆le ABC satisfy 0 < A, B, C < π and A + B + C = π,
=⇒ C = π − (A + B). Replacing C, we get
f (A, B) = cos A cos B cos(π − (A + B)) = − cos A cos B cos(A + B), 0 < A, B < π.
fA = − cos B[cos A(− sin(A + B)) + cos(A + B)(− sin A)] = cos B sin(2A + B).

Download From : www.EasyEngineering.net


Download From : www.EasyEngineering.net

136 Engineering Mathematics - I

fB = − cos A[− sin(A + 2B)] = cos A sin(A + 2B).


r = fAA = 2 cos B cos(2A + B).
s = fAB = cos A cos(A + 2B) + sin(A + 2B)(− sin A) = cos(2A + 2B).
t = fBB = 2 cos A cos(A + 2B).
For stationary points, solve fA = 0, fB = 0.

i.e., cos B sin(2A + B) = 0 and cos A sin(A + 2B) = 0.

cos B = 0 or sin(2A + B) = 0 and cos A = 0 or sin(A + 2B) = 0

ww =⇒ B =
π
2
or 2A + B = π or 2π

w.E
and
A=
π
or A + 2B = π or 2π.

Different possibilities asy 2

π π
case (i) Let B = and A = .
2 2
En
⇒ A+B=π
=⇒ C = 0 not possible. gin
π
case (ii) If B = and A + 2B = π.
2
⇒ A + π = π ⇒ A = 0 not possible.
π
case (iii) If B = and A + 2B = 2π.
ee rin
2
⇒ A + π = 2π ⇒ A = π not possible.
π g.n
case (iv) A = , 2A + B = π.
2
⇒ π + B = π ⇒ B = 0 not possible.
π
case (v) A = , 2A + B = 2π ⇒ B = π not possible.
2
et
case (vi) If 2A + B = π, A + 2B = π.
Subtracting A − B = 0 ⇒ A = B.
π
∴ 3A = π ⇒ A = .
3
π π
=⇒ B = , C = .
3 3
case (vii) If 2A + B = π and A + 2B = 2π ⇒ A − B = −π not possible.

Download From : www.EasyEngineering.net


Download From : www.EasyEngineering.net

Function of Several Variables 137

Finally 2A + B = 2π and A + 2B = 2π ⇒ A − B = 0 ⇒ A = B.
2π 2π
3A = 2π ⇒ A = ,B = .
3 3
2π 2π π
A+B= + = 4 > π not possible.
3 3 3 π π
∴ The only stationary point is , .
π π 3 3
At , ,
3 3
π 1
r = 2 cos cos π = 2 (−1) < 0.
3 2
π
t = 2 cos cos π = −1.
3

wws = cos
2
 4π 
3

= cos π +
π
3
 −1 2
1
=− .
2
1 3

w.E
rt − s = (−1)(−1) − = 1 − = > 0.
π π 2 4 4
∴ , is a maximum point.
3 3
π π π π π 1
∴ Maximum value of f is f ,

asy 3 3
= cos cos cos = .
3 3 3 8

En
Example 2.82. A flat circular plate is heated so that the temperature at any
point (x, y) is U(x, y) = x2 + 2y2 − x. Find the coldest point on the plate. [Jan 2005]
Solution. Given U = x2 + 2y2 − x.
gin
ee
U x = 2x − 1 Uy = 4y.

rin
r = U xx = 2. s = U xy = 0. t = Uyy = 4.
g.n
rt − s2 = 8 > 0 and r = 2 > 0.
∴ All points are minimum points.
At minimum, U x = 0 and Uy = 0.
et
1
U x = 0 ⇒ 2x − 1 = 0 ⇒ x = .
2
Uy = 0 ⇒ 4y = 0 ⇒ y = 0.
1 
∴ The minimum point is , 0 .
2
1 
∴ The coldest point on the plate is , 0 .
2

Download From : www.EasyEngineering.net


Download From : www.EasyEngineering.net

138 Engineering Mathematics - I

2.8 Constrained Maxima and Minima - Lagrange’s Method

Lagrange’s Method
Let f (x, y, z) be the function for which the extreme values are to be found subject to
the condition
φ(x, y, z) = 0. (1)

Construct the auxiliary function F(x, y, z) = f (x, y, z) + λφ(x, y, z), where λ is an


undetermined parameter independent of x, y, z which is called the Lagrange’s

wwmultiplier. Any relative extremum of f (x, y, z) subject to (1) must occur at a


stationary point of F(x, y, z).

w.E
The stationary points of F are given by
∂F
∂x
= 0,
∂F
∂y
⇒ f x + λφ x = 0, fy + λφy = 0, fz + λφz = 0, φ(x, y, z) = 0.
= 0,
∂F
∂z
= 0,
∂F
∂λ
= 0.

fx
=
fy
φ x φy φz
=
fz

asy
= −λ and φ(x, y, z) = 0.
Solving these equations we can find the values of x, y, z which are stationary

En
points of F and the values of f at these points give the maximum and minimum
values of f (x, y, z).
gin


Worked Examples

ee
Example 2.83. Find the maximum value of xm yn z p subject to x + y + z = a.

rin [Jan 2009]


Solution. Let f = xm ym z p .
φ = x + y + z − a = 0. g.n (1)

We have to maximise f subject to (1).


Let F = f + λφ where λ is the Lagrange’s multiplier.
et
F = xm yn z p + λ(x + y + z − a).

F x = mxm−1 yn z p + λ, Fy = nxm yn−1 z p + λ, Fz = pxm yn z p−1 + λ.

To find the stationary points, solve F x = 0, Fy = 0, Fz = 0, φ = 0.

F x = 0 ⇒ mxm−1 yn z p = −λ.

Download From : www.EasyEngineering.net


Download From : www.EasyEngineering.net

Function of Several Variables 139

Fy = 0 ⇒ nxm yn−1 z p = −λ.

F x = 0 ⇒ pxm yn z p−1 = −λ.

From the above three equations we get

mxm−1 yn z p = nxm yn−1 z p = pxm yn z p−1 .


m n p m+n+ p m+n+ p
Dividing by xm yn z p we get, = = = = .
x y z x+y+z a
ma na pa
x= ,y = ,z = .

wwThe stationary point is


m+n+ p
ma
,
m+n+ p
na
,
pa
m+n+ p
!
.

w.E
∴ Max. value of f =
m+n+ p m+n+ p m+n+ p
am+n+p mm nn p p
(m + n + p)m+n+p
.

asy
Example 2.84. Find the minimum value of x2 yz3 subject to 2x + y + 3z = a.
[Jan 2007]
Solution. Given f = x2 yz3 .
En
gin
φ = 2x + y + 3z − a = 0. (1)

ee
Let F = f + λφ where λ is the Lagrange’s multiplier.

F = x2 yz3 + λ(2x + y + 3z − a).


rin
F x = 2xyz3 + 2λ, Fy = x2 z3 + λ, Fz = 3x2 yz2 + 3λ.
g.n
To find the stationary points, solve F x = 0, Fy = 0, Fz = 0, φ = 0.

F x = 0 ⇒ 2xyz3 + 2λ = 0 ⇒ xyz3 = −λ.


et
Fy = 0 ⇒ x2 z3 = −λ.

Fz = 0 ⇒ 3x2 yz2 + 3λ = 0 ⇒ x2 yz2 = −λ.

Therefore
xyz3 = x2 z3 = x2 yz2

Download From : www.EasyEngineering.net


Download From : www.EasyEngineering.net

140 Engineering Mathematics - I

xyz3 = x2 z3 ⇒ y = x.

x2 z3 = x2 yz2 ⇒ y = z.

x = y = z.
a
(1) ⇒ 2x + x + 3x = a ⇒ 6x = a ⇒ x = = y = z.
6
a a a
∴ The stationary point is , , .
6 6 6
 a 2 a  a 3 a6  a 6
Minimum value = = 6 = .

ww 6 6 6 6 6
Example 2.85. If u = x2 + y2 + z2 where ax + by + cz − p = 0, find the stationary value

w.E
of u.
Solution. Given f = x2 + y2 + z2 .
[Jan 2006]

asy φ = ax + by + cz − p = 0. (1)

En
Let F = f + λφ where λ is the Lagrange’s multiplier.

gin
F = x2 + y2 + z2 + λ(ax + by + cz − p).

ee
F x = 2x + aλ, Fy = 2y + bλ, Fz = 2z + cλ.

To find the stationary points, solve F x = 0, Fy = 0, Fz = 0, φ = 0.


rin
2x + λa = 0 ⇒ x =
−aλ
2
⇒ ax =
−a2 λ
2
. g.n
Similarly
by =
−b2 λ
2
, cz =
−c2 λ
2
.
et
−a2 λ b2 λ c2 λ
(1) ⇒ − − =p
2 2 2
a2 + b2 + c2 −2p
λ = −p ⇒ λ = 2 .
2 a + b2 + c2
ap bp cp
∴x= 2 2 2
,y = 2 2 2
,z = 2 .
a +b +c a +b +c a + b2 + c2

Download From : www.EasyEngineering.net


Download From : www.EasyEngineering.net

Function of Several Variables 141

Stationary value of u is

a2 p2 b2 p2 c2 p2
u= + +
(a2 + b2 + c2 )2 (a2 + b2 + c2 )2 (a2 + b2 + c2 )2

p2 (a2 + b2 + c2 ) p2
= = .
(a2 + b2 + c2 )2 a2 + b2 + c2

Example 2.86. The temperature T at any point (x, y, z) in space is T = 400xyz2 .


Find the highest temperature on the surface of the unit sphere x2 + y2 + z2 = 1.
[Jan 2005]

wwSolution. We have to maximize

w.E T = 400xyz2 (1)

asy
subject to φ = x2 + y2 + z2 − 1 = 0.

Consider F = T + λφ, where λ is the Lagrange multiplier.


(2)

En
F = 400xyz2 + λ(x2 + y2 + z2 − 1).

gin
F x = 400yz2 + 2λx, Fy = 400xz2 + 2λy, Fz = 400xyz + 2λz.

F x = 0 ⇒ 400yz2 + 2λx = 0 ⇒ −λ =
ee
To find the stationary points, solve F x = 0, Fy = 0, Fz = 0, φ = 0.
200yz2
x
.
rin
g.n
200xz 2
Fy = 0 ⇒ 400xz2 + 2λy ⇒ −λ = .
y
Fz = 0 ⇒ 800xyz + 2λz = 0 ⇒ −λ = 400xy.

200yz2 200xz2

Taking
x
=

x
y
= 400xy.
200yz2 200xz2
=
y
we get x2 = y2 ⇒ y = ±x.
et
200yz2 z2 √
Taking = 400xy we get = 2x ⇒ z2 = 2x2 ⇒ z = ± 2x.
x x
200xz2 z2 √
Taking = 400xy we get = 2y ⇒ z2 = 2y2 ⇒ z = ± 2y.
y y
Substituting in x2 + y2 + z2 = 1 we get
1 1
x2 + x2 + 2x2 = 1 ⇒ 4x2 = 1 ⇒ x2 = ⇒ x = ±
4 2

Download From : www.EasyEngineering.net


Download From : www.EasyEngineering.net

142 Engineering Mathematics - I

1 √ 1 1
∴ y = ± ,z = ± 2 = ± √ .
2 2 2
1 1 1
The stationary points are given by x = ± , y = ± , z = ± √ .
2 2 2
To have maximum value, we must have xy positive.
 1 1 1   1 1 −1   −1 −1 1   −1 −1 −1 
∴ The points are , , √ , , , √ , , , √ , , , √ .
2 2 2 2 2 2 2 2 2 2 2 2
111 0
∴ Maximum T = 400 = 50 C.
222
Example 2.87. Find the shortest and longest distance from the point (1, 2, −1) to

wwthe sphere x2 + y2 + z2 = 24 using Lagrange’s method of constrained maxima and


minima. [Jun 2011, Jan 2002]

w.E
Solution. Let P(x, y, z) be any point on the sphere.
Let A be the point (1, 2, −1).

asy
q
AP = (x − 1)2 + (y − 2)2 + (z + 1)2 .

En
Let f (x, y, z) = (x − 1)2 + (y − 2)2 + (z + 1)2 . (1)

gin
AP is minimum or maximum if f is minimum or maximum.
∴ The problem is now reduced to minimise or maximise f subject to
φ(x, y, z) = x2 + y2 + z2 − 24 = 0.
Consider the auxiliary function ee
F = f + λφ = (x − 1)2 + (y − 2)2 + (z + 1)2 + λ(x2 + y2 + z2 − 24) rin
where λ is the
Lagrange’s multiplier.
g.n
F x = 2(x − 1) + 2λx, Fy = 2(y − 2) + 2λy, Fz = 2(z + 1) + 2λz.
To find the stationary points solve
F x = 0, Fy = 0, Fz = 0, φ = 0.
et
x−1 1
F x = 0 ⇒ 2(x − 1) + 2λx = 0 ⇒ x − 1 = −λx ⇒ −λ = =1− .
x x
y−2 2
Fy = 0 ⇒ 2(y − 2) + 2λy = 0 ⇒ y − 2 = −λy ⇒ −λ = =1− .
y y
z+1 1
Fz = 0 ⇒ 2(z + 1) + 2λz = 0 ⇒ z + 1 = −λz ⇒ −λ = =1+ .
z z
1 2 1
∴1− =1− =1+ .
x y z

Download From : www.EasyEngineering.net


Download From : www.EasyEngineering.net

Function of Several Variables 143

1 2 1 2
Taking 1 − = 1 − we get = ⇒ y = 2x.
x y x y
1 1
Taking 1 − = 1 + we get z = −x.
x z
2 1 −y
Taking 1 − = 1 + we get z = = −x.
y z 2
We have x2 + y2 + z2 = 24 ⇒ x2 + 4x2 + x2 = 24 ⇒ 6x2 = 24 ⇒ x2 = 4 ⇒ x = ±2.
When x = 2, y = 4, z = −2, the first point is (2, 4, −2). Let this point be P1 .
When x = −2, y = −4, z = 2, the second point is (−2, −4, 2). Let this point be P2 .
√ √ √ √ √
P1 A = 1 + 4 + 1 = 6, P2 A = 9 + 36 + 9 = 54 = 3 6

ww∴ Shortest distance = 6



Longest distance = 3 6.

w.E
Example 2.88. A rectangular box open at the top is to have a volume of 32cc.
Find the dimensions of the box which requires least amount of material for its
construction.
asy [Jun 2012, Dec 2011, Jun 2010, Jan 2005]

En
Solution. Let the dimensions of the box be Length = x, Breadth = y, height = z.
Given: Volume = 32cc.

gin
=⇒ xyz = 32, x, y, z > 0. (1)

ee
We have to minimize the amount of material used for the construction of the box.
Let S be the surface area of the box whose top is open

rin
∴ S = xy + 2xz + 2yz

g.n (2)

et
By Lagrange’s method
Let F = s + λφ = xy + 2yz + 2xz + λ(xyz − 32) where λ is the Lagrange’s multiplier.
F x = y + 2z + λyz, Fy = x + 2z + λxz, Fz = 2y + 2x + λxy.
To find the stationary points, solve

F x = 0, Fy = 0, Fz = 0, φ = 0

F x = 0 ⇒ y + 2z + λyz = 0

=⇒ y + 2z = −λyz

Download From : www.EasyEngineering.net


Download From : www.EasyEngineering.net

144 Engineering Mathematics - I

y 2z
⇒ + = −λ
yz yz
1 2
⇒ + = −λ (3)
z y
Fy = 0 ⇒ x + 2z + λxz = 0
=⇒ x + 2z = −λxz

i.e., xy + 2yz = −λxyz.


xy 2yz
⇒ + = −λ
xyz xyz

ww ⇒
1 2
+ = −λ
z x
(4)

w.E
Fz = 0 ⇒ 2y + 2x + λxy = 0
=⇒ 2x + 2y = −λxy

asy 2xz + 2yz = −λxyz.


2xz 2yz

En⇒ +
xyz xyz
= −λ

gin

2 2
+ = −λ
y x
(5)

(3) − (4) ⇒

ee
2 2
− =0
y x
1 1 rin
=
y x
⇒x=y g.n (6)

(3) − (5) ⇒
1 2
− =0
z x
et
1 2
=
z x
⇒ x = 2z (7)

From (6)and(7) we obtain x = y = 2z.


(1)⇒ 2z.2z.z = 32 ⇒ 4z3 = 32 ⇒ z3 = 8 ⇒ z = 2.

Download From : www.EasyEngineering.net


Download From : www.EasyEngineering.net

Function of Several Variables 145

∴ x = 4, y = 4.
The stationary point is (4, 4, 2).
The dimensions are 4cm, 4cm, 2cm.

Example 2.89. Find the dimensions of the rectangular box open at the top of
maximum capacity whose surface area is 432 sq.m. [Jun 2013]
Solution. Let the dimensions of the box be x, y, z.
Given, surface area= 432.

ww xy + 2xz + 2yz = 432 (1)

w.E
Let V be the volume of the box.
We have to maximize V.

By lagrange’s method
asy V = xyz (2)

En
F = V + λφ, where λ is the lagrange multiplier.

gin
F = xyz + λ(xy + 2xz + 2yz − 432).

ee
F x = yz + λ(y + 2z).

rin
Fy = xz + λ(x + 2z).

Fz = xy + λ(2x + 2y).

Fλ = xy + 2xz + 2yz − 432.


For stationary points, F x = 0, Fy = 0, Fz = 0, Fλ = 0. g.n
F x = 0 ⇒ yz + λ(y + 2z) = 0 et
⇒ xyz + λ(xy + 2xz) = 0. (1)

Fy = 0 ⇒ xz + λ(x + 2z) = 0

⇒ xyz + λ(xy + 2yz) = 0. (2)

Fz = 0 ⇒ xy + λ(2x + 2y) = 0

Download From : www.EasyEngineering.net


Download From : www.EasyEngineering.net

146 Engineering Mathematics - I

xyz + λ(2xz + 2yz) = 0. (3)

(1) + (2) + (3) ⇒


3xyz + λ(2xy + 4xz + 4yz) = 0

3xy + 2λ(xy + 2xz + 2yz) = 0

3xyz + 2λ × 432 = 0

3xyz = −864λ
3xyz xyz
λ=− =−

ww 864 288
Substituting the value of λ in F x = 0 we get

w.E yz −
xyz
288
(y + 2z) = 0

asy 1−
x
288
(y + 2z) = 0

En 1=
x
288
(y + 2z)

gin xy + 2xz = 288 (4)

ee
Fy = 0 ⇒ xz −

y
xyz
288
(x + 2z) = 0

rin
y
1−
288
(x + 2z) = 0

y g.n
1−
288
(x + 2z) = 0, 1 =

xy + 2yz = 288
288
(x + 2z)
et (5)

xyz
Fz = 0 ⇒ xy − (2x + 2y) = 0
288
z z
1− (2x + 2y) = 0, 1 = (2x + 2y)
288 288

2xz + 2yz = 288 (6)

Download From : www.EasyEngineering.net


Download From : www.EasyEngineering.net

Function of Several Variables 147

(4) − (5) ⇒ 2z(x − y) = 0, z , 0

∴ x−y=0

⇒ x = y.

(5) − (6) ⇒ xy − 2xz = 0

x(y − 2z) = 0, x , 0

∴ y − 2z = 0

y = 2z.

ww ∴ x = y = 2z

w.E (4) ⇒ 2z × 2z + 2 · 2z · z = 288

4z2 + 4z2 = 288

asy z2 = 36.

z = ±6.

En z = −6 is not possible

gin ∴ z = 6.

ee
∴ y = 2z = 12

x = 2z = 12

rin
The dimensions of the box to have maximum capacity are

g.n
Length = 12m

Breadth = 12m
et
Height = 6m.

Download From : www.EasyEngineering.net


Download From : www.EasyEngineering.net

ww
w.E
asy
En
gin
ee rin
g.n
et

Download From : www.EasyEngineering.net


Download From : www.EasyEngineering.net

3 Integral Calculus

ww3.1 The definite integral

w.E
Integration as the limit of a sum

asy y = f (x)

·········En
n rectangles
f (xk )

gin ∆x
x1 = a x2 x3

ee
xn+1 = b

rin
Consider the graph of the positive function y = f (x). Let us find the area
under the curve y = f (x),between the x−axis and the ordinates x = a and x = b. g.n
Divide this area into n rectangles of equal width ∆x =
b−a
n
. Let the x− co ordinates
at the left hand side of the rectangles be x1 = a, x2 , x3 , . . . xn+1 = b. Consider a typical
rectangle, the kth one with height f (xk ). The area of this rectangle is f (xk )∆x. The
et
sum of all the areas of the n rectangles is

f (x1 )∆x + f (x2 )∆x + · · · + f (xn )∆x.


n
P
In summation convention form, this can be written as f (xk )∆x. This
k=1
gives an estimate of the area under the curve but it is not exact. To improve the

Download From : www.EasyEngineering.net


Download From : www.EasyEngineering.net

150 Engineering Mathematics - I

estimate we must take a large number of very thin rectangles. This can be
achieved by allowing n → ∞ and making ∆x → 0.
n
P
∴ Area under the curve = lim f (xk )∆x.
n=∞ k=1
The lower and upper limits on the sum correspond to the first and the
last rectangle where x = a and x = b respectively. Hence the above limit can be
b
P
written as lim f (x)∆x. Since the number of rectangles increase without bound,
∆x→0 x=a
we drop the subsript k from xk and write f (x) which is the value of f at a typical
value of x. If this can actually be found, it is called the definite integral of f (x)

ww Rb
from x = a and x = b and it is written as f (x)dx.
a

w.E
Rb n
P b
P
∴ f (x)dx = lim f (xk )∆x = lim f (x)∆x.
n=∞ k=1 ∆x→0 x=a
a
Note. If we approximate each strip by a rectangle that has the same base as the

asy
strip and whose height is the same as the right edge of the strip; (i.e., xk is taken
at the right end points of the kth rectangle and f (xk ) as the height ) then also the
above result will be achieved.
En
gin
Definition of a definite integral. If f is a function defined for a ≤ x ≤ b, we
divide the interval [a, b] into n subintervals of equal width ∆x =
b−a
. We let

ee
n
x0 (= a), x1 , x2 , · · · xn (= b) be the end points of these subintervals and if we choose

rin
x1∗ , x2∗ , · · · xn∗ as any sample ponits in these subintervals, so that xi∗ lies in the ith
subinterval [xi−1 , xi ]. Then the definite integral of f from a to b is defined as
Rb
f (x)dx = lim
n
P
g.n
f (xi∗ )∆x, provided that the limit exists and gives the same value
n=∞ i=1

et
a
for all possible choices of sample points. If it does exist, we say that f is
integrable on [a, b].

Rb n
f (xi∗ )∆x can also be written as, for ∈> 0, there is an
P
Result (1). f (x)dx = lim
n=∞ i=1
a
Rb n
f (xi∗ )∆x <∈ for every integer n > N and for
P
integer N such that f (x)dx −
a i=1
every choice of xi∗ in [xi−1 , xi ].

Download From : www.EasyEngineering.net


Download From : www.EasyEngineering.net

Integral Calculus 151

n
f (xi∗ )∆x is called the Reimann sum.
P
Result (2). The sum
i=1

Theorem 1. If f is continuos on [a, b], (or) if f has only a finite number of jump
Rb
discontinuities, then f is integrable on [a, b].[i.e., f (x)dx exists].
a
Rb n
P
Theorem 2.If f is integrable on [, b], then f (x)dx = lim f (xi )∆x where ∆x =
n=∞ i=1
a
b−a
and xi = a + i∆x.
n ✎ ☞
Worked Examples

ww ✍

R1

w.E
Example 3.1. Using the area property evaluate

areas of the upper approximating rectangles approaches .


0
1
3
x2 dx. Show that the sum of the

asy
Solution. Consider the function y = f (x) = x2 . Divide the area under the curve
y = x2 between x = 0 and x = 1 into 4 rectangles of equal width ∆x =
1−0 1
= .

En
The width of each rectangle is .
1
# " 4# "
4 4

gin
" # " #
1 1 1 1 3 3
The subintervals are 0, , , , , and , 1 .
4 4 2 2 4 4

ordinates at x = 0, , & .
4 2 4 ee
The height of the rectangles is approximated to the value of the
1 1 3

rin
g.n
! ! !
1 1 3
∴ Area of the required portion = f (0)∆x + f ∆x + f ∆x + f ∆x.
4 2 4

et
!2 !2 !2
2 1 1 1 1 1 3 1
=0 . + . + . + .
4 4 4 2 4 4 4
" #
1 1 1 9
= 0+ + +
4 16 4 16
" #
1 1+4+9
=
4 16
1 14 7
= × = .
4 16 32

If the height of the rectangles is approximated to the value of the

Download From : www.EasyEngineering.net


Download From : www.EasyEngineering.net

152 Engineering Mathematics - I

ordinates at the right edge of each rectangle then,


! ! !
1 1 3
Area = f ∆x + f ∆x + f ∆x + f (1)∆x.
4 2 4
!2 !2 !2
1 1 1 1 3 1 1
= . + . + . + 12 ·
4 4 2 4 4 4 4
" # " #
1 1 1 9 1 1 + 4 + 9 + 16 1 30 15
= + + +1 = = × = .
4 16 4 16 4 16 4 16 32
Now, let us divide the area " into # "n rectangles
# " each of their
# with width
1−0 1 1 1 2 n−1 n

ww∆x =
n
= . The subintervals are 0, , , , · · · ,
n n n n
, =1 .
n n
The heights of the rectangle are the values of the function f (x) = x2 at
1 2 3 n

w.E
the points , , · · · .
n n n n
!2
1 1
!2
2 1
!2
3 1  n 2 1
∴ Area =
n n
. +

1 12 22 32
"
asyn n
. +
n n
. + ··· +

n2
#
n n
.

En
= + + + · · · +
n n2 n2 n2 n2
1 h i
= 3 12 + 22 + 32 + · · · + n2
n "
= 3
1 n(n + 1)(2n + 1)gin #
=
(n + 1)(2n + 1)
n
lim (Area) = lim
n=∞ n=∞
6
(n + 1)(2n + 1)
6n2
ee
n (1 + 1n )(2 + 1n ) 2 1
2
6n2

rin
= lim
n=∞ 6n2
= = .
6 3
g.n
Properties of definite integrals.

Ra
et
The following properties can be easily proved using Riemann sums.
Rb
(i) f (x)dx = − f (x)dx.
b a

Ra
(ii) f (x)dx = 0.
a

Rb
(iii) cdx = c(b − a),where c is any constant.
a

Download From : www.EasyEngineering.net


Download From : www.EasyEngineering.net

Integral Calculus 153

Rb Rb Rb
(iv) [ f (x) + g(x)]dx = f (x)dx + g(x)dx.
a a a

Rb Rb
(v) c f (x)dx = c f (x)dx,where c is any constant.
a a

Rb Rb Rb
(vi) [ f (x) − g(x)]dx = f (x)dx − g(x)dx.
a a a

Rb Rc Rb
(vii) If a < c < b then f (x)dx = f (x)dx + f (x)dx.

ww a

Comparison properties of the integral.


a c

w.E(i) If f (x) ≥ 0 for a ≤ x ≤ b, then


Rb
a
f (x)dx ≥ 0.

(ii) If f (x) ≥ g(x) for a ≤ x ≤ b, then


asy Rb
a
f (x)dx ≥
Rb
a
g(x)dx.

En
(iii) If m ≤ f (x) ≤ M for a ≤ x ≤ b, then m(b − a) ≤
Rb
f (x)dx ≤ M(b − a).
Proof. Given m ≤ f (x) ≤ M
gin a

Rb Rb
ee
Integrating between a and b we get
Rb
mdx ≤ f (x)dx ≤ Mdx
a a a
rin
g.n
Rb Rb Rb
m dx ≤ f (x)dx ≤ M dx
a a a
Rb

The first fundamental theorem of calculus


m(b − a) ≤
a
f (x)dx ≤ M(b − a).
et
Statement. If f is continuous on [a, b], then the function g defined by
Rx
g(x) = f (t)dt, a ≤ x ≤ b
a

is continuous on [a, b] and is differentiable on (a, b) and g′ (x) = f (x).


Proof.Choose x and x + b ∈ (a, b).

Download From : www.EasyEngineering.net


Download From : www.EasyEngineering.net

154 Engineering Mathematics - I

y
Zx+h Zx
Now, g(x + h) − g(x) = f (t)dt − f (t)dt
a a
Zx Zx+h Zx
= f (t)dt + f (t)dt − f (t)dt m M

a x a
Zx+h
= f (t)dt. 0 x u v x+h x
x

ww g(x + h) − g(x) 1 x+h


R
Hence, for h , 0, we have = f (t)dt. (1)
h h x
Let us assume that h > 0. Since f is continuous on [x, x + h], by the

w.E
extreme value theorem, there exists numbers u and v in [x, x + h] such that
f (u) = m and f (v) = M, where m and M are the absolute minimum and maximum

asy
values of f on [x, x + h]. By the comparison property of the definite integral we
have

En mh ≤
x+h
R
f (t)dt ≤ Mh.

gin
x
x+h
R
f (u)h ≤ f (t)dt ≤ f (v)h.

ee
x
Since h > 0, dividing throughout by h we get
x+h

rin
R
f (u) ≤ h1 f (t)dt ≤ f (v). (2)
x

g.n
(2) can be proved in a similar way for h < 0.
Now, allowing h → 0 and since u and v lie in [x, x + h], we have u → x
and v → x. Since f is continuous at x,we have

∴ lim f (u) = lim f (u) = f (x)


h→0 u→x
et
and lim f (v) = lim f (v) = f (x).
h→0 v→x

By (2) we have
Zx+h
1
lim f (u) ≤ lim f (t)dt ≤ lim f (v)
h→0 h→0 h h→0
x

Download From : www.EasyEngineering.net


Download From : www.EasyEngineering.net

ww
w.E
a syE
ngi
nee
rin
g.n
et

**Note: Other Websites/Blogs Owners Please do not Copy (or) Republish


this Materials, Students & Graduates if You Find the Same Materials with
EasyEngineering.net Watermarks or Logo, Kindly report us to
easyengineeringnet@gmail.com Download From : www.EasyEngineering.net
Download From : www.EasyEngineering.net

Integral Calculus 155

g(x + h) − g(x)
f (x) ≤ lim ≤ f (x) [by (1)]
h→0 h
i.e., f (x) ≤ g′ (x) ≤ f (x).

By Squeeze Theorem we get

g′ (x) = f (x). (3)

Since every differentiable function is continuous,


we obtain g is also continuous on (a, b).

ww If x = a or x = b, from (3) we get the one sided limits at a and b.


This shows that g is continuous on [a, b].

w.E
Rx √
Example 3.2. Find the derivative of the function g(x) = 1 + t2 dt.
√ 0
Solution. Since f (t) = 1 + t2 is continuous on [0, x], by part 1 of the Fundamental

asy
theorem of calculus we get g′ (x) = 1 + x2

En
 4 
d Rx 
Example 3.3. Find  sec tdt.
dx 1
Solution. Let u = x 4

 x4 gin  u 

ee
Z  Z
d  d 


∴  sec tdt =  sec tdt
dx   dx 

rin

1 1
 u 
Z
d   du

=
1
du
 sec tdt ·
dx
g.n
[By Chain rule]

= sec u · 4x3

= sec x4 · 4x3 .
[by FTC 1]
et
Second Fundamental theorem of calculus
Rb
Statement.If f is continuous on [a, b], then f (x)dx = F(b) − F(a) where F is any
a
antiderivative of f . [i.e, a function such that F ′ = f ].
Rx
Proof. Let g(x) = f (t)dt.
a
By the First Fundamental theorem of calculus we have g′ (x) = f (x). i.e,

Download From : www.EasyEngineering.net


Download From : www.EasyEngineering.net

156 Engineering Mathematics - I

g is an antiderivative of f .
If F is any other antiderivative of f on [a, b], we know that F and g differ
by a constant.
∴ F(x) = g(x) + c for a < x < b.
Since both F and g are continuous on [a, b] the above relation holds when x = a and
x = b.
i.e lim F(x) = F(a) & lim F(x) = F(b).
x→a+ x→b−
Also lim g(x) = g(a) & lim g(x) = g(b).

ww
x→a+ x→b−
Hence we obtain F(x) = g(x) + c for all x in [a, b] (1)
Ra
When x = a, we have g(a) = f (t)dt = 0.

w.E Using (1) for x = b and x = a we get


a

asy
F(b) − F(a) = [g(b) + c] − [g(a) + c]

= g(b) − g(a)

En = g(b) =
Zb
f (t)dt.

gin a
Result. The First and Second Fundamental Theorem of Calculus are combined

Statement of Fundamental Theorem of Calculus.


ee
together is called the Fundamental Theorem of Calculus.

rin
Suppose f is continuous on [a, b]
Rx
(i) If g(x) = f (t)dt, then g′ (x) = f (x).
g.n
et
a
Rb
(ii) f (x)dx = F(b) − F(a), where F is any antiderivative of f . (i.e., F ′ = f .)
a
Note (1). The Fundamental theorem of calculus says that differentiation and
integration are inverse processors.
Note (2). By part(ii) of the Fundamental theorem of calculus we have
Rb
f (x)dx = F(b) − F(a) where F ′ = f .
a
Rb
i.e F ′ (x)dx = F(b) − F(a).
a

Download From : www.EasyEngineering.net


Download From : www.EasyEngineering.net

Integral Calculus 157

The above result is sometimes called as the Net Change Theorem.


Net Change Theorem
Statement. The integral of a rate of change is the net change.
Rb
i.e F ′ (x)dx = F(b) − F(a).

a ☞
Worked Examples
✍ ✌
R3
Example 3.4. Evaluate (x2 − 2x)dx by using Reimann sum by taking right end
0
points as the sample points. Hence verify it by using fundamental theorem of

wwcalculus.
Solution. Let f (x) = x2 − 2x.

w.E Divide the interval [0, 3] into n sub intevals with equal width
∆x =
3−0 3
= .
n n "
asy
3
n
# "
3 6
# "
The intervals are 0, , , , , , · · ·
n n
6 9
n n
# "
3n − 3 3n
n
,
n
#
.

En
The sample points xi∗ are the right end points of the sub intervals.
3 6 9
∴ xi∗ = , , , · · · .
3n

gin
n n n n
The corresponding f (xi∗ ) = xi∗ 2 − 2xi∗ are calculated and tabulated as
follows.

xi∗ 3
n
ee 6
n
9
n ··· 3n
n rin
f (xi∗ )
By Reimann’s sum we have,
9
n2
− 6
n
36
n2
− 12
n
81
n2
− 18
n ··· 9n2
n2
− 6n
n
g.n
Z3

0
(x2 − 2x)dx = lim
n=∞
n
X

i=1
f (xi∗ )∆x
et
9n2 6n 3
" ! ! ! ! #
9 6 3 36 12 3 81 18 3
= lim − + − + − + · · · + −
n=∞ n2 n n n2 n n n2 n n n2 n n
" 3 2
! 3 2
! 3 2
!
3 3 3 3 3 3
= lim 3
· 1 − 2 · 2 + 3 · 22 − 2 · 4 + 3 · 32 − 2 · 6
n=∞ n n n n n n
3 32
!#
3 2
+ · · · + 3 · n − 2 · 2n
n n

Download From : www.EasyEngineering.net


Download From : www.EasyEngineering.net

158 Engineering Mathematics - I

33  2 2 2 2
 32
= lim 1 + 2 + 3 + · · · + n − lim (2 + 4 + 6 + · · · + 2n)
n=∞ n3 n=∞ n2
27 h n(n + 1)(2n + 1) i 9 h n(n + 1) i
= lim 3 − lim 2 · 2
n=∞ n 6 n=∞ n 2
3 1 1
h n (1 + n )(2 + n ) i h n (1 + 1n ) i 27
2
27
= lim − 9 lim = × 2 − 9 = 0.
6 n=∞ n3 n=∞ n2 6

By the Fundamental Theorem of Calculus we have


Z3 !3
2 x3 x2 27
(x − 2x)dx = −2· = − 9 = 9 − 9 = 0.
3 2 0 3

ww 0

w.E
Example 3.5. Evaluate the integral

R1 √
R1 √
0
1 − x2 dx interms of areas.


Solution.
0
asy
1 − x2 dx represents the area under the curve y = 1 − x2 between
the x−axis and the lines x = 0 and x = 1.

p
En y

gin
Now, y = 1 − x2

y2 = 1 − x2
0 1x
2 2
x + y = 1.
ee
This is a circle with centre at the origin and radius = 1 unit. Since we rin
need the area between x = 0 and x = 1, the required area is the area of the portion
1 π g.n
of the circle that lies in the first quadrant = · π × 12 = .


R1 √
0
π
1 − x2 dx = .
4
4 4
et
R3
Example 3.6. Evaluate the integral (x − 1)dx interms of areas.
0
R3
Solution. (x − 1)dx represents the area under the curve y = (x − 1) between the
0
x−axis and the lines x = 0 and x = 3.

Download From : www.EasyEngineering.net


Download From : www.EasyEngineering.net

3 Integral Calculus

ww3.1 The definite integral

w.E
Integration as the limit of a sum

asy y = f (x)

·········En
n rectangles
f (xk )

gin ∆x
x1 = a x2 x3

ee
xn+1 = b

rin
Consider the graph of the positive function y = f (x). Let us find the area
under the curve y = f (x),between the x−axis and the ordinates x = a and x = b. g.n
Divide this area into n rectangles of equal width ∆x =
b−a
n
. Let the x− co ordinates
at the left hand side of the rectangles be x1 = a, x2 , x3 , . . . xn+1 = b. Consider a typical
rectangle, the kth one with height f (xk ). The area of this rectangle is f (xk )∆x. The
et
sum of all the areas of the n rectangles is

f (x1 )∆x + f (x2 )∆x + · · · + f (xn )∆x.


n
P
In summation convention form, this can be written as f (xk )∆x. This
k=1
gives an estimate of the area under the curve but it is not exact. To improve the

Download From : www.EasyEngineering.net


Download From : www.EasyEngineering.net

150 Engineering Mathematics - I

estimate we must take a large number of very thin rectangles. This can be
achieved by allowing n → ∞ and making ∆x → 0.
n
P
∴ Area under the curve = lim f (xk )∆x.
n=∞ k=1
The lower and upper limits on the sum correspond to the first and the
last rectangle where x = a and x = b respectively. Hence the above limit can be
b
P
written as lim f (x)∆x. Since the number of rectangles increase without bound,
∆x→0 x=a
we drop the subsript k from xk and write f (x) which is the value of f at a typical
value of x. If this can actually be found, it is called the definite integral of f (x)

ww Rb
from x = a and x = b and it is written as f (x)dx.
a

w.E
Rb n
P b
P
∴ f (x)dx = lim f (xk )∆x = lim f (x)∆x.
n=∞ k=1 ∆x→0 x=a
a
Note. If we approximate each strip by a rectangle that has the same base as the

asy
strip and whose height is the same as the right edge of the strip; (i.e., xk is taken
at the right end points of the kth rectangle and f (xk ) as the height ) then also the
above result will be achieved.
En
gin
Definition of a definite integral. If f is a function defined for a ≤ x ≤ b, we
divide the interval [a, b] into n subintervals of equal width ∆x =
b−a
. We let

ee
n
x0 (= a), x1 , x2 , · · · xn (= b) be the end points of these subintervals and if we choose

rin
x1∗ , x2∗ , · · · xn∗ as any sample ponits in these subintervals, so that xi∗ lies in the ith
subinterval [xi−1 , xi ]. Then the definite integral of f from a to b is defined as
Rb
f (x)dx = lim
n
P
g.n
f (xi∗ )∆x, provided that the limit exists and gives the same value
n=∞ i=1

et
a
for all possible choices of sample points. If it does exist, we say that f is
integrable on [a, b].

Rb n
f (xi∗ )∆x can also be written as, for ∈> 0, there is an
P
Result (1). f (x)dx = lim
n=∞ i=1
a
Rb n
f (xi∗ )∆x <∈ for every integer n > N and for
P
integer N such that f (x)dx −
a i=1
every choice of xi∗ in [xi−1 , xi ].

Download From : www.EasyEngineering.net


Download From : www.EasyEngineering.net

Integral Calculus 151

n
f (xi∗ )∆x is called the Reimann sum.
P
Result (2). The sum
i=1

Theorem 1. If f is continuos on [a, b], (or) if f has only a finite number of jump
Rb
discontinuities, then f is integrable on [a, b].[i.e., f (x)dx exists].
a
Rb n
P
Theorem 2.If f is integrable on [, b], then f (x)dx = lim f (xi )∆x where ∆x =
n=∞ i=1
a
b−a
and xi = a + i∆x.
n ✎ ☞
Worked Examples

ww ✍

R1

w.E
Example 3.1. Using the area property evaluate

areas of the upper approximating rectangles approaches .


0
1
3
x2 dx. Show that the sum of the

asy
Solution. Consider the function y = f (x) = x2 . Divide the area under the curve
y = x2 between x = 0 and x = 1 into 4 rectangles of equal width ∆x =
1−0 1
= .

En
The width of each rectangle is .
1
# " 4# "
4 4

gin
" # " #
1 1 1 1 3 3
The subintervals are 0, , , , , and , 1 .
4 4 2 2 4 4

ordinates at x = 0, , & .
4 2 4 ee
The height of the rectangles is approximated to the value of the
1 1 3

rin
g.n
! ! !
1 1 3
∴ Area of the required portion = f (0)∆x + f ∆x + f ∆x + f ∆x.
4 2 4

et
!2 !2 !2
2 1 1 1 1 1 3 1
=0 . + . + . + .
4 4 4 2 4 4 4
" #
1 1 1 9
= 0+ + +
4 16 4 16
" #
1 1+4+9
=
4 16
1 14 7
= × = .
4 16 32

If the height of the rectangles is approximated to the value of the

Download From : www.EasyEngineering.net


Download From : www.EasyEngineering.net

152 Engineering Mathematics - I

ordinates at the right edge of each rectangle then,


! ! !
1 1 3
Area = f ∆x + f ∆x + f ∆x + f (1)∆x.
4 2 4
!2 !2 !2
1 1 1 1 3 1 1
= . + . + . + 12 ·
4 4 2 4 4 4 4
" # " #
1 1 1 9 1 1 + 4 + 9 + 16 1 30 15
= + + +1 = = × = .
4 16 4 16 4 16 4 16 32
Now, let us divide the area " into # "n rectangles
# " each of their
# with width
1−0 1 1 1 2 n−1 n

ww∆x =
n
= . The subintervals are 0, , , , · · · ,
n n n n
, =1 .
n n
The heights of the rectangle are the values of the function f (x) = x2 at
1 2 3 n

w.E
the points , , · · · .
n n n n
!2
1 1
!2
2 1
!2
3 1  n 2 1
∴ Area =
n n
. +

1 12 22 32
"
asyn n
. +
n n
. + ··· +

n2
#
n n
.

En
= + + + · · · +
n n2 n2 n2 n2
1 h i
= 3 12 + 22 + 32 + · · · + n2
n "
= 3
1 n(n + 1)(2n + 1)gin #
=
(n + 1)(2n + 1)
n
lim (Area) = lim
n=∞ n=∞
6
(n + 1)(2n + 1)
6n2
ee
n (1 + 1n )(2 + 1n ) 2 1
2
6n2

rin
= lim
n=∞ 6n2
= = .
6 3
g.n
Properties of definite integrals.

Ra
et
The following properties can be easily proved using Riemann sums.
Rb
(i) f (x)dx = − f (x)dx.
b a

Ra
(ii) f (x)dx = 0.
a

Rb
(iii) cdx = c(b − a),where c is any constant.
a

Download From : www.EasyEngineering.net


Download From : www.EasyEngineering.net

Integral Calculus 153

Rb Rb Rb
(iv) [ f (x) + g(x)]dx = f (x)dx + g(x)dx.
a a a

Rb Rb
(v) c f (x)dx = c f (x)dx,where c is any constant.
a a

Rb Rb Rb
(vi) [ f (x) − g(x)]dx = f (x)dx − g(x)dx.
a a a

Rb Rc Rb
(vii) If a < c < b then f (x)dx = f (x)dx + f (x)dx.

ww a

Comparison properties of the integral.


a c

w.E(i) If f (x) ≥ 0 for a ≤ x ≤ b, then


Rb
a
f (x)dx ≥ 0.

(ii) If f (x) ≥ g(x) for a ≤ x ≤ b, then


asy Rb
a
f (x)dx ≥
Rb
a
g(x)dx.

En
(iii) If m ≤ f (x) ≤ M for a ≤ x ≤ b, then m(b − a) ≤
Rb
f (x)dx ≤ M(b − a).
Proof. Given m ≤ f (x) ≤ M
gin a

Rb Rb
ee
Integrating between a and b we get
Rb
mdx ≤ f (x)dx ≤ Mdx
a a a
rin
g.n
Rb Rb Rb
m dx ≤ f (x)dx ≤ M dx
a a a
Rb

The first fundamental theorem of calculus


m(b − a) ≤
a
f (x)dx ≤ M(b − a).
et
Statement. If f is continuous on [a, b], then the function g defined by
Rx
g(x) = f (t)dt, a ≤ x ≤ b
a

is continuous on [a, b] and is differentiable on (a, b) and g′ (x) = f (x).


Proof.Choose x and x + b ∈ (a, b).

Download From : www.EasyEngineering.net


Download From : www.EasyEngineering.net

154 Engineering Mathematics - I

y
Zx+h Zx
Now, g(x + h) − g(x) = f (t)dt − f (t)dt
a a
Zx Zx+h Zx
= f (t)dt + f (t)dt − f (t)dt m M

a x a
Zx+h
= f (t)dt. 0 x u v x+h x
x

ww g(x + h) − g(x) 1 x+h


R
Hence, for h , 0, we have = f (t)dt. (1)
h h x
Let us assume that h > 0. Since f is continuous on [x, x + h], by the

w.E
extreme value theorem, there exists numbers u and v in [x, x + h] such that
f (u) = m and f (v) = M, where m and M are the absolute minimum and maximum

asy
values of f on [x, x + h]. By the comparison property of the definite integral we
have

En mh ≤
x+h
R
f (t)dt ≤ Mh.

gin
x
x+h
R
f (u)h ≤ f (t)dt ≤ f (v)h.

ee
x
Since h > 0, dividing throughout by h we get
x+h

rin
R
f (u) ≤ h1 f (t)dt ≤ f (v). (2)
x

g.n
(2) can be proved in a similar way for h < 0.
Now, allowing h → 0 and since u and v lie in [x, x + h], we have u → x
and v → x. Since f is continuous at x,we have

∴ lim f (u) = lim f (u) = f (x)


h→0 u→x
et
and lim f (v) = lim f (v) = f (x).
h→0 v→x

By (2) we have
Zx+h
1
lim f (u) ≤ lim f (t)dt ≤ lim f (v)
h→0 h→0 h h→0
x

Download From : www.EasyEngineering.net


Download From : www.EasyEngineering.net

Integral Calculus 155

g(x + h) − g(x)
f (x) ≤ lim ≤ f (x) [by (1)]
h→0 h
i.e., f (x) ≤ g′ (x) ≤ f (x).

By Squeeze Theorem we get

g′ (x) = f (x). (3)

Since every differentiable function is continuous,


we obtain g is also continuous on (a, b).

ww If x = a or x = b, from (3) we get the one sided limits at a and b.


This shows that g is continuous on [a, b].

w.E
Rx √
Example 3.2. Find the derivative of the function g(x) = 1 + t2 dt.
√ 0
Solution. Since f (t) = 1 + t2 is continuous on [0, x], by part 1 of the Fundamental

asy
theorem of calculus we get g′ (x) = 1 + x2

En
 4 
d Rx 
Example 3.3. Find  sec tdt.
dx 1
Solution. Let u = x 4

 x4 gin  u 

ee
Z  Z
d  d 


∴  sec tdt =  sec tdt
dx   dx 

rin

1 1
 u 
Z
d   du

=
1
du
 sec tdt ·
dx
g.n
[By Chain rule]

= sec u · 4x3

= sec x4 · 4x3 .
[by FTC 1]
et
Second Fundamental theorem of calculus
Rb
Statement.If f is continuous on [a, b], then f (x)dx = F(b) − F(a) where F is any
a
antiderivative of f . [i.e, a function such that F ′ = f ].
Rx
Proof. Let g(x) = f (t)dt.
a
By the First Fundamental theorem of calculus we have g′ (x) = f (x). i.e,

Download From : www.EasyEngineering.net


Download From : www.EasyEngineering.net

156 Engineering Mathematics - I

g is an antiderivative of f .
If F is any other antiderivative of f on [a, b], we know that F and g differ
by a constant.
∴ F(x) = g(x) + c for a < x < b.
Since both F and g are continuous on [a, b] the above relation holds when x = a and
x = b.
i.e lim F(x) = F(a) & lim F(x) = F(b).
x→a+ x→b−
Also lim g(x) = g(a) & lim g(x) = g(b).

ww
x→a+ x→b−
Hence we obtain F(x) = g(x) + c for all x in [a, b] (1)
Ra
When x = a, we have g(a) = f (t)dt = 0.

w.E Using (1) for x = b and x = a we get


a

asy
F(b) − F(a) = [g(b) + c] − [g(a) + c]

= g(b) − g(a)

En = g(b) =
Zb
f (t)dt.

gin a
Result. The First and Second Fundamental Theorem of Calculus are combined

Statement of Fundamental Theorem of Calculus.


ee
together is called the Fundamental Theorem of Calculus.

rin
Suppose f is continuous on [a, b]
Rx
(i) If g(x) = f (t)dt, then g′ (x) = f (x).
g.n
et
a
Rb
(ii) f (x)dx = F(b) − F(a), where F is any antiderivative of f . (i.e., F ′ = f .)
a
Note (1). The Fundamental theorem of calculus says that differentiation and
integration are inverse processors.
Note (2). By part(ii) of the Fundamental theorem of calculus we have
Rb
f (x)dx = F(b) − F(a) where F ′ = f .
a
Rb
i.e F ′ (x)dx = F(b) − F(a).
a

Download From : www.EasyEngineering.net


Download From : www.EasyEngineering.net

Integral Calculus 157

The above result is sometimes called as the Net Change Theorem.


Net Change Theorem
Statement. The integral of a rate of change is the net change.
Rb
i.e F ′ (x)dx = F(b) − F(a).

a ☞
Worked Examples
✍ ✌
R3
Example 3.4. Evaluate (x2 − 2x)dx by using Reimann sum by taking right end
0
points as the sample points. Hence verify it by using fundamental theorem of

wwcalculus.
Solution. Let f (x) = x2 − 2x.

w.E Divide the interval [0, 3] into n sub intevals with equal width
∆x =
3−0 3
= .
n n "
asy
3
n
# "
3 6
# "
The intervals are 0, , , , , , · · ·
n n
6 9
n n
# "
3n − 3 3n
n
,
n
#
.

En
The sample points xi∗ are the right end points of the sub intervals.
3 6 9
∴ xi∗ = , , , · · · .
3n

gin
n n n n
The corresponding f (xi∗ ) = xi∗ 2 − 2xi∗ are calculated and tabulated as
follows.

xi∗ 3
n
ee 6
n
9
n ··· 3n
n rin
f (xi∗ )
By Reimann’s sum we have,
9
n2
− 6
n
36
n2
− 12
n
81
n2
− 18
n ··· 9n2
n2
− 6n
n
g.n
Z3

0
(x2 − 2x)dx = lim
n=∞
n
X

i=1
f (xi∗ )∆x
et
9n2 6n 3
" ! ! ! ! #
9 6 3 36 12 3 81 18 3
= lim − + − + − + · · · + −
n=∞ n2 n n n2 n n n2 n n n2 n n
" 3 2
! 3 2
! 3 2
!
3 3 3 3 3 3
= lim 3
· 1 − 2 · 2 + 3 · 22 − 2 · 4 + 3 · 32 − 2 · 6
n=∞ n n n n n n
3 32
!#
3 2
+ · · · + 3 · n − 2 · 2n
n n

Download From : www.EasyEngineering.net


Download From : www.EasyEngineering.net

158 Engineering Mathematics - I

33  2 2 2 2
 32
= lim 1 + 2 + 3 + · · · + n − lim (2 + 4 + 6 + · · · + 2n)
n=∞ n3 n=∞ n2
27 h n(n + 1)(2n + 1) i 9 h n(n + 1) i
= lim 3 − lim 2 · 2
n=∞ n 6 n=∞ n 2
3 1 1
h n (1 + n )(2 + n ) i h n (1 + 1n ) i 27
2
27
= lim − 9 lim = × 2 − 9 = 0.
6 n=∞ n3 n=∞ n2 6

By the Fundamental Theorem of Calculus we have


Z3 !3
2 x3 x2 27
(x − 2x)dx = −2· = − 9 = 9 − 9 = 0.
3 2 0 3

ww 0

w.E
Example 3.5. Evaluate the integral

R1 √
R1 √
0
1 − x2 dx interms of areas.


Solution.
0
asy
1 − x2 dx represents the area under the curve y = 1 − x2 between
the x−axis and the lines x = 0 and x = 1.

p
En y

gin
Now, y = 1 − x2

y2 = 1 − x2
0 1x
2 2
x + y = 1.
ee
This is a circle with centre at the origin and radius = 1 unit. Since we rin
need the area between x = 0 and x = 1, the required area is the area of the portion
1 π g.n
of the circle that lies in the first quadrant = · π × 12 = .


R1 √
0
π
1 − x2 dx = .
4
4 4
et
R3
Example 3.6. Evaluate the integral (x − 1)dx interms of areas.
0
R3
Solution. (x − 1)dx represents the area under the curve y = (x − 1) between the
0
x−axis and the lines x = 0 and x = 3.

Download From : www.EasyEngineering.net


Download From : www.EasyEngineering.net

Integral Calculus 159

y
The required area = A1 − A2 .

A1 is a triangle with base = 2units A1


x=3

and height = 2units. A2


x
  x=0
Put x = 3 in y = x − 1.

1
∴ Area of A1 = × 2 × 2 = 2.
2

ww
A2 is a triangle with base = 1 unit and height = 1unit.
1 1
∴ Area of A2 = × 1 × 1 = .
2 2

w.E
R3 1 3
(x − 1)dx = 2 − = .
0 2 2

Example 3.7. Prove that


asy
Rb
a
xdx =
b2 − a2
2
by interpretting interms of areas.

Solution.
Rb
En [A.U Nov 2016]
xdx represents the area under the curve y = x between the x−axis
a
and the lines x = a and x = b.
gin
2
1
= b·b− a·a
1
1
2ee
Required area = Area of ∆OCD− Area of ∆OAB
1
= × OD × CD − × OB × AB
y
rin C

=
2 2
b2 a2 b2 − a2
− = g.n A

a
b

et
2 2 2 b−a
Zb 0 a B Db x
b2 − a2
∴ xdx = .
2
a
n
(xi3 + xi sin xi )∆x as an integral on the interval [0, π].
P
Example 3.8. Express lim
n→∞ i=1
Solution. According to Riemann sum,the limit
n Rπ
lim (xi3 + xi sin xi )∆x = (x3 + x sin x)dx.
P
n→∞ i=1
0

Example 3.9. Evaluate the Riemann sum for f (x) = x3 − 6x, taking the sample

Download From : www.EasyEngineering.net


Download From : www.EasyEngineering.net

160 Engineering Mathematics - I

R3
points to be the right end points and a = 0, b = 3, n = 6. Also Evaluate (x3 − 6x)dx.
0
3−0 1
Solution. Divide the interval [0, 3] into 6 subintervals with equal width = .
h i h i h i h i h i h i 6 2
Hence, the intervals are 0, 21 , 21 , 1 , 1, 23 , 23 , 2 , 2, 52 , 25 , 3
1 3 5
The right end points are , 1, , 2, , 3.
2 2 2
The value of f (x) at these points are as follows.

x(k) x3 −6x f (xk ) f (xk )∆x


1 1
-3 - 23 - 23

ww 2
1
3
1
8

27
−6 −5
8

- 45
16
- 52
- 45

w.E 2 8 −9 8 16
2 8 -12 -4 -2
5 125 5 5

asy 2 8 −15 8 16
9
3 27 −18 9 2

En
Reimann Sum =
n
X
f (xk )∆xk

gin =−
k=1
23 5 45
− − −2+
5
+
9

ee =
16 2 16

16
−140 + 77 −63
16 2
−23 − 40 − 45 − 32 + 5 + 72

rin
g.n
= = .
16 16
R3
Let us evaluate (x3 − 6x)dx.

∆x =
3−0 3
n
= .
n
0
Divide the interval [0, 3] into n subintervals with equal width et
h i h i h i h i
Hence the intervals are 0, 3n , n3 , n6 , 6n , n9 , · · · 3n−3 3n
n , n
The sample points xi∗ are the right end points of the sub intervals
∴ xi∗ = n3 , 6n , n9 , · · · 3n
n.
The corresponding f (xi∗ ) = xi3 − 6xi are calculated and tabulated as
follows.

Download From : www.EasyEngineering.net


Download From : www.EasyEngineering.net

Integral Calculus 161

3 6 9 3n
xi∗ n n n ··· n
33 3 63 6 93 9 33 n3 3n
f (xi∗ ) n3
−6· n n3
−6· n n3
−6· n ··· n3
−6· n

By Reimann sum
Z3 n
X
(x3 − 6x)dx = lim f (xi∗ )∆x
n=∞
0 i=1

33 63 33 n3
" ! ! ! #
3 3 6 3 3n 3
= lim −6· + 3 −6· + ··· + −6·
n=∞ n3 n n n n n n3 n n
" 4 2
#
3 3

ww

3 3 3 3
= lim 4 1 + 2 + 3 + · · · + n − 6 · 2 (1 + 2 + 3 + · · · + n)
n=∞ n n
 34 n(n + 1) 2
 !
2
!
3 n(n + 1) 

w.E
= lim  4 −6· 2 
n=∞ n 2 n 2
" 4 2
3 n (n + 1)2 32 n(n + 1)
#
= lim 4 −6· 2

asy
n=∞ n 4 n 2
 !2 !
 4 1 2 1 
 4 n 1 + n 1 + 
n 32 n 

En
 3
= lim  4 · −6· 2 
n=∞  n
 4 n 2 

gin

 !2 !
 81 1 1 
= lim  · 1 + − 27 · 1 + 

=
n=∞ 4

81
4
− 27 =ee n
81 − 108
4
=− .
27
4
n

rin
g.n
Indefinite Integrals.
The Fundamental theorem of
calculus gives the relation between

et
R
antiderivatives and integrals. The notation f (x)dx is traditionally used for an
Rb
antiderivative of f and is called an indefinite integral. Thus f (x)dx = F(x)
a
means F ′ (x) = f (x).
d R
[F(x)] = f (x) then f (x)dx = F(x)
dx Z
d
In other words if [F(x)] = f (x), then f (x)dx = F(x).
dx
d
Also we have [F(x) + c] = f (x) then
R dx
f (x)dx = F(x) + c,where c is called the constant of integration.

Download From : www.EasyEngineering.net


Download From : www.EasyEngineering.net

162 Engineering Mathematics - I

With this notation, we can derive the basic results in integration, using the
basic results in differentiation.
Basic results.
d n
1. We have (x ) = nxn−1 .
dx
d xn+1
!
(or) = xn .
dx n + 1
R xn+1
∴ xn dx = + c.
n+1

ww 2.
d
dx
(log x) = ⇒
1
x
R 1
x
dx = log x + c.

w.E3.
d x
dx
R
(e ) = e x ⇒ e x dx = e x + c.

bx

asy
R
4. b x dx = + c.
log b

En
R
5. sin xdx = − cos x + c.

gin
R
6. cos xdx = sin x + c.
R
7.

8.
R
sec2 xdx = tan x + c.

cosec2 xdx = − cot x + c. ee rin


g.n
R
9. sec x tan xdx = sec x + c.

et
R
10. cosecx cot xdx = −cosecx + c.
R 1
11. √ dx = sin−1 x + c.
1 − x2
R 1
12. dx = tan−1 x + c.
1 + x2
R 1
13. √ dx = sec−1 x + c.
x x2 −1
R
14. sinh xdx = cos hx + c.

Download From : www.EasyEngineering.net


Download From : www.EasyEngineering.net

Integral Calculus 163

R
15. cosh xdx = sin hx + c.
R 1 √
16. √ dx = cos h−1 x + c or log(x + x2 − 1) + c.
x2 − 1
R 1 √
17. √ dx = sin h−1 x + c or log(x + x2 + 1) + c.
+1 x2
Basic properties of indefinite integrals.
If f (x) and g(x) are functions of x and k is a constant, then the following
properties are true.

ww
R R R
(i) [ f (x) + g(x)]dx = f (x)dx + g(x)dx + c.
R R R
(ii) [ f (x) − g(x)]dx = f (x)dx − g(x)dx + c.

w.E
(iii)
R
k f (x)dx = k
R
f (x)dx + c.
✎ ☞

Example 3.10. Evaluate (10x4 − 2 sec2 x)dx. asy


Z
Worked Examples
✍ ✌

Solution.
Z Z Z
En
(10x − 2 sec x)dx = 10 x dx − 2 sec2 xdx + c
4 2 4

Z2 gin
= 10√·
x5
−12!tan x + c = 2x5 − 2 tan x + c.

ee
Example 3.11. Evaluate 2 5
x − 3 x + 2 dx.
x

rin
1

Z2 ! Z2 Z2 Z2
2 √1 2 1
x−2 dx
Solution.
1
x − 3 x + 2 dx =
x
x dx − 3 x 2 dx +
1
 3 2
1 1
g.n
3 2

et
!2
x−1
!
x  x 2 
= − 3  3  +
 
3 1 2
−1 1
1
!2
1 3 3 2h 3 i 1
= [2 − 1 ] − 3 × 2 − 1 −
2
3 3 x 1
1 √ 1
= [8 − 1] − 2[2 2 − 1] − [ − 1]
3 2
7 √ 1
= −4 2+2+
3 2
21 + 12 + 3 √ 36 √ √
= −4 2= − 4 2 = 6 − 4 2.
6 6

Download From : www.EasyEngineering.net


Download From : www.EasyEngineering.net

164 Engineering Mathematics - I

Z3
Example 3.12. Evaluate (x3 − 6x)dx.
0
Z3 Z3 Z3 !3 !3
3 3 x4 x2
Solution. (x − 6x)dx = x dx − 6 xdx = −6
4 0 2 0
0 0 0
1
= [34 − 0] − 3[32 − 0]
4
1 81 81 − 108 27
= × 81 − 27 = − 27 = =− .
4 4 4 4

wwExample 3.13. Evaluate


Z2
3
2x − 6x + 2
3
x +1
!
dx.

w.E
Solution.
Z2
3
2x − 6x + 2
3
0
! Z2
3
Z2
dx = 2 x dx − 6 xdx + 3
Z2
2
1
dx

asy
x +1 x +1
0 0 0 0
!2 !2
x4 x2  2
+ 3 tan−1 x

En
=2· −6
4 0 2 0
0

1  
= [24 − 0] − 3[22 − 0] + 3 tan−1 2 − tan−1 0

gin 2
= 8 − 12 + 3 tan−1 2 = 3 tan−1 2 − 4.

Example 3.14. Evaluate


Zπ/6
cos 2 x
 
2
ee
dx.
rin
0

g.n
Solution.
Zπ/6

0
cos2 x
 
2
dx =
1
2
Zπ/6

0
 π/6
Z
(1 + cos x)dx

Zπ/6


et
1  
=  dx + cos xdx
2 
0 0
1  π/6
  π/6 
= x dx + sin x
2 0 0
1 π  π  1 "π 1# π 1
= − 0 + sin − sin 0 = + = + .
2 6 6 2 6 2 12 4

Download From : www.EasyEngineering.net


Download From : www.EasyEngineering.net

Integral Calculus 165

Z9 √
2x2 + x2 x − 1
Example 3.15. Evaluate dx.
x2
1
Z9 √ Z9 √
2x2 x2 x 2x2 x2 x
!
+ −1 1
Solution. dx = + − 2 dx
x2 x2 x2 x
1 1
Z9  
1
= 2 + x 2 − x−2 dx
1
Z9 Z9 Z9

ww
1
=2 dx + x dx −
2 x−2 dx
1 1 1
 9  x 23 9

w.E
  !9
x−1
= 2 x +  3  −

 

1
2 1
−1 1
!
2 3   1

asy = 2[9 − 1] + 9 2 − 1 +
2
3
8
9
−1

En
= 16 + × 26 −
3 9
52 8 144 + 156 − 8 292
= 16 + − = = .
Z
1 gin 3 9 9 9

ee
Example 3.16. Evaluate 2
dx
sin 2x
Z x cos2
sin x + cos2 x
Z
1
Solution.
sin2 x cos2 x
dx =
sin2 x cos2 x
sin2 x
dx

cos2 x rin
g.n
Z Z
= 2
dx + dx
Z sin x cos Z
2x sin2 x cos2 x
= sec2 xdx + cosec2 xdx

= tan x − cot x + c.
et
3.2 The Substitution Rule
R
Integrals of functions of the form f (ax + b)dx.
This type of integrals can be evaluated by making a substitution ax + b = t.
R
For example consider (2x + 3)2 dx.

Download From : www.EasyEngineering.net


Download From : www.EasyEngineering.net

166 Engineering Mathematics - I

Let t = 2x + 3
dt = 2dx
dt
∴ dx =
2Z
1 t3
Z Z
2 2 dt 1 1
Now, (2x + 3) dx = t · = t2 dt = · + c = · (2x + 3)3 + c.
2 2 2 3 6
Standard results.
Z
1. Evaluate (ax + b)n dx.

ww Solution. Let ax + b = t.
dt
adx = dt ⇒ dx = .

w.E Z
(ax + b)n dx =
Z
tn = ·
a
a
dt 1 tn+1
a n+1
+c=
(ax + b)n+1
a(n + 1)
+ c.

asy
In a similar way, the following results can be easily derived.
Z

En
1 log(ax + b)
2. dx = + c.
ax + b a
eax+b
gin
Z
3. eax+b dx = + c.
a

4.
Z

Z
sin(ax + b)dx = −
a
sin(ax + b)
ee
cos(ax + b)
+ c.

rin
5.
Z
cos(ax + b)dx =
a
tan(ax + b)
+ c.

g.n
6.

7.
Z
sec2 (ax + b)dx =

cosec2 (ax + b)dx = −


a
+ c.

cosec(ax + b)
a
+ c.
et
Z
sec(ax + b)
8. sec(ax + b) tan(ax + b)dx = + c.
a
Z
cos(ax + b)
9. cosec(ax + b) cot(ax + b)dx = − + c.
a
✎ ☞
Worked Examples
✍ ✌

Download From : www.EasyEngineering.net


Download From : www.EasyEngineering.net

Integral Calculus 167

Z
Example 3.17. Evaluate sin2 3xdx.
Z Z
2 1
Solution. sin 3xdx = (1 − cos 6x)dx
2
"Z Z #
1
= dx − cos 6xdx
2
" #
1 sin 6x
= x− +c
2 6
x sin 6x
= − + c.
2 12

wwExample 3.18. Evaluate


Z
cos mx cos nxdx.

w.E
Solution. case(i) when m , n.
Z Z
1

asy
cos mx cos nxdx =

=
1
2
Z
(cos(m + n)x + cos(m − n)x) dx
Z
cos(m + n)xdx + cos(m − n)x dx
!

En 2
1 sin(m + n)x sin(m − n)x
!

gin
= + + c.
2 m+n m−n
case(ii) If m = n
Z
cos mx cos nxdx =

=
Z

Z
ee cos mx cos mxdx

cos2 mxdx rin


=
Z
1
(cos 2mx + 1)dx g.n
=
1
2
"Z

"
2
Z
cos 2mxdx + dx

1 sin 2mx
#
#
et
= +x +c
2 2m
sin 2mx x
= + + c.
4m 2

Z
Example 3.19. Evaluate cos3 2xdx.

Download From : www.EasyEngineering.net


Download From : www.EasyEngineering.net

168 Engineering Mathematics - I

Z Z !
3 1 3
Solution. cos 2xdx = cos 6x + cos 2x dx
4 4
Z Z cos 3θ = 4 cos3 θ − 3 cos θ
1 3
= cos 6xdx + cos 2xdx 1 3
4 4 cos3 θ = cos 3θ + cos θ
1 sin 6x 3 sin 2x 4 4
= · + · +c 3 1 3
4 6 4 2 cos 2x = cos 6x + cos 2x.
sin 6x sin 2x 4 4
= +3 + c.
24 8
Z
Example 3.20. Evaluate sin4 xdx.

ww
Z Z  2
Solution. sin4 xdx = sin2 x dx
Z !2
1 − cos 2x

w.E =

=
1
4
Z
2
2
dx.

(1 + cos 2x − 2 cos 2x)dx


cos2 θ =
1 + cos 2θ

1 +
2
cos 4x

=
1
4asy
Z
1+
1 + cos 4x
2
!
− 2 cos 2x dx
cos2 2x =
2
.

En
Z
1 1 1
= (1 + + cos 4x − 2 cos 2x)dx
4 2 2

=
1
Z
gin
3 1
!
+ cos 4x − 2 cos 2x dx

ee
4 2 2
" Z Z Z #
1 3 1
=
4 2
"
dx +
2
1 3 1 sin 4x
cos 4xdx − 2 cos 2xdx

sin 2x
#
3 rin
sin 4x sin 2x
= +
4 2 2 4
−2
2
+c= x+
8 32

4
g.n
+ c.

et
Z
Example 3.21. Evaluate sin2 4xdx.
Z Z !
2 1 − cos 8x
Solution. sin 4xdx = dx.
2
2 sin2 x = 1 − cos 2x
Z !
1 1
= − cos 8x dx
2 2
"Z Z # 2 sin2 4x = 1 − cos 8x
1
= dx − cos 8xdx 1 − cos 8x
2 sin2 4x =
" # 2
1 sin 8x x sin 8x
= x− +c= − + c.
2 8 2 16

Download From : www.EasyEngineering.net


Download From : www.EasyEngineering.net

Integral Calculus 169

x2
Z
Example 3.22. Evaluate dx.
(ax + b)3
t−b
Solution. Let ax + b = t ⇒ ax = t − b ⇒ x =
a
dt
adx = dt ⇒ dx = .
a
Z t − b 2
x2
Z
a dt
dx =
(ax + b)3 t 3 a
2
Z t2 + b − 2b t
1 a2 a dt

ww =

=
a
1
Z
+
t3
1 b2 1 2b 1
· −
!
dt

w.E =
a
1
a
"Z
t a2 t3
1
t
dt + 2
b2
a
Z
a t2

t−3 dt −
2b
a
Z
t−2 dt
#

asy
=
1
a
"
b
log t + 2 ·
2

a (−2)
t −2

2b
a
×
t−1
(−1)
#
+c

=
1
aEn
"
b2 1 2b 1
log t − 2 · 2 +
2a t a t
#
+c

=
1
a
"
gin
log(ax + b) − 2 ·
b2 1
a (ax + b)2
+
2b 1
a (ax + b)
#
+c

1
ee
= log(ax + b) − 3 ·
a
b2 1
+
2b
2a (ax + b)2 a2 (ax + b)
1

rin
+c

g.n
R

3.3 Evaluation of integrals of the form f (g(x))g (x)dx.

= g(x) du = g′ (x)dx.
Method. Let u Z
Now,
Z
f (g(x))g′ (x)dx =
This can be evaluated by earlier methods.
f (u)du. et
✎ ☞
Worked Examples
✍ ✌
Z p
Example 3.23. Evaluate 2x 1 + x2 dx.
Solution. Let 1 + x2 = u.
2xdx = du.

Download From : www.EasyEngineering.net


Download From : www.EasyEngineering.net

170 Engineering Mathematics - I

3

Z Z Z
p 1 u2 2 3
∴ 2
2x 1 + x dx = udu = u 2 du = 3 + c = (1 + x2 ) 2 + c.
2
3

Z p
Example 3.24. Evaluate x2 1 − 4x3 dx.
Solution. Let u = 1 − 4x3 .
du = −4 × 3x2 dx.
du
x2 dx = −

ww ∴
Z
2
p
12
3
x 1 − 4x dx =
Z
√ −du
u
12
!

w.E
Z
1 1
=− u 2 du
12
3
1 u2

asy =−
12 32
1 2
+c

3 1 3
(1 − 4x3 ) 2 + c = − (1 − 4x2 ) 2 + c.

En=−
12 3 18

Z gin 1
Example 3.25. Evaluate

Solution.
Z
x
1
−x
(1 + e ) (1 + e )
(1 +
dx =
e Z
ee
x )(1 + e−x )
dx.
1
1
(1 + e x )(1 + x )
dx
rin
=
Z
ex
e

(1 + e x )(1 + e x )
dx
g.n
ex
et
Z
= x )2
dx 1 + ex = t
Z (1 + e
dt
= e x dx = dt
t 2
t−1
Z
1 1
= t−2 dt = +c=− +c=− + c.
−1 t 1 + ex

Z
Example 3.26. Evaluate x3 cos(x4 + 2)dx.

Download From : www.EasyEngineering.net


Download From : www.EasyEngineering.net

Integral Calculus 171

Solution. Let x4 + 2 = t.

4x3 dx = dt
dt
x3 dx =
Z Z4
dt
∴ x3 cos(x4 + 2)dx = cos t
4
sin(x4 + 2)
Z
1 1
= cos tdt = sin t + c = + c.
4 4 4

ww
w.E
Example 3.27. Evaluate
Z

x
1 − 4x2
dx.

asy
En
Solution. Let 1 − 4x2 = t.

−8xdx = dt gin
Z
x
xdx = −

dx =
dt
Z8
1
√ −
dt
ee
!
rin
g.n
∴ √
1 − 4x2 t 8
Z
1 1
=− t− 2 dt
8
1 t2 1
1

=− 1 +c=− ×2 t+c=−
8 2 8
1p
4
1 − 4x2 + c. et
x3
Z
Example 3.28. Evaluate √ dx. [A.U. Dec. 2015]
4 + x2

Download From : www.EasyEngineering.net


Download From : www.EasyEngineering.net

172 Engineering Mathematics - I

Solution. Let 4 + x2 = t.

2xdx = dt
dt
xdx =
2
x3 x2
Z Z
∴ √ dx = √ xdx
4 + x2 Z 4 + x2
t − 4 dt
= √
t 2
Z 
1 1 1

= t 2 − 4t− 2 dt

ww 2
 3
1  t 2
=  3 − 4 1  + c
1 
t 2 

w.E
2 2 2
"  3 #
1 2 p
= 4 + x2 2 − 8 4 + x2 + c.
2 3

Example 3.29. Evaluate asy


Z p
1 + x2 x5 dx.

Solution. Let 1 + x2 = t.
En
2xdx = dt
dt gin
ee
xdx =
2

rin
Z p Z p
5
∴ 2
1 + x x dx = 1 + x2 x4 · xdx

g.n
Z p
= 1 + x2 (x2 )2 · xdx

et
Z
dt
= t(t − 1)2 ·
2
√ 2
Z
1
= t(t + 1 − 2t)dt
2
Z 
1 5 1 3

= t 2 + t 2 − 2t 2 dt
2
Z Z Z !
1 5 1 3
= t 2 dt + t 2 dt − 2 t 2 dt
2
 7 3 5 
1  t 2 t 2 t 2 
=  7 + 3 − 2 5  + c.
2 2 2 2

Download From : www.EasyEngineering.net


Download From : www.EasyEngineering.net

Integral Calculus 173

" #
1 2 2 27 2 2 23 4 2 25
= (1 + x ) + (1 + x ) − (1 + x ) + c
2 7 3 5
1 7 1 3 2 5
= (1 + x2 ) 2 + (1 + x2 ) 2 − (1 + x2 ) 2 + c.
7 3 5

ex
Z
Example 3.30. Evaluate x dx.
e −1
2
x
Solution. Let e 2 = t.
dx

ww
x
e2 · = dt
2
x x
ex
Z Z
e2 e2
dx = dx

w.E
∴ x x
e2 − 1 e2 − 1
Z x
e2 x
= x · e 2 dx
Z e2 − 1
=
1+t
tasy 2dt

En
Z !
1
=2 + 1 dt
t

gin
"Z Z #
1
=2 dt + dt + c
t

 x ee
= 2[log t + t] + c
x 
= 2[log e 2 − 1) + e 2 − 1 ] + c.
rin
g.n
et
Z
1
Example 3.31. Evaluate dx.
Z Z 1 + tan x
1 1
Solution. dx = dx
1 + tan x sin x
1+
Z cos x Z
cos x 1 2 cos x
= dx = dx
sin x + cos x 2 sin x + cos x
Z
1 sin x + cos x + cos x − sin x
= dx
2 sin x + cos x
Z !
1 cos x − sin x
= 1+ dx
2 sin x + cos x

Download From : www.EasyEngineering.net


Download From : www.EasyEngineering.net

174 Engineering Mathematics - I

Z Z
1 1 cos x − sin x
= dx + dx
2 Z 2 sin x + cos x
1 1 dt
= x+ t = sin x + cos x
2 2 t
1 1 dt = (cos x − sin x)dx
= x + log(t) + c
2 2
x 1
= + log(sin x + cos x)) + c.
2 2
Z
tan x
Example 3.32. Evaluate dx.
sec x + cos x
sin x

ww
Z Z
tan x cos x
Solution. dx = dx
sec x + cos x 1
+ cos x
Z cos x

w.E =
sin x
Z 1 + cos x
−dt
2
dx

asy=
1 + t2
= − tan−1 (t) + c
t = cos x

dt = − sin xdx

En = − tan−1 (cos x) + c −dt = sin xdx

Example 3.33. Evaluate


Z4 √
gin
2x + 1dx.

Solution. Let 2x + 1 = t.
0

2dx = dt ⇒ dx =
ee dt
. rin
When x = 0, t = 1.
2
g.n
When x = 4, t = 9.


Z4 √
2x + 1dx =
Z9
√ dt
t
2
et
0 1
Z9
1 1
= t 2 dt
2
1
 3 9
1  t 2  1 2 3 1 1 26
=  3  = × [9 2 − 1] = [27 − 1] = × 26 = .
2 2 2 3 3 3 3
1

Download From : www.EasyEngineering.net


Download From : www.EasyEngineering.net

Integral Calculus 175

Z2
1
Example 3.34. Evaluate dx.
(3 − 5x)2
1
Solution. Let 3 − 5x = t.
dt
−5dx = dt ⇒ dx = − .
5
When x = 0, t = −2.
When x = 2, t = −7.
Z2 Z−7 !
1 1 dt
∴ = −
(3 − 5x)2 t2 5

ww 1

1
−2
Z−7
t−2 dt

w.E
=−
5
−2
!−7
1 t−1

asy
=−
5 −1 −2
!−7 ! " #
1 1 1 −1 1 1 −2 + 7 1 5 1

En
= = + = = · = .
5 t −2 5 7 2 5 14 5 14 14

Example 3.35. Evaluate


Ze

ginlog x
x
dx.

Solution. Let t = log x.


1
dt = dx.
1

ee rin
g.n
x
When x = 1, t = log 1 = 0.
When x = e, t = log e = 1.


Ze

1
log x
x
dx =
Z1

0
tdt =
t2
!1

2 0 2
1 1
= −0= .
2 et
Integrals of symmetric functions
Theorem. Suppose f is continuous on [−a, a]
Ra Ra
(i) If f is even, then f (x)dx = 2 f (x)dx.
−a 0
Ra
(ii) If f is odd, then f (x)dx = 0.
−a

Download From : www.EasyEngineering.net


Download From : www.EasyEngineering.net

176 Engineering Mathematics - I

Za Z0 Za
Proof. We have f (x)dx = f (x)dx + f (x)dx
−a −a 0
Z−a Za
=− f (x)dx + f (x)dx.
0 0
In the first integral let us make the substitution
t = −x.

dt = −dx.

ww When x = 0, t = 0.

When x = −a, t = a.

w.E ∴
Za

−a
f (x)dx = −
Za
f (−t)(−dt) +
0
Za
f (x)dx.
0

= asy
Za
f (−t)dt +
Za
f (x)dx.
0
Za
En 0
Za
=
0
(i) If f is even, then f (−x) = f (x). ginf (−x)dx +
0
f (x)dx.


Za

−a
f (x)dx =
Za
f (x)dx +
0
(ii) If f is odd then f (−x) = − f (x)
ee
Za
f (x)dx. = 2
0
Za
f (x)dx.
0
rin

Za
f (x)dx =
Za
− f (x)dx +
Za
f (x)dx = 0. g.n
−a

Example 3.36. Evaluate


0

Z2
(x6 + 1)dx.
0

et
−2

Solution. Let f (x) = x6 + 1.

f (−x) = (−x)6 + 1 = x6 + 1.

∴ f(x) is even.

Download From : www.EasyEngineering.net


Download From : www.EasyEngineering.net

Integral Calculus 177

By the properties of definite integrals


Z2 Z2
∴ f (x)dx = 2 f (x)dx
−2 0
Z2
=2 (x6 + 1)dx
0
 2
Z2 

Z
= 2  x6 dx + dx
 
 

ww
0 0
 x7 2
 !  " 7 #

2  2
= 2   + (x)0  = 2 −0+2−0
7 0 7

w.E =2
"
128
7
#
+2 =2
"
128 + 14
7
#
=
2 × 142 284
7
=
7
.

Example 3.37. Evaluate


asy Z1
tan x
1 + x2 + x4
dx.

Solution. Let f (x) =


tan x
1 + x2 + x4
.
En
−1

f (−x) =
tan(−x)
1 + (−x)2 + (−x)4gin
=
− tan x
1 + x2 + x4
∴ f (x) is odd. ee
= − f (x).

By the properties of definite integrals rin


Z1
f (x)dx = 0 g.n
i.e,
Z1
−1

tan x
1 + x2 + x4
dx = 0.
et
−1

3.4 Integration of rational functions by Partial Fractions.

Techniques of resolving a given fraction into partial fractions.


p(x)
A rational function is generally of the form where p(x) and q(x) are
q(x)

Download From : www.EasyEngineering.net


Download From : www.EasyEngineering.net

178 Engineering Mathematics - I

algebraic expressions.
If the degree of p(x) is less than the degree of q(x) then the fraction
p(x)
is called a proper fraction.
q(x)
p(x)
If the degree of p(x) ≥ degree of q(x), then the given fraction is
q(x)
improper.
p(x)
While resolving a fraction into partial fractions, the following
q(x)
technique can be adopted.

wwCase (i) Let


p(x)
q(x)
be proper and if q(x) is expressed as linear factors.
1+x

w.E
For example, if the given fraction is , then we can express
(x − 1)(x + 2)(x − 3)
the fraction in the following way
1+x A B C
= + + .

p(x) asy
(x − 1)(x + 2)(x − 3) x − 1 x + 2 x − 3

Case (ii) Let


q(x)
En
be proper and q(x) contains repeated linear factors.
x2 + 2x − 3

x2 + 2x − 3 gin
Consider the following example

=
A
(x − 1)3 (2x + 3)
+
B
+
.This must be written as
C
+
D
.

Case (iii) Let


3
(x − 1) (2x + 3)
p(x)
q(x)
x

ee
− 1 (x − 1) 2 (x − 1) 3 2x +3

rin
be proper and q(x) contains nonfactorisable second degree
factors.
Consider the following example
(x −
x2
1)2 (x2
+ 3x + 2
+ 4)(x2 + 9) g.n
. This can be resolved
into partial fractions as follows.
x2 + 3x + 2
2 2 2
=
A
+
B
(x − 1) (x + 4)(x + 9) x − 1 (x − 1) 2
Cx + D Ex + F
+ 2
x +4
+ 2
x +9
. et
Methodology for improper fractions.
Case (i) If degree of p(x) = degree of q(x).
x2 + x + 1
Example 1. Consider .
(x − 1)(x − 2)
This must be resolved as
x2 + x + 1 B C
= A+ + .
(x − 1)(x − 2) x−1 x−2

Download From : www.EasyEngineering.net


Download From : www.EasyEngineering.net

Integral Calculus 179

"
p(x)
= A constant + the usual methods adopted for proper fractions. ]
q(x)
x3 + 2x + 4 B Cx + D
Example 2. 2
= A+ + 2 .
(x − 1)(x + 4) x − 1 x +9
Case (ii) If the degree of p(x) is one more than that of q(x), then
p(x)
= A First degree expression + usual method adopted in proper functions.
q(x)
x4 + x + 4 C Dx + E
Example 1. 2
= Ax + B + + 2 .
(x − 1)(x + 2) x−1 x +2
x3 + 2x + 3 C D
Example 2. 2
= Ax + B + + .
(x + 2) x + 2 (x + 2)2

wwCase (iii) If the degree of p(x) is 2 more than that of q(x), then
p(x)
q(x)
= A second degree expression + usual method adopted in proper functions.

w.E
Example 1.
x7 + 2x + 4
2 2
(x + 2)(x − 1) (x + 1)
= Ax2 + Bx + C +
D
+
E
+
F
x + 2 x − 1 (x − 1) 2
Gx + H
+ 2
This procedure can be extended for any improper fraction depending on the nature
x +1
.

of the fraction
p(x)
q(x)
Standard results.
.
asy
1. Evaluate
R 1
En
gin
dx.
x2
+ a2
Solution. Let x = a tan θ.


Z
1
x2 + a2
dx = a sec2 θdθ.

dx =
Z

a
1 ee
2 tan2 θ + a2
· a sec2 θdθ
rin
g.n
Z
1
= · a sec2 θdθ
a2 (tan2 θ + 1)

et
Z
1 1
= 2
· sec2 dθ
a
Z sec θ
1
= dθ
a
1 1 −1  x 
= θ + c = tan + c.
a a a
R 1
2. Evaluate dx.
x − a2
2

1 1 A B
Solution. Let = = +
x2 −a2 (x − a)(x + a) x − a x + a

Download From : www.EasyEngineering.net


Download From : www.EasyEngineering.net

180 Engineering Mathematics - I

A(x + a) + B(x − a)
=
(x − a)(x + a)
∴ 1 = A(x + a) + B(x − a).

Put x = a.
1
1 = A · 2a ⇒ A = .
2a
Put x = −a.
1
1 = B · (−2a) ⇒ B = − .
2a "
1 −1 #

ww
1 2a 2a 1 1 1
∴ 2 = + = −
x − a2 x − a x + a 2a x − a x + a
Z Z !
1 1 1 1

w.E ∴
x2 − a2
dx =

=
1
2a
1
x−a

2a x − a x + a
Z
dx −
Z
dx

x+a
1
dx
!

=
1 
2a asy 
log(x − a) − log(x + a) + c =
1
2a
log
 x − a
x+a
+ c.

3. Evaluate
R 1
dx. En
a2
− x2
Solution. Let 2
1
a −x 2
=
gin
1
=
A
+
B
(a − x)(a + x) a − x a + x
=
A(a + x) + B(a − x)
(a − x)(a + x)

Put x = a.
1 = A · 2a ⇒ A = .
1
ee
∴ 1 = A(a + x) + B(a − x).

rin
Put x = −a.
2a

1 g.n
1 = B · (2a) ⇒ B = .

∴ 2
1
2a

=
1
2a
+
1
2a
=
1
"

a − x2 a − x a + x 2a a − x a + x
1
+
1
# et
Z Z !
1 1 1 1
∴ dx = + dx
a2 − x2 2a a − x a + x
Z Z !
1 1 1
= dx + dx
2a a−x a+x
" #
1 log(a − x)
= + log(a + x) + c
2a −1

Download From : www.EasyEngineering.net


Download From : www.EasyEngineering.net

Integral Calculus 181

1  
log(a + x) − log(a − x) + c
=
2a
1 a + x
= log + c.
2a a−x
Z
1
Type 1. Evaluation of integrals of the form 2
dx
ax + bx + c
Method. Express the denominator to any one of the forms x2 − a2 , x2 + a2 or a2 − x2
and then apply the correct formula.
✎ ☞
Worked Examples
✍ ✌

ww
Z
1
Example 3.38. Evaluate 2
dx.
Z xZ + 8x − 7
1 1
Solution. dx = dx

w.E 2
x + 8x − 7
=
Z
1
(x + 4)2 − 23
2
(x + 4) − 7 − 16
dx

asy
Z
1
= √ dx
(x + 4) − ( 23)2
2
√ 

En

1  x + 4 − 23 
= √ log  √  + c.
2 23 x + 4 + 23

Example 3.39. Evaluate


Z
1
Z1 + x − xgin 2
dx.

ee
Z
1 1
Solution. 2
dx = dx
1+x−x 1 − (x2 − x)
=
Z

1
1 − (x − 21 )2 − 41
 dx
rin
=
Z
1
1 − (x − 21 )2 + 14
dx g.n
=
Z

Z
5
1
4 − (x − 2 )
1
1 2
dx et
= √ dx
5 2
( 2 ) − (x − 21 )2
 √5 
1
1 
2 + x − 2 
= √ log  √  + c
2 25 2
5
− x + 1
2
√ 
1  5 + 2x − 1 
= √ log  √  + c.
5 5 − 2x + 1

Download From : www.EasyEngineering.net


Download From : www.EasyEngineering.net

182 Engineering Mathematics - I

Z
1
Example 3.40. Evaluate 2
dx.
Z 2x
Z −x+5
1 1
Solution. dx = ! dx
2x2 − x + 5 x 5
2 x2 − +
2 2
Z
1 1
= 2
dx
2 1
!
5 1
x− + −
4 2 16
Z
1 1
= !2 dx
2 1 40 − 1
x− +

ww =
1
Z
4
1
!2
16
dx

w.E
2 1 39
x− +
4 16
Z
1 1
=  √ 2 dx

asy 2
x−
1
4
! 2
 39 
+ 
4


En
 
1 1  x − 1  2 4x − 1
!
−1  4 −1
= √ tan  √  + c = √ tan √ +c
2 39

gin
39 39 39
4 4

ee
Type 2. Evaluation of integrals using partial fractions.

rin
Example 3.41. Evaluate
Z
x+5
x2 + x − 2
dx.
g.n
Solution. Let 2
x+5
x +x−2
=

=
(x +
x+5
2)(x − 1)
=
x
A(x − 1) + B(x + 2)
(x + 2)(x − 1)
A
+ 2
+
x
B
− 1
.
et
∴ x + 5 = A(x − 1) + B(x + 2).
Put x = 1.
6 = 3B ⇒ B = 2.
Put x = −2.
3 = −3A ⇒ A = −1.

Download From : www.EasyEngineering.net


Download From : www.EasyEngineering.net

Integral Calculus 183

x+5 1 2
∴ =− +
Z x2
+ x − 2 Zx + 2 x − 1 !
x+5 1 2
∴ dx = − + dx
x2 + x − 2 x+2 x−1
Z Z
1 1
=− dx + 2 dx = − log(x + 2) + 2 log(x − 1) + c.
x+2 x−1

x2 + 2x − 1
Z
Example 3.42. Evaluate dx.
2x3 + 3x2 − 2x
x2 + 2x − 1 x2 + 2x − 1 x2 + 2x − 1
Solution. = = .

ww Let
2x3 + 3x2 − 2x x(2x2 + 3x − 2) x(x + 2)(2x − 1)
x2 + 2x − 1 A
= +
B
+
C

w.E
x(x + 2)(2x − 1) x x + 2 2x − 1
A(x + 2)(2x − 1) + Bx(2x − 1) + Cx(x + 2)
=
x(x + 2)(2x − 1)

asy
∴ x2 + 2x − 1 = A(x + 2)(2x − 1) + Bx(2x − 1) + Cx(x + 2).
Put x = 0.

En
1
−2A = −1 ⇒ A = .
2
Put x = −2.

gin
B(−2)(−5) = 4 − 4 − 1 ⇒ 10B = −1 ⇒ B =
−1
10
.

Put x = .
1 5 1
1
2
C· · = +1−1⇒
2 2 4
1 1
ee
5C 1
4
= ⇒C= .
1
4
1
5
rin
∴ 3
x2 + 2x − 1
2x + 3x2 − 2x
= 2 − 10 + 5 .
x x + 2 2x − 1 g.n
2

et
Z Z Z Z
x + 2x − 1 1 1 1 1 1 1
∴ dx = dx − dx + dx
2x3 + 3x2 − 2x 2 x 10 x+2 5 2x − 1
1 1 1 log(2x − 1)
= log x − log(x + 2) + · +c
2 10 5 2
1 1 1
= log x − log(x + 2) + · log(2x − 1) + c.
2 10 10

sec2 x
Z
Example 3.43. Evalutate dx. [A.U. Dec. 2015]
tan2 x + 3 tan x + 2
Solution. Let tan x = t.

Download From : www.EasyEngineering.net


Download From : www.EasyEngineering.net

184 Engineering Mathematics - I

sec2 xdx = dt.

sec2 x
Z Z
1
∴ 2
dx = 2
dt
tan x + 3 tan x + 2 Z t + 3t + 2
1
= dt
(t + 1)(t + 2)
Z
(t + 2) − (t + 1)
= dt
(t + 1)(t + 2)
Z !
1 1

ww =

=
Z
1

(t + 1) t + 2
dt −
Z
dt

1
dt + c

w.E (t + 1) t+2
= log(t + 1) − log(t + 2) + c

asy
!
t+1
= log +c
t+2

En
!
tan x + 1
= log + c.
tan x + 2

gin
Example 3.44. Evaluate
3x + 1
Z
3x + 1
ee
(x − 1)2 (x + 3)
A
dx.
B C rin
g.n
Solution. Let 2
= + 2
+
(x − 1) (x + 3) x − 1 (x − 1) x+3
A(x − 1)(x + 3) + B(x + 3) + C(x − 1)2

et
=
(x − 1)2 (x + 3)
∴ 3x + 1 = A(x − 1)(x + 3) + B(x + 3) + C(x − 1)2 .
Put x = 1.
4B = 4 ⇒ B = 1.
Put x = −3.
−1
16C = −8 ⇒ C = .
2
Equating the coefficients of x2 on both sides we get,
1
A + C = 0 ⇒ A = −C = .
2

Download From : www.EasyEngineering.net


Download From : www.EasyEngineering.net

Integral Calculus 185

1 1
3x + 1 1
∴ = 2 + − 2
(x − 1)2 (x + 3) x − 1 (x − 1)2 x + 3
Z Z Z Z
3x + 1 1 1 1 1 1
∴ dx = dx + dx − dx
(x − 1)2 (x + 3) 2 x−1 (x − 1)2 2 x+3
Z
1 1
= log(x − 1) + (x − 1)−2 dx − · log(x + 3) + c
2 2
1  (x − 1)−1
= log(x − 1) − log(x + 3) + +c
2 ! −1
1 x−1 1
= log − + c.

ww
2 x+3 x−1

w.E 2x2 − x + 4
Z
Example 3.45. Evaluate dx.
x3 + 4x
2
2x − x + 4 2x − x + 4 A Bx + C A(x2 + 4) + (Bx + C)x
2
Solution. Let 3 = = + 2 = .
x + 4x
asy
x(x2 + 4) x x +4 x(x2 + 4)
∴ 2x2 − x + 4 = A(x2 + 4) + (Bx + C)x.
Put x = 0.
4A = 4 ⇒ A = 1. En
gin
Equating the coefficients of x2 on both sides we get,

C = −1.
2x2 − x + 4 1
ee
A + B = 2 ⇒ 1 + B = 2 ⇒ B = 1.
Equating the coefficients of x on both sides we get,

rin
g.n
x−1 1 x 1
∴ 3
= + 2 = + 2 − 2 .
x + 4x x x + 4Z x x + 4 Z x + 4
2x2 − x + 4 In the second integral
Z Z
1 x 1
∴ 3
x + 4x
= dx +
x Z
= log x +
1
1 dt
t 2
2
x +Z4

1
dx −
1
x + 22
2
 x 
2
x +4
dx
dx

et
let x2 + 4 = t

2xdx = dt
= log x + log t − tan−1 +c dt
2 2 2 xdx = .
2
1 2 1 −1  x 
= log x + log(x + 4) − tan + c.
2 2 2

Z
2
Example 3.46. Evaluate dx.
(1 − x)(1 + x2 )

Download From : www.EasyEngineering.net


Download From : www.EasyEngineering.net

186 Engineering Mathematics - I

2 A Bx + C
Solution. Let 2
= +
(1 − x)(1 + x ) 1 − x 1 + x2
A(1 + x2 ) + (Bx + C)(1 − x)
= .
(1 − x)(1 + x2 )
∴ 2 = A(1 + x2 ) + (Bx + C)(1 − x).
Put x = 1.
2A = 2 ⇒ A = 1.
Equating the coefficients of x2 on both sides we get,
A − B = 0 ⇒ A = B = 1.

ww Equating the coefficients of x on both sides we get,


B−C = 0

w.E

2
B = C ⇒ C = 1.
=
1
+
x+1
=
1
+
x
+
1


2
2
(1 − x)(1 + x )
dx =
asy
(1Z− x)(1 + x2 ) 1 − x Z1 + x2 1 −Zx 1 + x2 Z
1
1−x
dx +
Z x +1
2
x
dx +
1 + x2

2
1
x +1
dx In the second integral

=
En
log(1 − x)
−1
+
1 dt
t 2
+ tan−1 x + c let 1 + x2 = t

gin 1
= − log(1 − x) + log t + tan−1 x + c
2
1
2xdx = dt
dt
xdx = .

ee
= − log(1 − x) + log(1 + x2 ) + tan−1 x + c.
2

rin
2

g.n
Z
10
Example 3.47. Evaluate using partial fractions dx.
(x − 1)(x2 + 9)

Solution. Let
10
=
A
(x − 1)(x2 + 9) x − 1
+ 2
x +9
∴ 10 = A(x2 + 9) + (Bx + C)(x − 1).
=
(x − 1)(x2 + 9)
. et
[A.U. Dec. 2015]
Bx + C A(x2 + 9) + (Bx + C)(x − 1)

Put x = 1.
10A = 10 ⇒ A = 1.
Equating the coefficients of x2 on both sides we get,
A + B = 0 ⇒ B = −A = −1.
Equating the constants on both sides we get,

Download From : www.EasyEngineering.net


Download From : www.EasyEngineering.net

Integral Calculus 187

9A − C = 10 ⇒ C = 9A − 10 = 9 − 10 = −1.

10 1 −x − 1
∴ 2
= + 2
(x − 1)(x + 9) x − 1 x + 9
1 x 1
= − 2 − 2 .
Z Zx − 1 x + 9Z x + 9 Z
10 1 x 1
∴ 2
dx = dx − 2
dx − 2
dx
(x − 1)(x + 9) x−1 x +9 x +9
Z
1 2x 1 −1  x 
= log(x − 1) − dx − tan +c

ww 2
1
x2 + 9
= log(x − 1) − log(x2 + 9) − tan−1
2
1
3
3
 x
3
3
+c

w.E 1 x
p  
= log(x − 1) − log( x2 + 9) − tan−1 +c
! 3 3
(x − 1) 1  x
− tan−1

asy = log √
x2 + 9 3 3
+ c.

En
4x2 − 3x + 2
Z
Example 3.48. Evaluate
gin
4x2 − 4x + 3
4x2 − 3x + 2
dx.

Solution. The integrand 2


4x − 4x + 3
of the Nr and Dr same. ee
is an improper fraction with the degrees

∴By the method of partial fractions we have


rin
4x2 − 3x + 2
2
4x − 4x + 3
= A+ 2
Bx + C
4x − 4x + 3
A(4x2 − 4x + 3) + Bx + C g.n
[∵4x3 − 4x + 3 is a nonfactorizable

=
4x2 − 4x + 3
∴ 4x2 − 3x + 2 = A(4x2 − 4x + 3) + Bx + C.
Equating the coefficients of x2 .
second degree factor]

et
4A = 4 ⇒ A = 1.
Equating the coefficients of x
−4A + B = −3 ⇒ −4 + B = −3 ⇒ B = 4 − 3 = 1.
Equating the constants
3A + C = 2 ⇒ 3 + C = 2 ⇒ C = 2 − 3 = −1.

Download From : www.EasyEngineering.net


Download From : www.EasyEngineering.net

188 Engineering Mathematics - I

4x2 − 3x + 2 x−1
∴ 2
=1+ 2
4x − 4x + 3 4x − 4x + 3
x−1
=1+
4(x2 − x + 3/4)
x−1 x−1
=1+   =1+  
1 2 3 1
4 (x − 2 ) + 4 − 4 4 (x − 21 )2 + 21

Z  
4x2 − 3x + 2
Z
 x−1 
∴ dx = 1 +    dx
4x2 − 4x + 3 4 (x − 21 )2 + 21
Z Z
1 x−1

ww = dx +

1
Z
4 (x − 12 )2 + 21
x − 12 − 21
dx

w.E = x+

= x+
4
1
Z
(x − 12 )2 + 21
x − 12
dx

dx −
1
Z
1
dx.

asy = x+
4
1
Z
1
(x − 2 )2 + 2
1 dt 1

1
Z
8
1
1
(x − 2 )2 + 12

dx

En
1
4 t+ 2 2 8 (x − 2 )2 + ( √1 )2
1
2

gin
In the second integral

1
!2

ee
Put x −

1
2
!
2 x − dx = dt
2
=t

rin
g.n
!
1 dt
x − dx = .
2 2

= x+
1
8
1 1 1

log(t + ) − 1 tan−1  1  + c
2 8 √
2
! √

2
2

 x − 1 
et
2 −1 2x − 1 √
!
1 1 2 1
= x+ log (x − ) + − tan × 2 +c
8 2 2 8 2
! !
1 2 1 1 1 −1 2x − 1
= x+ log x + − x + − √ tan √ +c
8 4 2 4 2 2
4x2 − 4x + 3
! !
1 1 2x − 1
= x+ log − √ tan−1 √ + c.
8 4 4 2 2

Download From : www.EasyEngineering.net


Download From : www.EasyEngineering.net

Integral Calculus 189

x4 − 2x2 + 4x + 1
Z
Example 3.49. Evaluate dx. [A.U Nov 2016].
x3 − x2 − x + 1
Solution. The integrand is an improper fraction in which the degree of the Nr is
1 more than that of the Dr. Hence, by the method of partial fractions we have
x4 − 2x2 + 4x + 1 x4 − 2x2 + 4x + 1 x4 − 2x2 + 4x + 1
= =
x3 − x2 − x + 1 x2 (x − 1) − (x − 1) (x − 1)(x2 − 1)
x4 − 2x2 + 4x + 1 x4 − 2x2 + 4x + 1
= = .
(x − 1)(x − 1)(x + 1) (x − 1)2 (x + 1)
x4 − 2x2 + 4x + 1 C D E
Let 2
= Ax + B + + 2
+
(x − 1) (x + 1) x − 1 (x − 1) x+1

ww =
(Ax + B)(x − 1)2 (x + 1) + C(x − 1)(x + 1) + D(x + 1) + E(x − 1)2
(x − 1)2 (x + 1)

w.E
∴ x4 − 2x2 + 4x + 1 = (Ax + B)(x − 1)2 (x + 1) + C(x − 1)(x + 1) + D(x + 1) + E(x − 1)2 .
Put x = 1 : 2D = 1 − 2 + 4 + 1 = 4 ⇒ D = 2.
Put x = −1 : 4E = 1 − 2 + 4 + 1 = 4 ⇒ 4E = −4 ⇒ E = −1.

asy
Equating the coefficients of x4 on both sides we get,
A = 1.

En
Put x = 0 : B − C + D + E = 1
B − C + 2 − 1 = 1.
B − C = 0. gin (1)

ee
Put x = 2 : (2A + B) · 3 + 3C + 3D + E = 16 − 8 + 8 + 1
6A + 3B + 3C + 3D + E = 17.
rin
6 + 3B + 3C + 6 − 1 = 17
3B + 3C = 17 − 11 = 6 g.n
B + C = 2.
(1) + (2) ⇒ 2B = 2
(1) ⇒ B = C = 1.
⇒ B = 1. et (2)

Download From : www.EasyEngineering.net


Download From : www.EasyEngineering.net

190 Engineering Mathematics - I

x4 − 2x2 + 4x + 1 1 2 1
∴ 3 2
= x+1+ + 2

x −x −x+1 x − 1 (x − 1) x+1
Z 4
x − 2x2 + 4x + 1
Z Z Z Z Z
1 1 1
∴ 3 2
dx = xdx + dx + dx + 2 2
dx − dx
x −x −x+1 x−1 (x − 1) x+1
x2
Z
= + x + log(x − 1) + 2 (x − 1)−2 dx − log(x + 1) + c
2
x2 (x − 1)−1
= + x + log(x − 1) + 2 − log(x + 1) + c
2 −1
x2
!
x−1 2
= + x + log − + c.
2 x+1 x+1

wwExample 3.50. Evaluate


Z
x3
(x − 1)(x − 2)
dx.

w.E
Solution. The integrand is an improper fraction in which the degree of the Nr is
1 more than that of the Dr.

x3
(x − 1)(x − 2)
asy
∴ By the method of partial fractions we have
= Ax + B +
C
+
x−1 x−2
D

=
En
(Ax + B)(x − 1)(x − 2) + C(x − 2) + D(x − 1)
(x − 1)(x − 2)

gin
∴ x3 = (Ax + B)(x − 1)(x − 2) + C(x − 2) + D(x − 1).

Put x = 2 : D = 8.
ee
Put x = 1 : −C = 1 ⇒ C = −1.

Equating the coefficients of x3 on both sides we get,


rin
A = 1.
Put x = 0 : 2B − 2C − D = 0 g.n
2B + 2 − 8 = 0.
2B − 6 = 0
et
2B = 6 ⇒ B = 3.
x3 1 8
∴ = x+3− +
(x − 1)(x − 2) x−1 x−2
Z
x 3 Z Z Z
1
Z
1
∴ dx = xdx + 3 dx − dx + 8 dx
(x − 1)(x − 2) x−1 x−2
x2
= + 3x − log(x − 1) + 8 log(x − 2) + c.
2

Download From : www.EasyEngineering.net


Download From : www.EasyEngineering.net

Integral Calculus 191

x3 + x
Z
Example 3.51. Evaluate dx.
x−1
Solution. The integrand is an improper fraction in which the degree of the Nr is
2 more than that of the Dr.
∴ By the method of partial fractions we have
x3 + x D (Ax2 + Bx + C)(x − 1) + D
= Ax2 + Bx + C + = .
x−1 x−1 x−1
∴ x3 + x = (Ax2 + Bx + C)(x − 1) + D.
Put x = 1 : D = 2.

ww Equating the coefficients of x3 on both sides we get,


A = 1.

w.E Equating the coefficients of x2 on both sides we get,


−A + B = 0.
B = A = 1.

asy
Put x = 0 : −C + D = 0
C = D = 2.
En

x3 + x
x−1 gin
= x2 + x + 2 +
2
x−1
.


Z 3
x +x
x−1
dx =

=
Z
x2 dx +

x3 x2
+
Z

ee Z
xdx + 2 dx + 2
Z

+ 2x + 2 log(x − 1) + c.
1
x−1
dx

rin
3 2

g.n
3.5 Integration of irrational functions et
Standard Results

Z
1
1. Evaluate √ dx.
a2 − x2
Solution. Let x = a sin θ ⇒ dx = a cos θdθ.

Download From : www.EasyEngineering.net


Download From : www.EasyEngineering.net

192 Engineering Mathematics - I

Z Z
1 1
∴ √ dx = p · a cos θdθ
a2 − x2 a 2 − a2 sin2 θ
Z
1
= q · a cos θdθ
2 2
a (1 − sin θ)
Z
1
= √ · a cos θdθ
a 2 cos2 θ
Z Z
1
= · a cos θdθ = dθ + c = θ + c
a cos θ
Z
1  x
√ dx = sin−1 + c.

ww a2 − x2 a

w.E2. Evaluate
Z
√ dx.
1

asy
+ x2 a2
Solution. Let x = a sin hθ

En
dx = a cos hθdθ.

Z
1 gin Z
1
∴ √
2
a +x 2
dx =

=
Z
ee

p
a + a2 sin h2 θ
2
1
a2 (1 + sin h2 θ)
· a cos hθ dθ

· a cos hθ dθ
rin
=
Z

a
1
2 cos h2 θ
· a cos hθ dθ
g.n
et
Z Z
1
= · a cos hθ dθ = dθ + c = θ + c
a cos hθ
Z
1 −1 x
  p
√ dx = sin h + c = log(x + x2 + a2 ) + c.
a2 − x2 a

Z
1
3. Evaluate √ dx.
x2 − a2

Download From : www.EasyEngineering.net


Download From : www.EasyEngineering.net

Integral Calculus 193

Solution. Let x = a cos hθ

dx = a sin hθdθ.
Z Z
1 1
∴ √ dx = √ · a sin hθ dθ
2
x −a 2 a 2 cos h 2 θ − a2
Z
1
= p · a sin hθ dθ
a2 (cos h2 θ − 1)
Z
1
= √ · a sin hθ dθ
Z a2 sin h2 θ
1
= · a sin hθ dθ

ww =
Z
a sin hθ
dθ + c = θ + c

w.E Z

1
a2 − x2
dx = cos h −1 x
 
a
p
+ c = log(x + x2 − a2 ) + c.

4. Evaluate
Z p
asy
a2 − x2 dx.

En
Solution. Let x = a sin θ

gin
dx = a cos θdθ.

ee
Z p Z p
∴ 2 2
a − x dx = a2 − a2 sin2 θ · a cos θ dθ

=
Z q
a2 (1 − sin2 θ) · a cos θ dθ
rin
=
Z p
a2 cos2 θ · a cos θ dθ
g.n
et
Z
= a2 cos2 θ dθ
Z
1 + cos 2θ
= a2 dθ
2
a2
Z
= (1 + cos θ)dθ
2
a2
" Z Z #
= dθ + cos 2θdθ
2
a2
" #
sin 2θ
= θ+ +c
2 2

Download From : www.EasyEngineering.net


Download From : www.EasyEngineering.net

194 Engineering Mathematics - I

a2
"  x #
−1 1
= sin + · 2 sin θ cos θ + c
2 a 2
a2  x  a2 x p
= sin−1 + · · cos2 θ + c
2 a 2 a
a2 −1 x
  ax p
= sin + 1 − sin2 θ + c
2 a 2
a2   ax r x2
−1 x
= sin + 1− 2 +c
2 a 2 a
a2  x  ax √a2 − x2
= sin−1 + +c
2 a 2 a

ww Z p
a2 − x2 dx =
a2
2
sin−1
 x x p
a
+
2
a2 − x2 + c.

w.E5. Evaluate
Z p
a2 + x2 dx.
Solution. Let x = a sin hθ

asy
dx = a cos hθdθ.

En
Z p Z p
∴ a2 + x2 dx = a2 + a2 sin h2 θ · a cos hθ dθ

gin
Z p
= a2 (1 + sin h2 θ) · a cos hθ dθ

ee
Z p
= a2 cos h2 θ · a cos hθ dθ

rin
Z
= a2 cos h2 θ dθ

g.n
Z
1 + cos h2θ
= a2 dθ
2
a2
et
Z
= (1 + cos h2θ)dθ
2
a2
" Z Z #
= dθ + cos h2θdθ
2
a2
" #
sin h2θ
= θ+ +c
2 2
a2
" #
−1 x
  1
= sin h + · 2 sin hθ cos hθ + c
2 a 2
a 2  x  a2 x p
= sin h−1 + · · cos h2 θ + c
2 a 2 a

Download From : www.EasyEngineering.net


Download From : www.EasyEngineering.net

Integral Calculus 195

a2  x ax p
= sin h−1 + 1 + sin h2 θ + c
2 a 2
a2   ax r x2
−1 x
= sin h + 1+ 2 +c
2 a 2 a
a2  x  ax √a2 + x2
= sin h−1 + +c
2 a 2 a
a2  x x p
= sin h−1 + a2 + x2 + c.
2 a 2
a2
Z p p xp 2
a2 + x2 dx = log(x + x2 + a2 ) + a + x2 + c.
2 2

ww 6. Evaluate
Z p
x2 − a2 dx.

w.E Solution. Let x = a cos hθ


dx = a sin hθdθ.


Z p
2
asy 2
x − a dx =
Z p
a2 cos h2 θ − a2 · a sin hθ dθ

En
Z p
= a2 (sin h2 θ − 1) · a sin hθ dθ

gin
Z p
= a2 sin h2 θ · a sin hθ dθ

ee
Z
2
=a sin h2 θ dθ

=a 2
Z
cos h2θ − 1
2

rin
=
a2
2
Z
(cos h2θ − 1)dθ
g.n
=

=
a2
2
"Z
cos h2θdθ − dθ

a2 sin h2θ
" #
−θ +c
Z #

et
2 2
a2   x 
= sin hθ cos hθ − cos h−1 +c
2 a
a2  p 2
x −1 x
 
= sin h θ · − cos h +c
2 a a
ax p 2 a2 −1 x
 
= cosh θ − 1 − cosh c
2 2 a

Download From : www.EasyEngineering.net


Download From : www.EasyEngineering.net

196 Engineering Mathematics - I

r
ax x2 a2 −1 x
 
= 2
− 1 − cosh c
2 a 2 a

ax x2 − a2 a2  x
= − cosh−1 c
2 a 2 a
xp 2 a2  x
= x − a2 − cosh−1 c.
2 2 a
a2
Z p
xp 2 p
x2 − a2 dx = x − a2 − log(x + x2 − a2 ) + c.
2 2
Z
1

ww3.5.1 Type I. Evaluation of integrals of the form √


ax2 + bx + c
Evaluation procedure. By the method of completing the square technique,
dx

w.E √ √ √ √
express ax2 + bx + c in any one of the forms a2 − x2 , x2 − a2 or a2 + x2 and
apply the correct formula.

asy
✎ ☞
Worked Examples
✍ ✌

En
Z
1
Example 3.52. Evaluate √ dx.
x2 + 16x + 100

Solution.
Z

2
1
dx = gin
Z
p
2
1
dx

ee
x + 16x + 100 (x + 8) + 100 − 64
Z
1
= dx

rin
p
(x + 8)2 + 36
Z
1
= p

(x + 8)2 + 62
p
dx
 g.n
et
= log (x + 8) + (x + 8)2 + 62 + c
 p 
= log (x + 8) + x2 + 16x + 100 + c.
Z
1
Example 3.53. Evaluate √ dx.
9 + 8x − x2
Z Z
1 1
Solution. √ dx = p dx
9 + 8x − x 2 9 − (x2 − 8x)
Z
1
= p dx
9 − (x − 4)2 − 16


Download From : www.EasyEngineering.net


Download From : www.EasyEngineering.net

Integral Calculus 197

Z
1
= p dx
9 − (x − 4)2 + 16
Z
1
= p dx
25 − (x − 4)2
Z
1
= p dx
52 − (x − 4)2
!
−1 x − 4
= sin + c.
5
Z
1

wwExample 3.54. Evaluate

Z

Z
2
9x + 24x
dx.

w.E
1 1
Solution. √ dx = q   dx
9x2 + 24x 9 x2 + 24x 9
Z
1 1
=
asy
3
q
2
x + 3 8x
dx

En
Z
1 1
= q dx
3 4 2

16
x+ 3 − 9

=
1
3
Z
gin q
1
2  2 dx

1
3

ee
= log  x +
x + 43 − 34

4
3
!
+
s
x+
4
3
!2
16



rin
−  + c
9

1
= log 
3

 3x + 4

3
! r 
8 
+ x2 + x + c.
3 g.n
3.5.2 Type II. Evaluation of integrals of the form
Z

ℓx + m
et dx
ax2 + bx + c
d  2 
Evaluation procedure. Assume express ℓx + m = A · ax + bx + c + B. Find
dx
the values of A and B and then substitute for ℓx + m in the integral which will be
evaluated easily.
✎ ☞
Worked Examples
✍ ✌

Download From : www.EasyEngineering.net


Download From : www.EasyEngineering.net

198 Engineering Mathematics - I

Z
3x + 1
Example 3.55. Evaluate √ dx.
2x2 + x + 3
d
Solution. Let 3x + 1 = A (2x2 + x + 3) + B
dx
= A(4x + 1) + B.
Equating the coefficients of x on both sides we get,
3
4A = 3 ⇒ A = .
4
Equating the constants
3
A+B=1⇒ +B=1
4

ww 3 1
B=1− = .
4 4

w.E 3
∴ 3x + 1 = (4x + 1) + .
4
1
4


Z
3x + 1
dx =
3
4
√ asy
Z (4x + 1) + 1
4 dx.

En
2
2x + x + 3 2
Z 2x + x + 3 Z
3 4x + 1 1 1
= √ dx + √ dx.

=
4
3
Z 2x 2+x+3

gin−1
4

(2x2 + x + 3) 2 d(2x2 + x + 3) +
2x 2+x+3
1
Z
1
! dx

ee
s
4 4 x 3
2 x2 + +
2 2

=
1
3 (2x2 + x + 3) 2
+
1

Z
1
rin
dx
1

g.n
s
4 4 2 !2
2 1 3 1
x+ + −
4 2 16
3
4
p
2
= × 2 2x + x + 3 + √
1
4 2
Z
s
x+
1
4
1
!2
+
24 − 1
16
dx
et
Z
3 p 1 1
= 2x2 + x + 3 + √ dx
!2  √ 2
s
2 4 2 1  23 
 
x+ + 
4 4

 s 
!2
3p 2 1  1 1 23 
= 2x + x + 3 + √ log  x + + x+ +  + c
2 4 2 4 4 16

Download From : www.EasyEngineering.net


Download From : www.EasyEngineering.net

Integral Calculus 199

 r 
3p 2 1  4x + 1 2
x 3 
= 2x + x + 3 + √ log  + x + +  + c.
2 4 2 4 2 2
Z
1
Example 3.56. Evaluate √ dx by using trignometric substitution. Hence
a2 − x2
6x + 5
use it in evaluating dx. [A.U Nov 2016]
6 + x − 2x2
Solution. For the first part, refer Result(1). Let us evaluate the second part.
d
Let 6x + 5 = A (6 + x − 2x2 ) + B
dx
= A(1 − 4x) + B.

ww Equating the coefficients of x on both sides we get


6
−4A = 6 ⇒ A = − = − .
3

w.E 4 2
Equating the constants,we get
A+B=5⇒− +B=1
3

asy
2
3 13
B=5+ = .
2 2
3
En
∴ 6x + 5 = − (1 − 4x) + .
2
13
2


Z

6x + 5
6 + x − 2x 2
dx =
Z −3
2

(1
6

+ gin
4x)
x −
+
2x
13

2
2
dx.

=−

=−
3
2
3
Z

Z

ee
1 − 4x
6 + x − 2x 2
−1
dx +
13
2
Z

(6 + x − 2x2 ) 2 d(6 + x − 2x2 ) +


1
6 + x −Z2x2
13
dx.

rin 1
dx

g.n
q
2 2 2(3 + 2x − x2 )
1
3 (6 + x − 2x2 ) 2

et
Z
13 1
=− 1
+ √ q dx
2 2 2 2 3 − (x2 − 2x )
Z
p 13 1
= −3 6 + x − 2x2 + √ q o dx
2 2
n
1 2 1
3 − (x − 4 ) − 16
Z
p 13 1
= −3 6 + x − 2x2 + √ q dx
2 2 3 − (x − 14 )2 + 16 1
Z
p
2
13 1
= −3 6 + x − 2x + √ q dx
2 2 ( 74 )2 − (x − 14 )2

Download From : www.EasyEngineering.net


Download From : www.EasyEngineering.net

200 Engineering Mathematics - I

 x − 14 
 
p
2
13 −1 
= −3 6 + x − 2x + √ sin  7  + c
2 2 4
!
p 13 −1 4x − 1
= −3 6 + x − 2x2 + √ sin + c.
2 2 7
Z
4x − 3
Example 3.57. Evaluate √ dx.
x2 + 2x − 1
d
Solution. Let 4x − 3 = A (x2 + 2x − 1) + B
dx
= A(2x + 2) + B.

ww Equating the coefficients of x on both sides we get,


2A = 4 ⇒ A = 2.

w.E Equating the constants


2A + B = −3 ⇒ 4 + B = −3 ⇒ B = −3 − 4 = −7.


Z

4x − 3
dx =
Z
asy
∴ 4x − 3 = 2(2x + 2) − 7.
2(2x + 2) − 7
√ dx.
2
x + 2x − 1
=2 √
2

En
Z x + 2x − 1
2x + 2
dx − 7
Z

1
dx.
Z x
gin
2 + 2x − 1
−1
= 2 (x2 + 2x − 1) 2 d(x2 + 2x − 1) − 7
x 2 + 2x − 1
Z
1
dx

ee
p
(x + 1)2 − 1 − 1
1
(x2 + 2x − 1) 2

rin
Z
1
=2 1
−7 p dx
2 (x + 1)2 − 2

g.n
Z
p
2
1
= 4 x + 2x − 1 − 7 q  √ 2 dx
2
(x + 1) − 2
p

p
2





p
r


2
= 4 x + 2x − 1 − 7 log  x + 1 + (x + 1) − 2  + c
 et

 √ 2 

= 4 x2 + 2x − 1 − 7 log x + 1 + x2 + 2x − 1 + c.
Z
x
Example 3.58. Evaluate √ dx.
3 − 2x − x2
d
Solution. Let x = A (3 − 2x − x2 ) + B
dx
= A(−2 − 2x) + B.

Download From : www.EasyEngineering.net


Download From : www.EasyEngineering.net

Integral Calculus 201

Equating the coefficients of x on both sides we get,


1
−2A = 1 ⇒ A = − .
2
Equating the constants
−2A + B = 0 ⇒ 1 + B = 0 ⇒ B = −1.

1
∴ x = − (−2 − 2x) − 1.
2
Z Z − 1 (−2 − 2x) − 1
x 2
∴ √ dx = √ dx.

ww 3 − 2x − x 2

=−
1
2
Z 3 − 2x − x

3
−2 − 2x
− 2x −
2

x 2
dx −
Z

3 −
1
2x − x 2
dx.

w.E =−
1
2
Z
−1
(3 − 2x − x2 ) 2 d(3 − 2x − x2 ) −
Z
p
1
3 − (x2 + 2x)
dx

asy
 1 Z
2 2
1 3 − 2x − x 1
=−
2 1
− p dx
3 − (x + 1)2 − 1


En
2
Z
p
2
1
= − 3 − 2x − x − dx
3 − (x + 1)2 + 1
p
gin
= − 3 − 2x − x2 −
Z
p
1
4 − (x + 1)2
dx
p
= − 3 − 2x − x −
p
2

2
ee Z
p
1
22 − (x + 1)2
−1 x + 1
!
dx

rin
g.n
= − 3 − 2x − x − sin + c.
2

3.5.3 Type III. Evaluation of integrals of the form


Z √
ax2 + bx + c dx et
Evaluation procedure. By completing the square form, express ax2 + bx + c as
any of x2 − a2 , a2 + x2 , a2 − x2 and apply the appropriate formula.
✎ ☞
Worked Examples
✍ ✌

R √
Example 3.59. Evaluate x2 − 4x + 6 dx.

Download From : www.EasyEngineering.net


Download From : www.EasyEngineering.net

202 Engineering Mathematics - I

Z p Z p
Solution. x2 − 4x + 6 dx = (x − 2)2 + 6 − 4 dx
Z p
= (x − 2)2 + 2 dx
Z q √
= (x − 2)2 + ( 2)2 dx

!q
x−2
= (x − 2)2 + ( 2)2
2

( 2)2 √
q !
+ 2 2
log (x − 2) + (x − 2) + ( 2) + c
2

ww
!
x−2 p 2  p
2

= x − 4x + 6 + log (x − 2) + x − 4x + 6 + c.
2

w.E
3.5.4 Type IV.Integration by trigonometric substitution.

asy
Certain integrals can be easily evaluated by trigonometric substitution.
There are three cases.

En √

gin
(i) Any integral involving quantities of the form
the substitution x = a sin θ.
a2 − x2 can be evaluated by

ee
(ii) Integrals involving functions of the form
substitution x = a tan θ or x = a sin hθ.

rin
a2 + x2 can be evaluated by the

√ g.n
(iii) Integrals involving functions of the form
substitution x = a sec θ or x = a cos hθ.

✎ ☞
et
x2 − a2 can be evaluated by the

Worked Examples
✍ ✌

Z √
9 − x2
Example 3.60. Evaluate dx.
√ x2 √
Solution. Since 9 − x2 is of the form a2 − x2 , we can evaluate this integral by
the substitution.

Download From : www.EasyEngineering.net


Download From : www.EasyEngineering.net

Integral Calculus 203

x = 3 sin θ. ⇒ dx = 3 cos θdθ.


Z √ Z p
9 − x2 9 − 9 sin2 θ
∴ dx = 3 cos θdθ
x2 9 sin2
θ
q
Z 9(1 − sin2 θ)
= 3 cos θdθ
√ 9 sin2 θ
cos2 θ
Z
= 2
cos θdθ
Z sin θ
cos θ
= cos θdθ
sin2 θ

ww =
Z
cos2 θ
2
Z sin θ

w.E =
Z
cot2 θdθ

= (cosec2 θ − 1)dθ

asy=
Z
cosec2 θdθ − dθ
Z

En
= − cot θ − θ + c = −
cos θ
−θ+c

=− gin

cos2 θ
− sin
sin θ
−1 x
 
+c

ee
sin θ 3
p
1 − sin2 θ −1 x
 
=−
r
sin θ
x2
− sin
3
+c

rin
=−
1−
x
9
− sin−1 x
 
3
+c
g.n
=−
x
s3
3 9 − x2
9
− sin−1
 x
3
+c
et
1p  x
=− 9 − x2 − sin−1 + c.
x 3
R 1
Example 3.61. Evaluate √ dx.
x2 x2 + 4 √
Solution. Since the integral involves one factor of the form x2 + 4,we can
evaluate this integral by the substitution

Download From : www.EasyEngineering.net


Download From : www.EasyEngineering.net

204 Engineering Mathematics - I

x = 2 tan θ ⇒ dx = 2 sec2 θdθ.


Z Z
1 1
∴ √ dx = √ · 2 sec2 θdθ
x2 x2 +4 4 tan2 θ 2
4 tan θ + 4
sec2 θ
Z
1
=
2
p  dθ
tan2 θ 4 tan2 θ + 1
sec2 θ
Z
1
= √ dθ
4 tan2 θ sec2 θ
sec2 θ
Z
1
= dθ
4 tan2 θ sec θ

ww =
1
4
Z
sec θ
tan2 θ

w.E cos2 θ
Z
1 1
= · dθ
4 cos θ sin2 θ
Z
1 cos θ 1

asy
= · dθ
4 sin θ sin θ
Z
1
= cosecθ · cot θ dθ

En 4
1
= − cosecθ + c

gin
=−
4
1p
4
cosec2 θ + c

=−

=−
1
ee
1p
4r
1 + cot2 θ + c

4
1+ 2 +c
x
tan θ = , cot θ = 2/x rin
=−
4
s
1 x2 + 4
x

c −

x2 + 4
+ c.
2

g.n
et
+ =
4 x2 4x

R x
Example 3.62. Evaluate dx. √
+4 x2 √
Solution. Since the integrand involves one factor of the form x2 + 4, we can
evaluate this integral by the substitution
x = 2 tan θ ⇒ dx = 2 sec2 θdθ.
Z Z
x 2 tan θ
∴ √ dx = √ · 2 sec2 θdθ
x2 + 4 4 tan2 θ + 4

Download From : www.EasyEngineering.net


Download From : www.EasyEngineering.net

Integral Calculus 205

tan θ sec2 θ
Z
=4 p dθ
4(tan2 θ + 1)
tan θ sec2 θ
Z
=4 √ dθ
2 sec2 θ
tan θ sec2 θ
Z
=2 dθ
sec θ
Z
= 2 tan θ · sec θ dθ

= 2 sec θ + c

ww
p
= 2 sec2 θ + c
p
= 2 1 + tan2 θ + c

w.E r
=2 1+

x2
4
+c

asy =2
4 + x2
2
p
+ c = 4 + x2 + c.

En
Note. This, integral can be evaluated easily by the substitution 4 + x2 = t.

gin
x3 + 1
Z
Example 3.63. Evaluate √ dx.
1 − x2 √

make the substitution


x = sin θ [a = 1]
ee
Solution. Since the integrand contains one factor of the form a2 − x2 , let us

rin
dx = cos θdθ
g.n
x3 + 1 sin3 θ + 1
et
Z Z
√ dx = p cos θdθ
1 − x2 1 − sin2 θ
sin3 θ + 1
Z
= √ · cos θdθ
cos2 θ
sin3 θ + 1
Z
= · cos θdθ
cos θ
Z
= (sin3 θ + 1) dθ
Z Z
= sin3 θ dθ + dθ

Download From : www.EasyEngineering.net


Download From : www.EasyEngineering.net

206 Engineering Mathematics - I

Z !
3 1
= sin θ − sin 3θ dθ + θ
4 4
Z Z
3 1
= sin θdθ − sin 3θdθ + θ
4 4
!
3 1 − cos 3θ
= (− cos θ) − +θ+c
4 4 3
3p 2 1
=− cos θ + (4 cos3 θ − 3 cos θ) + sin−1 x + c
4 12
3p 1 3 1p 2
=− 1 − sin2 θ + (cos2 θ) 2 − cos θ + sin−1 x + c
4 3 4
3p 1 1p

ww
3
=− 1 − x2 + (1 − sin2 θ) 2 − 1 − sin2 θ + sin−1 x + c
4 3 4
3p 1 3 1p
=− 1 − x2 + (1 − x2 ) 2 − 1 − x2 + sin x +c

w.E 1
4
3
p
3 4
= (1 − x2 ) 2 − 1 − x2 + sin−1 x + c.
3

Example 3.64. Evaluate


Z

1
asy
x3 x2 − 9
dx.

En

Solution. Since the integrand contains one factor of the form a2 − x2 , we can
make the substitution
x = 3 sec θ [∴ a = 3]
gin
dx = 3 sec θ · tan θdθ
Z

1
dx =
Z ee √
1
rin
· 3 sec θ tan θdθ

g.n
x3 x2 −9 Z33 sec3 θ · 9 sec2θ−9
1 1
= p sec θ tan θdθ
9 sec3 θ · 9(sec2 θ − 1)
=

=
1
9
1
Z

3
1

Z sec θ × 3 tan θ
1
2
sec θ tan θdθ

sec θ tan θdθ


et
27 sec 3 θ tan θ
Z
1 1
= 2θ

27 sec
Z
1
= cos2 θdθ
27
Z
1 1 + cos 2θ
= dθ
27 2

Download From : www.EasyEngineering.net


Download From : www.EasyEngineering.net

Integral Calculus 207

"Z Z #
1
= dθ + cos 2θdθ
54
" #
1 sin 2θ
= θ+ +c
54 2
1 1
= θ+ sin θ cos θ + c
54 54
1  x 1 p 1
= sec−1 + sin2 θ · +c
54 3 54 sec θ
1  x  1 p 3
= sec−1 + 1 − cos2 θ · + c
54 3 54 x
r

ww 1 x 1 9 3
 
= sec−1 + 1− 2 · +c
54 3 54 x x

1  x  1 x −9 2

w.E =

Z
54
sec−1
3
+
18 x2
+ c.

asy
1
Example 3.65. Evaluate 3
dx.
(a2 + x2 ) 2 √
Solution. Since the integrand contains a factor of the form a2 + x2 , we can make
the substitution
En
x = a tan θ
gin
⇒ dx = a sec2 θdθ.

ee
Z Z
1 1
∴ 3
dx = 3
· a sec2 θdθ

rin
(a2 + x2 ) 2 (a2 + a2 tan2 θ) 2
Z
1
= · a sec2 θdθ

g.n
3
(a2 (1 + tan2 θ)) 2
Z
1
= · a sec2 θdθ

et
3
(a2 sec2 θ) 2
Z
1
= · a sec2 θdθ
Za3 sec3 θ
1 1
= 2

a Z sec θ
1
= cos θ dθ
a2
1
= sin θ + c
a2
1 tan θ
= +c
a2 sec θ

Download From : www.EasyEngineering.net


Download From : www.EasyEngineering.net

208 Engineering Mathematics - I

1 x 1
= 2
· √ +c
a a sec2 θ
x 1
= 3 √ +c
a 1 + tan2 θ
x 1 x a x
= 3 q +c= 3 √ +c= √ + c.
a 2 a a2 + x2 a2 a2 + x2
1 + ax2

3.6 Integration by parts

ww Every differentiation rule has a corresponding integration rule. The rule


that correspond to the product for differentiation is called the integration by parts.

w.E From differential calculus we have


d
dx
dv
(uv) = u + v
dx
du
dx

Z
d
(uv)dx =
asy
Integrating both sides with respect to x
Z
dv
Z
du
u dx + v dx
dx
uv =
Z
En
dx
Z
udv + vdu
dx

Z Z
gin
udv = uv − vdu.

Priority rules for choosing u. ee rin


1. Inverse functions.

2. logarithmic functions. g.n


3. Polynomials in x. et
4. Any other function other than the above three.

Download From : www.EasyEngineering.net


Download From : www.EasyEngineering.net

Integral Calculus 209

✎ ☞
Worked Examples
✍ ✌
Z R1
Example 3.66. Evaluate tan−1 xdx and hence deduce the value of tan−1 xdx
0
[A.U.Nov 2016]
Solution. Let u = tan−1 x Z dv = Z
dx
1
du = dx dv = dx ⇒ v = x.
1 + x2
Applying
Z integration
Z by parts
tan−1 dx = udv

ww Z
= uv − vdu

w.E
Z
−1 1
= (tan x) · x − x· dx In the second integral
Z 1 + x2
1 dt
= x · tan−1 x − put 1 + x2 = t

asy
1
t 2
= x · tan−1 x − log t + c 2xdx = dt

En
2 dt
−1 1 2 xdx = .
= x · tan x − log(1 + x ) + c. 2
2

Z1
gin
 1 1 h i1
Now,
0
ee
tan−1 x dx = x tan−1 x − log(1 + x2 )
0 2
1
0

= tan−1 1 − 0 − (log 2 − log 1)


2 rin
π 1
= − log 2.
4 2 g.n
Example 3.67. Evaluate

Solution. Let u = sin−1 x


Z
sin−1 xdx.

dv = dx
et
Z Z
1
du = √ dx dv = dx ⇒ v = x.
1 − x2
Applying
Z integration
Z by parts
−1
sin x dx = u dv
Z
= uv − v du

Download From : www.EasyEngineering.net


Download From : www.EasyEngineering.net

210 Engineering Mathematics - I

Z
1
= (x sin−1 x) − x· √ dx
Z 1 −!x2 In the second integral
1 −dt
= x · sin−1 x − √ put 1 − x2 = t
2
Z t
1 1
= x · sin−1 x + (t)− 2 + c − 2xdx = dt
2
 1 dt
−1 1  t 2  xdx = − .
= x · sin x +  1  + c 2
2 2
Z p
∴ sin−1 x dx = (x sin−1 x) + 1 − x2 + c.

wwExample 3.68. Evaluate


Z
x sin xdx.

w.E
Solution. Let u = x

du = dx
Z
dv = sin xdx

dv =
Z
sin xdx ⇒ v = − cos x.
Applying
Z
x sin xdx =
Z
asy
integration by parts
Z
udv = uv − vdu

En Z
= −x · cos x − − cos xdx = −x cos x + sin x + c.

Example 3.69. Evaluate


Z
gin
xe x dx.

Solution. Let u = x

du = dx
ee
Z
dv = e x dx

dv =
Z
e x dx ⇒ v = e x . rin
Applying
Z
x
integration
Z byZparts
g.n
et
xe dx = udv = uv − vdu
Z
= x · e − e x dx
x

= xe x − e x + c = e x (x − 1) + c.
Z
Example 3.70. Evaluate log xdx.

Solution. Let u = log x dv = dx


Z Z
1
du = dx dv = dx ⇒ v = x.
x

Download From : www.EasyEngineering.net


Download From : www.EasyEngineering.net

Integral Calculus 211

Applying
Z integration
Z by parts
Z
log xdx = udv = uv − vdu
Z
1
= x · log x − x · dx
x
Z
= x log x − dx + c = x log x − x + c = x(log x − 1) + c.
Z
log x 2 dx.

Example 3.71. Evaluate [A.U. Dec. 2015]

Solution. Let u = log x 2



dv = dx

ww Applying
1
du = 2(log x) dx
x
integration by parts
Z
dv =
Z
dx ⇒ v = x.

w.E Z
2
log x dx =
Z Z
udv = uv − vdu
Z
1

asy
2
= log x · x − x · 2 log x dx
x
Z
2
= x log x − log xdx

En
= x log x 2 − 2x(log x − 1) + c. [From the provious problem]


Example 3.72. Evaluate


Z
gin
xn log xdx.

Solution. Let u = log x


1
du = dx
ee
Z
dv = xn dx

dv =
Z
xn dx ⇒ v =
xn+1
. rin
x
Applying integration by parts
n+1

g.n
et
Z Z Z
n
x log xdx = udv = uv − vdu

xn+1
Z n+1
x 1
= · log x − · dx
n+1 n+1 x
xn+1
Z
1
= log x − xn dx + c
n+1 n+1
xn+1 1 xn+1
= log x − · +c
n+1 n + 1 !n + 1
xn+1 1
= log x − + c.
n+1 n+1

Download From : www.EasyEngineering.net


Download From : www.EasyEngineering.net

212 Engineering Mathematics - I

Z
Example 3.73. Evaluate x2 e x dx.

Solution. Let u = x2 dv = e x dx
Z Z
du = 2xdx dv = e x dx ⇒ v = e x .
Applying
Z integration
Z by Zparts
2 x
x e dx = udv = uv − vdu
Z
= x2 · e x − e x · 2x dx
Z

ww = x2 e x − 2 xe x dx

= x2 e x − 2 xe x − e x + c
 
[Refer Example 3.79]

w.E = x2 e x − 2xe x + 2e x + c = e x (x2 − 2x + 2) + c


Z
Example 3.74. Evaluate

Solution. Let u = tan−1 x asy


x2 tan−1 xdx.

dv = x2 dx

du =
1
dx EnZ
dv =
Z
x 2
dx ⇒ v =
x3
.
Z
1 + x2
Applying
Z integration
x2 tan−1 xdx =
gin
Z by parts
3

ee
udv = uv − vdu

x3
Z 3

rin
−1 x 1
= tan x − · dx
3 3 1 + x2
x3 x3
Z
1
=
3
x3
tan−1 x −
3
1
Z
1 + x2
x2
dx Let 1 + x2 = t
g.n
=

=
3
x3
3
tan−1 x −

−1
tan x −
3
1
3
Z
1 + x2
t − 1 dt
t
·
· xdx

2
2xdx = dt
dt
xdx = .
2
et
x3
Z !
−1 1 1
= tan x − 1 − dt
3 6 t
x 3 1
Z
1
Z
1
−1
= tan x − dt + dt
3 6 6 t
x3 1 1 x3 1 + x2 1
= tan−1 x − · t + · log t + c = tan−1 x − + log(1 + x2 ) + c.
3 6 6 3 6 6

Download From : www.EasyEngineering.net


Download From : www.EasyEngineering.net

Integral Calculus 213

+ x + x2
Z
−1 x1
Example 3.75. Evaluate etan dx. [A.U. Dec. 2015]
1 + x2
Solution. Let u = tan−1 x or x = tan udx
1
du = dx dx = sec2 u du.
1 + x2
Applying integration by parts

+ x + x2 1 + x2
Z Z !
tan−1 x 1 tan−1 x x
e dx = e + dx
1 + x2 1 + x2 1 + x2
Z
−1
 x 
= etan x 1 + dx

ww =
Z
−1
1Z+ x2
etan x dx + etan x
−1
 x 
1 + x 2
dx

w.E
Z Z
= eu sec2 u du + eu tan u du
Z Z
eu sec2 u du + tan u d eu

asy

=
Z Z
= eu sec2 u du + eu tan u − eu sec2 u du

En = eu tan u + c = xetan
−1 x
+ c.

gin
3.7 Trigonometric Integrals

Theorem 1.Prove the reduction formula


ee rin
R

π
1
sinn xdx = − cos x sinn−1 x +
n π
n−1R
n
sinn−2 xdx and deduce that
π g.n
et
R2 n n − 1 R2 n−2 R2 n
sin dx = sin xdx. Find also the general value of sin xdx. Hence
0 n 0 0
π π
R2 R2
evaluate sin xdx and sin8 xdx.
7
[A.U.Dec 2016]
0 Z 0 Z
n
Proof. Write sin xdx = sinn−1 x sin xdx.

Choose u = sinn−1 x dv = sin xdx


Z Z
du = (n − 1) sinn−2 x · cos xdx dv = sin xdx ⇒ v = − cos x.
Applying integration by parts

Download From : www.EasyEngineering.net


Download From : www.EasyEngineering.net

214 Engineering Mathematics - I

Z Z Z
sinn xdx = udv = uv − vdu
Z
= − sinn−1 x · cos x − − cos x · (n − 1) sinn−2 x cos xdx
Z
= − sinn−1 x · cos x + (n − 1) sinn−2 x cos2 xdx
Z
= − sinn−1 x · cos x + (n − 1) sinn−2 x(1 − sin2 x)dx
Z Z
= − sinn−1 x · cos x + (n − 1) sinn−2 xdx − (n − 1) sinn xdx

ww
Z Z Z
sinn xdx + (n − 1) sinn xdx = − sinn−1 x cos x + (n − 1) sinn−2 xdx
Z Z
sinn xdx = − sinn−1 x cos x + (n − 1)
sinn−2 xdx

w.E Z
sinn xdx =
n
−1 n−1
n
sin x cos x +
(n − 1)
n
Z
sinn−2 xdx

π
asy
Taking limits from 0 to everywhere we get
2
Z
π
2

sinn xdx =
−1 n−1
En
sin x cos x
! π2
n−1
Z2
π

sinn−2 xdx.

gin
+
n 0 n
0 0
π π

ee
Z 2 Z2
n−1 n−1
=0+ sinn−2 xdx = sinn−2 xdx.

rin
n n
0 0

Applying the same procedure again for the integral on the RHS we get
Z2
π

n−3
Z2
π

g.n
0


Z
sinn−2 xdx =

π
2

sinn xdx =
n−2

n−1 n−3
·
0
sinn−4 xdx

Z2
π

sinn−4 xdx
et
n n−2
0 0

Repeating this process n times we get


π
Z 2
n−1 n−3 n−5
sinn xdx = · · · · · · · · (ultimate integral).
n n−2 n−4
0

Download From : www.EasyEngineering.net


Download From : www.EasyEngineering.net

Integral Calculus 215

π π
R2 R2 π
If n is even, the ultimate integral is sinn−n xdx = dx = [x]02 = π2 .
0 0
π
2
R  π
If n is odd, the ultimate integral is sin xdx = − cos x 2 = −[cos π2 − cos 0]
0
0
= −[0 − 1] = 1.
π
R2
∴ The general value of sinn xdx is
π 0 n − 1 n − 3 n − 5 2
Z2 

 · · , · · · · 1, if n is odd
sinn xdx =  n n−2 n−4 3


 n−1 n−3 n−5 1 π
if n is even.

ww
· · ,··· · ,



0 n n−2 n−4 2 2
π

w.E
Z2
7−1 7−3 7−5 6 4 2 16
Now, sin7 xdx = · · ·1= · · ·1= .
7 7−2 7−4 7 5 3 35
0

asy
π
Z2
7 5 3 1 π 35π
Also, sin8 xdx = · · · · = .
8 6 4 2 2 256
0

En
Theorem 2.Prove the reduction formula
R
n 1 n−1
cos xdx = cos x sin x +
n
n−1R
n
n−2
gin π
R2
cos xdx. Hence reduce the value of cosn dx.

Proof. Write
Z
n
cos xdx =
Z
n−1
cos x cos xdx.

Choose u = cosn−1 x
ee 0

dv = cos xdx
rin
du = (n − 1) cosn−2 x · (− sin x)dx
Z
dv =
Z

g.n
cos xdx ⇒ v = sin x.
Applying
Z
n
integration
cos xdx =
Z by parts
udv = uv − vdu
Z

Z
= cos x · sin x − sin x · (n − 1) cosn−2 x(− sin x)dx
n−1
et
Z
= cosn−1 x · sin x + (n − 1) cosn−2 x sin2 xdx
Z
= cosn−1 x · sin x + (n − 1) cosn−2 x(1 − cos2 x)dx
Z Z
= cosn−1 x · sin x + (n − 1) cosn−2 xdx − (n − 1) cosn xdx

Download From : www.EasyEngineering.net


Download From : www.EasyEngineering.net

216 Engineering Mathematics - I

Z Z Z
cosn xdx + (n − 1) cosn xdx = cosn−1 x sin x + (n − 1) cosn−2 xdx
Z Z
n cosn xdx = cosn−1 x sin x + (n − 1)
cosn−2 xdx
Z Z
1 (n − 1)
cosn xdx = cosn−1 x sin x + cosn−2 xdx.
R n R n
If n is odd, the ultimate integral is cosn−(n−1) xdx = cos xdx = sin x.
R R
If n is even, the ultimate integral is cosn−n xdx = dx = x.
π
Now,applying limit from 0 to we obtain
π
2 π π

ww
Z2 ! π2 Z2 Z2
1 n − 1 n − 1
cosn xdx = · cosn−1 x sin x + cosn−2 xdx = cosn−2 xdx.
n 0 n n
0 0 0

w.E By repeatedly applying this process we obtain


π
R2
cosn xdx =
n−1 n−3 n−5
n
· ·
n−2 n−4
· · · · · · (ultimate integral)

asy
0
 π π π
If n is even, the ultimate integral is x 2 = − 0 = .
π
0 2 2

En
R2  π
If n is odd, the ultimate integral is cos xdx = sin x 2 = 1.
0
 n − 1 0n − 3 n − 5

gin
π
Z2 2
· · , · · · · 1, if n is odd



cosn xdx =  n n−2 n−4 3


∴ n−1 n−3 n−5 1 π

ee

· · , · · · · , if n is even.



0 n n−2 n−4 2 2
Result. By looking at the above two theorems, we obtain the following result
If n is odd (say n = 2k + 1),then
π π
rin
g.n
R2 2k+1 R2 2.4.6....2k
sin xdx = cos2k+1 xdx = .
0 0 3.5.7...(2k + 1)

et
If n is even (say n = 2k),then
π π
R2 2k R2 1.3.5....(2k − 1) π
sin xdx = cos2k xdx = · .
0 0 2.4.6.....(2k) 2
For example.
π π π
R2 7 R2 R2
sin xdx = cos dx = cos2.3+1 xdx
7 [n = 3]
0 0 0
2.4.6 16
= .
=
π π
3.5.7
π
35
R2 8 R2 R2
sin xdx = cos8 dx = cos2.4 xdx [n = 4]
0 0 0

Download From : www.EasyEngineering.net


Download From : www.EasyEngineering.net

Integral Calculus 217

1.3.5.7 π 35π
= × = .
2.4.6.8 Z 2 256
Evaluation of integrals of the form sinm x · cosn xdx
Evaluation procedure

(i) If the power of cosine is odd (say n = 2k + 1)then write


Z Z
m n
sin x cos xdx = sinm x · cos2k+1 xdx
Z
= sinm x · cos2k x cos xdx

ww =
Z
sinm x · (cos2 x)k cos xdx

w.E
Z
= sinm x · (1 − sin2 x)k cos xdx

Now make the substitution t = sin x, and hence the integral is evaluated
easily.
asy
(ii) If
En
Z the power of sineZis odd (say n = 2k + 1)then write
sinm x cosn xdx = sin2k+1 x · cosn x cos xdx

=
Z
gin
sin2k x · sin x cosn xdx
Z
ee
= (1 − cos2 x)k · cosn x sin xdx.

rin
Now make the substitution t = cos x, and hence the integral is evaluated
easily.
g.n
sin2 x =

cos2 x =
(1 − cos 2x)

(1 +
2
cos 2x)
et
(iii) If the powers of both sine and cosine are even, then use the relations

2
so that the resulting integrals can be evaluated easily.

(iv) If the powers of both sine and cosine are odd, then apply either case(i) or
case(ii).
✎ ☞
Worked Examples
✍ ✌

Download From : www.EasyEngineering.net


Download From : www.EasyEngineering.net

218 Engineering Mathematics - I

Z
Example 3.76. Evaluate sin5 x cos2 xdx.
Solution. Here the power of sine is odd.
R R
∴ We write sin5 x cos2 xdx = sin4 x · sin x cos2 xdx
R
= Z (sin2 x)2 · cos2 x sin xdx
= (1 − cos2 x)2 · cos2 x sin xdx
put cos x = t
Z
= (1 − t2 )2 · t2 (−dt) − sin xdx = dt
Z
= − (1 + t4 − 2t2 )t2 dt sin xdx = −dt.

ww Z
= − (t2 + t6 − 2t4 )dt

w.E
" 3
t7 t5 cos3 x cos7 x 2
#
t
=− + −2· +c=− − + cos5 x + c.
3 7 5 3 7 5

Example 3.77. Evaluate asyZ


sin6 x cos3 xdx.

En
Solution. Here the power of cosine is odd.

gin
R R
∴ we write sin6 x cos3 xdx = sin6 x · cos2 x · cos xdx
R
= Z sin6 x(1 − sin2 x) · cos xdx
=
Z
= (t6 − t8 )dt
ee
t6 (1 − t2 )dt put sin x = t

rin
cos xdx = −dt.
t7 t9
= − +c=
7 9
sin7 x sin9 x
7

9
+ c.
g.n
Example 3.78. Evaluate
Z
sin9 x cos5 xdx.
Solution. Here, the powers of both sine and cosine are odd.
et
R R
∴ we write sin9 x cos5 xdx = sin9 x · cos4 x · cos xdx
R
= Z sin9 x(1 − sin2 x)2 · cos xdx
= t9 (1 − t2 )2 dt put sin x = t
Z
= t9 (1 + t4 − 2t2 )dt cos xdx = dt.

Download From : www.EasyEngineering.net


Download From : www.EasyEngineering.net

Integral Calculus 219

Z
= (t9 + t13 − 2t11 )dt

t10 t14 t12 sin10 x sin14 x sin12 x


= + −2 +c= + − + c.
10 14 12 10 14 6
Z
Evaluation of integrals of the form tanm x · secn xdx
Evaluation procedure.

(i) If
Z the power of secant
Z is even (say n = 2k) then write the given integral as

ww m n
tan x sec xdx =

=
Z
tanm x · sec2k xdx

tanm x · sec2k−2 x sec2 xdx

w.E =
Z
tanm x · sec2(k−1) x sec2 xdx

asy
Z
= tanm x · (sec2 x)k−1 sec2 xdx put tan x = t
Z
=
Z
En
tanm x · (1 + tan2 x)k−1 sec2 xdx sec2 xdx = dt.

gin
tm (1 + t2 )k−1 dt, which can be evaluated easily.

(ii) If
Z the power of tangent
tanm x secn xdx =
Z

Z
ee
is odd (say m = 2k + 1)then write the given integral as
tan2k+1 x · secn xdx

tan2k x · tan x secn−1 x sec xdx rin


g.n
=
Z
= (tan2 x)k · secn−1 x sec x tan xdx put sec x = t
Z
= (sec2 x − 1)k secn−1 x sec x tan xdx
Z et
sec x tan xdx = dt.

= (t2 − 1)k · tn−1 dt, which can be evaluated easily.

✎ ☞
Worked Examples
✍ ✌
Z
Example 3.79. Evaluate tan6 x sec4 xdx.
Solution. Here, the power of secant is even. Hence we write the given integral as

Download From : www.EasyEngineering.net


Download From : www.EasyEngineering.net

220 Engineering Mathematics - I

Z Z
tan6 x sec4 xdx = tan6 x sec2 x sec2 xdx
Z put tan x = t
= tan6 x(1 + tan2 x) sec2 xdx
Z sec2 xdx = dt.
= t6 (1 + t2 )dt
t7 t9
+ +c =
7 9
7
tan x tan9 x
= + + c.
7 9
Z

wwExample 3.80. Evaluate tan5 x sec7 xdx.


Solution. Here, the power of tangent is odd. Hence we write the given integral

w.E
as Z
5 7
tan x sec xdx =
Z
tan4 x sec6 x sec x tan xdx

asy put sec x = t


Z
= (sec2 x − 1)2 sec6 x sec x tan xdx
Z sec x tan xdx = dt.

En
2 2 6
= (t − 1) t dt
Z
=

=
Z
gin
(t4 + 1 − 2t2 )t6 dt

(t10 + t6 − 2t8 )dt

=
t11 t7
11 7
ee
t9
+ −2 +c=
9
sec11 x sec7 x 2
11
+
7
− sec9 x + c.
9
rin
Some standard results.

1. Find
R
tan xdxZ g.n
Solution. tan xdx =

=
Z
Z
sin x
cos
1
x
dx

(−dt)
cos x = t

− sin xdx = dt
et
t
= − log t + c sin xdx = −dt.

= − log(cos x) + c (or)
!
−1 1
= log(cos x) + c = log + c = log(sec x) + c.
cos x

Download From : www.EasyEngineering.net


Download From : www.EasyEngineering.net

Integral Calculus 221

R
2. Find sec xdxZ Z
sec x + tan x
Solution. sec xdx = sec x · dx
sec x + tan x
Z
1 sec x + tan x = t
= (dt)
t (sec x tan x + sec2 x)dx = dt
= log t + c
sec x(sec x + tan x)dx = dt.
= log(sec x + tan x) + c.
R
3. Find cosecxdx
Z Z
cosecx + cot x

ww Solution. cosecxdx =

=
Z
cosecx ·
1
(−dt)
cosecx + cot x
dx
cosecx + cot x = t

w.E (−cosecx cot x − cosec2 x)dx = dt


t
= − log t + c
−cosecx(cosecx + cot x)dx = dt.

asy
= − log(cosecx + cot x) + c
Z
4. Find cot x dx.

Solution.
Z
cot x dx =
Z
En
cos x
dx =
Z
d(sin x)
= log(sin x) + c.
sin x

gin

sin x

Worked Examples

Example 3.81. Evaluate


Z Z
Z

ee
tan3 xdx.

rin
g.n
3
Solution. tan xdx = tan2 x tan xdx
Z
(sec2 x − 1) tan xdx

et
=
Z Z
2
= sec x tan xdx − tan xdx
Z
= tan xd(tan x) − log sec x

tan2 x
− log(sec x) + c.
=
2
Z
Example 3.82. Evaluate sec3 xdx.
R R
Solution. sec3 xdx = sec x · sec2 xdx

Download From : www.EasyEngineering.net


Download From : www.EasyEngineering.net

222 Engineering Mathematics - I

Let us apply integration by parts


Choose, u = sec x dv = sec2 xdx
Z Z
du = sec x tan xdx dv = sec2 xdx ⇒ v = tan x
Z Z Z
3
sec xdx = udv = uv − vdu
Z
= sec x · tan x − tan x · sec x · tan xdx
Z
= sec x tan x − sec x tan2 xdx

ww
Z
= sec x tan x − sec x(sec2 x − 1)dx
Z Z Z

w.E
3 3
sec xdx = sec x tan x − sec xdx + sec xdx
Z Z
sec3 xdx + sec3 xdx = sec x tan x + log(sec x + tan x) + c
Z
asy
2 sec3 xdx = sec x tan x + log(sec x + tan x) + c

En
Z
1
sec3 xdx = sec x tan x + log(sec x + tan x) + c.

2

3.8 Improper integrals gin


Definition of an improper integral Type-I
Rt
ee rin
(i) If
a
f (x)dx exists for every number t ≥ a, then
R∞ Rt g.n
a
f (x)dx = lim
t→∞
a
f (x)dx
provided this limit exists (as a finite number).

Rb
et
(ii) If f (x)dx exists for every number t ≤ b, then
t
Rb Rb
f (x)dx = lim f (x)dx
t→−∞
−∞ t
provided this limit exists (as a finite number).
R∞ Rb
The improper integrals f (x)dx and f (x)dx are called convergent if the
a −∞

Download From : www.EasyEngineering.net


Download From : www.EasyEngineering.net

Integral Calculus 223

corresponding limit exists and divergent if the limit does not exist.
R∞ Ra
(iii) If both f (x)dx and f (x)dx are convergent then we define
a −∞
R∞ Ra R∞
f (x)dx = f (x)dx + f (x)dx.
−∞ −∞ a
Here a is any real number.
✎ ☞
Worked Examples
✍ ✌

R∞

wwExample 3.83. Discuss the convergence of


0
e−x dx.

w.E
Solution.
Z∞
−x
e dx = lim
t→∞
Zt
e−x dx

asy
0 0
!t
e−x
!

−t 0
 1
= lim = − lim e − e = − lim t − 1 = −(−1) = 1
t→∞ −1 t→∞ t→∞ e

En
0
Z∞
∴ e−x dx is convergent.
0
gin
Example 3.84. Determine whether the integral

Solution.
Z∞
1
dx = lim
Zt
ee
1
dx
R∞ 1
1 x
2
dx is convergent or divergent.

rin
1
x2 t→∞

Zt
1
x2
g.n
= lim
t→∞

= lim
1
x−1
x−2 dx

!t
= − lim
1
!t
= − lim
1
!
− 1 = −(−1) = 1.
et
t→∞ −1 t→∞ x t→∞ t
1 1
Z∞
1
∴ dx is convergent.
x2
1

R∞ 1
Example 3.85. Determine the convergence of dx.
1 x

Download From : www.EasyEngineering.net


Download From : www.EasyEngineering.net

224 Engineering Mathematics - I

Z∞ Zt
1 1
Solution. dx = lim dx
x t→∞ x
1 1

= lim (log x)t1 = lim (log t − log 1) = lim log t = ∞


t→∞ t→∞ t→∞
i.e the limit does not exist, as a finite number.
R∞ 1
∴ dx is divergent.
1 x

R∞ 1

wwExample 3.86. Discuss the convergence of


Z∞
1
Zt
1
0 a2 + x2
dx.

w.E
Solution.
0
a2 + x2
dx = lim
t→∞
0
1  −1
a2 + x2
dx

 x t

asy
= lim tan
t→∞ a a 0
1  −1 t 
= lim tan − tan−1 0

En
t→∞ a a
1 −1
= (tan ∞ − 0)
a
=
R∞ 1
1 π
a 2
−0 =
 π
2a ginwhich is a finite number.

∴ 2
0 a +x
2
dx is convergent.
ee rin
R∞ dx
Example 3.87. Determine whether the integral √ convergent or divergent
x g.n
et
1
Z∞ Zt  1 t
dx 1  x 2  √
Solution. √ = lim x− 2 dx = lim  1  = 2 lim ( t − 1) = 2 · ∞ = ∞.
x t→∞ t→∞
2 0
t→∞
1 1
Here the limit does not exist as a finite number.
R∞ 1
∴ √ dx is divergent.
1 x

R∞
Example 3.88. Discuss the convergence of the integral log xdx.
1

Download From : www.EasyEngineering.net


Download From : www.EasyEngineering.net

Integral Calculus 225

Z∞ Zt
Solution. log x dx = lim log xdx
t→∞
1 1
Zt
 
1
 
= lim  log x · x t0 −

x · dx [using integration by parts]
 
t→∞  x 
1
h i
= 2 lim t log t − (x)t1 = 2 lim t log t − t + 1 = ∞
 
t→∞ t→∞
R∞
∴ log xdx is divergent.
1

wwExample 3.89. Determine whether the integral


Z∞
log x
x
dx is convergent or

w.E
1
divergent. Evaluate if the integral is convergent. [A.U. Dec. 2015]
Z∞ Zt Zt
log x log x
Solution.
1
x
asy
dx = lim
t→∞
1
x
dx = lim
t→∞
1
log xd(log x)

En
!t
(log x)2
= lim
t→∞ 2 1
1
gin 
= lim (log t)2 − (log 1)2
2 t→∞


ee
1
= lim (log t)2 = ∞, [which is not a finite number.]
2 t→∞


Z∞
log x
x
dx is divergent.
rin
1

R∞ 1 g.n
Example 3.90. For what values of p is the integral

Solution.
Z∞
1
xp
dx = lim
t→∞
Zt
x−p dx
1 x
p
dx convergent?

et
1 1
!t
x−p+1
= lim
t→∞ −p + 1
1
!
1 1−p 1 1
= lim (t − 1) = · −1 .
1 − p t→∞ 1 − p t p−1
1
If p > 1, then p − 1 > 0 and hence t p−1 → ∞ as t → ∞ and hence → 0.
t p−1

Download From : www.EasyEngineering.net


Download From : www.EasyEngineering.net

226 Engineering Mathematics - I

R∞ 1 1 1
∴ p
dx = (0 − 1) = , which is a finite number.
1 x 1− p p−1
R∞ 1
∴ p
dx is convergent if p > 1.
1 x
If p < 1, then p − 1 < 0 or 1 − p > 0.
1
Hence p−1 = t1−p → ∞ as t → ∞.
t
R∞ 1 1
∴ p
dx = (∞ − 1) = ∞.
1 x 1 − p
R∞ 1
∴ p
dx diverges, if p < 1.

ww 1 x

Also, when p = 1,
R∞ 1
1 x
dx = ∞ [Example 3.96]

w.E ∴
R∞ 1
1 x
p
dx is divergent if p = 1.
R∞ 1
Hence,
1 x
p

asy
dx is convergent if p > 1 and divergent if p ≤ 1.

Example 3.91. Discuss the convergence ofEn Z0


e x dx

R0 R0 gin −∞

Solution.
−∞
=
e x dx = lim
t→−∞

lim [e x ]0t
t→−∞
t
ee
e x dx

= lim [e0 − et ] = 1 − 0 = 1, which is a finite number


∴ The given integral is convergent.
t→−∞
rin
R0 g.n
et
1
Example 3.92. Evaluate dx.
−∞ (1 − 3x)2
Z0 Z0
1
Solution. 2
dx = lim (1 − 3x)−2 dx
(1 − 3x) t→−∞
−∞ t
#0 #0
(1 − 3x)−1
" "
1 1 1 1
= lim = lim = lim [1 − 0] = .
t→−∞ (−1)(−3) t
t→−∞ 3 1 − 3x
t 3 t→−∞ 3

R0
Example 3.93. Evaluate xe x dx.
−∞

Download From : www.EasyEngineering.net


Download From : www.EasyEngineering.net

Integral Calculus 227

Z0 Z0
x
Solution. xe dx = lim xd(e x )
t→−∞
−∞ t
Z0
 
 
x 0 x 
= lim  x · e t − e dx

t→−∞  
t
h i
= lim 0 − et − (e x )0t
t→−∞
h i h i
= lim −et − (1 − et ) = lim −et − 1 + et = lim (−1) = −1.
t→−∞ t→−∞ t→−∞

wwExample 3.94. Evaluate


Z∞
1
dx.

w.E
Solution.
Z∞
1
dx =
−∞
Z0
4 + x2

1
dx +
Z∞
1
dx
−∞
4 + x2

asy
−∞
4 + x2

Z0
0
4 + x2

Zt

En
1 1
= lim 2
dx + lim dx
t→−∞ 4+x t→−∞ 4 + x2
t 0

= lim
t→−∞ 2 gin
1 −1 x
tan
!0
+ lim
2 t t→∞ 2
1 −1 x
tan
!t

2 0

=
1
2 t→−∞
1  −1


ee
lim tan−1 (0) − tan−1
t 0 1

2 t 2 t→∞
 1


tan 0 − tan−1 (−∞) + tan−1 (∞) − tan−1 0


t
+ lim tan−1 − tan−1 (0)
2

t

rin0

g.n
2 2
1  π  1  π  1 π π π
= 0− − + −0 = + = .
2 2 2 2 2 2 2 2

Example 3.95. If a > 0, discuss the convergency or divergency of


Z∞ et sin xdx.
a
Z∞ Zt h it
Solution. sin xdx = lim sin xdx = lim − cos x = − lim [cos t − cos a].
t→∞ t→∞ a t→∞
a a
We know that cost is bounded and it lies between −1 and 1.
R∞
Hence the integral sin xdx oscillates finitely[It neither converges nor diverges].
a

Download From : www.EasyEngineering.net


Download From : www.EasyEngineering.net

228 Engineering Mathematics - I

Z0
Example 3.96. Discuss the convergence of x sin xdx.
−∞
Z0 Z0
Solution. x sin xdx = lim xd(− cos x)
t→−∞
−∞ t
Z0
 
 
0
= lim [−x cos x]t − − cos xdx
 
t→−∞  
t

= lim [0 − t cos t + (sin x)0t ]

ww
t→−∞

= lim (−t cos t + 0 − sin t)


t→−∞

w.E
R0
Since cos x and sin x oscillates between −1 and 1, x sin xdx oscillates finitely.
−∞

asy
Definition of an improper integral type II

Zb
En
(i) If f is continuous on [a, b) and is discontinuous at b, then
Zt

a
t→b
gin
f (x)dx = lim− f (x)dx
a

= lim
∈→0 ee b−∈
Z

a
if the limit exists (as a finite number).
f (x)dx

rin
(ii) If f is continuous on (a, b] and is discontinuous at a, then g.n
Zb

a
f (x)dx = lim+
t→a
Zb
f (x)dx
t
et
Zb
= lim f (x)dx
∈→0
a+∈
if the limit exists (as a finite number).
Rb
The improper integral f (x)dx is called convergent if the corresponding limit
a
exists and divergent if the limit does not exist.

Download From : www.EasyEngineering.net


Download From : www.EasyEngineering.net

Integral Calculus 229

Rc Rb
(iii) If f hss a discontinuity at c where a < c < b, and both f (x)dx and f (x)dx
a c
are convergent, then we define
Rb Rc Rb
f (x)dx = f (x)dx + f (x)dx.
a a c
✎ ☞
Worked Examples
✍ ✌
Z1
1
Example 3.97. Evaluate √ dx if possible.
x

ww
0
Solution. The integrand has an infinite discontinuity at the lower limit 0.
Z1 Z1  1 1
1 − 12  x 2  √

w.E
∴ √ dx = lim x dx = lim  1  = 2 lim (1 − ∈) = 2.
x ∈→0 ∈→0
2 ∈
∈→0
0 0+∈

Z3

asy
Example 3.98. Determine whether the integral
0
dx
√ is convergent or
x
divergent. Evaluate if it is convergent.

En
Solution. The integrand has an infinite discontinuity at the lower limit 0.
[A.U. Dec. 2015]


Z3
1
√ dx = lim
x ∈→0
Z3
1

∈→0 gin 1 3
 x 2  √ √ √
x− 2 dx = lim  1  = 2 lim ( 3 − ∈) = 2 3,
∈→0
0

ee
0+∈ 2 ∈

[which is a finite number.]



Hence, the given integral is convergent and its values is 2 3.
rin
Example 3.99. Find the value of the improper integral
Z5

1
g.n
dx if possible.

Solution. The integrand has an infinite discontinuity at the lower limit 2.

Z5 Z5 5
 (x − 2) 21 

2


x−2

√ √
et
1 − 21
∴ √ dx = lim (x − 2) dx = lim  1
 = 2 lim ( 3 − ∈) = 2 3.
x−2 ∈→0 ∈→0 ∈→0
2 2+∈ 2 2+∈

Z1
1
Example 3.100. Evaluate √ dx if possible.
1 − x2
0
Solution. The integrand has an infinite discontinuity at the upper limit 1.

Download From : www.EasyEngineering.net


Download From : www.EasyEngineering.net

230 Engineering Mathematics - I

Z1 Z1−∈
1 1
∴ √ dx = lim √ dx
1− x2 ∈→0 1 − x2
0 0

= lim (sin−1 x)1−∈


0
∈→0
  π
= lim sin−1 (1− ∈) − sin−1 0 = sin−1 1 − 0 = .
∈→0 2
π
Z2
Example 3.101. Discuss the convergence of sec xdx if possible.
0
Solution. The integrand has an infinite discontinuity at the upper limit π2 .

ww ∴
π
Z2
sec xdx = lim
π
Z2 −∈
sec xdx

w.E 0
∈→0
0

= lim (log(sec x + tan x))02


∈→0
π
−∈

asy
 π π
= lim log(sec( − ∈) + tan( − ∈)) − log(sec 0 + tan 0)
∈→0 2 2


En
π π
= log(sec( ) + tan( )) − log 1 = log ∞ = ∞.
π
2 2

gin
R2
∴ sec xdx diverges.
0

Example 3.102. Evaluate the improper integral


ee Z3

2

1
3−x
dx if possible.

rin [A.U.Nov 2016]


Solution. The integrand has an infinite discontinuity at the upper limit 3.
Z3 Z3−∈ g.n
et
3−∈
 (3 − x) 21 

1 − 21
∴ √ dx = lim (3 − x) dx = lim   
3−x ∈→0 ∈→0 − 12 2
2 2
p √ 
= lim (−2) 3 − (3− ∈) − 3 − 2
∈→0
√ 
= −2 lim 3 − 3+ ∈ − 1 = −2(0 − 1) = 2.
∈→0

Z1
1
Example 3.103. Evaluate 2
dx if possible.
x3
−1
Solution. The integrand has an infinite discontinuity at x = 0,which lies between

Download From : www.EasyEngineering.net


Download From : www.EasyEngineering.net

Integral Calculus 231

−1 and 1.
Z1 Z0 Z1
1 1 1
∴ 2
dx = 2
dx + 2
dx
x 3 x 3 x3
−1 −1 0
Z0−∈ Z1
2 2
= lim x− 3 dx + lim x− 3 dx
∈→0 ∈→0
−1 0+∈
 1 −∈  1 1
 x 3   x 3 
= lim  1  + lim  1 
∈→0 ∈→0
3 −1 1  3 ∈  

ww
1
= 3 lim (− ∈) 3 + 1 + 3 lim 1 − (∈) 3 = 3 × 1 + 3 × 1 = 6.
∈→0 ∈→0

Z3

w.E
Example 3.104. Evaluate the improper integral

Solution. The integrand has an infinite discontinuity at x = 1 which lies between


0
1
x−1
dx if possible.

0 and 3.
asy

Z3
1
x−1
dx =
En Z1
1
x−1
dx +
Z3
1
x−1
dx
0

gin 0
Z1−∈
1
1
Z3
1
= lim

ee∈→0


0

= lim log |x − 1|
∈→0
x−1
dx + lim
∈→0

1−∈
0
1+∈

∈→0
x−1
dx


rin
+ lim log |x − 1| 31+∈


= lim log | ∈ | − log | − 1| + log 2 − log | ∈ |


∈→0

g.n
= log 0 + log 2 − log 0

= −∞ + log 2 + ∞ et
which is an indeterminate

R3 1
∴ dx diverges.
0 x−1

Z1
1
Example 3.105. Evaluate dx if possible.
x
−1
Solution. The integrand has an infinite discontinuity at x = 0 which lies between

Download From : www.EasyEngineering.net


Download From : www.EasyEngineering.net

232 Engineering Mathematics - I

−1 and 1.
Z1 Z0 Z1
1 1 1
∴ dx = dx + dx
x x x
−1 −1 0
Z0−∈ Z1
1 1
= lim dx + lim dx
∈→0 x ∈→0 x
−1 0+∈
h i−∈ h i1
= lim log |x| + lim log |x|
∈→0 −1 ∈→0 −∈
 
= lim log |− ∈ | − log | − 1| + lim log 1 − log | ∈ |

ww
∈→0 ∈→0

= lim log | ∈ | − log(∈)
∈→0

w.E
!
|∈|
= lim log = lim log 1 = 0, which is a finite number
∈→0 |∈| ∈→0
R1 1
dx converges.

asy

−1 x

A comparison test for improper integrals.

En
Comparison theorem. Suppose that f and g are continuous functions with

R∞
f (x) ≥ g(x) ≥ 0 for x ≥ a.
R∞ gin
(i) If

(ii) If
a

R∞ R∞
ee
f (x)dx is convergent, then g(x)dx is convergent.

g(x)dx is divergent, then f (x)dx is divergent.


a

rin
a

a

Worked Examples



g.n
Example 3.106. Show that
Z∞

0
2
e−x dx is convergent.
et
R∞ R1 R∞
Solution. e−x2 dx = e −x2 dx +
2
e−x dx
0 0 1
R1
e −x2 dx =a definite value and hence it is convergent.
0
R∞ 2
Let us consider the integral e−x dx.
1

Download From : www.EasyEngineering.net


Download From : www.EasyEngineering.net

Integral Calculus 233

For all x ≥ 1,we have x2 ≥ x.


⇒ −x2 ≤ −x
2 2
⇒ e−x ≤ e−x ⇒ e−x ≥ e−x .
Z∞ Zt
−x t
Now e dx = lim e−x dx = lim −e−x 1
t→∞ t→∞
1 1
!

−t −1
 1 1
= − lim e − e =− 0− = , which is finite.
t→∞ e e
R∞
∴ e−x dx converges.

ww
1
R∞ 2
By comparison theorem e−x dx converges.
0

w.E ⇒
R∞
0
e−x2 dx is convergent.

asy
Z∞
1 + e−x
Example 3.107. Prove that the integral dx is divergent.
x

En
1
1 + e−x 1 e−x 1
Solution. We have for all x ≥ 1, = + >
x x x x
We know from Example 3.101,
gin
R∞ 1
1 x
dx diverges [∴ p = 1]

ee
R∞ 1 + e−x
∴ By comparison theorem, dx is divergent.
x

rin
1

Example 3.108. Determine the convergency or divergency of the integral

g.n
Z∞
1
3
dx, using comparison test.
x +1

et
1
1 1 1 1
Solution. For all x ≥ 1, we have, < 3 ⇒ 3 > 3 .
+1 x x3x x +1

R 1
We have from Example 3.101, 3
dx converges [p > 1].
1 x
R∞ 1
∴ By comparison theorem 3
dx is convergent.
1 x +1

Example 3.109. Determine the convergency or divergency of the integral


Z∞
cos2 x
dx.
x2
2

Download From : www.EasyEngineering.net


Download From : www.EasyEngineering.net

234 Engineering Mathematics - I

Solution. We know that for all x ≥ 2 cos2 x ≤ 1.


cos2 x 1
⇒ 2
≤ 2. [∴ x2 > 0]
x x
1 cos2 x
or 2 ≥ .
x x2
R∞ 1
By Example 3.101, 2
dx converges [p > 1].
2 x
R∞ cos2 x
∴ By comparison theorem dx also converges.
2 x2
Z∞
1

wwExample 3.110. Discuss the convergency or divergency of the integral

1 1
2
x + ex
dx.

w.E
Solution. For all x > 2, we have <
x + ex ex
= e−x .

⇒ e−x >
1
x + ex
.
Z∞
−x
Now e dx = lim
asy
Zt
t
e−x dx = lim −e−x 2
t→∞ t→∞

En
2 2
!

−t −2
 1 1
= − lim e − e =− 0− 2 = 2 which is finite.

gin
t→∞ e e
R∞
∴ e−x dx is convergent.

ee
2
R∞ 1
∴ By comparison theorem, dx is convergent.
2 x + ex

rin
g.n
et

Download From : www.EasyEngineering.net


Download From : www.EasyEngineering.net

4 Multiple Integrals

ww4.1 Evaluation of double integrals

Definition. Let f (x, y) be a single

w.E
valued and bounded function of two
independent variables x and y which
Y
R

asy
is defined in a closed region R of
the xy plane.

En
Divide the region
R into rectangles by drawing lines

Ai be the ith rectangle. gin


parallel to the coordinate axes. Let
Let (xi , yi )
be any point inside the ith rectangle
and ∆Ai be its area.
ee
Let there be
O

rin
X

n rectangles A1 , A2 , . . . , An which lie


completely inside R. g.n
Let us consider the sum
n
X
et
f (x1 , y1 )∆A1 + f (x2 , y2 )∆A2 + · · · + f (xn , yn )∆An = f (xi , yi )∆Ai . (1)
i=1

As n → ∞, the number of rectangles increase indefinitely such that the largest


linear dimension of the rectangle which is the diagonal of ∆Ai tends to zero. The
limit of the sum (1) if exists is defined to be the double integral of f (x, y) over the

Download From : www.EasyEngineering.net


Download From : www.EasyEngineering.net

236 Engineering Mathematics - I

"
region R and it is denoted by f (x, y)dA.
R
n
X "
i.e., lim f (xi , yi )∆Ai = f (x, y)dA.
n→∞
∆Ai →0 i=1 R

If A is a typical rectangle whose dimensions parallel to the coordinate axes are dx


and dy then dA = dxdy. Then the above double integral can be written as
" " "

ww f (x, y)dA =
R
f (x, y)dxdy =
R
f (x, y)dydx.
R

w.E
In cartesian coordinates the following cases are to be taken into account while
evaluating the double integrals.

Case (i) Consider the case where


asy Y R

the limits are constants.


region R be a rectangle given by En Let the

R = {(x, y) : a ≤ x ≤ b, c ≤ y ≤ d},
gin where
a, b, c, d are constants, then, the double
integral is given by
Z Z Zb Zd

ee
Zd
 b
Z
O


rin
X

g.n
  
f (x, y)dxdy =  f (x, y)dy dx or =
 
 f (x, y)dx  dy.

R  
a c c a

et
i.e., if the limits are constants, the order of integration, is immaterial, provided
proper limits are taken.

Case (ii) Consider the case when Y


the limits of x are constants and
the limits of y are functions of x. h(x)
In this case the region R is given
by R = {(x, y) : a ≤ x ≤ b, g(x) ≤ y ≤ h(x)}, g(x)

where a, b are constants. Now the X


x=a x=b
double integral becomes

Download From : www.EasyEngineering.net


Download From : www.EasyEngineering.net

Multiple Integrals 237

" Zb  Zh(x)


 

 
f (x, y)dxdy =  f (x, y)dy dx.
 
R a g(x)
i.e., if the limits of the innermost integral are functions of x, then the order of
evaluation of the integral is, first with respect to y and finally with respect to x.

Case (iii) Consider the case when


Y

wwthe limits of y are constants and


the limits of x are functions of y=d

w.E
y.
by
Now the region R is given
R = {(x, y) : g(y) ≤ x ≤ h(y), c ≤ y ≤ d},
g(y) h(y)

where c, d are constants.


integral now takes the form asy The double y=c

"
En Zd  Zh(y)
 


gin
 
f (x, y)dxdy =  f (x, y)dx dy.
 
R c g(y)

ee
i.e., if the limits of the innermost integral are functions of y, the order of
integration is first with respect to x and finally with respect to y.

rin
"

Worked Examples



g.n
Example 4.1. Evaluate

y = 2.
" Z2 Z2
R
dxdy over the region R bounded by x = 0, x = 2, y = 0,
et[Jan 1996]

Solution. dxdy = dxdy.


R 0 0
Since the limits are constants, the order of integration is immaterial.
" Z2 Z2 Z2
∴ dxdy = (x)20 dy = 2dy = 2 dy = 2[y]20 = 4.
R 0 0 0

Download From : www.EasyEngineering.net


Download From : www.EasyEngineering.net

238 Engineering Mathematics - I

Z1 Z2
Example 4.2. Evaluate x(x + y)dydx. [Jun 1996]
0 1

Z1 Z2 Z1 Z2
 
 
Solution. x(x + y)dydx =  x(x + y)dy dx
 

0 1 0 1
Z1  !2  Z1
y2 
!!
 2 2 2 1
=  x (y)1 + x  dx = x +x 2− dx
2 1 2
0 0

ww =
Z1
2
x +
3x
2
!
dx =
x
3
3 1
!

0
+
3 x2
2 2
!1

0
=
1 3 13
+ = .
3 4 12

w.E
0

Note. In some cases, the integrand can be expressed as the product of two

asy
quantities which are independent. In that case the evaluation of the double
integral is equivalent to the product of the integrals involving the independent

En
integrands as the following examples suggest.

Za Zb
gin dxdy
Example 4.3. Evaluate

Za Zb Za
1 1
ee xy

Zb
.

rin
g.n
dx dy dx dy
Solution. = = (log x)a1 (log y)b1 = log a log b.
x y x y
1 1 1 1

Example 4.4. Evaluate


Z1 Z1

dxdy
p
1 − x2 1 − y2
.
et
0 0

Z1 Z1 Z1 Z1
dxdy dx dy
Solution. √ p = √ p
1 − x2 1 − y2 1 − x2 1 − y2
0 0 0 0
π π π2
= (sin−1 x)10 (sin−1 y)10 = = .
22 4

Download From : www.EasyEngineering.net


Download From : www.EasyEngineering.net

Multiple Integrals 239

Z2 Z4 Z4 Z2
y
Example 4.5. Prove that (xy + e )dydx = (xy + ey )dxdy
1 3 3 1

Z2 Z4 Z2 !y=4
y y2
Solution. (xy + e )dydx = x + ey dx
2 y=3
1 3 1
Z2 
x 
= (16 − 9) + (e4 − e3 ) dx
2
1
Z2 Z2

ww =
7
2
1
xdx +
1
(e4 − e3 )dx

w.E 7
2 2 1
x2
+ (e4 − e3 )(x)21
=
!2

asy
7 21
= (4 − 1) + (e4 − e3 )(2 − 1) = + e4 − e3 . (1)
4 4
Z4 Z2 Z4 ! x=2
y
(xy + e )dxdy =
En
x2
y +e x
2
y
x=1
dy

gin
3 1 3
Z4 !
3 y
= y + e dy

=
ee3

3 y2
22
+ey
!4

3
2

rin
3
= (16 − 9) + e4 − e3 =
4
21
4
+ e4 − e3 .
g.n (2)

From (1) and (2) we have the required result.

Example 4.6. Evaluate


Z1 Z1+x2

dxdy
1 + x2 + y2
.
et
0 0
Solution. Since the limits of the innermost integral are functions of x, the order
of evaluation must be, first w.r.t. y and finally w.r.t. x.
√ √ 
Z1 Z1+x2 Z1  Z1+x2 
dxdy  dy 
2 2
= 2 2  dx
1+x +y 1 + x + y 


0 0 0 0

Download From : www.EasyEngineering.net


Download From : www.EasyEngineering.net

240 Engineering Mathematics - I

Z1 " # √1+x2
1 y
= √ tan−1 √ dx
1+x 2 1 + x2 0
0
Z1 Z1
1 π 1
= √ (tan−1 1)dx = √ dx
1+x 2 4 1 + x2
0 0
π p
1 π √
= log(x + 1 + x2 )0 = (log(1 + 2)).
4 4
√2 2
Za Za −y q

wwExample 4.7. Evaluate


0 0
a2 − x2 − y2 dxdy.

Solution. Since the limits of the innermost integral are functions of x, the order

w.E
of evaluation must be first w.r.t. x and finally w.r.t. y.

2 2
Za −y q

2 2
Za −y q

asy
Za Za
a2 − x2 − y2 dxdy = (a2 − y2 ) − x2 dxdy

En
0 0 0 0
Za  q  x= √a2 −y2
 x a2 − y2 x 
a2 − y2 − x2 + sin−1 p

gin
=   dy
2 2 a2 − y2 x=0
0
Za
=

π
0ee Za
a2 − y2 −1
2
sin (1)dy

rin
=
4
0
(a2 − y2 )dy

g.n
et
!a
π 2 y3
= a y−
4 3 0
π 3 a3 π 2a3 πa3
!
= a − = = .
4 3 4 3 6

Za Za2 +x2
dxdy
Example 4.8. Evaluate .
x2 + y2 + a2
0 0
Solution. Since the limits of the innermost integral are functions of x, the order
of integration must be first w.r.t. y and finally w.r.t. x.

Download From : www.EasyEngineering.net


Download From : www.EasyEngineering.net

Multiple Integrals 241

√ √
Za Za2 +x2 Za Za2 +x2
dxdy dy
∴ = dx.
x2 + y2 + a2 x2 + y2 + a2
0 0 x=0y=0

Za Za2 +x2
dy
= √ 2 dx.
x=0 y=0 x2 + a2 + y2
Za ! √a2 +x2
1 y
tan−1 √

ww
= √ dx.
x2 + a2 x2 + a2 0
x=0
Za

w.E
1
= √ (tan−1 1 − tan−1 0)dx
x2 + a2
x=0
Za Za
=
x=0

2
asy
x +a 2
π
4
π
dx = ·
4
1

x=0

1
x + a2
2
dx

En
π  p a
= log x + x2 + a2
4  0
π

gin
p  
= log a + 2a2 − log a
4
πh  √ 

ee
i
= log a + 2a − log a
4
√ 

rin

π a 1+ 2 π  √ 
= log = log 1 + 2 .
4 a 4
"
Example 4.9. Evaluate xydxdy over the positive quadrant of the circle
g.n
et
R
x2 + y2 = a2 . [Jan 2014, Jun 1996]
Solution. The region of integration is bounded by the coordinate axes y = 0, x = 0
and the circle x2 + y2 = a2 . Y
Since the innermost integration is w.r.t.
B(0, a)
x, divide the region into strips parallel
P Q
to the x-axis. Along a typical strip, the

q is constant but x varies


y−coordinate x=0
from 0 to a2 − y2 . Finally y varies from X
(0, 0) y=0 (a, 0)
0 to a.

Download From : www.EasyEngineering.net


Download From : www.EasyEngineering.net

242 Engineering Mathematics - I

√2 2 √
" Za Za −y Za " 2 # a2 −y2
x
∴ xydxdy = xydxdy = y dy
2 0
R y=0 x=0 y=0
Za #a
1 2 y2 y4 1 a4 a4 a4 a4
" " # " #
1 2 2 1
= y(a − y )dy = a − = − = 1− = .
2 2 2 4 0 2 2 4 4 2 8
y=0

wwExample 4.10. Evaluate


x y
bounded by + = 1.
a b
xydxdy over the region in the positive quadrant

w.E
Solution. The region of integration is bounded by the coordinate axes y = 0, x = 0
x y
and the line + = 1.
a b

asy
Since the innermost integration
is w.r.t. x, divide the region into strips
Y
(0, b) B

parallel to the x-axis. Along a typical


En x=0
P
x
a +
Q
y
b =1

strip PQ, x varies from 0 to a 1 − by . y


gin
 
(0, 0) y=0
X
A
varies from 0 to b. a(1− y ) (a, 0)


"

R
xydxdy =
Zb Z b

y=0 x=0
xydxdy
ee rin
g.n
Zb !a(1− by )
x2
= y· dy
2 0

et
y=0
Zb "  #
1 y 2
= y a2 1 − − 0 dy
2 b
y=0
Zb
a2 y2 2y
!
= y 1+ 2 − dy
2 b b
y=0
Zb
a2 y3 2y2
!
= y+ 2 − dy
2 b b
y=0

Download From : www.EasyEngineering.net


Download From : www.EasyEngineering.net

Multiple Integrals 243

#b
a2 y2 y4 2y3
"
= + −
2 2 4b2 3b 0
2
" 2 4 2b3
#
a b b
= + 2−
2 2 4b 3b
2
" 2 2 2b2
#
a b b
= + −
2 2 4 3
2
a 6b + 3b − 8b2
" 2 2
#
=
2 12
2
a b 2 a b2
2

ww =
2 12
=

Example 4.11. Evaluate


24
"
xydxdy where R is the region bounded by the

w.E R
parabola y2 = x, the x−axis and the line x + y = 2 lying on the first quadrant.

asy
Solution. Solving y2 = x and x + y = 2 we get the points of intersection.
y2 + y − 2 = 0 ⇒ (y + 2)(y − 1) = 0 ⇒ y = 1, −2. When y = 1, x = 1.

En Y

the x−axis . gin


Divide the region into strips parallel to
Along a typical strip, x (0, 2)

varies from
from 0 to 1.
y2
ee
to 2−y and finally y varies
P
(1, 1)

Q
(2, 1)
rin
y2 = x

" Z1 Z2−y Z1
x2
!2−y
g.n
x=2−y

R
xydxdy =

=
y=0 x=y2

Z1
y
xydxdy =


y=0

(2 − y)2 − y4 dy =
1
y
2 y2

Z1 
dy


y 4 + y2 − 4y − y4 dy
et
2 2
y=0 y=0
Z1 !1
1 
3 2 5
 1 y2 y4 y3 y6
= 4y + y − 4y − y dy = 4 + −4 −
2 2 2 4 3 6 0
0
! " #
1 4 1 4 1 1 24 + 3 − 16 − 2 1 9 3
= + − − = = × = .
2 2 4 3 6 2 12 2 12 8

Download From : www.EasyEngineering.net


Download From : www.EasyEngineering.net

244 Engineering Mathematics - I

"
Example 4.12. Evaluate xydxdy where A is the region bounded by x = 2a and
A
the curve x2 = 4ay. [Jan 2006]
Solution.

Y
Let us evaluate this double integral first
w.r.t. y and then w.r.t. x. Divide the
region into strips parallel to the y−axis.

ww PQ is one such strip.


Along this strip, y varies form 0 to
x2
x2 = 4ay Q x = 2a

w.E
and finally x varies from 0 to 2a.
4a
P
X

" asy
Z2a Z4a
x2

A
xydxdy =

En
x=0 y=0
xydydx

= x
2 0gin
x2
Z2a " 2 # 4a
y
dx

=
x=0
Z2a
x
ee
x4
32a 2
!
dx =
Z2a 5
x
32a2
dx
rin
g.n
x=0 x=0
6 2a a4
" #
1 x 1 6
= = (64a − 0) = .

et
32a2 6 0 32a2 × 6 3
"
Example 4.13. Evaluate xy(x + y)dxdy over the area between y = x2 and y = x.
R
Solution. Solving the two equations we get the points of intersection.
y = x2 , y = x.
=⇒ x2 = x =⇒ x2 − x = 0 =⇒ x(x − 1) = 0 =⇒ x = 0 & x = 1.
When x = 0, y = 0, when x = 1, y = 1.
∴ The points of intersection are (0, 0) and (1, 1).

Download From : www.EasyEngineering.net


Download From : www.EasyEngineering.net

Multiple Integrals 245

Y
Let us evaluate this double
integral, first w.r.t. y and then w.r.t. x.
y=x (1, 1)
Divide this region into strips parallel to
Q
y = x2
the y−axis. PQ is one such strip. Along
P
this strip, y varies from x2 to x and x X
(0, 0)
varies from 0 to 1.
" Z1 Zx
(x2 y + xy2 )dydx

ww
xy(x + y)dxdy =
R x=0 y=x2

Z1

w.E
!x !x !
y3 2 y2
= x +x dx
3 x2 2 x2
x=0

=
asy
Z1
x 3
3
6 x2 2
2
4
!
(x − x ) + (x − x ) dx

En
0
Z1
x4 x6 x4 x7
!

gin
= − + − dx.
2 2 3 3
0
!1

Example 4.14. Evaluate


=
x5 x7 x5 x8
− + −
10 14 15 24
"
ee 0
=
1

1
+
1

1
10 14 15 24 56

ydxdy over the region R bounded by y = x and


3
= .

rin
y = 4x − x2 .
R
g.n
Solution. The equation of the parabola is
y = −(x2 − 4x) = −((x − 2)2 − 4) = −(x − 2)2 + 4 et
⇒ y − 4 = −(x − 2)2
vertex is (2, 4) and it is open downwards.
Solving the two equations we get
x = 4x − x2 ⇒ x2 + x − 4x = 0 ⇒ x2 − 3x = 0 ⇒ x(x − 3) = 0 ⇒ x = 0, x = 3.
When x = 0, y = 0.
When x = 3, y = 12 − 9 = 3.

Download From : www.EasyEngineering.net


Download From : www.EasyEngineering.net

246 Engineering Mathematics - I

The points of intersection are (0, 0), (3, 3).


Y
Let us evaluate the double integral first
(2, 4)
w.r.t. y and then w.r.t. x. Divide the Q
(3, 3)
region into strips parallel to the y−axis.
y=x y = 4x − x2
PQ is one such strip. Along this strip, y
P
varies from x to 4x − x2 . Finally x varies X
(0, 0)
from 0 to 3.

ww " Z 2
Zx=3 4x−x Z3 2 !4x−x2
y

w.E
ydxdy = ydydx = dx
2 x
R x=0 y=x x=0
Z3 Z3 
1 1

asy
  
2 2 2
= (4x − x ) − x dx = 16x2 + x4 − 8x3 − x2 dx
2 2
x=0 0

En
!3
x3 x5 x4
35
# "
1 1 3 4
= 15 + −8 = 5×3 + −2×3
2 3 5 4 0 2 5
1 9
!
= 27 5 + − 6 =
2 5 2 5 gin
27 25 + 9 − 30
!
=
27 4 54
2 5
= .
5

4.2 Change of order of integration ee rin


g.n
Za2 y=g
Z 2 (x)
Consider the integral f (x, y)dydx.

et
x=a1 y=g1 (x)

To evaluate this integral, since the innermost integral has functions of x as


limits, first we integrate w.r.t. y and then we integrate w.r.t x. Sometimes, it is
very difficult to evaluate in this order. By changing the order of integration we
can easily evaluate. In this example the original order of integration is first w.r.t.
y and then w.r.t. x. By the change of order of integration we need to integrate
first w.r.t. x and then w.r.t. y. For this, divide the region into strips parallel to the
x-axis. Consider one such strip. The limits for x will be functions of y which are the

Download From : www.EasyEngineering.net


Download From : www.EasyEngineering.net

Multiple Integrals 247

values of x corresponding to the curves in which the ends of the strip lies. Finally
the values of y will be constants according to the movement of this strip along the
given region. If the first integration is with respect to y then divide the region into
strips parallel to the y−axis and then find the corresponding limits for y and x and
then evaluate.
✎ ☞
Worked Examples
✍ ✌

ww
2
Z1 Z x

Example 4.15. Change the order of integration in f (x, y)dydx. [Jan 2014]
0 0

w.E

Solution. The region of integration is given by y = 0, y = 2 x, x = 0, x = 1.
i.e., y = 0, y2 = 4x, x = 0, x = 1.

asy Y
By changing
integration, the first integration is w.r.t.En
the order of

y =
4 x
(1, 2)

gin
2
x=0
x. Hence, divide the region into strips P Q
x=1
parallel to the x−axis. PQ is one such
strip. Along this strip, x varies from
to 1. Finally y varies from 0 to 2.
y2
4 ee y=0

rin
X

g.n

2
Z1 Z x Z2 Z1
∴ f (x, y)dydx = f (x, y)dxdy.
0 0 y=0 2
x= y4

Za Za
x
et
Example 4.16. Change the order of integration in dxdy and hence
x2 + y2
0 y
evaluate. [Jan 2014, Jun 2013, Jun 2011]
Solution. The region of integration is given by x = y, x = a, y = 0, y = a.
i.e., the region of integration is bounded by the lines x = 0, x = a, y = 0, y = x
By changing the order of integration, the first integration is w.r.t y. The double

Download From : www.EasyEngineering.net


Download From : www.EasyEngineering.net

248 Engineering Mathematics - I

integral takes the form


Za Za "
x x
dxdy = dydx.
x + y2
2 x2 + y2
0 y R

Divide the shaded region into strips parallel to the y-axis. PQ is one such strip.
Along this strip, y varies from 0 to x and finally x varies from 0 to a.

ww Za Za
x
Za Zx
x
y=a
(a, a)

w.E

0 y
x + y2
2
dxdy =
x=0 y=0
x + y2
2
dydx

x=0
Q

x=a

y
Za

=
!x

x
asy
1 −1 y
= x tan dx
x x 0 X
x=0 (0, 0) P (a, 0)

=
Za
π
4 Enπ
dx = (x)a0 =
4
πa
4
.
x=0

gin
Z∞ Z∞
e−y
Example 4.17. Evaluate
0 x
eey
dydx by changing the order of integration.

Solution. The region of integration is y = x, y = ∞, x = 0, x = ∞. rin


[May 2011, Jan 1999]

Here, the first integration is w.r.t y and then we integrate w.r.t x.


g.n
By changing the order of integration,
the innermost integration must be
Y
et
w.r.t.x.
x=0
So, we divide the region of integration
y=x
into strips parallel to the x−axis.
X
Along a typical strip, x varies from 0 to (0, 0) y=0

y and finally y varies from 0 to ∞.

Download From : www.EasyEngineering.net


Download From : www.EasyEngineering.net

Multiple Integrals 249

Z∞  Zy −y 
 y 
Z∞ Z∞ Z∞ −y
 
e−y  e Z
e
 
dydx = dx dy =  dx dy
 
∴ 
y y  y
0 x y=0 x=0 y=0 0
Z∞ −y Z∞ !∞
e y e−y
= (x)0 dy = −y
e dy = = −[e−∞ − e0 ] = −[0 − 1] = 1
y −1 0
x=0 0

Z3 Z4−y
Example 4.18. Evaluate (x + y)dxdy by changing the order of integration.

ww 0 1
[Jan 2003]

w.E
p
Solution. The region of integration is bounded between x = 1, x = 4 − y and
y = 0, y = 3. i.e.,x = 1, x2 = 4 − y and y = 0, y = 3.
x = 1, x2 = −(y − 4) and y = 0, y = 3.

asy
The region is bounded between x = 1
and the parabola x2 = −(y−4) with vertex En Y

(0, 4) and y = 0 and y = 3. By changing


gin (0, 4)

ee
the order of integration the innermost y=3 (1, 3)

rin
integration is w.r.t.y. Hence, divide this
x=1 x2 = 4−y
region into strips parallel to the y−axis.
Along a typical strip y varies from 0 to
4 − x2 and finally x varies from 1 to 2.
(0, 0)
g.n
(1, 0)
y=0
(2, 0)
X

0

Z3 Z4−y
(x + y)dxdy =
1
4−x2
Z2 Z
(x + y)dydx =
x=1 y=0
Z2
xy +
y2
2 0
!4−x2
dx
x=1
et
Z2
(4 − x2 )2
!
2
= x(4 − x ) + dx
2
x=1
Z2
16 + x4 − 8x2
!
3
= 4x − x + dx
2
1

Download From : www.EasyEngineering.net


Download From : www.EasyEngineering.net

250 Engineering Mathematics - I

!2
x2 x4 1 x5 x3
= 4 − + 8x + −4
2 4 2 5 3 1
1 1 4
= 2(4 − 1) − (16 − 1) + 8(2 − 1) + (32 − 1) − (8 − 1)
4 10 3
15 31 28 840 − 225 + 186 − 560 241
=6− +8+ − = = .
4 10 3 60 60

b a2 −x2
Za Za
Example 4.19. Change the order of integration in x2 dydx and hence

wwevaluate it.
0


0
[Jan 2012]

w.E
b a2 −x2
Solution. The region of integration is given by y = 0, y = a , x = 0, x = a.
b2
i.e., y = 0, y2 = 2 (a2 − x2 )
a
y2
b2
=
x2 y2
1 −
x2
a2 asy
+
a2 b2
= 1.
After changing the order of integration, En
the first integration is w.r.t. x and finally
gin x=0
Y

w.r.t. y. Divide the region into strips


parallel to the x− axis. PQ is one such
strip.
q Along this strip x varies from 0 to
ee P
(0, b)

y=0
rin
Q x=a

X
2
a 1 − by2 . Finally y varies from 0 to b.

r
g.n
et
2
b a2 −x2 a 1− y2
Za Za Zb Z b

∴ x2 dydx = x2 dxdy y = b sin θ


0 0 y=0 x=0
r dy = b cos θdθ
2
Zb 3 a
! 1− y2
x b when y = 0, b sin θ = 0
= dy
3 0
y=0 ⇒θ=0
Zb when y = b, b sin θ = b
1 a3 2
= (b − y2 )3/2 dy π
3 b3 ⇒ sin θ = 1 ∴ θ =
0 2
Zb
a3
= 3 (b2 − y2 )3/2 dy
3b
0

Download From : www.EasyEngineering.net


Download From : www.EasyEngineering.net

Multiple Integrals 251

π
Z2
a3
= 3 (b2 − b2 sin2 θ)3/2 b cos θdθ
3b
0
π
Z2
a3
= 3 b3 (1 − sin2 θ)3/2 b cos θdθ
3b
0
π
Z2
a3 b
(cos2 θ)3/2 cos θdθ

ww
=
3
0
π
Z2

w.E =
a3 b
3
0
cos3 θ · cos θdθ

asy
π
Z2
a3 b
= cos4 θdθ
3

=
0
a3 b 3 1 π πa3 b
· · · = . En
3 4 2 2 16

gin Za

a+ Z a2 −y2

Example 4.20. Change the order of integration in

evaluate it.
ee 0 a−

a2 −y2

rin
xydxdy and then

[Jan 2000]
Solution. The region
q of integration
q is between
x = a − a2 − y2 , x = a + a2 − y2 , y = 0, y = a. g.n
i.e.,(x − a)2 = a2 − y2 , (x − a)2 = a2 − y2 , y = 0, y = a.
(x − a)2 + y2 = a2 , (x − a)2 + y2 = a2 , y = 0, y = a.
et
∴ The region of integration is the circle Y
y=a
with (a, 0) as centre and radius a and
between y = 0 and y = a.
y=0 X
When we change the order of (a, 0)

integration, the inner most integration


must be w.r.t y.

Download From : www.EasyEngineering.net


Download From : www.EasyEngineering.net

252 Engineering Mathematics - I

Hence, divide the region into strips parallel to y−axis. Along a typical strip, y
p
varies from 0 to a2 − (x − a)2 and finally x varies from 0 to 2a.
√2 2 √
2 2
Za a+ Z a −y Z2a aZ−(x−a)
xydxdy = xydydx
0
√ 2 2 x=0 y=0
a− a −y

Z2a a2 −(x−a)2
y2
!
= x dx
2 0
x=0

ww =
1
2
Z2a
2
x(a − (x − a) )dx =
1
2
2
Z2a
(a2 x − x(x2 − 2ax + a2 ))dx

w.E
0 0
Z2a  !2a !2a 
1 2 3 2 2 1  x3 x4 
= (a x − x + 2ax − a x)dx = 2a − 
2 2 3 0 4 0 

=
0
1 2a 3 16a4
"
8a − asy #
=
a4 16
" #
− 4 = a4 =
1 4 2a4
.

En
2 3 4 2 3 2 3 3

Example 4.21. Change


Za Za
y2
dydx. gin the order of integration and hence evaluate

ee
p
√ y4 − a2 x2
0 ax

Solution. The region of integration is given by y =
i.e., y2 = ax, y = a, x = 0 and x = a. By changing the order of integration, the first rin
ax, y = a, x = 0 and x = a.

integration is w.r.t. x.
g.n
Hence, we divide the region into strips
parallel to the x-axis. Along a typical
Y
y=a
et
y2 x=0 y=0
strip x varies from 0 to and finally y y2 = ax
a
varies from 0 to a. X
y2
Za Za Za Za
y2 y2
p dydx = p dxdy
√ y4 − a2 x2 y4 − a2 x2
0 ax y=0 x=0

Download From : www.EasyEngineering.net


Download From : www.EasyEngineering.net

Multiple Integrals 253

y2
Za Za
1 y2
= r  dxdy
a y2 2
y=0 x=0
a − x2

Za    ya2
1  2 −1  x 
= y sin  2  dy
a y
y=0 a x=0
Za
1
= y2 (sin−1 1)dy

ww
a
0
!a
π y3
=

w.E =
2a 3
π
2a 3
a3
=
πa2
6
.
0

asy
Note. While changing the order of integration, we may come across situations
where the region has to be divided into several parts and evaluation has to be

En
done accordingly. The following examples illustrate this aspect.

gin
Example 4.22. Change the order of integration and hence evaluate
Z1 Z2−y
xydxdy.

ee
Solution. The region of integration is given by x = 0, x = 2 − y,
rin
0 0
[Dec 2011]
y = 0, y = 1.
x+y=2
g.n
OACE.
The region of integration is
After changing the order of B(0, 2)
Y

x+y=2
et
integration, the first integration is w.r.t.
y and final integration is w.r.t. x. E Q C(1, 1)
y=1
Q′
Divide the region into two regions x=0 x=1
A(2, 0)
ODCE and DAC. X
O(0, 0) P D(1, 0) P′
Z1 Z2−y " " y=0
∴ xydxdy = xydydx+ xydydx.
0 0 ODCE DAC

Download From : www.EasyEngineering.net


Download From : www.EasyEngineering.net

254 Engineering Mathematics - I

Consider the region ODCE


Divide the region into strips parallel to the y−axis. PQ is one such strip.
Along this strip, y varies from 0 to 1 and finally x varies from 0 to 1.
" Z1 Z1
∴ xydydx = xydydx.
ODCE x=0 y=0
Z1 !1
y2
= x· dx
2 0
x=0

ww =
1
2
Z1
x(1 − 0)dx

w.E
0
Z1
1
= xdx
2

1 x2
= · asy
!1
0

En
2 2 0
1
= (1 − 0)
4
1
= .
4 gin
Consider the region DAC
ee
Divide this region into strips parallel to y−axis. P′ Q′ is one such strip.
Along this strip, y varies from 0 to 2 − x and finally x varies from 1 to 2. rin

"
xydydx =
Z2 Z2−x
xydydx.
g.n
DAC

=
x=1 y=0
Z2

y2
2
!2−x

0
dx.
et
x=1
Z2
1
= x(2 − x)2 dx.
2
1
Z2
1
= x(4 + x2 − 4x)dx.
2
0

Download From : www.EasyEngineering.net


Download From : www.EasyEngineering.net

Multiple Integrals 255

Z2
1
= (4x + x3 − 4x2 )dx.
2
0
 !2 !2 !2 
1  x2 x4 x3 
= 4 · + −4· 
2 2 1 4 1 3 1
!
1 1 4
= 2[4 − 1] + [16 − 1] − (8 − 1)
2 4 3
!
1 15 28
= 6+ −
2 4 3

ww =
1 72 + 45 − 112
2 12
!

w.E Z1 Z2−y
=
1
24
(5) =
5
24


asy
0 0
1
xydxdy = +
5
4 24
=
6 + 5 11
24
= .
24

En
Example 4.23. Change the order of integration in
Z1 Z2−y
xydxdy and hence

evaluate. gin 0 y
[Jan 2001]

ee
Solution. The region of integration is x = y, x = 2 − y, y = 0, y = 1.
i.e.,x = y, x + y = 2, y = 0, y = 1. Solving we get 2x = 2 ⇒ x = 1.
B is (1, 1), A is (2, 0). rin
Y g.n
The region of integration is OAB.
et
2
=

After the change of order of integration (0, 2)


y
x+

y
=

the innermost integration is w.r.t.y and


x

B(1, 1)
the next integration is w.r.t.x. x=0
Q
Q′
Divide this region into two regions OCB x=1
A(2, 0)
and CBA. X
O(0, 0) P C(1, 0) P′
Z1 Z2−y " " y=0
xydxdy = xydydx + xydydx.
0 y OCB CBA

Download From : www.EasyEngineering.net


Download From : www.EasyEngineering.net

256 Engineering Mathematics - I

Consider the region OCB


Divide this region into strips parallel to the y−axis.
One such strip is PQ.
Along this strip y varies from 0 to x.
Finally x varies from 0 to 1.

" Z1 Zx Z1 !x
y2
xydydx = xydydx = x dx
2 0
OCB x=0 y=0 x=0

ww =
1
2
Z1
2
x(x − 0)dx =
1
2
Z1
x3 dx

w.E =
1
2 4
0

x4
!1
0

asy
0
1
= .
8

Consider the region CBA


En
gin
Divide this region into strips parallel to the y−axis.
One such strip is P′ Q′ . Along this strip y varies from 0 to 2 − x and finally x varies
from 1 to 2.

" Z2 Z2−x
ee Z2
y2
!2−x
rin
CBA
xydydx = xydydx =
x=1 y=0
x
2 0
x=1
dx
g.n
=
1
2
Z2

x=1
Z2
x(2 − x)2 dx =
1
2

Z2

x=1
!2
x(4 + x2 − 4x)dx

!2 !2 
et
1 3 1  x2
2 x4 x3 
= (4x + x − 4x )dx = 4 + −4 
2 2 2 1 4 1 3 1
x=1
" # " #
1 1 4 1 15 28
= 2(4 − 1) + (16 − 1) − (8 − 1) = 6+ −
2 4 3 2 4 3
! !
1 72 + 45 − 112 1 5 5
= = = .
2 12 2 12 24

Download From : www.EasyEngineering.net


Download From : www.EasyEngineering.net

Multiple Integrals 257

Z1 Z2−y
1 5 3+5 8 1
∴ xydxdy = + = = = .
8 24 24 24 3
0 y

√a √
Z 2 Za2 −x2
Example 4.24. Evaluate y2 dydx by changing the order of integration.
0 x
p a
Solution. The region of integration is bounded by y = x, y = a2 − x2 , x = 0, x = √ .
2
a
i.e.,y = x, x2 + y2 = a2 , x = 0, x = √ .

wwThe required region is OAB. Draw AC


2
B(0, a)
Y
Q′

w.E
P′  
perpendicular to OB. C A √a , √a
2 2

Divide this region into two parts OAC P Q


X

asy
and CAB. After changing the order of O x= √a
2
x=0
integration, the innermost integration
must be w.r.t.x.
√a √
Z 2 Za2 −x2 " " En
∴ 2
y dydx = 2
y dxdy+
gin
y2 dxdy.

ee
0 x OAC CAB

Consider the region OAC


Divide this region into strips parallel to the x−axis. rin
Let PQ be one such strip.
a
Along this strip x varies from 0 to y and finally y varies from 0 to √ . g.n
"
y2 dxdy =
OAC
√a
Z 2 Zy
y2 dxdy =
y=0 0
√a
Z2
y
y2 (x)0 dy
2

y=0
et
√a
2 ! √a
y4 1 a4 a4
Z
2
3
= y dy = = = .
4 0 4 4 16
0

Consider the region CAB


Divide this region into strips parallel to the x−axis.

Download From : www.EasyEngineering.net


Download From : www.EasyEngineering.net

258 Engineering Mathematics - I

p
One such strip is P′ Q′ . Along this strip x varies from 0 to a2 − y2 and finally y
varies from √a to a.
2

"
2 2
Za Za −y Za √2 2
a −y
y2 dxdy = 2
y dxdy = 2
y (x)0 dy
CAB y= √a x=0 y= √a
2 2

Za q
= y2 a2 − y2 dy
√a

ww
2
" #
a 1 π
y = a sin θ, dy = a cos θdθ, when y = √ ⇒ sin θ = √ , θ =
2 2 4

w.E π

When y = a ⇒ a = a sin θ ⇒ sin θ = 1 ⇒ θ =
2
π π
Z2 Z2
=
π
4 asy
a2 sin2 θa cos θa cos θdθ = a4 sin2 θ cos2 θdθ
π
4

=
a4
Z
π
2

En 2 2
4 sin θ cos θdθ =
a4
Z
π
2

(2 sin θ cos θ)2 dθ


4
π
4
π gin 4
π
π
4

=
a4
4
 π
Z2

π
4
π
ee
(sin2 2θ)dθ =
a4
4
Z2

π
4
1 − cos 4θ
2

rin
g.n

4
Z2 Z2 
a  
=  dθ − cos 4θdθ
8  

et
π π
4 4
 !π 
a4  π2 sin 4θ 2 
= (θ) π − 
8  4 4 π
4

a4 π π 1
!
= − − (sin 2π − sin π)
8 2 4 4
a4 π πa4
= = .
8 4 32

a4 πa4 a4
∴ Value of the integral= + = [π + 2].
16 32 32

Download From : www.EasyEngineering.net


Download From : www.EasyEngineering.net

Multiple Integrals 259


Z1 Z2−x2
x
Example 4.25. Evaluate p dydx by changing the order of
x2 + y2
0 x
integration.
p
Solution. The region of integration is y = x, y = 2 − x2 , x = 0, x = 1.
i.e., y = x, x2 + y2 = 2, x = 0, x = 1. Y

x
After changing the order of √

y=
B(0, 2)
P′ Q′
integration the inner most integration C A(1, 1)

wwis w.r.t.x and the next integration is P Q

y=0
X
w.r.t. y. The given region is OAB. Draw O x=1
x=0

w.E
AC perpendicular to OB.
Divide this region into two parts OAC
and CAB.

Z1 Z2−x2
x asy "
x
"
x

0 x
p
x2 + y2
dydx =

En
OAC
p
x2 + y2
dxdy+
CAB
p
x2 + y2
dxdy

Consider the region OAC


gin
ee
Divide this region into strips parallel to the x−axis.
Let PQ be one such strip. Along this strip x varies from 0 to y and finally y varies
from 0 to 1.
rin
"
p
x
dxdy =
Z1 Zy
p
x
dxdy =
1
2
Z1 Zy
p
2x
g.n
dxdy
OAC
x2 + y2

=
1
2
y=0 x=0
Z1 Zy
x2

2
(x + y )
+ y2

2 − 21 2
y=0 x=0

d(x + y )dy =21


2
x2 + y2
Z1  2 et 1 y
 (x + y2 ) 2 
 1
2
 dy
y=0 0 y=0 0

Z1   Z1 √
2 21
= (2y ) − y dy = ( 2y − y)dy
y=0 y=0

√ y2 1
!1
y2 2 1 1 √
!
= 2 − = − = ( 2 − 1).
2 0 2 0 2 2 2

Download From : www.EasyEngineering.net


Download From : www.EasyEngineering.net

260 Engineering Mathematics - I

Consider the region CAB


Divide this region into strips parallel to the x−axis.
p
One such strip is P′ Q′ . Along this strip x varies from 0 to 2 − y2 and finally y

varies from 1 to 2.
√ √ √ √ 2
"
2
Z 2 Z2−y Z 2 2 2 1  2−y
x x 1  (x + y ) 
2 
p dxdy = p dxdy =  1
 dy
x2 + y2 x2 + y2 2 2
CAB y=1 x=0 y=1 0
√ √

ww
Z2 √ 2
y2
!
1
= (2 − y)dy =
2 2y −
2 1
y=1

w.E √ √


2−1 √ √
1
2
1
√ √
2( 2 − 1) − (2 − 1) = 2( 2 − 1) −
=
1
2

asy
Value of the integral =
√ 2
+ 2( 2 − 1) −
√ √ 2
2 − 1 + 2 2( 2 − 1) − 1
=

En 2 √
2−1+4−2 2−1 2− 2

=

gin 2
=
2
.

Za 2a−x

ee
Z
Example 4.26. Change the order of integration in the integral xydydx and

evaluate it.
rin
0

[Jan 2013]
x2
a

x2
Solution. The region of integration is given by y = , y = 2a − x, x = 0, x = a.
x2 = ay, x + y = 2a, x = 0, x = a.
a
g.n
After changing the order of
B(0, 2a)
Y et x2 = ay
integration, the first integration is w.r.t.
Q′
P′
y and the final integration is w.r.t. x. C A(a, a) 2a
y=
OAB is the required region. Divide this P Q x+
X
region into OAC and CAB. y=0 O x=a
x=0
Za 2a−x
Z " "
∴ xydydx = xydxdy+ xydxdy.
0 x2 OAC CAB
a

Download From : www.EasyEngineering.net


Download From : www.EasyEngineering.net

Multiple Integrals 261

Consider the region OAC


Divide this region into strips parallel to the x−axis. PQ is one such strip.

Along this strip, x varies from 0 to ay and finally y varies from 0 to a.

" Za Z ay
∴ xydxdy = xydxdy
OAC y=0 x=0
Za ! √ay
x2
= y dy
2 0
y=0

ww
Za
1
= y · (ay − 0)dy
2
0

w.E =
a
2
Za
2
y dy =
a y3
2 3
!a

0
=
a 3 a4
6
·a = .
6

asy
0
Consider the region CAB
Divide this region into strips parallel to the x−axis. P′ Q′ is one such strip.

En

"
xydxdy =
2a−y
Z2a Z
gin
xydxdy
CAB

=
Z2a ee
y=a x=0

y
x2
2
!2a−y
dy
rin
g.n
0
y=a
Z2a
1
=

=
2

1
a
Z2a
y(2a − y)2 dy

y(4a2 + y2 − 4ay)dy
et
2
a
Z2a
1
= (4a2 y + y3 − 4ay2 )dy
2
a
 !2a !2a !2a 
1  2 y2 y4 y3 
= 4a + − 4a · 
2 2 a 4 a 3 a 

Download From : www.EasyEngineering.net


Download From : www.EasyEngineering.net

262 Engineering Mathematics - I

!
1 2 2 2 1 4 4 4a 3 3
= 2a (4a − a ) + [16a − a ] − [8a − a ]
2 4 3
4 4
!
1 15a 28a
= 6a4 + −
2 4 3
1 72a + 45a − 112a4
4 4 a4 5a4
!
= = [72 + 45 − 112] = .
2 12 24 24
Za 2a−x
a4 5a4 4a4 + 5a4 9a4 3a4
Z
∴ xydydx = + = = = .
6 24 24 24 8
0 x2
a

wwExample 4.27. Change the order of integration in the integral


Z1 Z2−x
xydydx and

w.E
evaluate it.
Solution. In the above Example, put a = 1.
0 x2


Z1 Z2−x
asy
3
xydydx = .
8
0 x2

En
4.3 Double integral in polar coordinates
gin
ee
Usually the double integral in polar coordinates (r, θ) is of the form

Zθ2 Zf2 (θ)


rin
θ=θ1 r= f1 (θ)
f (r, θ)drdθ.

g.n
Result. Reduction
π
Z2
formula

cosn θdθ =
π
Z2
sinn θdθ
et
0 0
n−1n−3 2
= · · · 1 if n is odd
n n−2 3
n−1n−3 1π
= ··· if n is even.
n n−2 22
✎ ☞
Worked Examples
✍ ✌

Download From : www.EasyEngineering.net


Download From : www.EasyEngineering.net

Multiple Integrals 263

Zπ Za
Example 4.28. Evaluate rdrdθ. [Jan 2014]
0 0

Zπ Za Zπ !a Zπ
r2 a2 a2 π πa2
Solution. rdrdθ = dθ = dθ = [θ] = .
2 0 2 2 0 2
0 0 0 0

Zπ Z
sin θ

Example 4.29. Evaluate rdrdθ. [Jan 2014]


0 0

wwSolution.
Zπ Z
sin θ

rdrdθ =
Zπ 2 !sin θ
r

w.E 0 0
2 0

=
1
0

sin2 θdθ =
11π π
= .
2
asy 0
222 8

Zπ Z

En
cos θ

Example 4.30. Evaluate rdrdθ.

Zπ Z
cos θ Zπ 2 !cos θ
0

gin 0

Solution.
0 0
rdrdθ =
r
2 0

1
0


ee rin
=
2
0
cos2 θdθ

g.n
et
π
Z2
1 1π π
= 2 cos2 θ = = .
2 22 4
0

Z2 Zπ
Example 4.31. Evaluate r sin2 θdrdθ. [Jan 2013]
0 0

Z2 Zπ
 

r  sin2 θdθ dr

Solution. I =

 
0 0

Download From : www.EasyEngineering.net


Download From : www.EasyEngineering.net

264 Engineering Mathematics - I

 π 
Z2  Z2 
 2 
= r 2 sin θdθ dr
 
0 0
Z2 !2
1 π π r2 π
=2 r· · dr = · = · 4 = 4.
2 2 2 2 0 4
0

π
Z2 2Zcos θ
Example 4.32. Evaluate r2 drdθ.
−π 0

ww
2

π π π
Z2 !2 cos θ Z2 Z2
r3 1 8 16 2 32

w.E
Solution. I =
−π
2
3 0
dθ =
3
−π
2
8 cos3 θdθ = 2
3
0
cos3 θdθ =
3 3
= .
9

Example 4.33. Evaluate


Z2 aZcos θ p
asy
r a2 − r2 drdθ.
π

En
0 0

gin
π π
Z2 aZcos θ p Z2 aZcos θ
−1 1
Solution. 2 2
r a − r drdθ = (a2 − r2 ) 2 (−2rdr)dθ
2
0 0

ee =
−1
2
Z  2
0
π
2
0

3 a cos θ
 (a − r2 ) 2 
 3
2
 dθ
0 rin
g.n
0
π
Z  2
−1 2 2 2 2 3
2 3

(a − a cos θ) − (a ) dθ

et
= 2 2
2 3
0
π
Z 2
−1
= (a3 sin3 θ − a3 )dθ
3
0
 π π 
 Z2 Z2 
−1  3
sin3 θdθ − a3 dθ

= a
3  
0 0
−a3 2 −a3 2 π
! !
π
= 1− = −
3 3 2 3 3 2

Download From : www.EasyEngineering.net


Download From : www.EasyEngineering.net

Multiple Integrals 265

a3 π 2 a3 3π − 4 a3
! !
= − = = (3π − 4).
3 2 3 3 6 18
"
Example 4.34. Evaluate r3 drdθ where A is the area between the circles
A
r = 2 sin θ and r = 4 sin θ. [Jan 2006]
Solution. The region of integration is the shaded portion.

wwThe first integration is w.r.t. r.


Consider the radius vector PQ. In the Q

w.E
region r varies from 2 sin θ to 4 sin θ. θ x = 4 sin θ
P
varies from 0 to π. 4 x = 2 sin θ
2
"

asy
Zπ 4Zsin θ Zπ !4 sin θ θ
3 3 r4 r
r drdθ = r drdθ = dθ O
4 2 sin θ

En
A θ=0 r=2 sin θ θ=0

1
= (44 sin4 θ − 24 sin4 θ)dθ
4
θ=0
Zπ gin
=

1
1
4
θ=0

4
ee
(256 sin4 θ − 16 sin4 θ)dθ


rin
sin4 θdθ
=
4
θ=0
240 sin θdθ = 60

π
θ=0
g.n
= 60 × 2

3 1 π 45π
Z

θ=0
2

sin4 θdθ
et
= 120 = .
422 2
"
Example 4.35. Evaluate r3 drdθ over the area bounded between the circles
R
r = 2 cos θ and r = 4 cos θ.
Solution.

Download From : www.EasyEngineering.net


Download From : www.EasyEngineering.net

266 Engineering Mathematics - I

θ
We first integrate w.r.t. r. Consider the π
θ= 2 P
radial strip OP. Along the strip r varies
−π
from 2 cos θ to 4 cos θ. θ varies from to
2
π r
. O
2
π π
" Z2 4Zcos θ Z2 4 !4 cos θ
3 3 r
r drdθ = r drdθ = dθ
4 2 cos θ θ = − π2

ww
R −π 2 cos θ −π
2 2
π π
Z2 Z2
1 1
= (44 cos4 θ − 24 cos4 θ)dθ = (256 cos4 θ − 16 cos4 θ)dθ

w.E 4
−π
2
π
2 Z2
π
4
−π
2

asy
Z
1 3 1 π 45π
= 240 cos4 θdθ = 60(2) cos4 θdθ = 120 = .
4 422 2
−π 0
2

4.4 Change of Variables in double integralEn


gin
ee
The evaluation of the double integral can be made easy in several
occasions by changing the variables.
Case"(i) Change of variables from (x, y) to u and v.
rin
Let
R
f (x, y)dxdy be the given double integral.
g.n
Let x = g(u, v) and y = h(u, v). By this transformation the elementary area dxdy is
transformed to dxdy = |J|dudv where J =
transformation.
" "
∂(x, y)
∂(u, v)
is the Jacobian of the et
f (x, y)dxdy = F(u, v)|J|dudv.
R R

Case (ii) Change of variables from Cartesian to polar coordinates.


Let x = r cos θ, y = r sin θ.
∂(x, y)
Then dxdy = |J|drdθ, where J = .
∂(u, v)

Download From : www.EasyEngineering.net


Download From : www.EasyEngineering.net

Multiple Integrals 267


∂x ∂x cos θ −r sin θ
∂r ∂θ = r cos2 θ + r sin2 θ = r.
Now, J = ∂y ∂y
=
sin θ r cos θ
∂r ∂θ
∴ dxdy = rdrdθ. " "
f (x, y)dxdy = F(r, θ)rdrdθ.
R R
✎ ☞
Worked Examples
✍ ✌
Example

4.36. By changing into polar coordinates, evaluate the integral
Z 2
Z2a 2ax−x

ww 0 0
(x2 + y2 )dydx. [Jan 1999]

w.E
p
Solution. The limits for y are y = 0, y = 2ax − x2 ⇒ y2 = 2ax − x2
x2 + y2 − 2ax = 0 ⇒ (x − a)2 − a2 + y2 = 0 ⇒ (x − a)2 + y2 = a2 .

asy
The limits for x are x = 0, 2a.
En
The region of integration is the upper
semi circle, with centre (a, 0) and radius gin Y

a. In polar coordinates
x = r cos θ, y = r sin θ, dxdy = rdrdθ.
ee x= 0
r = 2a cos θ

rin
x = 2a

g.n
2 2 2
x + y − 2ax = 0 ⇒ r − 2ar cos θ = 0
O (a, 0) y=0
X
⇒ r(r − 2a cos θ) = 0 ⇒ r = 0, r = 2a cos θ.

The limits for r are r = 0, 2a cos θ.


π
θ varies from 0 to .
2
et
π π
Z2 2aZcos θ Z2 4 !2a cos θ
2 r
∴I= r rdrdθ = dθ
4 0
θ=0 0 0
π
2
3 1 π 3πa4
Z
1
= 24 a4 cos4 θdθ = 4a4 = .
4 422 4
0

Download From : www.EasyEngineering.net


Download From : www.EasyEngineering.net

268 Engineering Mathematics - I


Z2 Z2x−x2
Example 4.37. Transform the integral (x2 + y2 )dydx into polar coordinates
0 0
and hence evaluate it. [May 2011]
Solution. In the previous problem, put a = 1.


Z2 Z2x−x2
x
Example 4.38. Evaluate p dydx by changing into polar
x2 + y2
0 0

wwcoordinates.

Solution. Let I =
Z2

Z2x−x2
x
dydx.
[Jan 2001]

w.E
p
x2 + y2
0 0 p
The limits for y are y = 0, y = 2x − x2 ⇒ y2 = 2x − x2 . x varies from 0 to 2.

asy
x2 + y2 − 2x = 0 ⇒ (x − 1)2 − 1 + y2 = 0
⇒ (x − 1)2 + y2 = 1.

En
gin
It is a circle with centre (1, 0) and radius Y
=
2
+y
1) 2

1
= 1. The region of integration is the
ee os
θ

2c
(x

r=

upper semi circle. To change into polar


coordinates we have x= 0
rin x=2

x = r cos θ, y = r sin θ and dxdy = rdrdθ.


O (1, 0) g.n
(2, 0)
X

2
Now, x + y − 2x = 02
y=0

et
r2 − 2r cos θ = 0 ⇒ r(r − 2 cos θ) = 0

r = 0, r = 2 cos θ.

∴ Limits for r are r = 0, 2 cos θ.

Download From : www.EasyEngineering.net


Download From : www.EasyEngineering.net

Multiple Integrals 269

π
θ varies from 0 to .
2
π π
Z2 2Zcos θ Z2 !2 cos θ
r cos θ r2
I= √ rdrdθ = cos θ dθ
r2 2 0
0 0 0
π π
Z 2 Z2
1
= cos θ4 cos2 θdθ = 2 cos3 θdθ
2
0 0
2 4

ww = 2 .1 = .
3 3
Za Za
x2

w.E
Example 4.39. Evaluate
0 y
p
x2 + y2
Solution. The limits for x are x = y, x = a. Limits for y are y = 0, y = a.
dxdy by changing to polar coordinates.

asy
The shaded portion is the region of integration.
Changing into polar coordinates we have x = r cos θ, y = r sin θ, dxdy = rdrdθ.
When x = a, r cos θ = a ⇒ r =
a
cos θ
.
En
∴ r varies from 0 to
θ varies from 0 to .
π
a
cos θ
gin
I=
π a
Z4 Zcos θ 2 2
r cos θ
r
4

rdrdθ =
π
Z4
2
cos θ
! a
r3 cos θ
3 0

ee
Y
rin
g.n
y=a
θ=0 0 0
π π
Z4 Z4 r = cosa θ
1 a3 1
x

et
y=

2
= cos θ 3 dθ = sec θdθ x= 0
3 cos θ 3 x=a
0 0
a3  π a3  √ 
O X
= log(sec θ + tan θ) 04 = log( 2 + 1) . y=0
3 3

Example

4.40. Transform the integral into polar coordinates and hence evaluate
Za Za2 −x2 q
x2 + y2 dydx. [Jun 2012]
0 0
Solution.

Download From : www.EasyEngineering.net


Download From : www.EasyEngineering.net

270 Engineering Mathematics - I


The limits for y are y = 0, y = a2 − x2
y2 = a2 − x2 Y

2
π/
x2 + y2 = a2 . 2

=
2 =a

θ
x2 + y
The limits for x are x = 0, x = a. P
x=0
By changing into polar coordinates we

0
θ=
O X
have x = r cos θ, y = r sin θ, dxdy = rdrdθ. y= 0

wwIn the region of integration, r varies


from 0 to a and θ varies from 0 to .
π
2
x=a

w.E √
Za Za2 −x2 q
x2 + y2 dydx =
π
Z2 Za p
r2 rdrdθ
0 0

asy
θ=0 r=0

Z2 Za
π
Z2
π

r3
!a
π
Z2
a3 a3 π a3 π πa3
=
0
En 0
r2 drdθ =
0
3 0
dθ =
0
3
dθ =
3
· [θ]02 =
3 2
=
6
.

Example gin
4.41. Transform the integral

Za Za2 −x2
dxdy
into polar

ee
p
√ a2 − x2 − y2
0 ax−x2
coordinates and then evaluate it.
Solution.
rin [Jan 2012]

The limits for y are


Y
g.n
et
p p
y = ax − x2 y= a2 − x2 P

y2 = ax − x2 y2 = a2 − x2 X
O 
a

2,0
2 2 2 2 2
x + y − ax = 0 x +y =a ,

x2 + y2 = a2 is a circle with centre (0, 0),


radius= a. x2 + y2 − ax = 0 is a circle with
a  a
centre , 0 and radius .
2 2
The limits for x are x = 0, x = a.

Download From : www.EasyEngineering.net


Download From : www.EasyEngineering.net

Multiple Integrals 271

Consider the curve x2 + y2 − ax = 0


By changing into polar coordinates we have,

r2 − a · r cos θ = 0

r(r − a cos θ) = 0

r , 0 ⇒ r = a cos θ.

Consider the equation x2 + y2 = a2 . We have r = a.

ww In the region of integration we have r varies from a cos θ to a and θ varies


π
from 0 to .

w.E 2



Za Za2 −x2
p
dxdy
=
π
Z2 Za

rdrdθ

0 ax−x2
asy
a2 − x2 − y2 a2 − r2
θ=0 a cos θ

Z2
π
Z0

En
−tdt
= dθ
π t
Z2 θ=0 r=a sin θ
=−
0
(t)0a sin θ dθ
gin t2 = a2 − r2

=−
Z2

0
π

(0 − a sin θ)dθ ee 2tdt = −2rdr.

rin
rdr = −tdt

when r = a cos θ,

g.n
π
Z2
=a sin θdθ t2 = a2 − a2 cos2 θ
0

= a (− cos θ)0
 π 
π
2

= −a cos − cos 0 = −a(0 − 1) = a.


= a2 sin2 θ.
et
= a2 (1 − cos2 θ)

2 t = a sin θ.

when r = a. t = 0.
" q
Example 4.42. Evaluate a2 − x2 − y2 dxdy where R is the semicircle
R
x2 + y2 = ax in the first quadrant by changing to polar coordinates.

Download From : www.EasyEngineering.net


Download From : www.EasyEngineering.net

272 Engineering Mathematics - I

 a 2 a2  a 2 a2
Solution. x2 + y2 − ax = 0 ⇒ x − + y2 − =0⇒ x− + y2 = .
2 4 2 4
By changing into polar coordinates we get x = r cos θ, y = r sin θ, dxdy = rdrdθ.
r2 − ar cos θ = 0 ⇒ r(r − a cos θ) = 0 ⇒ r = 0, r = a cos θ.
Limits for r are 0, a cos θ.
π
Limits for θ are 0, .
2
π
" q Z2 aZcos θ p
Now I = a2 − x2 − y2 dxdy = a2 − r2 rdrdθ

ww 2
R
2 2
Let t = a − r ⇒ 2tdt = −2rdr ⇒ tdt = −rdr
0 r=0

w.E when r = 0, t = a and when r = a cos θ, t = a sin θ


π π π

asy
Z2 aZsin θ Z2 3 !a sin θ Z2
t −1
I= −ttdtdθ = − dθ = (a3 sin3 θ − a3 )dθ
3 a 3
a

En
0 0 0
3 2 3 4 − 3π a3
! ! !
−1 3 2 3 π −a π −a
= a .1 − a = .1 − = = (3π − 4).

gin
3 3 2 3 3 2 3 6 18
R∞ R∞ 2 +y2 ) R∞ 2
Example 4.43. Evaluate e−(x dxdy and hence evaluate e−x dx.

Solution. Let I =
Z∞ Z∞
0 0

e−(x
ee
2 +y2 )
dxdy.
0
[Jan 2014, Jun 2011, Jan 2010, Jan 2004]

rin
0 0
Y g.n
Since x varies from 0 to ∞ and y varies
from 0 to ∞, the region of integration is
the entire first quadrant. x=0
r
P et
Changing to polar coordinates we get
θ
x = r cos θ, y = r sin θ and dxdy = rdrdθ. O
X
y=0
Here, r varies from 0 to ∞ and θ varies
π
from 0 to .
2

Download From : www.EasyEngineering.net


Download From : www.EasyEngineering.net

Multiple Integrals 273

π
Z∞ Z∞ Z2 Z∞
2 +y2 ) 2 dt
∴I= e−(x dxdy = e−r rdrdθ Let r2 = t, 2rdr = dt ⇒ rdr =
2
0 0 0 0
π
Z Z∞2
dt
= e−t dθ When r = 0, t = 0, when r = ∞, t = ∞
2
0 0
π
Z2 !∞
1 e−t
= dθ

ww
2 −1 0
0
π
Z2
1 1 π π

w.E Z∞
=−
2
0
(0 − 1)dθ = (θ)02 = .
2 4

asy
2
To find e−x dx.
0

We have,
Z∞ Z∞
e−(x
2 +y2 )

En
dxdy =
π
4
0
Z∞ Z∞
0

2 2
e−x e−y dxdy =
gin π

0
Z∞
0

−x2
e dx
Z∞
e−y dy =
2
ee4

π
rin
0
 ∞
0
2
4

g.n
et
Z
π

−x 2
 e dx =
 
4
0
Z∞ √
−x2 π
e dx = .
2
0

" s
1 − x2 − y2
Example 4.44. Evaluate dxdy over the positive quadrant of the
1 + x2 + y2
circle x2 + y2 = 1.
Solution.

Download From : www.EasyEngineering.net


Download From : www.EasyEngineering.net

274 Engineering Mathematics - I

By changing into polar coordinates we


get x = r cos θ, y = r sin θ and dxdy = rdrdθ. Y

2
π/
Now x2 + y2 = 1

θ=
=⇒ r2 = 1 P
x=0
i.e., r = 1. For the region of integration,

1
r=

0
θ=
we have r varies from 0 to 1 and θ varies θ
X
π O y=0
from 0 to .

ww 2

" s

w.E Let I =
π
1 − x2 − y2
1 + x2 + y2
dxdy

=
Z2 Z1 r

asy
1 − r2
1 + r2
rdrdθ

En
0 0
π
Z Z12
r(1 − r2 )
=
0 0

gin
1 − r4
drdθ

ee
π
Z Z1
 2

r r3 
 
= √ − √  drdθ

rin

 1 − r4 1 − r4 
0 0
π 

g.n
Z2  Z1 Z1

 1 2r 1 −4r 3 
= √ dr + √ dr dθ
 
2 1 − r4 4 1 − r4 

et
0 0 0
π
Z  Z1 Z1
 2

 1 dt 1 4
d(1 − r ) 

=  √ dr + √  dθ [t = r2 ]
2 4

1 − t2 1 − r4 
0 0 0
π
Z  2 1 1
 
 1 −1 1 1  (1 − r4 ) 2  
=  (sin t)0 +    dθ
2 4 1
0 2 0
π
Z 2 !
1π 1
= + (0 − 2) dθ
22 4
0

Download From : www.EasyEngineering.net


Download From : www.EasyEngineering.net

Multiple Integrals 275

π
Z2 !
π 1
= − dθ
4 2
0
!
π 1 π
= − (θ)02
4 2
!
π 1 π
= −
4 2 2
π2 π
= − .
8 4

wwExample 4.45. By transforming into polar coordinates evaluate


x2 y2
x2 + y2
dxdy
"

w.E
over the annular region between the circles x2 + y2 = a2 and x2 + y2 = b2 , (b > a)

Y
[Jan 2013]
Solution. By changing

asy
coordinates we get x = r cos θ, y = r sin θ
into polar
b

En
x=0
and dxdy = rdrdθ, x2 + y2 = r2 . a
In the region of integration we X

gin
O y=0
have r varies from a to b and θ varies
from 0 to 2π.

ee rin
"
x2 y2
x2 + y2
dxdy =
Z2π Zb
r2 cos2 θ · r2 sin θ
r2
2
· rdrdθ g.n
=
θ=0 r=a
Z2π Zb
r3 cos2 θ sin2 θdrdθ
et
θ=0 r=a
Z2π !b
2 2r4
= cos θ sin θ · dθ.
4 r=a
0
Z2π
1
= cos2 θ sin2 θ(b4 − a4 )dθ
4
0

Download From : www.EasyEngineering.net


Download From : www.EasyEngineering.net

276 Engineering Mathematics - I

Z2π
b4 − a4
= (sin θ cos θ)2 dθ
4
0
Z2π !2
b4 − a4 sin 2θ
= dθ
4 2
0
Z2π
b4 − a4
= sin2 2θdθ.
16
0
Z2π

ww =
b4 − a4
16
0
1 − cos 4θ
2

w.E
 2π
Z2π

4 4
b − a 
Z 
=  dθ − cos 4θdθ
32  
0 0

asy #2π 

b4 − a4  2π sin 4θ  b4 − a4 (b4 − a4 )π
"
= [θ]0 −  = (2π) =
32 4 0 32 16

En
4.5 Area enclosed by plane curves
gin "



ee
Result. Area of a region R is given by

Worked Examples
R
dxdy.


✌ rin
Example 4.46. Find the area of the circle of radius a by double integration.
g.n
Solution.
The circle is x2 + y2 = a2 .
By symmetry Area = 4× area in the first
and finally x varies from 0 to a. et [Jan 2006]


quadrant. Za Za2 −x2 Za √
2 2

Divide the region in the first quadrant ∴ Area = 4 dydx = 4 (y)0 a −x dx


x=0 y=0 x=0
into strips parallel to the y-axis. Along a
√ Za !a
p xp 2 a2 −1  x 
typical strip, y varies from 0 to a2 − x2 =4 a2 − x2 dx =4 a − x2 + sin
2 2 a 0
0

Download From : www.EasyEngineering.net


Download From : www.EasyEngineering.net

Multiple Integrals 277

a2 −1 a2 π
!
=4 sin (1) = 4 = πa2 .
2 2 2

Y
(0, a) √
y = a2 − x 2

x=0

ww O
y= 0 (a, 0)
X

w.E x2 +y2 = a2

x2 y2

asy
Example 4.47. Find the area of the ellipse +
a2 b2
= 1 using double integration.
[Jun 2013]
Solution.
En
Consider the area in the first quadrant.
gin Y

ee
(0, b) y = ab a2 − x 2
Divide this area into strips parallel to
x=0
the y-axis. Along a typical strip, y varies
from 0 to
bp 2
a − x2 and finally x varies rin
y=0 (a, 0)
X
a
from 0 to a.
g.n
By symmetry, Area of the ellipse = 4× Area in the first quadrant.

Za
b
a

Za
2 −x2
Za b

a2 −x2
Za
bp 2
et
Area = 4 dydx = 4 a
(y)0 dx = 4 a − x2 dx
a
x=0 y=0 x=0 0
Za p !a
4b 2 2
4b x p 2 2
a2 −1  x 
= a − x dx = a −x + sin
a a 2 2 a 0
0
4b a2 π
= sin−1 (1) = 2ab = πab.
a 2 2

Download From : www.EasyEngineering.net


Download From : www.EasyEngineering.net

278 Engineering Mathematics - I

Example 4.48. Using double integral, find the area bounded by the parabola
y2 = 4ax and x2 = 4ay [Jun 2013, May 2011, Jan 2010]
Solution.

Y
To find the points of intersection, solve x2 = 4ay

the two equations.


A(4a, 4a)
2
y = 4ax (1)

ww x2 = 4ay (2)
y=0 O
X

w.E x42−y264a3 x = 0
(2)2 ⇒ x4 = 16a

= x(x
3
64a3 ) = 0
16a2 −· 4ax x=0
y2 = 4ax
x=
= 0,
asy
x33
64a x− 64a = 0
3

x3 = 64a3 ⇒ x = 4a.

En
gin
When x = 0, y = 0. One point of intersection is O(0, 0).
When x = 4a, 4ay = 16a2
16a2
y=
4a
= 4a.
ee
∴ The other point of intersection is A(4a, 4a).
The shaded portion is the required region. Divide the region into strips rin
parallel to the x−axis. Along one such strip, x varies form
y2
4a
p
to 4ay and finally y
g.n
et
varies form 0 to 4a. √
Z4a Z4ay
∴ Required area = dxdy
y=0 2
x= y4a

Z4a √
4ay
= [x] y2 dy
4a
y=0
Z4a " p
y2
#
= 4ay − dy
4a
0

Download From : www.EasyEngineering.net


Download From : www.EasyEngineering.net

Multiple Integrals 279

Z4a Z4a
√ 1/2 1
=2 a· y dy − y2 dy
4a
0 0
 4a !4a
√  y3/2  1 y3
= 2 a ·  3  −
  ·
2 0
4a 3 0
√ 2 1
= 2 a × · (4a)3/2 − 64a3
3 12a

4 a √ 16a2
= 8a a −
3 3

ww =
32a
3
2

16a
3
2
=
16a2
3
Sq.units

w.E
Example 4.49. Find the area common to y2 = 4x and x2 = 4y using double
integration. [Dec 2011]

asy
Solution. Put a = 1 in the previous example.

Example 4.50. Find by double integration, the area between the parabolas
3y2 = 25x and 5x2 = 9y.
En [Jun 2012]

gin
Solution. To find the points of intersection, solve the two equations.

25
y2 =

x2 = y
3
9
5
x

ee
(1)

(2)
x2 = 95 y
Y

rin
g.n
2 4 81 2
(2) ⇒ x = y A(3, 5)
25
81 × ✚✚
25
✚ 4=

25x

x4 = 27x

x4 − 27x = 0
3
x

O(0, 0)
et X

x(x3 − 27) = 0 y2 = 25
3 x

x = 0, x3 − 27 = 0

x3 = 27

x = 3.

Download From : www.EasyEngineering.net


Download From : www.EasyEngineering.net

280 Engineering Mathematics - I

when x = 0, 9y = 0 ⇒ y = 0.
One point of intersection is O(0, 0).
where x = 3, 9y = 5 × 9
y = 5.
∴ The other point of intersection is (3, 5).
The shaded portion is the region of integration. Divide the region √ into strips
3y2 3 y
parallel to the x−axis. Along one such strip, x varies from to √ and finally y
25 5

wwvaries from 0 to 5. √
Z5 Z 5
3 y

Z5 √
3 y

w.E
5
∴ Area = dxdy = (x) 3y2
dy
25
0 3y2 0
25

Z5

asy
3y2
!
3 √
= √ y− dy
5 25
0

= √  3  −
5
3  y3/2  3
En
·
y3
!5
2 √
= √ [5 5] −
1
· 53 = 10 − 5 = 5 Sq.units.
5 2 0 25 3 0

gin 5 25

Example 4.51. Find the smaller of the areas bounded by the ellipse 4x2 + 9y2 = 36
and the straight line 2x + 3y = 6.
ee
Solution. Solving the given two equations, we get the points of intersection.
rin
[Jan 2012]

Y
2) g.n
et
,
2 2 B(0
4x + 9y = 36 (1) 4x2 +9y2 = 36

2 x+
2x + 3y = 6 (2) 3y=
6

O
X
From (2), 3y = 6 − 2x A(3
, 0)
6 − 2x
i.e., y = .
3
Substituting in (1) we get

9(6 − 2x)2
4x2 + = 36
9

Download From : www.EasyEngineering.net


Download From : www.EasyEngineering.net

Multiple Integrals 281

4x2 + 36 + 4x2 − 24x = 36

8x2 − 24x = 0

x2 − 3x = 0

x(x − 3) = 0 ⇒ x = 0 or x = 3.

6
When x = 0, (2) ⇒ y = = 2.
3
One point of intersection is (0, 2)
6−6

wwWhen x = 3, y =
3
= 0.
The other point of intersection is (3, 0).

w.E
The shaded portion is the smaller area. Divide the regionrinto
the y−axis. Along one such strip, y varies from
6 − 2x
3
to 4 −
strips parallel to
4 2
9
x and finally x
varies from 3 to 0.
asy
En
q
4− 94 x2
Z0 Z0 q
  4− 94 x2
Z
∴ Area = dxdy = y 6−2x dx
x=3 y= 6−2x
3
gin 3
3

Z0  r

ee
!
 4 2 6 − 2x 
=  4 − x −  dx
3
Z0 r
9 3

Z0 Z0 rin
=
3
4
9
(9 − x2 )dx −
3
2dx +
2
3
3
xdx
g.n
=

=
3 2
2
"
2 xp

"
0−

#
2
9 −1  x 
9 − x + sin
2
#0

3 3
1
− 2(0 − 3) + [0 − 9] =
0
− 2[x]3 +

−3π
" #0
2 x2
3 2 3

+6−3=3−

et
3 22 3 2 2
 π  3(2 − π)
=3 1− = Sq.units.
2 2

Example 4.52. Find the smaller of the areas bounded by y = 2 − x and x2 + y2 = 4


using double integral.

Download From : www.EasyEngineering.net


Download From : www.EasyEngineering.net

282 Engineering Mathematics - I

Solution. Solving the given two equations, we get the points of intersection.

We have x2 + y2 = 4 ⇒ x2 + (2 − x)2 = 4 ⇒ x2 + 4 + x2 − 4x = 4 ⇒ 2x2 − 4x = 0

⇒ x2 − 2x = 0 ⇒ x(x − 2) = 0 ⇒ x = 0, x = 2.

Y
When x = 0, y = 2, when x = 2, y = 0.
(0, 2)
The points of intersection are (0, 2) and y = 2− x

ww(2, 0). The shaded portion is the required


region. Divide this region into strips
x=0

O X

w.E
y= 0 (2, 0)
parallel to the y-axis. Along one such

typical strip, y varies from 2−x to 4 − x2

asy
and finally x varies from 0 to 2.

Now area =
"
dxdy. =
En
Z2 Z4−x2 Z2 √
4−x2
Z2 p
dydx = (y)2−x dx = ( 4 − x2 − (2 − x))dx
R

xp gin
x=0 y=2−x

4
4 − x2 + sin−1
 x !2
x=0

− 2(x)20 +
x 2 2
!
x=0

ee
=
2 2 2 0 2 0

rin
4 π
= 2 sin−1 (1) − 2(2 − 0) + = 2 − 4 + 2 = π − 2.
2 2

Example 4.53. Find the area bounded by the parabola y2 = 4 − x and y2 = 4 − 4x as


a double integral and evaluate it. g.n [Jan 2001]
Solution. The parabola y2 = 4 − x = −(x − 4). Its vertex is at (4, 0).
Consider the parabola y2 = 4 − 4x = −4(x − 1) . Its vertex is at (1, 0).
et
Both the parabolas are open to the left. Let us solve the two equations to find the
points of intersection, y2 = 4 − x and y2 = 4 − 4x.
∴ 4 − x = 4 − 4x ⇒ 4x − x = 0 ⇒ 3x = 0 ⇒ x = 0 ⇒ y2 = 4 ⇒ y = ±2.

Download From : www.EasyEngineering.net


Download From : www.EasyEngineering.net

Multiple Integrals 283

Y
The points of intersection are
(2, 0), (−2, 0). The shaded portion is the
required region. Divide this region into (1, 0) (4, 0)
O X
strips parallel to the x-axis. Along one
y2
typical strip, x varies from 1 − to 4−y2 .
4
Finally y varies from −2 to 2.

ww " x=4−y2
Z2 Z2 Z2
y2
Z !!
4−y2 2
Now area = dxdy. = dydx = (x) 2 dy = 4−y − 1− dy
1− y4 4

w.E
R y=−2 2 y=−2 y=−2
x=1− y4

Z2 !2 !2
y2 y3 1 y3
!
2 2 2
= 4−y −1+ dy = 4(y)−2 − − (y)−2 +
y=−2
1 asy4

1
3 −2

= 4(2 + 2) − (8 + 8) − (2 + 2) + (8 + 8) = 16 −
16
−4+
16
4 3 −2

= 16 − 4 −
3
16 4
+ = 12 −En 12
12
= 12 − 4 = 8.
3 12

3 3
gin
3

Example 4.54. Find the area of the region bounded by y = x − 2 and y2 = 2x + 4.

The vertex is (−2, 0).


ee
Solution. The parabola is y2 = 2x + 4 = 2(x + 2).

rin
To find the points of intersection, let us solve the two equations.
The curves are y = x − 2, y2 = 2x + 4. g.n
(x − 2)2 = 2x + 4.

x2 − 4x + 4 − 2x − 4 = 0

x2 − 6x = 0
et
x(x − 6) = 0 ⇒ x = 0, x = 6.
When x = 0, y = −2.
When x = 6, y = 4.

Download From : www.EasyEngineering.net


Download From : www.EasyEngineering.net

284 Engineering Mathematics - I

Y
The points of intersection are (0, −2) and
(6, 4). The shaded portion is the required
(6, 4)
region. Divide the region into strips
parallel to the x-axis. Along one such
X
y2 − 4 (−2, 0)
O
strip, x varies from to y + 2 and
2 (0, −2)
finally y varies from −2 to 4.

ww " Z4 Zy+2 Z4

w.ENow, Area =
R
dxdy =
y=−2 2
x= y 2−4
dxdy =
y=−2
(x)
y+2
2
x= y 2−4
dy

=
Z4
y+2−asy
y2
2
!!
− 2 dy =
Z4
y+2−
y2
2
!
+ 2 dy =
Z4
y+4−
y2
2
!
dy

y2
−2

y3
!4
En1
−2

1
−2

gin
= + 4y − = (16 − 4) + 4(4 + 2) − (64 + 8) = 6 + 24 − 12 = 18.
2 6 −2 2 6
Area as double integral in polar coordinates

ee✎
!
Result. Area in polar coordinates is given by rdrdθ.

Worked Examples
R

rin

Example 4.55. Find the area of the cardioid r = a(1 − cos θ).

g.n
Solution.

θ
et
Since the curve is symmetric about the
initial line, Area = 2× area above the P
θ=π
initial line. r
O θ=0
Along this region, r varies from 0 to
a(1 − cos θ) and θ varies from 0 to π. r = a(1−cos θ)

Download From : www.EasyEngineering.net


Download From : www.EasyEngineering.net

Multiple Integrals 285

Zπ Z θ)
a(1−cos Zπ !a(1−cos θ) Zπ
r2
Area = 2 rdrdθ = 2 dθ = a2 (1 − cos θ)2 dθ
2 0
θ=0 r=0 0 0
Zπ Zπ
1 + cos 2θ
= a2 (1 − cos2 θ − 2 cos θ)dθ = a2 (1 + − 2 cos θ)dθ
2
0 0
Zπ !π !
3 cos 2θ 3 1 sin 2θ
=a 2
( + − 2 cos θ)dθ = a2 (θ)π0 + − 2(sin θ)π0
2 2 2 2 2 0
0

ww 3πa2
!

= a2 = .
2 2

w.E
Example 4.56. Find the area of one loop of the lemniscate of Bernoulli
r2 = a2 cos 2θ.

asy
Solution. The curve is symmetric about both the axes. One loop lies to the right
of y-axis and symmetric about the x-axis.


En
∴ Area = 2× area of the portion which lies above the axis.
π
Along the region r varies√from 0 to a cos 2θ and θ varies from 0 to .

∴ Area = 2
π
Z4 a Zcos 2θ
rdrdθ gin 4

ee θ


4

θ=0 r=0
π/

s
co
θ=

π
!a √cos 2θ a2

rin
4 =
P
r2
r2
Z
=2 dθ
2

g.n
0
θ=0 r
π
O
Z 4

=a
0

2
a2 cos 2θdθ

sin 2θ
! π4
=
a2  π  a2
sin − 0 = .
θ = −π/4

et
2 0 2 2 2

Example 4.57. Find the area between r = 2 cos θ andπ r = 4 cos θ. π


Z2 Z2
Solution.π r=4 cos θ 1
Z2 Z
π
Z2 !4 cos θ = (16 cos2 θ − 4 cos2 θ)dθ = 6 cos2 θdθ
r 2 2
−π −π
Area = rdrdθ = dθ 2 2
2 2 cos θ
θ= −π
2
r=2 cos θ θ= −π
2

Download From : www.EasyEngineering.net


Download From : www.EasyEngineering.net

286 Engineering Mathematics - I

π
θ
Z2 π
θ= P
2 1π 2
=6×2 cos θdθ = 12 × = 3π.
22
0
O
r

θ = − π2

wwExample 4.58. Find the area inside the circle r = a sin θ but lying outside the
cardioid r = a(1 − cos θ). [Jan 2009]

w.E
Solution. The equation of the given curves are r = a sin θ, r = a(1 − cos θ). Solving,
we get a sin θ = a(1 − cos θ) ⇒ sin θ = 1 − cos θ.

asy
⇒ sin θ + cos θ = 1 ⇒ (sin θ + cos θ)2 = 1.
sin2 θ + cos2 θ + 2 sin θ cos θ = 1 ⇒ 1 + sin 2θ = 1

En
π
⇒ sin 2θ = 0 ⇒ 2θ = 0 or 2θ = π ⇒ θ = 0, θ = .
2
θ
The shaded portion is the required
region. Consider the radius vector OP. gin θ = π/2

The limits for r and θ as OP traverses


the required region is as follows.
ee r
θ=π
rin
P
Q

r = a sin θ

varies from a(1 − cos θ) to a sin θ and θ


π
varies from 0 to .
2
O

g.n
θ=0
r

Area =
Z
π
2 Z sin θ
r=a

θ=0 r=a(1−cos θ)
rdrdθ =
Z
π
2

θ=0
2 a sin θ
r
2
!

a(1−cos θ)

r = a(1−cos θ)

et
π π
Z2  Z2 
1 2 2 2 2 a2  
= a sin θ − a (1 − cos θ) dθ = sin2 θ − 1 − cos2 θ + 2 cos θ dθ
2 2
0 0
2 2 a2
!
a 1π π 1π a  π 
= − − + 2.1 = 2− = (4 − π).
2 22 2 22 2 2 4

Download From : www.EasyEngineering.net


Download From : www.EasyEngineering.net

Multiple Integrals 287

Example 4.59. Find the area which is inside the circle r = 3a cos θ and outside the
cardioid r = a(1 + cos θ). [Jan 2013]
Solution. The shaded portion is the required area. Solving the two equations we
get the points of intersection, r = 3a cos θ and r = a(1 + cos θ).

3
θ

π/
=
θ
∴ 3a cos θ =a(1 + cos θ)
r
3 cos θ =1 + cos θ (3a/2,0) (3a, 0)

ww 2 cos θ =1
1 π π
Z3
π
3aZcos θ
π
Z3 "
r2
#3a cos θ

w.E
Required area
2
⇒ θabove
cos θ==2 × area = , − θ .== 2
3 3
π
θ=0 r=a(1+cos θ)
rdrdθ. = 2
θ=0
2 r=a(1+cos θ)

asy
Z 3

=2 [9a2 cos2 θ − a2 (1 + cos θ)2 ]dθ

En
0
π
Z3
=2
0
π gin
(9a2 cos2 θ − a2 − a2 cos2 θ − 2a2 cos θ)dθ

= 2a2
Z3

0
π
ee
(9 cos2 θ − 1 − cos2 θ − 2 cos θ)dθ

rin
g.n
Z3
= 2a2 (8 cos2 θ − 2 cos θ − 1)dθ

et
0
π
Z3 !
2 8(1 + cos 2θ)
= 2a − 2 cos θ − 1 dθ
2
0
π
Z3
= 2a2 (4 + 4 cos 2θ − 2 cos θ − 1)dθ
0
π
Z3
= 2a2 (3 + 4 cos 2θ − 2 cos θ)dθ
0

Download From : www.EasyEngineering.net


Download From : www.EasyEngineering.net

288 Engineering Mathematics - I

 π π π 
 Z3 Z3 Z3 
2 
= 2a 3 dθ + 4 cos 2θdθ − 2 cos θdθ

 
0 0 0
 ! π3 
2
 π
3
sin 2θ π
3

= 2a 3(θ)0 + 4 − 2(sin θ)0 
2 0
π !  π !
2
 2π
= 2a 3 − 0 + 2 sin − sin 0 − 2 sin − sin 0
3 3 3
 √ √ 
3 3 
= 2a2 π + 2 ·  = 2πa2 Sq.units.

−2·
2 2

ww4.6 Triple integral in Cartesian coordinates

w.E
Let the function f (x, y, z) be defined in a specific region V in space for all x, y and
z with proper limits. The triple integral of f (x, y, z) w.r.t. x, y and z is defined as
Zz1 Zy1 Zx1
f (x, y, z)dxdydz. asy
z0 y0 x0

En
The evaluation of the triple integral is as follows.

gin
(i) If all the limits are constants, then the integration can be performed in any
order with proper limits.

(ii) Consider
g2 (z) fZ
Zb Z 2 (y,z)

ee
f (x, y, z)dxdydz.
a y=g1 (z) x= f1 (y,z)
rin
g.n
First we integrate with respect to x treating y and z as constants. Next we integrate
the resulting function of y and z w.r.t.y treating z as constant. Finally integrate the
resulting function of z w.r.tz.

Z1 Z2 Z2

Worked Examples



et
Example 4.60. Evaluate x2 yzdxdydz.
0 0 1
Solution.
Z1 Z2 Z2 Z1 Z2 !2
2 x3
x yzdxdydz = yz dydz
3 1
0 0 1 0 0

Download From : www.EasyEngineering.net


Download From : www.EasyEngineering.net

Multiple Integrals 289

Z1 Z2
1
= yz(8 − 1)dydz
3
0 0
Z1 !2 Z1
7 y2 7
= z dz = z4dz
3 2 0 6
0 0
!1
28 z2
=
6 2 0
28 1 7
= = .
6 2 3

ww Za Zb Zc

w.E
Example 4.61. Find

Solution.
0 0 0
(x2 + y2 + z2 )dxdydz. [Jun 2009]

Za Zb Zc asy Za Zb
x3
!c

0 0 0
2 2

En
2
(x + y + z )dxdydz =
0 0
3
+ y2 x + z2 x dydz
0

gin =
Za Zb
c3 2 2
!
+ cy + cz dydz

ee
3
0 0
Za  3

rin
!b 
 c b y3 2 b

=  3 (y)0 + c 3 + cz (y)0  dz
0

=
0
Za
c3 b cb3
+ 2
!
+ cz b dz g.n
=
3
0

(z)0 +
3

bc3 a cb3 a
3
3

(z)0 + bc
z3
3
!a

0
et
abc3 ab3 c a3 bc abc 2
= + + = (a + b2 + c2 ).
3 3 3 3

log 2Zx Z
Z x+y

Example 4.62. Evaluate e x+y+z dxdydz. [Jan 2009]


0 0 0

Download From : www.EasyEngineering.net


Download From : www.EasyEngineering.net

290 Engineering Mathematics - I

Solution.
log 2Zx Z
Z x+y log 2Zx Z
Z x+y log 2Zx
Z
 x+y 
 Z 
e x+y+z dxdydz = e x ey ez dxdydz = e x ey  ez dz dxdy
 
 
0 0 0 0 0 0 0 0 0
Z 2Zx
log Z 2Zx
log
x+y
= e x ey (ez )0 dxdy = e x ey (e x+y − 1)dxdy
0 0 0 0
Z 2Zx
log

= (e2x+2y − e x ey )dxdy

ww =
0 0
Z 2 Zx
log 


e2x e2y dy −
Zx
x y 


e e dy dx

w.E
 
0 0 0
log 2 !x
e2y
Z !
e2x x y x

asy
= − e (e )0 dx
2 0
0
Z 2
log

En e2x 1
! !
2x x x
= e − − e (e − 1) dx
2 2

gin 0
log 2
Z
e4x e2x 2x x
!

ee
= − − e + e dx
2 2
0

=
Z 2
log
e4x 3e2x
− x
!
+ e dx rin
0

1 e4x
!log 2
2 2

3 e2x
!log 2
g.n
et
log 2
= − + (e x )0
2 4 0 2 2 0
1 3
= [16 − 1] − (4 − 1) + (2 − 1)
8 4
15 9
= − +1
8 4
15 − 18 + 8 5
= = .
8 8
Z1 Z1−xZx+y
Example 4.63. Evaluate ez dzdydx. [Jan 1996]
0 0 0

Download From : www.EasyEngineering.net


Download From : www.EasyEngineering.net

Multiple Integrals 291

Solution.
Z1 Z1−xZx+y Z1 Z1−x Z1 Z1−x
z z  x+y
e x+y − 1 dydx

e dzdydx = e 0 dydx =
0 0 0 0 0 0 0
1
Z Z1−x Z1  Z1−x Z1−x 
 
e x ey − 1 dydx =
 x
ey −
 
= e dy dx
 
0 0 0 0 0
Z1  
1−x
= e x ey 0 − (y)1−x
0 dx

ww
0
Z1   Z1
x 1−x
= e (e − 1) − (1 − x) dx = (e − e x − 1 + x)dx

w.E 0

= e(x)10 − (e x )10 − (x)10 +


x2
2
!1
0

= e − (e − 1) − 1 +
1
2

asy 1 1
=e−e+1−1+ = .
2 2
0

En
Z3 Z1 Z xy

gin
Example 4.64. Evaluate xydzdydx. [Jan 2001]
1 1 0
x
Solution.

Z3 Z1 Z xy
xydzdydx =
Z3 Z1
ee

xy
xy(z)0 dydx =
Z3 Z1

xy xydydx =
Z3 Z1
3 3
x 2 y 2 dydx
rin
g.n
1 1 0 1 1 1 1 1 1
x x x x

Z3  5 1
 y 2  Z3  ! 25 
3 2 3 
 1 

et
= x  5  dx =
2 x 2 1 −  dx
2 1
5 x 
1 x 1
Z3  5 3 ! 
2 1 2  x 2 
3

= x − dx =  5  − (log x)31 
2
5 x 5 2 1
1
2 2 √
! !
2 2 5
= (3 2 − 1) − log 3 = (9 3 − 1) − log 3 .
5 5 5 5
2
Z1 Z1−x (x+y)
Z
Example 4.65. Evaluate xdzdydx. [Jan 1996]
0 0 0

Download From : www.EasyEngineering.net


Download From : www.EasyEngineering.net

292 Engineering Mathematics - I

Solution.
2
Z1 Z1−x (x+y)
Z Z1 Z1−x Z1 Z1−x
(x+y)2
xdzdydx = x(z)0 dydx = x(x + y)2 dydx
0 0 0 0 0 0 0
Z1 3 1−x
! Z1
(x + y) 1
= x dx = x((x + 1 − x)3 − x3 )dx
3 0 3
0 0
Z1 Z1  !1 !1 
1 1 3 4 1  x2 x5 
= x(1 − x )dx = (x − x )dx =  −

ww

3 3 3 2 0 5 0
0 0
! !
1 1 1 1 5−2 1
= − = = .

w.E log 2Zx x+log


Z Z y
3 2 5 3 10 10

Example 4.66. Evaluate

Solution. asy
e x+y+z dzdydx.
0 0 0
[Jun 2013]

log 2Zx x+log


Z y En
log 2Zx

gin
Z Z
 x+y+z  x+log y
e x+y+z dzdydx = e 0 dydx
0 0 0 0 0

=ee Z 2Zx
log

0 0
 
e x+y+x+log y − e x+y dydx.

rin
=
Z 2Zx
log
 
e2x ey+log y − e x · ey dydx
g.n
=
0 0
Z 2Zx
log

0 0
 
e2x · ey · elog y − e x ey dydx
et
Z 2Zx
log
 
= e2x yey − e x ey dydx
0 0
log 2
 Zx Zx

Z 
 2x y x y 
= e ye dy − e e dy  dx


0 0 0

Download From : www.EasyEngineering.net


Download From : www.EasyEngineering.net

Multiple Integrals 293

log 2
 Zx

Z 
 2x y x y x
= e yd(e ) − e (e )0  dx


0 0
log 2
Z 

Zx
 

   
 2x  y  x
 y   − e x (e x − 1)

= 
e 

 ye 0 − e dy  
 dx

   

0 0
Z 2
log
 
x 
= e2x xe x − ey 0 − e2x + e x dx
0
log 2

ww
Z
= {e2x (xe x − e x + 1) − e2x + e x }dx
0

w.E =
Z 2
log
 
xe3x − e3x + e2x − e2x + e x dx

asy
0
log 2
Z  
= xe3x − e3x + e x dx

En 0
Z 2
log log 2 log 2

gin
Z Z
3x 3x
= xe dx − e dx + e x dx.
0 0 0

=
Z
ee
log 2

0
xd
e3x
3
!

e3x
3 0
!log 2
log 2
+ ex 0

rin
=
xe3x
3 0
!log 2 Z

log 2
e3x
3
1
3

dx − e3 log 2 − 1 + (elog 2 − 1)g.n
= log 2 ·
8
30

!
e
1
0
3x
!log 2
− [8 − 1] + (2 − 1)
39
et
8 8 1 7
= log 2 − − − +1
3 9 9 3
!
8 7 7
= log 2 − + −1
3 9 3
!
8 7 + 21 − 9
= log 2 −
3 9

Download From : www.EasyEngineering.net


Download From : www.EasyEngineering.net

294 Engineering Mathematics - I

8 19
= log 2 − .
3 9
$
dxdydz
Example 4.67. Evaluate p for all positive values of x, y, z for
1 − x2 − y2 − z2
which the integral is real. [Jan 2013, Jan 2012]
Solution. The integral is real for all values of x2 + y2 + z2 < 1.
√ √
$ 1−x 2 2
Z1 Z1−x2 Z −y
dxdydz 1
∴ p = p dzdydx
1 − x2 − y2 − z2 1 − x2 − y2 − z2

ww 0 y=0

Z1 Z1−x2
z=0

 −1 z
 √1−x2 −y2


w.E = sin p  dydx


1 − x2 − y2 0
0 0

Z Z1−x2
1

asy =
0 0
(sin−1 1 − sin−1 0)dydx

En

Z1 Z1−x2
π
= dydx

gin 0
Z1
0
2

ee

π 2
= [y]0 1−x dx
2

π
Z1 p
0

rin
=
2
1 − x2 dx
0
g.n
et
" #1
π xp 2
1 −1
= 1 − x + sin x
2 2 2 0
" #
π 1 −1
= sin 1 − 0
2 2
π 1 π π2
= · · = .
2 2 2 8
√ √ 2 −x2 −y2
Za Za2 −x2 aZ
1
Example 4.68. p dzdydx. [Dec 2011]
a2 − x2 − y2 − z2
0 0 0

Download From : www.EasyEngineering.net


Download From : www.EasyEngineering.net

Multiple Integrals 295

Solution.
√2 2 2
 √a2 −x2 −y2
√ √
Za Za2 −x2 aZ−x −y Za Za2 −x2
1  −1 z 
p dzdydx = sin p  dydx
a2 − x2 − y2 − z2 a2 − x2 − y2 0

0 0 0 0 0

Za Za2 −x2  
= sin−1 1 − sin−1 0 dydx
0 0

Za Za2 −x2
π
= dydx.

ww 0

π
Za
0

[y]0 a
2

2 −x2

w.E =
2

π
0
Za p
dx

asy =
2
0
a2 − x2 dx

#a

En
" 2
π a −1 x xp 2 2
= sin + a −x
2 2 a 2 0

gin π a2 −1
" #
= sin 1 − 0
2 2

Example 4.69. Evaluate


$ ee dxdydz
=
πa2 π π2 a2
4 2
· =
8
.

, where V is the region bounded by rin


V
x = 0, y = 0, z = 0, and x + y + z = 1.
(x + y + z + 1)3

g.n
[Jan 2014, Dec 2011]
Solution. The limits for x, y and z are as follows.

z : 0 to 1 − x − y
et
y : 0 to 1 − x

x : 0 to 1

$ Z1 Z1−x 1−x−y
Z
dzdydz
∴ = (1 + x + y + z)−3 dzdydx
(1 + x + y + z)3
V 0 0 0

Download From : www.EasyEngineering.net


Download From : www.EasyEngineering.net

296 Engineering Mathematics - I

Z1 Z1−x" #1−x−y
(1 + x + y + z)−2
= dydx
−2 0
0 0
Z1 Z1−x
1 
=− 2−2 − (1 + x + y)−2 dydx
2
0 0
Z1 Z1−x !
1 1 −2
=− − (1 + x + y) dydx
2 4
0 0
Z1  #1−x 

ww (1 + x + y)−1
"
1  1 1−x 
=−  [y]0 −  dx
2 4 −1 0
0

w.E =−
1
2
Z1

0
1
4
!
(1 − x) + 2−1 − (1 + x)−1 dx

asy !1
 
1  1 1 1 x2 1 1 1

= −  (x)0 − · + [x]0 − (log(1 + x))0 
2 4 4 2 0 2

=−
En
1 1 1 1
2 4 8 2
− + − log 2
!

=−
1 2−1+4
2 gin8
− log 2
!

=−
1 5
2 8
$
ee !
1
− log 2 = log 2 − .
2
5
16

rin
g.n
Example 4.70. Find the value of xyzdxdydz through the positive spherical
octant for which x2 + y2 + z2 ≤ a2 . [Jan 2014, Jun 2010]
Solution. The limits for x, y, z are as follows.
q
z : 0 to a2 − x2 − y2
p
et
y : 0 to a2 − x2

x : 0 to a.

√ √
$
2 −x2 −y2
aZ
Za Za2 −x2
∴ xyzdxdydz = xyzdzdydx.
x=0 y=0 z=0

Download From : www.EasyEngineering.net


Download From : www.EasyEngineering.net

Multiple Integrals 297

√ √
Za Za2 −x2 a2 −x2 −y2
z2
" #
= xy · dydx.
2 0
x=0 y=0

Za Za2 −x2
1
= xy · (a2 − x2 − y2 )dydx
2
0 0

Za Za2 −x2
1  
= a2 xy − x3 y − xy3 dydx
2
0 0
# √a2 −x2

ww =
1
2
Za "
a2 x ·
0
y
2
2 y2
− x3 − x
2
y4

4 y=0
dx

w.E =
1
2
Za
a2 x 2
2
2 x3 2
2
2 x 2
4
2 2
!
(a − x ) − (a − x ) − (a − x ) dx

asy
0
Za
a4 x a2 x3 a2 x3 x5 x 4
!
1 4 2 2
= − − + − (a + x − 2a x ) dx

En
2 2 2 2 2 4
0
Za
a4

gin x5 a4 x5 a2 3
!
1 2 3
= x−a x + − x− + x dx
2 2 2 4 4 2
0

ee
!a 4 a
!a !a !a !a !
1 a4 x2 1 x6 a4 x2 1 x6 a2 x4
!
2 x
= −a + − − +

rin
2 2 2 0 4 0 2 6 0 4 2 0 4 6 0 2 4 0
1  a6✁ a6✁ a6 a6✁ a6 a6✁
 
=  ✁ − ✁ + − ✁ − + ✁ 

=
2 ✁4 ✁4
1
×a 6 1

12 ✁8
1
!
24 ✁8
g.n
=
2
a6 2 − 1
2
6
24
12 24
!
et
a
= .
48

Download From : www.EasyEngineering.net


Download From : www.EasyEngineering.net

298 Engineering Mathematics - I

4.7 Volume as triple integral

Result. If V is the volume bounded by a definite$ region D in space, then the


volume is calculated by the triple integral V = dxdydz with proper limits.
D

x2 y2 z2
Example 4.71. Find the volume of the ellipsoid + + = 1 using triple
a2 b2 c2

ww
integral.
Solution. We know that the ellipsoid is symmetric about the
$ coordinate planes.

w.E
∴ Volume of the ellipsoid = 8×Volume in the first octant = 8

x2 y2 z2
The ellipsoid is given by 2 + 2 + 2 = 1
V
dxdydz.

a
asy
b c r
In the first octant, z varies from 0 to c 1−
x2 y2

a2 b2
r
y varies from 0 to b 1 − 2
x2
En
gin
a
and x varies from 0 to a.

∴ Volume = 8
a
q
Z Z a
2
b 1− x2 c
r

eeZ a
2
1− x2 − y2
2
b

dzdydx
rin
x=0 y=0
q
b 1− x
Za Z a2
2
z=0

r g.n
et
2 2
c 1− x2 − y2
a b
=8 (z)z=0 dydx
x=0 y=0
q
2
b 1− x
Za Z a2 r
x2 y2
= 8c 1− − dydx
a2 b2
x=0 y=0
q
2
b (1− x )
Za Z a2 s !
1 x2
= 8c b2 1 − 2 − y2 dydx
b a
x=0 y=0

Download From : www.EasyEngineering.net


Download From : www.EasyEngineering.net

Multiple Integrals 299

q
2
 b 1− x2
Za  2 x2
s a
 b (1 − ) 2
! 
8c a2 −1 y y x 
=  sin q  +2 b2 1 − 2 − y2  dx
b 2 x 2 a
b2 1 −
 
x=0 a2 0
Za b2 1 − 
x2
8c a2
= sin−1 1dx
b 2
0
Za
x2
!
π
= 4bc 1 − 2 dx
2 a
0
!a

ww = 2πbc x − 2

x3
3a 0
a  4πabc

w.E = 2πbc a −
3
=
3
.

x2 y2 z2

1
asy
Note. The volume of the portion of the ellipsoid 2 + 2 + 2 = 1 which lies in
a b c
1 4πabc πabc
the first octant is × Volume of the ellipsoid = × = .
8

En 8 3 6

gin
Example 4.72. Find the volume of the sphere x2 +y2 +z2 = a2 using triple integrals.
[Jan 2006]

ee
Solution. We know that the sphere is symmetric about the coordinate planes.
∴ Volume of the sphere = 8× Volume of the portion of the sphere
that lies in the first octant. rin
The sphere is given by x2 + y2 + z2 = a2 .
g.n
et
p
In the first octant z varies from 0 to a2 − x2 − y2 .

y varies from 0 to a2 − x2 .
and x varies from 0 to a.
$
Volume of the sphere = 8 × dzdydx.
V
√ √ 2 −x2 −y2
Za Za2 −x2 aZ

=8 dzdydx
x=0 y=0 z=0

Download From : www.EasyEngineering.net


Download From : www.EasyEngineering.net

300 Engineering Mathematics - I


Za Za2 −x2 √2 2 2
a −x −y
=8 (z)0 dydx
x=0y=0

Za Za2 −x2 q
=8 a2 − x2 − y2 dydx
x=0 y=0

Za ! a2 −x2
a2 − x2 y y
q
=8 sin−1 √ + 2 2
a −x −y2 dx
2 a2 − x2 2 0
x=0
Za

ww =8
x=0
a2 − x2 −1
2
sin (1)dx

w.E =4
π
2
Za

x=0
(a2 − x2 )dx

asy
= 2π a x −
x3
2
3 0
!a

En 3
= 2π a −
a
3
3
!
=
4πa3
3
.

gin
Note
ee
The volume of the portion of the sphere that lies in the first octant is
1 1 4πa3 πa3 rin
8
× Volume of the sphere = ×
8 3
=
6
.

g.n
Example 4.73. Find the volume of the tetrahedron bounded by the plane
x y z
et
+ + = 1 and the coordinate planes. [Jun 2006]
a b c
Solution. Let D be the region in space bounded by the tetrahedron.
x y z
The region of integration is given by + + = 1, x = 0, y = 0, z = 0.
a b c  x y
Along the region of integration z varies from 0 to c 1 − − .
a b
 x
y varies from 0 to b 1 − .
a

Download From : www.EasyEngineering.net


Download From : www.EasyEngineering.net

Multiple Integrals 301

and x varies from 0 to a.


x x y
Za b(Z1− a ) c(1−
Za−b)
∴ Volume = dzdydx.
x=0 y=0 z=0
1− ax
Za b(Z )
c(1− x − y )
= (z)0 a b dydx.
x=0 y=0
x
Za b(Z1− a ) 
x y

ww =
x=0
Za
y=0
c 1 − − dydx.
a b

!b(1− ax )

w.E =c
x=0
y−
xy y2
a

2b 0
dx.

asy
Za n  x x  x 1 2  x 2 o
=c b 1− − b 1− − b 1− dx
a a a 2b a

En
x=0
Za 
bc x 2
= 1− dx
2

gin
x=0
a
 a
 1 − x 3 

=

=
bc 

−abc
ee
  a  
2  3 −1 
a

[0 − 1]
0

rin
=
6
6
abc
. g.n
Example 4.74. Find the volume of the portion of the cylinder x2 +y2 = 1 intercepted
between the plane x = 0 and the paraboloid x2 + y2 = 4 − z.
et[Jun 2012]
Solution. The surfaces are x2 + y2 = 1, x = 0 and x2 + y2 = 4 − z.
For the required volume integral
x varies from 0 to 1

y varies from 0 to 1 − x2
z varies from 0 to 4 − x2 − y2 .

Download From : www.EasyEngineering.net


Download From : www.EasyEngineering.net

ww
w.E
a syE
ngi
nee
rin
g.n
et

**Note: Other Websites/Blogs Owners Please do not Copy (or) Republish


this Materials, Students & Graduates if You Find the Same Materials with
EasyEngineering.net Watermarks or Logo, Kindly report us to
easyengineeringnet@gmail.com Download From : www.EasyEngineering.net
Download From : www.EasyEngineering.net

302 Engineering Mathematics - I

√ 2 −y2
Z1 Z1−x2 4−x
Z
Volume = 4 dzdydx
x=0 y=0 z=0

2
Z1 Z1−x
4−x2 −y2
= 4 [z]0 dydx
x=0 y=0

Z1 Z1−x2
=4 (4 − x2 − y2 )dydx

ww =4
x=0 y=0
Z1  √
2

2
4(y)0 1−x − x2 (y)0 1−x −
 y3
! √1−x2 

 dx.

w.E x=0
3 0 

x = sin θ.
Z1

asy
(1 − x2 )3/2
p !
2 2
=4 (4 − x ) 1 − x − dx. dx = cos θdθ.
3

En
0
When x = 0, sin θ = 0 ⇒ θ = 0.
Zπ/2
(cos2 θ)3/2
!
π
(4 − sin2 θ) cos θ − When x = 1 sin θ = 1 ⇒ θ = .

gin
=4 cos θdθ
3 2
0
Zπ/2

ee
cos3 θ
!
2
=4 4 cos θ − sin θ cos θ − cos θdθ
3
0

rin
=4
Zπ/2
4 cos2 θ − sin2 θ cos2 θ −
cos4 θ
3
!
dθ g.n
=4
0
Zπ/2
4 cos2 θ − sin2 θ(1 − sin2 θ) −
cos4 θ
3
!

et
0
Zπ/2
cos4 θ
!
2 2 4
=4 4 cos θ − sin θ + sin θ − dθ
3
0
 π/2
Zπ/2 Zπ/2 Zπ/2

 Z 
1
= 4 4 cos2 θdθ − sin2 θdθ + sin4 θdθ − cos4 θdθ
 
 3 
0 0 0 0

Download From : www.EasyEngineering.net


Download From : www.EasyEngineering.net

Multiple Integrals 303

" #
1 π 1 π 3 1 π 1 3 1 π
=4 4· · − · + · · − · · ·
2 2 2 2 4 2 2 3 4 2 2
" #
1 π 3 1
=4· · 4−1+ −
2 2 4 4
" #
3 1
7=π 3+ −
4 4
" #
1
=π 3+
2

= Cubic units.

ww 2

Example 4.75. Find the volume bounded by the cylinder x2 +y2 = 4 and the planes

w.E
y + z = 4 and z = 0.
Solution.
[May 2011]

asy
En
The given surface is x2 + y2 = 4, z = 0 and
y + z = 4.
In the region, x varies from −2 to 2,
√ √ gin
ee
y varies from − 4 − x2 to 4 − x2 , z
varies from 0 to 4 − y.
Z2

Z4−x2 Z4−y
rin
∴ Volume =

x=−2 y=− 4−x2 z=0
dzdydx

g.n
et

Z2 Z4−x2
4−y
= [z]0 dydx.

x=−2 y=− 4−x2

Z2 Z4−x2
= (4 − y)dydx.

x=−2 y=− 4−x2

Z2  √ " 2 # √4−x2 


2 y 
4(y) √4−x 2 −
  dx.
= √
− 4−x 2 − 4−x2 
x=−2

Download From : www.EasyEngineering.net


Download From : www.EasyEngineering.net

304 Engineering Mathematics - I

Z2 "  p p  1 #
2 2 2 2
= 4 4 − x + 4 − x − 4 − x − (4 − x ) dx.
2
−2
Z2 p
= 8 4 − x2 dx.
−2
Z2 p
=8×2 4 − x2 dx.
0

ww
" p #2
x 2
4 −1  x 
= 16 4 − x + sin
2 2 2 0

w.E
h i
= 16 2 sin−1 1
π
= 16 × 2 ×
2
= 16π Cubic units.
asy
En
Example 4.76. Find the volume of the solid bounded by the cylinders x2 + y2 = a2
and x2 + z2 = a2 .
gin
Solution. The surfaces are the cylinders x2 + y2 = a2 and x2 + z2 = a2 .

In the given region,


ee rin
x varies from −a to a,
√ √ g.n
y varies from − a2 − x2 to a2 − x2 and



z varies from − a2 − x2 to a2 − x2 .


et
Za Za2 −x2 Za2 −x2
Volume = dzdydx.
√ √
x=−a y=− a2 −x2 z=− a2 −x2

Za Za2 −x2 √
2 2
= [z] √a −x dydx.
− a −x2
2

x=−a y=− a2 −x2

Download From : www.EasyEngineering.net


Download From : www.EasyEngineering.net

Multiple Integrals 305


Za Za2 −x2  p p 
= a2 − x2 + a2 − x2 dydx.
x=−a y=− √a2 −x2

Za Za2 −x2 p
= 2 a2 − x2 dydx
x=−a y=− √a2 −x2

Za p √
2 2
=2 a2 − x2 (y) √a −x
2 2
dx
− a −x
x=−a

ww =2
Za p
2
a −x 2
p
2 2
p
2 2

a − x + a − x dx

w.E
x=−a
Za p p
=2 a2 − x2 · 2 a2 − x2 dx.

asy
x=−a
Za !a #
x3
"
2 2 2 a
=4 (a − x )dx. = 4 a · (x)−a −
x=−a
"
1 3
#
En "
3 −a

2a3
#
4a3 16a3

gin
2 3 3
= 4 a (a + a) − (a + a ) = 4 2a − =4× = Cubic units.
3 3 3 3

and x2 = 4ay and the plane z = 0 and z = 3. ee


Example 4.77. Find the volume of the region bounded by the surfaces y2 = 4ax

Solution. Let us find the common points of the region bounded by


rin
y2 = 4ax g.n (1)

and x2 = 4ay.

(2) ⇒ y =
x2
4a
.
et (2)

Substituting in (1) we get


x4
= 4ax.
16a2
x4 = 64a3 x.

x4 − 64a3 x = 0

Download From : www.EasyEngineering.net


Download From : www.EasyEngineering.net

306 Engineering Mathematics - I

x(x3 − 64a3 ) = 0

x = 0, x3 = 64a3

x = 4a.

For the given region,


x varies from 0 to 4a,
x2 √
y varies from to 4ax,
4a
z varies from 0 to 3.

ww ∴ Volume =

Z4a Z4axZ3
dzdydx

w.E x=0 y= x2 z=0



4a

Z4a Z4ax

Z4a Z4ax

asy
3
= [z]0 dydx. = 3dydx.
x=0 y= x2 x=0 y= x2
4a 4a

=3 En
Z4a √
[y] x24ax dx.

gin
x=0
Z4a √
4a

x2
!
=3


0

= 3 2 a
ee"
4ax −

 √ x3/2 4a 1
#
4a


dx.

·
" 3 #4a 
x 
rin
g.n

3/2 0 4a 3 0 
!
√ 2 3/2 1 3
= 3 2 a (4a) − · 64a

=3

4 a
3
3

· 4a 4a −
12a
16a2
3
!
et
16a2
!
16a √ √
=3 a·2 a−
3 3
2 2 16a2
!
32a 16a
=3 − =3× = 16a2 Cubic units.
3 3 3
Example 4.78. Find the volume of the solid bounded by the surfaces y2 = x and
the planes x + y + z = 2 and z = 0.

Download From : www.EasyEngineering.net


Download From : www.EasyEngineering.net

Multiple Integrals 307

Solution. The given region is bounded by


y2 = x (1)

x+y+z=2 (2)

z = 0. (3)

Whenz = 0, (2) ⇒ x + y = 2

x = 2 − y.

Substituting in (1) we get

ww y2 = 2 − y

y2 + y − 2 = 0

w.E (y − 1)(y + 2) = 0

∴ y = −2, 1.

For the given region we have,


asy
y varies from −2 to 1,
x varies from y2 to 2 − y and En
z varies from 0 to 2 − x − y.
gin
∴ Volume =
Z1 Z2−y 2−x−y
Z
dzdxdy
y=−2 x=y2 z=0

Z1 Z2−y
ee rin
=
2−x−y
[z]0 dxdy
g.n
et
y=−2 x=y2

Z1 Z2−y
= (2 − x − y)dxdy
y=−2 x=y2

Z1  " 2 #2−y 
 2−y x 2−y 

= 2 · [x]y2 − − y[x]y2  dy
2 y2
y=−2
Z1 !
2 1 2 4 2
= 2{2 − y − y } − {(2 − y) − y } − y{2 − y − y } dy.
2
−2

Download From : www.EasyEngineering.net


Download From : www.EasyEngineering.net

308 Engineering Mathematics - I

Z1 !
1
= 4 − 2y − 2y2 − [4 + y2 − 4y − y4 ] − [2y − y2 − y3 ] dy
2
−2
Z1
y2 y4
!
2 2 3
= 4 − 2y − 2y − 2 − + 2y + − 2y + y + y dy
2 2
−2
Z1
3y2 y4
!
3
= 2 − 2y − +y + dy
2 2
−2
#1 " #1 " 4 #1 " #1
y2 3 y3 1 y5
"
y

ww = 2[y]1−2 −2
2−2

2 3 −2
1
+

1
+
4 −2 2 5 −2
1
= 2(1 + 2) − (1 − 4) − (1 + 8) + (1 − 16) + (1 + 32)

w.E =6+3− −
9 15 33
2 4
+
10
2 4 10

=9− −
2
asy
9 15 33 180 − 90 − 75 + 66 81
4
+
10
=
20
=
20
Cubic units.

En
gin
ee rin
g.n
et

Download From : www.EasyEngineering.net


Download From : www.EasyEngineering.net

5 Differential Equations

5.1 Introduction
wwA differential equation is an equation involving one dependent variable and its

w.E
derivatives with respect to one or more independent variables.
Ordinary Differential Equation

asy
An ordinary differential equation is an equation in which there is only one
independent variable and so the derivatives involved in it are ordinary
derivatives.
En
Partial differential equation
gin
A partial differential equation is an equation in which there are two or more

of them.
ee
independent variables and partial differential coefficients with respect to any one

rin
Examples
∂2 u
(i) 2 = a2 2 .
∂x
∂2 u
∂t g.n
(ii) x
2
2∂ u
∂x2
∂u
(iii) x + y
∂x
+ 2xy
∂u
∂y
∂2 u
∂x∂y
= 5u.
+ y
2
2∂ u
∂y2
= 0.
et
Order of a differential equation
The order of a differential equation is the order of the highest derivative that
occurs in it.
Degree of a differential equation
The degree of a differential equation is the degree of the highest derivative

Download From : www.EasyEngineering.net


Download From : www.EasyEngineering.net

310 Engineering Mathematics - I

occurring in it after the equation has been reduced to a form free from radicals
and fractions as far as the derivatives are concerned.
Examples
d2 x
(i) For the differential equation + a2 x = 0, the order is 2 and the degree is 1
dt2
 !2  23 2
 dy  = c d y , the order is 2 and degree is 2,

(ii) For the differential equation 1 +
dx dx2
 !2 3 2y 2
!
 dy  2 d
since it can be reduced to the form 1 +  = c .
dx  dx2

wwFormation of a differential equation


An ordinary differential equation can be formed by eliminating certain arbitrary

w.E
constants from a relation involving variables and constants.
applied mathematics, every geometrical and physical problems when translated
In the study of

asy
into a mathematical model will always give rise to a differential equation.


Worked Examples

En
Example 5.1. Form the differential equation of the family of straight lines passing
through the origin.
gin
Solution. The equation of the family of straight lines passing through the origin
is given by
ee y = mx.
rin (1)

Differentiating w.r.t. x, we get


dy g.n
Substituting the value of m in (1) we obtain
dx

!
= m.

et
dy
y= x
dx

which is the required differential equation.

Example 5.2. Obtain the differential equation of the family of circles


x2 + y2 + 2ax + r2 = 0 by eliminating the arbitrary constant a.

Download From : www.EasyEngineering.net


Download From : www.EasyEngineering.net

Differential Equations 311

Solution. The given equation is

x2 + y2 + 2ax + r2 = 0. (1)

Differentiating w.r.t. x, we get


dy
2x + 2y + 2a = 0
dx
dy
x+y = −a
dx !
dy
a=− x+y .
dx

ww Substituting the value of a in (1) we obtain


2 2 dy
!
+ r2 = 0

w.E
x + y − 2x x + y
dx
dy
x2 + y2 − 2x2 − 2xy + r2 = 0
dx

asy 2xy
dy
dx
= y2 − x2 + r2 ,

En
which is the required differential equation.

gin
Example 5.3. Find the differential equation of the family of circles with centre
(a, b) and radius r.
Solution. The equation of the circle is

(x − a)2 + (y − b)2 = r2 .
ee rin (1)

g.n
dy
Differentiating w.r.t. x, we get 2(x − a) + 2(y − b) =0
dx
dy
x − a + (y − b)
dx
= 0.

Again differentiating we get


d2 y dy dy
et (2)

1 + (y − b) + =0
dx2 dx dx  !2 
d2 y  dy 
(y − b) 2 = − 1 + 
dx dx 
 dy 2
1 + dx
y−b=− 2
.
d y
dx2

Download From : www.EasyEngineering.net


Download From : www.EasyEngineering.net

312 Engineering Mathematics - I

Substituting this in (2) we get


 dy 2
1 + dx dy
x−a− 2
=0
d y dx
dx2
 dy 2
dy 1 + dx
x−a= .
dx d2 y
dx2

Substituting the values of x − a and y − b in (1) we obtain


   2   dy 2 2
!2 1 + dy 2

ww dy
dx  2
d y
dx

dx2
2
+
1 +
 2
d y
dx
2 = r2
dx2

w.E 
1+
 dy 2 2
dx


1 + dy
!2 

 = r2

asy
 2 dx 
d2 y
dx2
  dy 2 3

En 1+

d2 y
dx
2 = r2

gin 
dx2

dy
!2  32 2
 = r d y ,

ee
 
1 +
dx dx2

which is the required differential equation.


rin
Example 5.4. Form the differential equation of the simple harmonic motion
x = a cos nt. g.n
Solution. x = a cos nt.
Differentiate w.r.t. t we get
dx
et
= −an sin nt.
dt
Again differentiating w.r.t. t we obtain
d2 x
= an2 (− cos nt) = −n2 a cos nt = −n2 x
dt2
d2 x
+ n2 x = 0,
dt2
which is the required differential equation.

Download From : www.EasyEngineering.net


Download From : www.EasyEngineering.net

Differential Equations 313

Solution of a differential equation


A solution of a differential equation is a relation between the variables that
satisfies the given differential equation. A solution of a differential equation is
also called as the integral of the equation.
Examples
d2 x
1. x = a cos nt is a solution of the differential equation + n2 x = 0.
dt2
d2 y dy
2. For the differential equation 2
+ 3 + 2y = 0, y = e−x and y = e−2x are
dx dx

wwsolutions.
General Solution

w.E
A general or complete solution of a differential equation is the one in which the
number of arbitrary constants is equal to the order of the differential equation.
Example. Consider

asy
y = A cos αx + B sin αx (1)

En
where A and B are the arbitrary constants.

gin
Differentiating w.r.t. x we get

dy
dx
d2 y
= −Aα sin αx + Bα cos αx.
ee
= −Aα2 cos αx − Bα2 sin αx. rin
dx2
= −α2 (A cos αx + B sin αx) = −α2 y.
g.n
d2 y
dx2
+ α2 y = 0.
et
Hence, (1) is a general solution of (2) as the number of arbitrary constants A, B is
(2)

the same as the order of (2).


Particular Solution
A particular solution is a solution that can be obtained by giving particular
values to the arbitrary constants in the general solution.
Example. y = 2 cos αx + sin αx is a particular solution of (2), since it can be derived

Download From : www.EasyEngineering.net


Download From : www.EasyEngineering.net

314 Engineering Mathematics - I

dy
from the general solution by giving initial values of 0, 2 and α to x, y and
dx
respectively.
Linear independent solutions
Two solutions y1 and y2 of the differential equation

d2 y dy
+ a1 (x) + a2 (x)y = 0 (1)
dx2 dx

wware said to be linearly independent, if there exist constants c1 and c2 such that
c1 y1 + c2 y2 = 0 implies c1 = 0 and c2 = 0.

w.E
If c1 and c2 are not both zero, then the two solutions y1 and y2 are said to be linearly
dependent.

asy
If y1 and y2 are any two solutions of (1) then the linear combination c1 y1 + c2 y2
where c1 and c2 are constants is also a solution of (1).

En
Example. y1 = e−x and y2 = e−2x are two independent solutions of the differential
equation

gind2 y dy
+ 3 + 2y = 0.

ee
dx 2 dx

Then, the general solution is y = c1 e−x + c2 e−2x .


Linear Differential Equation rin
g.n
A linear differential equation is an equation in which the dependent variable and

variable and derivative or product of derivatives.


Note.
et
its derivatives occur only in the first degree and there is no product of dependent

A differential equation which is not linear is called a non linear


differential equation.
Examples
d2 y
1. x 2 + y = x2 is linear.
dx
dy
2. y + x2 = 0 is nonlinear.
dx

Download From : www.EasyEngineering.net


Download From : www.EasyEngineering.net

Differential Equations 315

5.2 Linear differential equation with constant coefficients

The general form of the nth order linear ordinary differential equation with
constant coefficients is
dn y dn−1 y dy
a0 n
+ a1 n−1
+ · · · + an−1 + an y = Q(x) (1)
dx dx dx
where a0 , a1 , a2 , . . . , an are constants with a0 , 0. If Q(x) = 0 then
dn y dn−1 y dy
(1) ⇒ a0 n
+ a1 n−1
+ · · · + an−1 + an y = 0 (2)
dx dx dx

wwwhich is the homogeneous equation corresponding to (1).


The general solution of (2) is called the complementary function of (1) and it is

w.E
denoted by yc . The general solution of (2) contains n arbitrary constants. A solution
which contains no arbitrary constants is a particular solution.

asy
If y p is a particular solution of (1), then the general solution of (1) is y = yc + y p .

En
This is also called as the complete solution of the ordinary differential equation.
Computation of Complementary function
Let
d
dx
d2
dx
Then (1) becomes
dn
= D, 2 = D2 , . . . , n = Dn .
dx
gin
ee
(a0 Dn + a1 Dn−1 + · · · + an−1 D + an )y = Q(x)

rin (3)

g.n
The auxiliary equation is

a0 mn + a1 mn−1 + a2 mn−2 + · · · + an−1 m + an = 0.

Let m1 , m2 , . . . , mn be its roots.


Case (i) If m1 , m2 , . . . , mn are real and different then
et
yc = CF = c1 em1 x + c2 em2 x + · · · + cn emn x .

Case (ii) If some of the roots (say r) are equal, where r < n
i.e m1 = m2 = · · · = mr = m, then

C.F = yc = (c1 + c2 x + c3 x2 + · · · + cr xr−1 )emx + cr+1 emr+1 x + · · · + cn emn x .

Download From : www.EasyEngineering.net


Download From : www.EasyEngineering.net

316 Engineering Mathematics - I

In particular, if two roots are equal say m1 = m2 = m then,

yc = (c1 + c2 x)emx + c3 em3 x + · · · + cn emn x .

Case (iii) If two roots are complex, say m1 = α + iβ and m2 = α − iβ and the other
roots are real and different, then

yc = eαx (c1 cos βx + c2 sin βx) + c3 em3 x + · · · + cn emn x .

wwCase (iv) If m1 = m2 = α + iβ, m3 = m4 = α − iβ and the other roots are real and
different, then

w.E yc = [(c1 + c2 x) cos βx + (c3 + c4 x) sin βx]eαx + c5 em5 x + · · · + cn emn x .

asy
Computation of particular integral (y p )
Let f (D) = a0 Dn + a1 Dn−1 + · · · + an−1 D + an .

En
Then (3) can be written as f (D)y = Q(x).

gin
Now y p = P.I. =
1
f (D)
Q(x)

5.2.1 Type I

Let Q(x) = eαx .


ee rin
g.n
Case (i) If α is not equal to any of the mi (i = 1, 2, . . . , n), then f (α) , 0. In this case
eαx
y p = PI =
f (α)
(Replace D by α).
Case (ii) If α is equal to some of the m′i s( say m1 = m2 = · · · = mr ) then
f (D) = (D − α)r g(D) where g(α) , 0.
et
1 αx eαx xr
∴ y p = PI = e = .
(D − α)r g(D) g(α)r!

In particular, if r = 1, then

1 eαx
yp = eαx = x.
(D − α)g(D) g(α)

Download From : www.EasyEngineering.net


Download From : www.EasyEngineering.net

Differential Equations 317

1 αx eαx x2
When r = 2, y p = e = .
(D − α)2 g(D) g(α) 2!
1 αx eαx x3
When r = 3, y p = e = etc.
(D − α)3 g(D) g(α) 3!

✎ ☞
Worked Examples
✍ ✌
Example 5.5. Solve (D2 − 4)y = 1. [Dec 2013]
Solution. The auxiliary equation is

ww m2 − 4 = 0 ⇒ m2 = 4

∴ yc = c1 e2x + c2 e−2x .
⇒ m = ±2.

w.E yp =
1
=
1
D2 − 4 D2 − 4
e0x
=
e0x
0−4
1
=− .
4

asy 1
Solution is y = yc + y p = c1 e2x + c2 e−2x − .
4

En
Example 5.6. Solve (4D2 − 4D + 1)y = 4.
1 1
Solution. A.E is 4m2 − 4m + 1 = 0 ⇒ (2m − 1)2 = 0 ⇒ m = , .
[Jun 1996]

1
gin
yc = e 2 x (c1 + c2 x).
2 2

yp =
1
ee
4D2 − 4D + 1
(4) =
x
Solution is y = yc + y p = (c1 + c2 x)e 2 + 4.
1
4D2 − 4D + 1
4
e0x = = 4.
1

rin
Example 5.7. Find the particular integral of (D2 − 2D + 1)y = cos hx.
g.n
Solution.
PI =
1
D2 − 2D + 1
"
cos hx =
1
(D − 1)2
#
e x + e−x
2
!
et
[Jun 2013, Jun 2005]

1 1 1
= 2
ex + e−x
2 (D − 1) (D − 1)2
1 x x2 e−x
" #
= e +
2 2 (−1 − 1)2
" 2 x
e−x x2 e x e−x
#
1 x e
= + = + .
2 2 4 4 8

Download From : www.EasyEngineering.net


Download From : www.EasyEngineering.net

318 Engineering Mathematics - I

Example 5.8. Find the particular integral of (D2 − 4)y = cosh 2x. [May 2011]
Solution. 1
PI = cosh 2x
D2 −4 " 2x
e + e−2x
#
1
=
(D + 2)(D − 2) 2
" #
1 1 1
= e2x + e−2x
2 (D + 2)(D − 2) (D + 2)(D − 2)
1 x 2x
 x −2x 
= e + e
2 4 −4
x 2x

ww =

=
8
x
[e − e−2x ]

sinh 2x.

w.E
4

Example 5.9. Find the particular integral of (D3 − 1)y = e2x . [May 2005]
e2x e2x

asy
1
Solution. PI = 3 e2x = 3 = .
D −1 2 −1 7

Solution. En
Example 5.10. Find the particular integral of (D − 1)2 = sinh x. [Nov 2003]

gin
" x
e − e−x
#
1 1
PI = sinh x =
(D − 1)2 (D − 1)2 2

ee
" #
1 1 1
= 2
ex − e−x
(D − 1)2

rin
2 (D − 1)
1 x2 x 1 −x
" #
= e − e
2 2
x 2
= ex −
e−x
4

. g.n
4

Example 5.11. Solve


8

d2 y
dx 2
dy
+ 4 + 5y = −2 cosh x.
dx
et
Solution. The A.E is

m2 + 4m + 5 = 0 ⇒ (m + 2)2 + 5 − 4 = 0

⇒ (m + 2)2 = −1 ⇒ m + 2 = ±i ⇒ m = −2 ± i.

yc = e−2x (c1 cos x + c2 sin x).

Download From : www.EasyEngineering.net


Download From : www.EasyEngineering.net

Differential Equations 319

2
yp = [−(e x + e−x )]
2(D2
+ 4D + 5)
−1 1
= 2 ex − 2 e−x
D + 4D + 5 D + 4D + 5
−e x e−x
= −
1+4+5 1−4+5
−e x e−x
= − .
10 2

The general solution is y = yc + y p

ww y = e−2x (c1 cos x + c2 sin x) −


e x e−x
10

2
.

w.E
Example 5.12. Find the particular integral of (D + 2)(D − 1)2 y = e−2x + 2 sinh x.
[May 2006]
Solution.
PI =
1
asy
(D + 2)(D − 1)2
[e−2x + 2 sinh x]

=
1
(D + 2)(D − 1) 2
En
[e−2x + e x − e−x ]

=
1
(D + 2)(D − 1) 2
gin
e−2x +
1
(D + 2)(D − 1) 2
ex −
1
(D + 2)(D − 1)2
e−x

=
x −2x x2 x e−x
9

9
e + e −

e + e −
6

6
2
x −2x x x e−x
1(4)

4
.
ee rin
Example 5.13. Solve (D2 + 1)2 y = 0. g.n [May 2008]
Solution. A.E is (m2 + 1)2 = 0.
m2 = −1, m2 = −1 =⇒ m = ±i, m = ±i.
The complex roots are repeated.
et
∴ yc = CF = (c1 + c2 x) cos x + (c3 + c4 x) sin x.
y p = PI = 0.
Solution is y = yc + y p

y = (c1 + c2 x) cos x + (c3 + c4 x) sin x.

Download From : www.EasyEngineering.net


Download From : www.EasyEngineering.net

320 Engineering Mathematics - I

Example 5.14. Solve (D2 + 1)y = 0 given that y(0) = 0, y′ (0) = 1. [May 1996]
Solution. A.E is m2 + 1 = 0 =⇒ m2 = −1 =⇒ m = ±i.

CF = yc = e0x [c1 cos x + c2 sin x] = c1 cos x + c2 sin x.

y p = PI = 0.

Solution is y = yc + y p

y = c1 cos x + c2 sin x.

ww Given: y(0) = 0.

w.E =⇒ c1 = 0
y′ = −c1 sin x + c2 cos x.

asy
Given: y′ (0) = 1 =⇒ c2 = 1.
Hence, solution is, y = sin x.

1 En
Example 5.15. Find the particular integral of (D2 − 4D + 4)y = 2 x .
1
[Dec 2012]
Solution. P.I = 2
D − 4D + 4
2x
2x =
gin
(D − 2)2
e x log 2

=
(log 2 − 2)2
.

ee
Example 5.16. Find the particular integral of (D2 − 4)y = 3 x .
1 1 1 3x rin [May 2007]

g.n
x x log 3 x log 3
Solution. PI = 2 3 = 2 e = e = .
D −4 D −4 (log 3)2 − 4 (log 3)2 − 4
d2 y

et
dy
Example 5.17. Solve + 6 + 5y = e2x .
dx2 dx
Solution. (D2 + 6D + 5)y = e2x .
A.E is, m2 + 6m + 5 = 0 ⇒ (m + 1)(m + 5) = 0 ⇒ m = −1, −5.

yc = c1 e−x + c2 e−5x .
1 2x 1 2x e2x e2x
yp = e = e = = .
D2 + 6D + 5 (D + 1)(D + 5) 3(7) 21
e2x
Solution is, y = yc + y p = c1 e−x + c2 e−5x + .
21

Download From : www.EasyEngineering.net


Download From : www.EasyEngineering.net

Differential Equations 321

d3 y d2 y dy
Example 5.18. Solve + 2 + = e2x .
dx3 dx2 dx
Solution. The A.E is

m3 + 2m2 + m = 0 ⇒ m(m2 + 2m + 1) = 0

⇒ m(m + 1)2 = 0 ⇒ m = 0, m = −1, −1.

yc = c1 e0x + (c2 + c3 x)e−x

yc = c1 + e−x (c2 + c3 x).

ww yp =
D3
1
+ 2D2 + D
e2x

w.E
1
= e2x
D(D + 1)2
e2x e2x
= = .
2(3)2
asy
18
The general solution is y = yc + y p

En
y = c1 + e−x (c2 + c3 x) +
e2x
18
.

gin
Example 5.19. Solve (D2 − 2D + 1)y = (e x + 1)2 . [ Dec 2010]
Solution. AE is m2

ee
− 2m + 1 = 0 =⇒ (m −

yc = e x (c1 + c2 x)
1
1)2 = 0 =⇒ m = 1, 1.

rin
yp =
D2− 2D + 1
1
(e x + 1)2

(e2x + 1 + 2e x ) g.n
et
= 2
(D − 1)
1 1 1
= 2
e2x + 2
e0x + 2 ex
(D − 1) (D − 1) (D − 1)2
e2x x2
= + 1 + 2 ex
1 2
= e2x + 1 + x2 e x .

Solution is y = yc + y p

y = e x (c1 + c2 x) + e2x + 1 + x2 e x .

Download From : www.EasyEngineering.net


Download From : www.EasyEngineering.net

322 Engineering Mathematics - I

5.2.2 Type II

Q(x) = sin ax or cos ax where a is a constant.

1 1
y p = P.I. = sin ax or cos ax.
f (D) f (D)

Working rule. Replace D2 by −a2 and evaluate till D is eliminated.


If f (D) = D2 + a2 , then f (D) = 0 when D2 is replaced by −a2 . In this case
Z
1 x −x

wwand
P.I. = 2
D +a 2
sin ax =
2
sin axdx =
2a
cos ax.

w.E 2
1
D +a 2
cos ax =
x
2
Z
cos axdx =
x
2a
sin ax.

Another method
asy
1 1 iax
P.I. =
En
f (D)
sin ax = Imaginary Part of
f (D)
e

and
P.I. =
1gincos ax = Real Part of
1 iax
e

which can be evaluated using Type I.


Note
f (D)
ee f (D)

rin
Suppose f (D) = 0 when D2 is replaced by −a2 , then f (D) is of the form f (D2 ).
Then, P.I. =
1
cos ax =
cos ax
. If f (−a2 ) = 0 then P.I. = ′
x cos ax g.n
if f ′ (−a2 ) , 0.
2
f (D ) 2
f (−a )
x2 cos ax
If f ′ (−a2 ) = 0, then P.I. = ′′
f (−a2 )
f (−a2 )
et
if f ′′ (−a2 ) , 0, and so on. In a similar way the
particular integral for sin ax can be evaluated replacing cos ax.
✎ ☞
Worked Examples
✍ ✌

Example 5.20. Find the particular integral of (D2 + 4)y = sin 2x.
[Dec 2009, Dec 2011]
Solution.

Download From : www.EasyEngineering.net


Download From : www.EasyEngineering.net

Differential Equations 323

Z
1 x
PI = 2
sin 2x = sin 2xdx [∵ f (−a2 ) = 0]
D +4 2
!
x − cos 2x
=
2 2
x
= − cos 2x.
4

Example 5.21. Find the particular integral of (D2 + 1)y = sin x sin 2x. [May 1997]
Solution. 1
PI = (sin x sin 2x)
D2+1" #
−1 cos 3x − cos x

ww = 2

=
D +1
−1
"
1
2

cos 3x − 2
1
cos x
#

w.E =
2 D2 + 1
"
−1 cos 3x
2 −32 + 1 2

x
Z
D +1

cos xdx
#

=−
"

2 −8 asy
1 cos 3x x
− sin x
2
#

=
cos 3x x sin x
16
+
4 En.

Example 5.22. Find the particular


1 x
Z gin
integral of (D2 + 1)y = sin x.
−x
[Jun 2010]
Solution. P.I = 2
D +1
sin x =
2
ee
sin xdx =
2
cos x.

rin
Example 5.23. Find the particular integral of (D2 + 4D + 2)y = sin 3x. [May 1998]

g.n
Solution. 1 1 1
PI = 2
sin 3x = sin 3x = sin 3x
D + 4D + 2 −9 + 4D + 2 4D − 7
(4D + 7) 4 cos 3x(3) + 7 sin 3x
=

=
(4D + 7)(4D − 7)
12 cos 3x + 7 sin 3x
16(−9) − 49
sin 3x =
16D2 − 49
et
12 cos 3x + 7 sin 3x
=
−144 − 49
1
=− [12 cos 3x + 7 sin 3x].
193

Example 5.24. Find the particular 2


Z integral of (D + 4)y = cos 2x. [May 2001]
1 x x sin 2x x sin 2x
Solution. PI = 2 cos 2x = cos 2xdx = = .
D +4 2 2 2 4

Download From : www.EasyEngineering.net


Download From : www.EasyEngineering.net

324 Engineering Mathematics - I

Example 5.25. Find the particular integral of (D2 + 1)2 y = sin 2x.
1 sin 2x sin 2x
Solution. PI = 2 2
sin 2x = 2
= .
(D + 1) (−4 + 1) 9

Example 5.26. Find the particular integral of (D2 + 1)y = sin2 x.


Solution. 1 1 1 − cos 2x
!
2
PI = 2 sin x = 2
D +1 D +1 2
" #
1 1 1
= − cos 2x
2 D2 + 1 D2 + 1
" #

ww
1 1 0x cos 2x
= e −
2 D2 + 1 −4 + 1
" #
1 cos 2x

w.E =
2
1+
1 cos 2x
= +
2 6
3
.

asy
Example 5.27. Find the particular integral of (D"2 + 1)y = sin(2x + 5).

En
#
1 sin(2x + 5) sin(2x + 5)
Solution. PI = 2 sin(2x + 5) = =− .
D +1 −4 + 1 3

gin
Example 5.28. Find the particular integral of (D2 + 4D + 8)y = cos(2x + 3).
Solution.
PI =

=
1
D2 + 4D + 8
1
ee
cos(2x + 3) =

cos(2x + 3) =
1 1
1
−4 + 4D + 8
cos(2x + 3)

cos(2x + 3) rin
=
4D" + 4
1 (D − 1)
4 D + 1#
cos(2x + 3) g.n
et
4 (D − 1)(D + 1)
" #
1 −2 sin(2x + 3) − cos(2x + 3)
=
4 D2 − 1
" #
1 2 sin(2x + 3) + cos(2x + 3)
=−
4 −4 − 1
1
= [2 sin(2x + 3) + cos(2x + 3)] .
20

Example 5.29. Solve (D2 − 4D + 3)y = sin 3x cos 2x. [May 2007]
Solution. The A.E is, m2 − 4m + 3 = 0 ⇒ (m − 1)(m − 3) = 0 ⇒ m = 1, 3.

Download From : www.EasyEngineering.net


Download From : www.EasyEngineering.net

Differential Equations 325

yc = c1 e x + c2 e3x .
1
yp = sin 3x cos 2x
D2− 4D + 3
1 1
= 2
(sin 5x + sin x)
2 D − 4D + 3
1 1 1 1
= 2
sin 5x + 2
sin x
2 D − 4D + 3 2 D − 4D + 3
= PI1 + PI2 .
1 1 1 1
P.I1 = 2
sin 5x = sin 5x
2 D − 4D + 3 2 (−25 − 4D + 3)

ww =
1 1
2 (−22 − 4D)
sin 5x =
−1 1
4 (11 + 2D)
sin 5x

w.E =−

=−
1 (2D − 11)
4 (2D + 11)(2D − 11)
sin 5x
1 2 × 5 cos 5x − 11 sin 5x

=−
4
asy
4D2 − 121
1 10 cos 5x − 11 sin 5x
4 4(−25) − 121
=−
4 En
1 10 cos 5x − 11 sin 5x
−221
=
884 gin
10 cos 5x − 11 sin 5x
.

ee
" #
1 1 1 1
P.I2 = sin x = sin x
2 D2 − 4D + 3 2 (−1) − 4D + 3
=
1 1
2 2 − 4D
sin x =
1 1
4 1 − 2D
sin x
rin
=
1 (1 + 2D)
4 (1 − 4D2 )
sin x
g.n
=

=
1 sin x + 2 cos x
4

20
5
sin x + 2 cos x
.
et
The general solution is y = yc + y p
10 cos 5x − 11 sin 5x sin x + 2 cos x
y = c1 e x + c2 e3x + + .
884 20
Example 5.30. Solve (D2 + 1)y = sin2 x. [May 2006]
1 − cos 2x
Solution. (D2 + 1)y = .
2

Download From : www.EasyEngineering.net


Download From : www.EasyEngineering.net

326 Engineering Mathematics - I

The A.E is m2 + 1 = 0 ⇒ m2 = −1 ⇒ m = ±i

yc = e0x (c1 cos x + c2 sin x) = c1 cos x + c2 sin x


1 1 1
yp = (1 − cos 2x) = − cos 2x
2(D2 + 1) + 1) 2(D2+ 1) 2(D2
1 cos 2x 1 1
= e0x − = + cos 2x.
2(D2 + 1) 2(−4 + 1) 2 6

The general solution is y = yc + y p

ww y = c1 cos x + c2 sin x +
1 1
+ cos 2x.
2 6

w.E
Example 5.31. Solve the equation (D2 + 16)y = cos3 x.
Solution. A.E.is
[Dec. 2010]

asy
m2 + 16 = 0 ⇒ m2 = −16 ⇒ m = ±4i

En
yc = e0x (c1 cos 4x + c2 sin 4x)

gin
yc = c1 cos 4x + c2 sin 4x.

1
ee rin
yp =
D2
cos3 x
+ 16 "
1 1
#
g.n
cos 3θ = 4 cos3 θ − 3 cos θ
= 2

=
D + 16 4
1
"
1
4 D2 + 16
"
(cos 3x + 3 cos x)

cos 3x + 3 2
1
D + 16
# "
cos x
#

#
1
et
4 cos3 θ = cos 3θ + 3 cos θ

cos3 θ = (cos 3θ + 3 cos θ)


4
1 cos 3x 3 cos x 1 cos 3x 3 cos x
= + = +
4 −9 + 16 −1 + 16 4 7 15

cos 3x 3 cos x
yp = + .
28 20
cos 3x 3 cos x
Solution is y = yc + y p = c1 cos 4x + c2 sin 4x + +
28 20

Download From : www.EasyEngineering.net


Download From : www.EasyEngineering.net

Differential Equations 327

Example 5.32. Solve (D2 − 3D + 2)y = 2 cos(2x + 3) + 2e x . [Dec 2011, May 2005]
Solution. The A.E is
m2 − 3m + 2 = 0 ⇒ (m − 1)(m − 2) = 0 ⇒ m = 1, 2.
yc = c1 e x + c2 e2x .
1 1
yp = 2 cos(2x + 3) + 2 2e x
D2 − 3D + 2 D − 3D + 2
= PI1 + PI2 .
1 1
PI1 = 2 cos(2x + 3) = 2 cos(2x + 3)

ww D2 − 3D + 2 −4 − 3D + 2
1 1
=2 cos(2x + 3) = −2 cos(2x + 3)
−2 − 3D 2 + 3D

w.E = −2

= −2
(2 − 3D)
4 − 9D2
cos(2x + 3)
2 cos(2x + 3) + 6 sin(2x + 3)

1
asy 4 + 36
= − 2[cos(2x + 3) + 3 sin(2x + 3)]
20
1
En
= − [cos(2x + 3) + 3 sin(2x + 3)].
10
PI2 = 2
1
gin
(D − 1)(D − 2)
e x = 2x
ex
−1
= −2xe x .

ee
The general solution is y = yc + y p
1
y = c1 e x + c2 e2x − [cos(2x + 3) + 3 sin(2x + 3)] − 2xe x .
10
rin
Example 5.33. Solve (D2 + 16)y = e−3x + cos 4x.
Solution. The A.E is g.n
m2 + 16 = 0 ⇒ m2 = −16 ⇒ m = ±4i.
yc = e0x (c1 cos 4x + c2 sin 4x) = c1 cos 4x + c2 sin 4x.
1 1 e−3x x
Z
et
yp = 2 e−3x + 2 cos 4x = + cos 4xdx
D + 16 D + 16 9 + 16 2
e−3x x sin 4x e −3x x sin 4x
= + = + .
25 2 4 25 8
The general solution is y = yc + y p .
e−3x x sin 4x
y = c1 cos 4x + c2 sin 4x + + .
25 8

Download From : www.EasyEngineering.net


Download From : www.EasyEngineering.net

328 Engineering Mathematics - I

5.2.3 Type III

Q(x) = xm where m is a positive integer.


xm xm 1
PI = = = [1 ± g(D)]−1 xm .
f (D) k[1 ± g(D)] k
Expand [1 ± g(D)]−1 using binomial series expansion upto Dm and operate on xm
term by term, we obtain the required particular integral.
Z
1 1
Note. It is important to note that f (x) = f (x)dx. 2 f (x) means integrate
D D
f (x) w.r.t x twice and so on.

wwThe following results will be useful for our discussion.


(i) (1 + x)−1 = 1 − x + x2 − x3 + x4 − · · ·

w.E
(ii) (1 + x)−2 = 1 − 2x + 3x2 − 4x3 + · · ·
(iii) (1 − x)−1 = 1 + x + x2 + x3 + x4 + · · ·

asy
(iv) (1 − x)−2 = 1 + 2x + 3x2 + 4x3 + · · · .

En
Example 5.34. Find the particular integral of (D2 + D)y = x2 + 2x + 4. [Jun 2000]
Solution.

gin
1 1
PI = (x2 + 2x + 4) = (x2 + 2x + 4)
D2
+D D(D + 1)
1 1

1 2
Z
1
ee
= (1 + D)−1 (x2 + 2x + 4) = (1 − D + D2 − · · · )(x2 + 2x + 4)
D D
1
= (x + 2x + 4 − 2x − 2 + 2) = (x2 + 4)
D D
rin
2
= (x + 4)dx

x3
[∵
D
stands for integration]
g.n
=

1
3
+ 4x.

Example 5.35. Find the particular integral of (D2 + 5D + 1)y = x2 .


et[May 2002]
Solution. PI = 2 x2 = (1 + (D2 + 5D))−1 x2
D + 5D + 1

= (1 − (D2 + 5D) + (D2 + 5D)2 + · · · )x2 = (1 − D2 − 5D + 25D2 + · · · )x2

= (1 − 5D + 24D2 )x2 = x2 − 5(2x) + 24(2)

= x2 − 10x + 48.

Download From : www.EasyEngineering.net


Download From : www.EasyEngineering.net

Differential Equations 329

Example 5.36. Solve (D3 − 3D2 − 6D + 8)y = x. [Dec 2006]


Solution. A.E is m3 − 3m2 − 6m + 8 = 0
(m − 1)(m + 2)(m − 4) = 0 ⇒ m = 1, −2, 4.
yc = c1 e x + c2 e−2x + c3 e4x .
1
yp = x
D3 − 3D2 − 6D + 8
1
= h 3 2
ix
8 1 + D −3D8 −6D
#−1

ww D3 − 3D2 − 6D
"
1
= 1+ x
8 8
D3 − 3D2 − 6D
! !
1

w.E =

=
8
1
"
1−

6
8
#
1 + D + ··· x =
1
"
+ ··· x

x+
6
#

=
8
1
8
"
8
3
x+ .
4
#
asy 8 8

En
The general solution is y = yc "+ y p #

gin
y = c1 e x + c2 e−2x + c3 e4x +
1
8
3
x+ .
4

Solution. The A.E is m3 − m2 ee


Example 5.37. Solve (D3 − D2 − D + 1)y = 1 + x2 .

⇒ (m − 1)(m2 − 1) = 0 ⇒ m = 1, 1, −1.
−m+1=0⇒ m2 (m

rin
− 1) − (m − 1) = 0
[Apr 2007]

yc = c1 e −x

1
x
+ e (c2 + c3 x).
g.n
yp =

= 3
−D3

2
1
−D+1

D −D −D+1
(1 + x2 )
D2
e0x + 3
1
2
D −D −D+1
x2
et
x2
=1+ = 1 + [1 − (D + D2 − D3 )]−1 x2
1 − (D + D2 − D3 )
= 1 + [1 + (D + D2 − D3 ) + (D + D2 − D3 )2 + · · · ]x2

= 1 + [1 + D + D2 + D2 + · · · ]x2

= 1 + [x2 + 2x + 4] = x2 + 2x + 5.

Download From : www.EasyEngineering.net


Download From : www.EasyEngineering.net

330 Engineering Mathematics - I

The general solution is y = yc + y p


y = c1 e−x + e x (c2 + c3 x) + x2 + 2x + 5.

Example 5.38. Solve (D2 + D)y = x2 + 2x + 4. [Dec 2008]


Solution. The A.E is
m2 + m = 0 ⇒ m(m + 1) = 0 ⇒ m = 0, −1.
yc = c1 + c2 e−x .

ww yp =
1
D2 + D
(x2 + 2x + 4) =
1
D(1 + D)
(x2 + 2x + 4)

w.E 1
= (1 + D)−1 (x2 + 2x + 4)
D
1
= [1 − D + D2 − · · · ](x2 + 2x + 4)
D
1
asy
= [x2 + 2x + 4 − (2x + 2) + 2]
D
1
= [x2 + 4]
D En
gin
Z
= (x2 + 4)dx

=
x3
3
+ 4x.
ee rin
g.n
The Solution is y = yc + y p

y = c1 + c2 e−x +
x3
3
+ 4x.
et
5.2.4 Type IV

Q(x) = eax g(x) where g(x) may be xm or sin ax or cos ax. In this case
1 ax ax g(x)
P.I = e g(x) = e . This will fall into any of the previous three types
f (D) f (D + a)
which can be evaluated by the known methods.

Download From : www.EasyEngineering.net


Download From : www.EasyEngineering.net

Differential Equations 331

5.2.5 Type V

Q(x) = xm cos ax or xm sin ax. In this case

1 m 1 m
P.I = x cos ax or x sin ax.
f (D) f (D)
1 iax m 1 m iax
= R.P e x or I.P x e .
f (D) f (D)
1 1
= R.P eiax xm or I.P eiax xm
f (D + ia) f (D + ia)

wwwhich can be evaluated by earlier methods.




Worked Examples

w.E
Example 5.39. Find the particular integral of (D2 + 4D + 4)y = xe−2x . [May 2005]
Solution.

asy
1 1
PI = xe−2x = xe−2x
D2+ 4D + 4 (D + 2)2
1 1
= e−2x
1 1 En
(D − 2 + 2) 2
x = e−2x 2 x
1
Z
D
= e−2x
DD
−2x 1 x
x = e−2x
2
!
gin
e−2x
D
Z
xdx

x2 dx

ee
=e =
D 2 2

=
e−2x x3
2 3
=
x3 e−2x
6
.
rin
Example 5.40. Find the particular integral of
1 1
(D2+ 1)y =
1
xe x .
g.n
[Jun 2003]

et
Solution. P.I = xe x = e x x = ex 2 x
2
D +1 2
(D + 1) + 1 D + 2D + 1 + 1
1 1
= ex 2 x = ex h 2
ix
D + 2D + 2 2 1 + D +2D
2
#−1
ex D2 + 2D
"
= 1+ x
2 2
ex D2 + 2D ex
" #
= 1− + · · · x = [1 − D · · · ]x
2 2 2
ex
= [x − 1].
2

Download From : www.EasyEngineering.net


Download From : www.EasyEngineering.net

332 Engineering Mathematics - I

Example 5.41. Find the particular integral of y′′ + 2y′ + 5y = e−x cos 2x.[May 2005]
Solution. P.I = 1 1
2
e−x cos 2x = e−x 2
cos 2x
D + 2D + 5 (D − 1) + 2(D − 1) + 5
1 1
= e−x 2 cos 2x = e−x 2 cos 2x
DZ− 2D + 1 + 2D − 2 + 5 D +4
x x sin 2x
= e−x cos 2xdx = e−x
2 2 2
e−x x sin 2x
= .
4
Example 5.42. Find the particular integral of (D2 − 2D + 2)y = e x cos x. [Dec 2010]

wwSolution. P.I = 1
D2 − 2D + 2
e x cos x.
1

w.E = ex ·

= ex · 2
D +✟
2
(D + 1) − 2(D + 1) + 2

✟+ 1 −✟
1
✟− 2
cos x.

cos x.

asy ✁+2 ✁
2D 2D
1 cos x
= ex · 2 cos x. = e x · x
D
Z +1 2D
=
xe x
2
En
cos xdx =
xe x sin x
2
.

Solution. 1 gin
Example 5.43. Find the particular integral of (D2 − 2D + 4)y = e x cos x. [Dec 2001]
1
PI =
D2
= ex 2
− 2D + 4
1 ee
e x cos x = e x

D + 2D + 1 − 2D − 2 + 4
2
(D + 1) − 2(D + 1) + 4
cos x = e x 2
1
D +3
cos x

cos x
rin
= ex
cos x
−1 + 3
=
e x cos x
2
.
g.n
Example 5.44. Find the particular integral of (D − 1)2 y = e x sin x.

Solution. PI =
1
2
e x sin x = e x
1
sin x
et
[May 2003, Jun 2012]

(D − 1) (D + 1 − 1)2
1 1 1
= e x 2 sin x = e x sin x
DZ DD
1 1
= ex sin xdx = e x (− cos x)
D D
Z
= −e x cos xdx = −e x sin x.

Download From : www.EasyEngineering.net


Download From : www.EasyEngineering.net

Differential Equations 333

Example 5.45. Solve (D2 + 5D + 4)y = e−x sin 2x. [Dec 2012, May 2011]
Solution. A.E is m2 + 5m + 4 = 0
(m + 1)(m + 4) = 0 ⇒ m = −1, −4.

yc = c1 e−x + c2 e−4x .
1
yp = e−x sin 2x.
D2+ 5D + 4
−x 1
=e · 2
sin 2x.
(D − 1) + 5(D − 1) + 4

ww = e−x 2
1
D − 2D + 1 + 5D − 5 + 4
= e−x · 2
1
sin 2x.

w.E
sin 2x.
D + 3D
1
= e−x · sin 2x.
−4 + 3D
= e−x ·
asy3D + 4
(3D + 4)(3D − 4)
sin 2x.

En
3D + 4
= e−x · sin 2x.
9D2 − 16

gin
3 × 2 cos 2x + 4 sin 2x
= e−x
9 × (−4) − 16
6 cos 2x + 4 sin 2x
= e−x

=−
e−x
−52
−52
ee
(6 cos 2x + 4 sin 2x).
rin
Solution is y = yc + y p
y = c1 e−x + c2 e−4x −
e−x
(6 cos 2x + 4 sin 2x) g.n
52

Example 5.46. Solve (D2 − 2D + 2)y = e2x x2 + 5 + e−2x .


Solution. A.E. is
et[Jun 2009]

m2 − 2m + 2 = 0 ⇒ (m − 1)2 + 2 − 1 = 0

(m − 1)2 + 1 = 0 ⇒ (m − 1)2 = −1

(m − 1) = ±i ⇒ m = 1 ± i.

Download From : www.EasyEngineering.net


Download From : www.EasyEngineering.net

334 Engineering Mathematics - I

yc = e x (c1 cos x + c2 sin x).


1
yp = (e2x x2 + 5 + e−2x )
D2
− 2D + 2
1 5 1
= 2 e2x x2 + 2 + 2 e−2x
D − 2D + 2 D − 2D + 2 D − 2D + 2
= PI1 + PI2 + PI3 .
1 1
PI1 = e2x x2 = e2x x2
D2
− 2D + 2 2
(D + 2) − 2(D + 2) + 2
1 1
= e2x 2 x2 = e2x 2 x2

ww
D + 4 + 4D − 2D − 4 + 2 D + 2D + 2
1 e2x  D2 + 2D −1 2
= e2x 2
x 2
= 1 + x
2(1 + D +2D ) 2 2

w.E
2
2x 2
!2
e D + 2D D2 + 2D
= [1 − + + . . . ]x2
2 2 2

=
e2x 2 1
2
e2x 2
asy
[x − .2 − 2x + 2]
2
e2x 2
=
2
5 En
[x − 1 − 2x + 2] =
5
2
(x − 2x + 1).

PI2 = 2
D − 2D + 2
1 gin
e0x = .
2
e−2x e−2x
PI3 = 2
D − 2D + 2
The solution is y = yc + y p ee
e−2x =
4+4+2

e2x 2
=
10
.

5 e−2x rin
y = e x (c1 cos x + c2 sin x) +

d2 y
2
(x − 2x + 1) + +
2 10
.

g.n
Example 5.47. Solve
dx2
− 4y = x sinh x.
Solution. (D2 − 4)y = x sinh x
A.E. is
et
[Apr 2006]

m2 − 4 = 0 ⇒ m2 = 4 ⇒ m = ±2.
yc = c1 e2x + c2 e−2x
1 1
y p = PI = x sinh x = x(e x − e−x )
D2 − 4 2(D2 − 4)
1 1 1 1
= xe x − xe−x
2 D2 − 4 2 D2 − 4

Download From : www.EasyEngineering.net


Download From : www.EasyEngineering.net

Differential Equations 335

ex 1 e−x 1
= x − x
2 (D + 1)2 − 4 2 (D − 1)2 − 4
ex 1 e−x 1
= 2
x − 2
x
2 D + 2D + 1 − 4 2 D − 2D + 1 − 4
ex 1 e−x 1
= x − x
2 D2 + 2D − 3 2 D2 − 2D − 3
ex 1 e−x 1
= 2
x− x
2 (−3)(1 − D +2D ) 2 (−3)(1 − D2 −2D )
3 3
ex  D2 + 2D −1 e−x  D2 − 2D −1
= 1− x+ 1− x
−6 3 6 3

ww =
−e x h
6
−e x h
1+
D2 + 2D
3
2 i e−x h
i
+ ··· x +
2i
e−x h
6
1+
D2 − 2D
3
i
+ ··· x

w.E P.I. =
6
x+
3
+

The solution is y = yc + y p
6
x− .
3

asy
y = c1 e2x + c2 e−2x −
ex h
6
x+
2 i e−x h
3
+
6
x−
2i
3

Example 5.48. Solve


d2 y
dx2
+ 2
dy
dxEn
+ y =
e−x
x2
. [Jun 2013, May 1989 ]

gin
Solution. A.E is m2 + 2m + 1 = 0 ⇒ (m + 1)2 = 0 ⇒ m = −1, −1.

yc = e−x (c1 + c2 x).

yp =
1
2
(D + 1) x
e−x
2
= e−xee 1 1
(D − 1 + 1) x2
2
rin
g.n
! Z
1 1 1
= e−x 2 2 = e−x x−2 dx
D x D

et
h −1 i Z
−x 1 x −x 1
=e = −e dx
D −1 x
y p = −e−x log x.

The general solution is y = yc + y p

y = e−x (c1 + c2 x) − e−x log x.

Example 5.49. Solve (D4 − 1)y = e x cos x. [Apr 2009]


Solution. A.E is m4 − 1 = 0 ⇒ (m2 − 1)(m2 + 1) = 0

Download From : www.EasyEngineering.net


Download From : www.EasyEngineering.net

336 Engineering Mathematics - I

⇒ (m − 1)(m + 1)(m2 + 1) = 0 ⇒ m = 1, −1, m2 = −1 ⇒ m = ±i.

yc = C1 e x + C2 e−x + C3 cos x + C4 sin x.


1 1 1
yp = e x cos x = e x cos x = e x 2 cos x
D4−1 4
(D + 1) − 1 (D + 2D + 1)2 − 1
1
= ex 4 cos x
D + 4D + 1 + 4D3 + 2D2 + 4D − 1
2
1
= ex 4 cos x
D + 4D + 6D2 + 4D
3
1 cos x
= ex cos x = e x

ww yp = −
1 − 4D − 6 + 4D
e x cos x
5
.
−5

w.E The general solution is y = yc + y p .

asy
y = C1 e x + C2 e−x + C3 cos x + C4 sin x −
e x cos x
5
.

Solution. (D + 2)2 y = e−2x sin x En


Example 5.50. Solve (D + 2)2 y = e−2x sin x. [Jun 2009]

(D2 + 4D + 4)y = e−2x sin x.


gin
yc = e−2x (c1 + c2 x).
ee
A.E. is (m + 2)2 = 0 ⇒ m = −2, −2.

rin
yp =
1
(D + 2)2
e−2x sin x
1 g.n
= e−2x
(D − 2 + 2)2
1
= e−2x 2 sin x
D
sin x
et
sin x
= e−2x
−1
= −e−2x sin x.

The general solution is y = yc + y p


y = e−2x (c1 + c2 x) − e−2x sin x.

Download From : www.EasyEngineering.net


Download From : www.EasyEngineering.net

Differential Equations 337

Example 5.51. Solve (D2 + 4D + 3)y = e−x sin x + xe3x . [Jun 2010, Jan 2002]
Solution. A.E is m2 + 4m + 3 = 0 ⇒ (m + 1)(m + 3) = 0 ⇒ m = −1, −3.

yc = c1 e−x + c2 e−3x .
1
yp = (e−x sin x + xe3x )
D2
+ 4D + 3
1 1
= e−x sin x + xe3x
(D + 1)(D + 3) (D + 1)(D + 3)
e−x e3x
= sin x + x
(D − 1 + 1)(D − 1 + 3) (D + 3 + 1)(D + 3 + 3)

ww =
e−x
D(D + 2)
sin x +
e3x
(D + 4)(D + 6)
x

w.E = PI1 + PI2 .

PI1 = e−x
1
sin x = e−x
1
sin x

= e−x asy
D2
+ 2D
(2D + 1)
2
4D − 1
sin x = e−x
2D − 1
2 cos x + sin x
−5
=
−e−x
5 En
(2 cos x + sin x).

PI2 = e3x
1
gin
(D + 4)(D + 6)
x

= 2

=
e3x

1
D + 10D + 24
1
24 1 + D2 +10D
x
ee
x
rin
=
e3x 
1+
2
24
D + 10D −1
x g.n
=

=
24
e3x h
24
e3x h
1−

x−
10 i
24
D2 + 10D  D2 + 10D 2
24
+
24
i
− ··· x et
24 24
e3x h 5i
= x− .
24 12

The general solution is y = yc + y p


e−x e3x h 5i
y = c1 e−x + c2 e−3x − (2 cos x + sin x) + x− .
5 24 12

Download From : www.EasyEngineering.net


Download From : www.EasyEngineering.net

338 Engineering Mathematics - I

Example 5.52. Solve (D2 − 4D + 3)y = e x cos 2x. [Jun 2012]


Solution. A.E. is

m2 − 4m + 3 = 0

(m − 1)(m − 3) = 0

m = 1, 3.

yc = c1 e x + c2 e3x .
1
yp = e x cos 2x.

ww D2 − 4D + 3
= ex · 2
1
(D + 1) − 4(D + 1) + 3
cos 2x

w.E = ex · 2
1
D + 2D + 1 − 4D − 4 + 3
1
cos 2x

asy = ex 2

= ex
D − 2D
1
cos 2x.

cos 2x

En
=−
e
−4 − 2D
x 1
cos 2x

=−gin2 D+2
ex D−2
2 (D − 2)(D + 2)
cos 2x

=−
2
e
ee
e x −2 sin 2x − 2 cos 2x

x
−4 − 4
= − 2(sin 2x + cos 2x)
!

rin
16
ex
= − (sin 2x + cos 2x). g.n
et
8
Solution is y = yc + y p
ex
= c1 e x + c2 e3x − (sin 2x + cos 2x).
8
Example 5.53. Solve (D2 − 3D + 2)y = x cos x. [Jan 2007]
Solution. The A.E. is m2 − 3m + 2 = 0 ⇒ (m − 2)(m − 1) = 0 ⇒ m = 1, 2.
yc = C1 e x + C2 e2x .
1 1
yp = x cos x = 2 x( R.P. eix )
D2 − 3D + 2 D − 3D + 2

Download From : www.EasyEngineering.net


Download From : www.EasyEngineering.net

Differential Equations 339

1
= R.P. eix x
i)2
(D + − 3(D + i) + 2
1
= R.P. eix 2 x
D − 1 + 2Di − 3D − 3i + 2
1
= R.P. eix 2 x
D − 3D + 2Di + 1 − 3i
1
= R.P. eix h ix
D2
(1 − 3i) 1 + 1−3i + D(2i−3)
1−3i
eix h D(2i − 3) D2 i−1
= R.P. 1+ + x
1 − 3i 1 − 3i 1 − 3i

ww = R.P.
eix h
1 − 3i
eix h
1−
D(2i − 3) i
1 − 3i
2i − 3 i
x

w.E = R.P.

= R.P.
1 − 3i
eix h
(1 − 3i)2
x−
1 − 3i
x(1 − 3i) − (2i − 3)
i

= R.P. asy
eix
1 + 9 − 6i
h
x − 3ix − 2i + 3
i

= R.P.
eix h
10 − 6i En
x + 3 − i(2 + 3x)
i

= R.P.
(10 + 6i)eix h
136 ginx + 3 − i(2 + 3x)
i

= R.P.

= R.P.
ee
(10 + 6i)(cos x + i sin x)(x + 3 − i(2 + 3x))
136

136 rin
(10 cos x − 6 sin x + i(10 sin x + 6 cos x))(x + 3 − i(2 + 3x))

=
(10 cos x − 6 sin x)(x + 3) + (10 sin x + 6 cos x)(2 + 3x))
136
.
g.n
The solution is y = yc + y p .

y = C1 e x + C2 e2x +
et
(10 cos x − 6 sin x)(x + 3) + (10 sin x + 6 cos x)(2 + 3x))
.
136

Example 5.54. Solve (D2 + 1)2 y = x2 cos x. [May 2002]


Solution. A.E. is (m2 + 1)2 = 0 ⇒ m2 = −1, −1 ⇒ m = ±i, ±i.

yc = (C1 + C2 x) cos x + (C3 + C4 x) sin x.

Download From : www.EasyEngineering.net


Download From : www.EasyEngineering.net

340 Engineering Mathematics - I

1 1
yp = x2 cos x = 2 x2 (R.P. eix )
(D2+ 1) 2 (D + 1)2
1
= R.P. eix x2
((D + i)2 + 1)2
1
= R.P. eix 2 x2
(D − 1 + 2Di + 1)2
1
= R.P. eix 2 x2
(D + 2Di)2
1
= R.P. eix  x
2
D 2

2
(2Di) 1 + 2i

ww = R.P. eix

1 + 2i
−4D2
D −2

x2

w.E = R.P.
−eix h
4D
−e
2
ix
1 − 2
D
2i
+ 3
3D2
D2
−4
i
+ · · · x2

asy
h i
= R.P. 1 + iD − + · · · x2
4D2 4
eix h 2 3 i

En
= −R.P. x + i2x − 2
4D2 4
eix h 2 3i

gin
= −R.P. x + 2ix −
4D2 2
ix
e 1
Z 
3
= −R.P. x2 + 2ix − dx

= −R.P. ee
4 D
eix 1  x3
4 D 3
eix
Z  3
x
+ ix2 − x
3
2


3 
2

rin
= −R.P.
4
ix
e x 4
3
+ ix2 − x dx

x 3 2
3x i
2
g.n
et
h
= −R.P. +i −
4 12 3 4
 cos x + i sin x h x4 x3 3x2 i
= −R.P. +i −
4 12 3 4
1 hh x4 3x2 i x3 i
yp = − − cos x − sin x .
4 12 4 3

The solution is y = yc + y p

1 hh x4 3x2 i x3 i
y = (C1 + C2 x) cos x + (C3 + C4 x) sin x − − cos x − sin x .
4 12 4 3

Download From : www.EasyEngineering.net


Download From : www.EasyEngineering.net

Differential Equations 341

d2 y dy
Example 5.55. Solve 2
− 2 + y = xe x sin x. [Dec 2013, May 2010]
dx dx
2 2
Solution. A.E. is m − 2m + 1 = 0 ⇒ (m − 1) = 0 ⇒ m = 1, 1.

yc = e x (C1 + C2 x).
1
yp = xe x sin x
(D − 1)2
1
= ex x sin x
(D + 1 − 1)2
1
= e x 2 x sin x

ww =e x1
DZ

D
x sin xdx

w.E = ex
D
1

1h
Z
xd(− cos x)
Z

asy
i
= ex − x cos x − − cos xdx
D
1h i
= ex

En
− x cos x + sin x
D
h Z Z i
x
=e − x cos xdx + sin xdx

=e −x
h Z
gin
xd(sin x) − cos x
i

x
h (
ee
Z
= e − x sin x − sin xdx − cos x
h
= e x − x sin x − cos x − cos x
i
) i

rin
h i
= −e x x sin x + 2 cos x . g.n
The solution is y = yc + y p
h i
et
y = e x (C1 + C2 x) − e x x sin x + 2 cos x .

Example 5.56. Solve (D2 − 4D + 4)y = x2 e2x cos 2x. [Jan 2007]
Solution. A.E. is (m − 2)2 = 0 ⇒ m = 2, 2.

yc = e2x (C1 + C2 x).

Download From : www.EasyEngineering.net


Download From : www.EasyEngineering.net

342 Engineering Mathematics - I

1 1
yp = 2
x2 e2x cos 2x = e2x x2 cos 2x
(D − 2) (D + 2 − 2)2
Z
1 1
= e2x 2 x2 cos 2x = e2x x2 cos 2xdx
DZ D
1  sin 2x 
= e2x x2 d
D 2
2 Z
2x 1 x sin 2x sin 2x
h i
=e − 2xdx
D 2 2
1 h x2 sin 2x Z i
= e2x − x sin 2xdx
D 2

ww
2 Z
2x 1 x sin 2x
h  − cos 2x i
=e − xd
D 2 2
1 h x2 sin 2x  −x cos 2x Z − cos 2x i

w.E = e2x

= e2x
D 2

2
1 h x2 sin 2x x cos 2x 1 sin 2x i
+ −

2
dx

= e2x
D
1
D
2

2asy +
2
h x2 sin 2x x cos 2x sin 2x i
2

2 2

4
=
e2x h
2
Z
2
En
x sin 2xdx +
Z
x cos 2xdx −
1
2
Z
sin 2xdx
i

=
e2x h
2
Z
x2 d
gin
 − cos 2x  Z
2
+ xd
 sin 2x  1  − cos 2x i
2

2 2

=
e2x

4
h

e2x h
− x2 cos 2x + 2 xd
Z

ee
− x2 cos 2x + cos 2x(2x)dx + x sin 2x − sin 2xdx +
Z  sin 2x 
+ x sin 2x +
Z

cos 2x
+rin
cos
cos 2x i

2x i
2

=
e
4
2x h
2
Z
− x2 cos 2x + x sin 2x − sin 2xdx + x sin 2x + cos 2x
2
g.n
2
i

=
4
e2x h
4
e2x h
− x2 cos 2x + x sin 2x +
cos 2x

3
2
+ x sin 2x + cos 2x
i
i
et
= − x2 cos 2x + 2x sin 2x + cos 2x
4 2
e2x h i
yp = 4x sin 2x + (3 − 2x2 ) cos 2x .
8
The solution is y = yc + y p
e2x h i
y = e2x (C1 + C2 x) + 4x sin 2x + (3 − 2x2 ) cos 2x .
8

Download From : www.EasyEngineering.net


Download From : www.EasyEngineering.net

Differential Equations 343

Example 5.57. Solve (D2 − 2D)y = x2 e x cos x. [Jan 2002]


Solution. A.E. is m2 − 2m = 0 ⇒ m(m − 2) = 0 ⇒ m = 0, 2.

yc = C1 + C2 e2x .
1 1
yp = x2 e x cos x = e x x2 cos x
D2 − 2D (D + 1)2 − 2(D + 1)
1 1
= ex 2 x2 cos x = e x 2 x2 cos x
D + 2D + 1 − 2D − 2 D −1
= ex (R.P. eix )x2
D2 − 1

ww = R.P. e x eix
1
(D + i)2 − 1
1
x2

w.E = R.P. e x eix 2


D − 1 + 2Di − 1
= R.P. e x eix 2
1
x2
x2

asy
D − 2 + 2Di
1
= R.P. e x eix  2
 x2
(−2) 1 − D +2Di

En
2
x
e e ix  2
D + 2Di −1 2
= -R.P. 1− x

= -R.P.
2

2 gin
e x eix 
1+
2
D2 + 2Di  D2 + 2Di 2
2
+
2

+ · · · x2

= -R.P.

= -R.P.
2 ee
e x eix  2 1
x + (2 + 4ix) + (−2)
e x eix  2
2
x + 1 + 2ix − 2



rin
= -R.P.
2
e x eix  2
x − 1 + 2ix

g.n
et
2
ex  
= -R.P. (cos x + i sin x) x2 − 1 + 2ix
2
−e x
= [cos x(x2 − 1) − 2x sin x]
2
ex
= [(1 − x2 ) cos x + 2x sin x].
2
The solution is y = yc + y p
ex
y = C1 + C2 e2x + [(1 − x2 ) cos x + 2x sin x].
2

Download From : www.EasyEngineering.net


Download From : www.EasyEngineering.net

344 Engineering Mathematics - I

Example 5.58. Solve (D2 − 4D + 4)y = 8(e2x + sin 2x + x2 ).


Solution. A.E is m2 − 4m + 4 = 0 ⇒ (m − 2)2 = 0 ⇒ m = 2, 2.

yc = e2x (c1 + c2 x).


1
P.I = (8e2x + 8 sin 2x + 8x2 )
(D − 2)2
1 1 1
= 2
8e2x + 2
8 sin 2x + 8x2
(D − 2) (D − 2) (D − 2)2
= PI1 + PI2 + PI3 .

ww PI1 =
1
(D − 2)2
1
8e2x
= 8
x2 2x
2
e = 4x2 e2x .
1

w.E PI2 = 2
D − 4D + 4
1
= − 8 sin 2x
4D
8 sin 2x =
−4 − 4D + 4
8 sin 2x

asy
= −2 sin 2xdx =
2 cos 2x
2
= cos 2x.

PI3 =
1
(D − 2)2
En
8x2 =
8 1
4 1 − D 2
x2 = 2 1 −
D −2 2
2
x

gin
2
D D2
+ . . . x2

=2 1+2 +3
2 4
2 3 
= 2 x + 2x + 2
4
= 2x2 + 4x + 3.
ee rin
The general solution is y = yc + y p
g.n
y = e2x (c1 + c2 x) + 4x2 e2x + cos 2x + 2x2 + 4x + 3.

Example 5.59. Solve (D2 + 4)2 y = cos 2x.


et
Solution. The A.E is (m2 + 4)2 = 0 ⇒ m = ±2i, ±2i.

yc = (c1 + c2 x) cos 2x + (c3 + c4 x) sin 2x.


1
yp = cos 2x
(D2
+ 4)2
1
=x 2
cos 2x[ See note on Page 134]
2(D + 4)2D

Download From : www.EasyEngineering.net


Download From : www.EasyEngineering.net

Differential Equations 345

x 1
= cos 2x
4 D(D2 + 4)
x 1
= 3
cos 2x
4 D + 4D
x x
= cos 2x
4 (3D2 + 4)
x2 cos 2x
!
=
4 −12 + 4
−x2
= cos 2x.
32

ww The general solution is y = yc + y p


y = (c1 + c2 x) cos 2x + (c3 + c4 x) sin 2x − x2
32 cos 2x.

w.E
An important result to remember
Let us evaluate
1
f (x).

Let
1
D−a
D−a
f (x) = y.
asy
En
Operating on both sides by D − a we obtain
⇒ (D − a)y = f (x) ⇒ Dy − ay = f (x).

R R gin
This is a linear equation of the first order, where P = −a, Q = f (x).

ee
pdx −adx
I.F. = e =e = e−ax .
The solution is
Z
rin
g.n
y × I.F = Q × I.Fdx
Z
−ax
ye = f (x)e−ax dx

y=eax
Z
e−ax f (x)dx. et
d2 y
Example 5.60. Solve + a2 y = tan ax.
dx2
Solution. A.E is m2 + a2 = 0 ⇒ m = ±ai.

yc = c1 cos ax + c2 sin ax.


1 1
yp = tan ax = tan ax
D2 +a2 (D + ai)(D − ai)

Download From : www.EasyEngineering.net


Download From : www.EasyEngineering.net

346 Engineering Mathematics - I

1 h 1 1 i
= − tan ax
2ai D − ai D + ai
1 h 1 1 i
= tan ax − tan ax .
2ai D − ai D + ai
1
PI1 = tan ax
D −Zai Z
aix −iax
h 1 ax
i
=e tan axe dx ∵ f (x) = e f (x)e−ax dx
D−a
Z
= eaix tan ax(cos ax − i sin ax)dx

sin2 ax
Z

ww
iax
=e (sin ax − i )dx
cos ax
1 − cos2 ax 
 − cos ax Z
= eiax −i dx

w.E = eiax
a
 − cos ax i
a
cos ax
− log(sec ax + tan ax) + i
a
sin ax 
a
=
−eiax 
a
asy
cos ax − i sin ax + i log(sec ax + tan ax)
−1  iax −iax


En

= e e + ieiax log(sec ax + tan ax)
a
−1 

gin

= 1 + ieiax log(sec ax + tan ax) .
a

changing i into −i we get


1
D + ai
tan ax =

−1 1
−1 
a ee
1 − ie−iax log(sec ax + tan ax)

1 1


rin
yp =
2ai a
−1
(1 + ieiax log(sec ax + tan ax)) +
2ai a
1
= 2 (−i + eiax log(sec ax + tan ax)) + 2 (−i − e−iax log(sec ax + tan ax))
g.n
(1 − ie−iax log(sec ax + tan ax))

2a
1 h iax
2a
= 2 (i − e log(sec ax + tan ax) − i − e−iax log(sec ax + tan ax))
2a
1
i
et
= − 2 [(cos ax + i sin ax) log(sec ax + tan ax)
2a
+ (cos ax − i sin ax) log(sec ax + tan ax)]
−1
= log(sec ax + tan ax)2 cos ax
2a2
−1
y p = 2 cos ax log(sec ax + tan ax).
a

Download From : www.EasyEngineering.net


Download From : www.EasyEngineering.net

Differential Equations 347

Solution is y = yc + y p .

−1
y = c1 cos ax + c2 sin ax + cos ax log(sec ax + tan ax).
a2

5.3 Linear differential equations with variable coefficients

5.3.1 Cauchy’s homogeneous linear differential equation

The general form of the nth order linear D.E with variable coefficients is

ww a0 xn
dn y
dx n
dn−1 y dy
+ a1 xn−1 n−1 + · · · + an−1 x + an y = Q(x)
dx dx

w.E
where a0 , a1 , . . . , an are constants with a0 , 0.
This can be reduced to a linear D.E with constant coefficients as follows.
Let x = ez , ⇒ z = log x.
asy dy dy dz dy 1
= =

En dx dz dx dz x
dy dy

gin x =
dx dz

d2 y
ee
=
d  dy  d  1 dy 
dx2 dx dx
= 2 +
=
dx x dz
−1 dy 1 d  dy 
x dz x dx dz rin
=− 2
−1 dy 1 d  dy  dz
+
x dz x dz dz dx g.n
=− 2

=− 2
−1 dy 1 d2 y 1
+
x dz x dz2 x
−1 dy 1 d2 y
+ 2 2
et
x dz x dz
d 2y d 2y dy
x2 2 = 2 − .
dx dz dz

d 2 d2
Let θ = ,θ = 2.
dz dz
dy dy
Now, θy = =x = xDy
dz dx

Download From : www.EasyEngineering.net


Download From : www.EasyEngineering.net

348 Engineering Mathematics - I

Then xD = θ.
d2 y
x2 = θ2 y − θy = θ(θ − 1)y
dx2
x2 D2 y = θ(θ − 1)y.

Similarly x3 D3 = θ(θ − 1)(θ − 2)y etc.

On substitution in (1), it will be reduced to a D.E with constant coefficients, which


can be evaluated as usual.
✎ ☞

wwExample 5.61. Reduce x2



d2 y
Worked Examples

− 3x
dy

+ 3y = x into a differential equation with

w.E
constant coefficients.
Solution. The given equation is
dx2 dx
[May 2007]

asy
(x2 D2 − 3xD + 3)y = x. (1)

Let x = ez or z = log x.
En
gin
d
Then, xD = θ and x2 D2 = θ(θ − 1) where θ = .
dz
Now (1) gives (θ(θ − 1) − 3θ + 3)y = ez .

(θ2 − 4θ + 3)y = ez
ee
i.e, (θ2 − θ − 3θ + 3)y = ez

rin
d2 y dy
− 4 + 3y = ez , g.n
et
dz 2 dz
which is a linear second order differential equation with constant
coefficients.
d2 y dy
Example 5.62. Solve x + = 0. [Jun 2006]
dx2 dx
d2 y dy
Solution. The given equation is x 2 + = 0.
dx dx
Multiplying by x, we get
d2 y dy
x2 2
+x = 0.
dx dx

Download From : www.EasyEngineering.net


Download From : www.EasyEngineering.net

Differential Equations 349

Let x = ez or z = log x.
d d2 d
Now x = θ, then x2 2 = θ(θ − 1) where θ = .
dx dx dz
The given equation becomes

(θ(θ − 1) + θ) y = 0

(θ2 − θ + θ)y = 0

θ2 y = 0.

The A.E is m2 = 0 =⇒ m = 0, 0

ww ∴ yc = e0z (c1 + c2 z) = c1 + c2 z

w.E yc = c1 + c2 log x.

y p = 0.

asy
The general solution is y = yc + y p

En
y = c1 + c2 log x.
d2 y dy
Example 5.63. Solve x2 + 4x + 2y = 0. [Jun 2013, May 2008]
dx 2
Solution. Let x = ez or z = log x.
d d2
dx
gin d
Let x
dx dx
The given equation is reduced to
(θ(θ − 1) + 4θ + 2)y = 0
ee
= θ. Then, x2 2 = θ(θ − 1) where θ = .
dz

rin
(θ2 − θ + 4θ + 2)y = 0
(θ2 + 3θ + 2)y = 0. g.n
A.E is m2 + 3m + 2 = 0
(m + 1)(m + 2) = 0, m = −1, m = −2.
et
yc = c1 e−z + c2 e−2z

= c1 e− log x + c2 e−2 log x


−1 ) −2 c1 c2
= c1 elog(x + c2 elog x = c1 x−1 + c2 x−2 = + 2.
x x
y p = 0.

Download From : www.EasyEngineering.net


Download From : www.EasyEngineering.net

350 Engineering Mathematics - I

The general solution is y = yc + y p


c1 c2
y= + 2.
x x
2
d y dy
Example 5.64. Solve x2 2 − x + y = 0. [Jun 2009, May 2003]
dx dx
Solution. Let x = ez or z = log x.
dy d
Define x = θy where θ = .
dx dz
d2 y
Then, x2 2 = θ(θ − 1)y.
dx
∴ The given equation becomes

ww θ(θ − 1)y − θy + y = 0
(θ2 − θ − θ + 1)y = 0

w.E (θ2 − 2θ + 1)y = 0


A.E is m2 − 2m + 1 = 0 ⇒ (m − 1)(m − 1) = 0 ⇒ m = 1, 1.

asy
yc = (c1 + c2 z)ez .

En
= (c1 + c2 log x)x.

y p = 0.

The solution y = yc + y p gin


i.e., y = (c1 + c2 log x)x.
ee rin
Example 5.65. Transform the equation x2 y′′ + xy′ = x into a linear differential
equation with constant coefficients.
Solution. Let x = ez or z = log x. g.n
[May 2011, Dec 2011, Jun 2010]

Let x
d
dx
d2
dx
The given equation is reduced to
d
= θ. Then, x2 2 = θ(θ − 1) where θ = .
dz et
(θ(θ − 1) + θ)y = ez

(θ2 − θ + θ)y = ez

θ2 y = ez
d2 y
= ez ,
dz2

Download From : www.EasyEngineering.net


Download From : www.EasyEngineering.net

Differential Equations 351

which is the required linear equation.

Example 5.66. Reduce the equation (x2 D2 + xD + 1)y = log x into an ordinary
differential equation with constant coefficients. [Dec 2010]
Solution. Let x = ez or z = log x.
d d2
Let x = θ. Then, x2 2 = θ(θ − 1) where θ = d
dz .
dx dx
The given equation is reduced to

(θ(θ − 1) + θ + 1)y = z

ww (θ2 − θ + θ + 1)y = z

(θ2 + 1)y = z

w.E d2 y
dz2
+ y = z,

asy
which is the required linear equation.
2

coefficients. En
Example 5.67. Convert (x2 D2 + xD + 7)y =
x
into an equation with constant
[Dec 2009]
Solution. Let x = ez or z = log x.
d d gin
Define D =

Then x
d
dx
dx
,θ = .

= θ. x 2d
2

dx2
dz
ee
= θ(θ − 1).
The given equation is reduced to rin
(θ(θ − 1) + θ + 7)y =
2
ez
= 2e−z g.n
(θ2 − θ + θ + 7)y = 2e−z

(θ2 + 7)y = 2e−z


et
d2 y
+ 7y = 2e−z ,
dz2
which is the required differential equation with constant coefficients.
2
Example 5.68. Convert (3x2 D2 + 5xD + 7)y = x log x into an equation with constant
coefficients [Dec 2013]

Download From : www.EasyEngineering.net


Download From : www.EasyEngineering.net

352 Engineering Mathematics - I

Solution. Let x = ez or z = log x.


d d
Let D = ,θ= .
dx dz
d d2
Then, x = θ, x2 2 = θ(θ − 1).
dx dx
The given equation is reduced to
2
(3 · θ(θ − 1) + 5θ + 7)y = ·z
ez
(3θ2 − 3θ + 5θ + 7)y = 2e−z z

(3θ2 + 2θ + 7)y = 2e−z z

ww 3
d2 y
dz 2
dy
+ 2 + 7y = 2e−z z,
dz

w.E which is the required equation.

Example 5.69. Solve (x2 D2 + xD + 1)y = 0. [Jun 2005, May 2002]

d asy
Solution. Let x = ez or z = log x.
d

En
Define D = ,θ = .
dx dz
d d2
Then, x = θ. x2 2 = θ(θ − 1)
dx dx
gin
The given equation is reduced to

(θ(θ − 1) + θ + 1)y = 0

(θ2 − θ + θ + 1)y = 0
ee rin
(θ2 + 1)y = 0.

A.E is m2 + 1 = 0. g.n
m2 = −1

m = ±i.
et
yc = e0z (c1 cos z + c2 sin z) = c1 cos(log x) + c2 sin(log x)

y p = 0.

The general solution is y = yc + y p

y = c1 cos(log x) + c2 sin(log x).

Download From : www.EasyEngineering.net


Download From : www.EasyEngineering.net

Differential Equations 353

Example 5.70. Solve x2 y′′ − xy′ + y = x.


Solution. (x2 D2 − xD + 1)y = x.
Let x = ez or z = log x.
d d
Define D = ,θ = .
dx dz
Then, xD = θ, x2 D2 = θ(θ − 1).
Now, the given equation is reduced to

(θ(θ − 1) − θ + 1)y = ez

ww (θ2 − θ − θ + 1)y = ez

(θ2 − 2θ + 1)y = ez .

w.E A.E is m2 − 2m + 1 = 0 ⇒ (m − 1)2 = 0 ⇒ m = 1, 1.

asy
yc = ez (c1 + c2 z) = x(c1 + c2 log x).

yp = 2
1
θ − 2θ + 1
En
ez =
1
(θ − 1)2
ez
=
z2 z
2
e

=
x(log x)2
2
gin
ee
The general solution isy = yc + y p
x
i.e., y = x(c1 + c2 log x) + (log x)2 .
2
rin
Example 5.71. Solve x 2d
dx 2
2y dy
+ 4x + 2y = x log x.
Solution. Let x = ez , or z = log x.
dx
g.n
[Dec 2006, May 2003]

Define D =
dx
d
,θ = .
d
dz
Then, xD = θ, x2 D2 = θ(θ − 1).
et
Now, the given equation is reduced to

(θ(θ − 1) + 4θ + 2)y = ez z

(θ2 − θ + 4θ + 2)y = ez z

(θ2 + 3θ + 2)y = ez z.

Download From : www.EasyEngineering.net


Download From : www.EasyEngineering.net

354 Engineering Mathematics - I

A.E is, m2 + 3m + 2 = 0 ⇒ (m + 1)(m + 2) = 0 ⇒ m = −1, −2.

c1 c2
yc = c1 e−z + c2 e−2z = + 2.
x x
1 1
yp = ez z = ez z
θ2
+ 3θ + 2 2
(θ + 1) + 3(θ + 1) + 2
1 1
= ez 2 z = ez 2 z
θ + 2θ + 1 + 3θ + 3 + 2 θ + 5θ + 6
1 ez  5θ + θ2 −1
= ez  2
 z = 1 + z
6 1 + 5θ+θ 6 6
6

ww ez 5θ + θ2 ez  5θ 

= 1− + ··· z = 1− z
6 6 6 6
z
e  5θ  e  z 5  x 5

w.E
= z− z = z − .1 = log x − .
6 6 6 6 6 6

Solution is y = yc + y p

i.e., y =
x asy
c1 c2 x 
+ 2 + log x − .
x 6
5
6

En
Example 5.72. Solve (x2 D2 − xD + 1)y =
 log x 2
. [Jun 2005]

gin
x
z
Solution. Let x = e , or z = log x.
d d
Define D = ,θ = .
dx dz
Then, xD = θ, x2 D2 = θ(θ − 1).
ee
Now, the given equation is reduced to
rin
(θ(θ − 1) − θ + 1)y = (ze−z )2
g.n
(θ2 − θ − θ + 1)y = z2 e−2z

(θ2 − 2θ + 1)y = z2 e−2z

(θ − 1)2 y = z2 e−2z .
et
A.E is m2 − 2m + 1 = 0 ⇒ (m − 1)2 = 0 ⇒ m = 1, 1.

yc = ez (c1 + c2 z) = x(c1 + c2 log x).


1 1 1
yp = 2
z2 e−2z = e−2z 2
z2 = e−2z z2
(1 − θ) (1 − (θ − 2)) (1 − θ + 2)2

Download From : www.EasyEngineering.net


Download From : www.EasyEngineering.net

Differential Equations 355

1 2 e−2z z2 e−2z  θ −2 2


= e−2z z = = 1 − z
(3 − θ)2 9 1− θ 2
  9 3
3
e−2z  2 θ2 e−2z  2 2 1
= 1 + θ + 3 + · · · z2 = z + 2z + 2
9 3 9 9 3 3
e−2z  4 2 
= z2 + z +
9 3 3
1  4 2
y p = 2 (log x)2 + log x + .
9x 3 3
The solution is y = yc + y p .

ww 1  4 2
y = x(c1 + c2 log x) + 2 (log x)2 + log x + .
9x 3 3

w.E
Example 5.73. Solve x2
d2 y
dx 2
Solution. Let x = ez , or z = log x.
dy
dx x
1
+ 4x + 2y = x2 + 2 . [Jun 2013, May 2008]

asy
d d
Define D = ,θ = .
dx dz
Then, xD = θ, x2 D2 = θ(θ − 1).

En
Now, the given equation is reduced to

gin
(θ(θ − 1) + 4θ + 2)y = e2z + e−2z

(θ2 − θ + 4θ + 2)y = e2z + e−2z

ee
(θ2 + 3θ + 2)y = e2z + e−2z .

A.E is m2 + 3m + 2 = 0 ⇒ (m + 1)(m + 2) = 0 ⇒ m = −1, −2. rin


yc = c1 e−z + c2 e−2z =
c1 c2
x
+ 2.
x g.n
yp =

=
θ2
1
+ 3θ + 2
1
(e2z + e−2z )

e2z +
1
e−2z
et
(θ + 1)(θ + 2) (θ + 1)(θ + 2)
e2z ze−2z
= +
12 −1
x 2 log x
yp = − 2 .
12 x
c1 c2 x2 log x
Solution is y = yc + y p = + 2+ − 2 .
x x 12 x

Download From : www.EasyEngineering.net


Download From : www.EasyEngineering.net

356 Engineering Mathematics - I

Example 5.74. Solve (x2 D2 − 3xD + 4)y = x2 given that y(1) = 1, y′ (1) = 0.
Solution. Let x = ez , or z = log x.
d d
Define D = dx , θ = dz .
Then, xD = θ, x2 D2 = θ(θ − 1).
Now, the given equation is reduced to

(θ(θ − 1) − 3θ + 4)y = e2z

(θ2 − θ − 3θ + 4)y = e2z

ww (θ2 − 4θ + 4)y = e2z

(θ − 2)2 y = e2z .

w.E A.E is m2 − 4m + 4 = 0 ⇒ (m − 2)2 = 0 ⇒ m = 2, 2.

asy
yc = e2z (c1 + c2 z)

= x2 (c1 + c2 log x).

yp =
1
Ene2z
=
z2 2z (log x)2 2
e = x .
(θ − 2)2
Solution is y = yc + y p .gin 2 2

ee
y = x2 (c1 + c2 log x) +

When x = 1, y = 1 which gives c1 = 1


x2
2
(log x)2 .

rin
1 x2 1
y′ = x2 c2 + (c1 + c2 log x)2x + (log x)2 x + 2 log x . g.n
x
When x = 1, y′ = 0. which implies

0 = c2 + 2
2 x

et
c2 = −2

Solution is
x2
y = x2 (1 − 2 log x) + (log x)2 .
2

Download From : www.EasyEngineering.net


Download From : www.EasyEngineering.net

Differential Equations 357

Example 5.75. Solve the differential equation (x2 D2 − xD + 4)y = x2 sin(log x)


[Jun 2012]
Solution. Let x = ez or z = log x.
d d
Let D = ,θ= .
dx dz
d d2
Then, x = θ, x2 2 = θ(θ − 1).
dx dx
The given equation is reduced to

(θ(θ − 1) − θ + 4)y = e2z sin z

ww (θ2 − θ − θ + 4)y = e2z sin z

(θ2 − 2θ + 4)y = e2z sin z.

w.E The A.E. is

asy
m2 − 2m + 4 = 0

(m − 1)2 + 4 − 1 = 0

(m − 1)2 + 3 = 0En
gin
(m − 1)2 = −3

m − 1 = ± 3i
ee

m = 1 ± 3i.
rin
z
√ √
yc = e (c1 cos 3z + c2 sin 3z)
√ √ g.n
yc = x(c1 cos 3 log x + c2 sin 3 log x).

yp =

2z
θ2
1
− 2θ + 4
e2z sin z
1
et
=e 2
sin z
(θ + 2) − 2(θ + 2) + 4
1
= e2z 2 sin z
θ + 4θ + 4 − 2θ − 4 + 4
1
= e2z 2 sin z
θ + 2θ + 4
1
= e2z sin z
−1 + 2θ + 4

Download From : www.EasyEngineering.net


Download From : www.EasyEngineering.net

358 Engineering Mathematics - I

1
= e2z sin z
2θ + 3
2θ − 3
= e2z sin z
(2θ − 3)(2θ + 3)
2θ − 3
= e2z 2 sin z
4θ − 9
2 cos z − 3 sin z
= e2z
−4 − 9
−e 2z −x2 
= (2 cos z − 3 sin z) = 2 cos(log x) − 3 sin(log x) .
13 13
Solution is y = yc + y p

ww = x(c1 cos
√ √
3 log x + c2 sin 3 log x) −
x2
13

2 cos(log x) − 3 sin(log x) .

w.E
Example 5.76. Solve (x2 D2 − 3xD + 4)y = x2 cos log x [Dec 2010]

Let D =
d
dx
,θ= .
d
dz asy
Solution. Let x = ez or z = log x.

Then, x
d
dx
d2
En
= θ, x2 2 = θ(θ − 1).
dx

gin
The given equation is reduced to

ee
(θ(θ − 1) − 3θ + 4)y = e2z cos z

(θ2 − θ − 3θ + 4)y = e2z cos z


rin
(θ2 − 4θ + 4)y = e2z cos z

(θ − 2)2 y = e2z cos z. g.n


The A.E. is (m − 2)2 = 0

m = 2, 2.
et
yc = e2z (c1 + c2 z)

= x2 (c1 + c2 log x).


1
yp = e2z cos z
(θ − 2)2
1
= e2z cos z
(θ + 2 − 2)2

Download From : www.EasyEngineering.net


Download From : www.EasyEngineering.net

Differential Equations 359

1
= e2z cos z
θ2Z
1
= e2z cos zdz
θ
1
= e2z sin z
θZ
= e2z sin zdz

= −e2z cos z = −x2 cos(log x).

Solution is y = yc + y p

ww = x2 (c1 + c2 log x) − x2 cos(log x).

w.E
Example 5.77. Solve (x2 D2 − 2xD − 4)y = x2 + 2 log x.
Solution. Let x = ez or z = log x.
[Jun 2010]

Define D =
d
d
dx
asy
,θ= .
2 d
d
dz
2
Then, x
dx
= θ, x
dx2
En= θ(θ − 1).
The given equation is now reduced to

gin
(θ(θ − 1) − 2θ − 4)y = e2z + 2z.

ee
(θ2 − θ − 2θ − 4)y = e2z + 2z.

(θ2 − 3θ − 4)y = e2z + 2z.


rin
The A.E. is
g.n
m2 − 3m − 4 = 0

(m − 4)(m + 1) = 0.
et
m = −1, 4.

yc = c1 e−z + c2 e4z
c1
= + c2 x4 .
x
1
yp = 2 (e2z + 2z)
θ − 3θ − 4

Download From : www.EasyEngineering.net


Download From : www.EasyEngineering.net

360 Engineering Mathematics - I

1 2
= e2z + 2 z
θ2 − 3θ − 4 θ − 3θ − 4
e2z 2
= +  z
4 − 6 − 4 −4 1 − θ2 −3θ
4
2z 2
!−1
e 1 θ − 3θ
= − 1− z
−6 2 4
−e2z 1 θ2 − 3θ
!
= − 1+ + ··· z
6 2 4
−e2z 1
" #
3
= − z−

ww =
−x
6

6
2 1
2
"
− log x − .
2
4
3
4
#

w.E Solution is y = yc + y p
c1 x2 1
"
3
#

asy
4
= + c2 x − − log x − .
x 6 2 4
d2 y dy
Example 5.78. Solve (x2
En
dx 2
+ 4x + 2y) = e x
dx

x2 gin
d2 y dy
(or)
log x
+ 4x + 2y = ee .
dx

Solution. Let x = ez , or z = log x.


d d
2

ee dx

rin
[Dec 2013]

Define D =
dx
,θ = .
dz
Then, xD = θ, x2 D2 = θ(θ − 1). g.n
Now, the given equation is reduced to
(θ(θ − 1) + 4θ + 2)y = ee
(θ2 − θ + 4θ + 2)y = ee
z
z
et
z
(θ2 + 3θ + 2)y = ee .
A.E is m2 + 3m + 2 = 0 ⇒ (m + 1)(m + 2) = 0 ⇒ m = −1, −2.

yc = (c1 e−z + c2 e−2z )


c1 c2
= + 2.
x x

Download From : www.EasyEngineering.net


Download From : www.EasyEngineering.net

Differential Equations 361

!
1 ez (θ + 2) − (θ + 1) ez 1 1 z
yp = e = e = − ee .
(θ + 1)(θ + 2) (θ + 1)(θ + 2) θ+1 θ+2
1 ez 1 ez
= e − e
θ+1 θ+2
y p = PI1 − PI2 .
1 ez 1 z
PI1 = e = ee
θ+1 θ − (−1)
Z Z
z ez z
=e−z
e e dz = e −z
ee d(ez )
z ex
= e−z ee =

ww
.
x
1 ez 1 z
PI2 = e = ee
θ+2 θ − (−2)

w.E =e−2z
Z
2z ez
e e dz = e −2z
Z
z
ez ee ez dz.

Let ez = t.
ez dz = dt. asy
PI2 = e −2z
Z
te dt = e En
t
td(et )
i
−2z
Z h

=e−2z
" Z #
gin
te − e dt = e−2z [tet − et ]
t t

z
= e−2z [ez ee − ee ] =
x − 1 x ex ex
ee
z 1
x2
[xe x − e x ]

rin
g.n
= e = − 2.
x2 x x
ex ex ex ex
Now P.I = PI1 − PI2 =
Solution is y = yc + y p .
y=
c1 c2 e x
x
+ 2 + 2.
x x
x

x
+ 2 = 2.
x x
et
5.3.2 Legendre’s linear differential equation

The general form of the Legendre’s linear differential equation is

dn y n−1 d
n−1 y dy
(ax + b)n + a1 (ax + b) + · · · + an−1 (ax + b) + an y = Q(x)
dxn dx n−1 dx

Download From : www.EasyEngineering.net


Download From : www.EasyEngineering.net

362 Engineering Mathematics - I

where a, b, ai ’s are constants.


d
It can be reduced to the previous type by the substitution ez = ax + b and θ = .
dz

⇒ z = log(ax + b)

dy
(ax + b) = aθy ⇒ (ax + b)D = aθ.
dx
d2 y
Then, (ax + b)2 = a2 θ(θ − 1)y ⇒ (ax + b)2 D2 = a2 θ(θ − 1) etc.
dx2
✎ ☞

ww ✍
Worked Examples

w.E
Example 5.79. Transform the equation (2x + 3)2 y′′ − (2x + 3)y′ + 2y = 6x into a
differential equation with constant coefficients. [May 2005]

Define θ = .
d
dz asy
Solution. Let 2x + 3 = ez or z = log(2x + 3).

En
Then, (2x + 3)D = 2θ and (2x + 3)2 D2 = 4θ(θ − 1).
Now, the given differential equation is reduced to

gin
(4θ(θ − 1) − 2θ + 2)y = 6
(ez − 3)

ee
(4θ2 − 4θ − 2θ + 2)y = 3(ez − 3)

(4θ2 − 6θ + 2)y = 3(ez − 3)


2

rin
4
d2 y
dz2
−6
dy
dz
+ 2y = 3(ez − 3), g.n
which is the required equation.

d2 y dy
et
Example 5.80. Transform the equation (2x + 3)2 2
− 2(2x + 3) − 12y = 6x into a
dx dx
differential equation with constant coefficients. [May 2015, Jun 2012]
Solution. Let 2x + 3 = ez or z = log(2x + 3).
d
Define θ = .
dz
d d2
Then, (2x + 3) = 2θ and (2x + 3)2 2 = 4θ(θ − 1).
dx dx

Download From : www.EasyEngineering.net


Download From : www.EasyEngineering.net

Differential Equations 363

Now, the given differential equation is reduced to


(ez − 3)
(4θ(θ − 1) − 2(2)θ − 12)y = 6
2
(4θ2 − 4θ − 4θ − 12)y = 3(ez − 3)

(4θ2 − 8θ − 12)y = 3(ez − 3)


d2 y dy
4 − 8 − 12y = 3(ez − 3),
dz2 dz
which is the required equation.

wwExample 5.81. Solve (x + 2)2


d2 y
dx 2
dy
− (x + 2) + y = 3x + 4.
Solution. Let x + 2 = ez ⇒ z = log(x + 2).
dx
[Jun 2008]

w.E Define θ = .
d
dz
d
Now (x + 2)
dx
d 2

asy
and (x + 2)2 2 = θ(θ − 1).
dx

En
The given equation is now reduced to

gin
(θ(θ − 1) − θ + 1)y = 3(ez − 2) + 4 = 3ez − 2

ee
(θ2 − θ − θ + 1)y = 3ez − 2

(θ2 − 2θ + 1)y = 3ez − 2

(θ − 1)2 y = 3ez − 2. rin


A.E is, (m − 1)2 = 0 ⇒ m = 1, 1. g.n
yc = ez (c1 + c2 z) = (x + 2)(c1 + c2 log(x + 2)).

yp =
1
(3ez − 2)
et
(θ − 1)2
1 1
=3 2
ez − 2 e0z
(θ − 1) (θ − 1)2
z2
= 3 ez − 2
2
3
y p = (log(x + 2))2 (x + 2) − 2.
2

Download From : www.EasyEngineering.net


Download From : www.EasyEngineering.net

364 Engineering Mathematics - I

The general solution is y = yc + y p .

3
y = (x + 2)(c1 + c2 log(x + 2)) + (log(x + 2))2 (x + 2) − 2.
2

Example 5.82. Solve (2x + 3)2 y′′ − (2x + 3)y′ − 12y = 6x. [Dec 2009]
d
Solution. Let 2x + 3 = ez and θ =
dz
Now (2x + 3)D = 2θ
and (2x + 3)2 D2 = 4θ(θ − 1).

ww The given equation is now reduced to

w.E 1
(4θ(θ − 1) − 2θ − 12)y = 6 (ez − 3)
2
(4θ2 − 4θ − 2θ − 12)y = 3(ez − 3)

asy
(4θ2 − 6θ − 12)y = 3(ez − 3)

En
2(2θ2 − 3θ − 6)y = 3(ez − 3)
3
(2θ2 − 3θ − 6)y = (ez − 3).

gin 2

A.E is, 2m2 − 3m − 6 = 0

3
ee
3
2(m2 − m − 3) = 0 ⇒ (m − )2 − 3 −
2 4
9
16
=0
rin
√ g.n
et
3 2 57 3 57
m− = ⇒m− =±
4 16 4 4
√ √ √
3 57 3 + 57 3 − 57
m= ± ⇒m= , .
4 4 4 4
 √   √ 
3+ 57 3− 57
4 z 4 z
yc = c1 e + c2 e

 √   √ 
3+ 57 3− 57
4 4
yc = c1 (2x + 3) + c2 (2x + 3) .

Download From : www.EasyEngineering.net


Download From : www.EasyEngineering.net

Differential Equations 365

!
1 3 z 1 3 z 9
PI = 2 (e − 3) = 2 e −
2θ − 3θ − 6 2 2θ − 3θ − 6 2 2
9
3 1
= 2
ez − 2 2 e0z
2 2θ − 3θ − 6 2θ − 3θ − 6
9
3 ez 2
= −
2 2 − 3! − 6 0 − 0 − 6
3 ez 9 3 3
= + = − (2x + 3) + .
2 −7 2(6) 14 4
Solution is y = yc + y p .

ww
 √   √ 
3+ 57
4
3− 57
4
3 3
y = c1 (2x + 3) + c2 (2x + 3) − (2x + 3) + .
14 4

w.E
Example 5.83. Solve (3x + 2)2
d2 y
dx2
dy
+ 3(3x + 2) − 36y = 3x2 + 4x + 1.
Solution. Let 3x + 2 = ez or z = log(3x + 2).
dx
[Jun 2013]

Define D =
d
dx
asy
,θ= .
d
d
dz
d2
Then, (3x + 2)
dx
En
= 3θ, (3x + 2)2 2 = 9θ(θ − 1)
The given equation is now reduced to
dx

(9θ(θ − 1) + 3 · 3θ − 36)y = 3 gin


ez − 2
!2
+4
ez − 2
!
+1
3
ee
3 3
4
(9θ2 − 9θ + 9θ − 36)y = (e2z + 4 − 4ez ) + (ez − 2) + 1
9
1
3
(9θ2 − 36)y = [e2z + 4 − 4ez + 4ez − 8] + 1 rin
2
3
1
9(θ − 4)y = [e2z − 4] + 1 g.n
3
1
(θ2 − 4)y = e2z −
27
1 2z 1
= e − .
4
+
27 9
1
et
27 27
The A.E. is m2 − 4 = 0

m2 = 4

m = ±2.

yc = c1 e2z + c2 e−2z

Download From : www.EasyEngineering.net


Download From : www.EasyEngineering.net

366 Engineering Mathematics - I

c2
= c1 (3x + 2)2 + .
(3x + 2)2
1
yp = (e2z − 1)
27(θ2 − 4)
" #
1 1 2z 1 0z
= · e − · e
27 θ2 − 4 θ2 − 4
" #
1 1 2z 1
= ·e −
27 (θ − 2)(θ + 2) −4
" 2z #
1 ze 1
= +
27 4 4

ww =
2
(3x + 2) log(3x + 2)
108
+
1
108

w.E Solution is y = yc + y p

y = c1 (3x + 2)2 +
c2
+
1 h
(3x + 2)2 108
(3x + 2) 2
log(3x + 2) + 1
i

Example 5.84. Solve (1 + x)2asy d2 y dy


+ (1 + x) + y = 4 cos[log(1 + x)].

En dx 2 dx
[Dec 2014, Dec 2011]

Define D =
d
dx
, θ= .
d
dz gin
Solution. Let 1 + x = ez or z = log(1 + x).

Then, (1 + x)
d
dx ee
d2
= θ, (1 + x)2 2 = θ(θ − 1).
dx
The given equation is now reduced to
rin
(θ(θ − 1) + θ + 1)y = 4 cos z.
g.n
(θ2 − θ + θ + 1)y = 4 cos z

(θ2 + 1)y = 4 cos z.

The A.E. is m2 + 1 = 0
et
m2 = −1

m = ±i.

yc = e0z (c1 cos z + c2 sin z)

Download From : www.EasyEngineering.net


Download From : www.EasyEngineering.net

Differential Equations 367

= c1 cos log(1 + x) + c2 sin log(1 + x).


4
yp = cos z
θ2
+1
z
= 4 cos z
2θZ
= 2z cos zdz

= 2z sin z

= 2 log(1 + x) sin log(1 + x).

ww Solution is y = yc + y p

y = c1 cos log(1 + x) + c2 sin log(1 + x) + 2 log(1 + x) sin log(1 + x).

w.E
5.4 Method of variation of parameters

Consider the equation asy


En
d2 y
dx 2
dy
+ P(x) + Q(x)y = R(x).
dx
(1)

gin
Let C.F = c1 y1 (x) + c2 y2 (x) where c1 and c2 are arbitrary constants, then y1 (x) and
y2 (x) are two independent solutions of

d2 y
dx2
ee dy
+ P(x) + Q(x)y = 0.
dx
rin (2)

g.n
By the method of variation of parameters, y p is evaluated by y p = u(x)y1 + v(x)y2

et
where u(x) and v(x) are evaluated by the following way.
Define the Wronskian of y1 and y2 by

y1 y2
W = , 0,
y′ y′
1 2
Z Z
y2 R(x) y1 R(x)
Then, u(x) = − dx and v(x) = dx.
W W
Then, the general solution is given by y = yc + y p .
✎ ☞
Worked Examples
✍ ✌

Download From : www.EasyEngineering.net


Download From : www.EasyEngineering.net

368 Engineering Mathematics - I

Example 5.85. Find the Wronskian of y1 , y2 of y′′ − 2y′ + y = e x log x. [Dec 2012]
Solution. The A.E. is
m2 − 2m + 1 = 0
(m − 1)2 = 0
m = 1, 1.

yc = e x (c1 + c2 x).

∴ y1 = e x , y2 = xe x .

ww y′1 = e x

y′2 = xe x + e x = e x (x + 1).

y1 y2 e x xe x

w.E

W[y1 , y2 ] = =
y′ y′ e x e x (x + 1)
1 2

asy
1 x
= e x × e x
1 x + 1

En
= e2x (x + 1 − x) = e2x

Example 5.86. Solve


d2 y
dx2 gin
+ a2 y = sec ax. [Jun 2012]

ee
Solution. A.E is m2 + a2 = 0 ⇒ m2 = −a2 ⇒ m = ±ai.

CF = yc = C1 cos ax + C2 sin ax.


rin
y1 = cos ax,

R(x) = sec ax.


y2 = sin ax.

g.n

W =

y1 y2 cos ax
=
sin ax

y′ y′ −a sin ax a cos ax
1 2
et
= a cos2 ax + a sin2 ax = a.

Now, y p = u(x)y1 + v(x)y2 .


Z Z
y2 R(x) sin ax sec ax
u(x) = − dx = − dx
W a
Z
1 1 log(sec ax)
=− tan axdx = −
a a a

Download From : www.EasyEngineering.net


Download From : www.EasyEngineering.net

Differential Equations 369

1
= − 2 log sec ax.
Za Z
y1 R(x) cos ax sec ax 1
v(x) = dx = dx = x.
W a a
−1 x
Hence, y p = u(x)y1 + v(x)y2 = 2 log sec ax cos ax + sin ax.
a a

The solution is y = yc + y p

cos ax x
y = C1 cos ax + C2 sin ax − 2
log sec ax + sin ax.
a a

wwExample 5.87. Solve by the method of variation of parameters,


d2 y
dx2
+ y = x sin x.
[Jun 2010, May 2002]

w.E
Solution. A.E is m2 + 1 = 0 ⇒ m2 = −1 ⇒ m = ±i.

asy
yc = c1 cos x + c2 sin x.

y1 = cos x, y2 = sin x.

En
Now, y p = u(x)y1 + v(x)y2 .

gin

y1 y2 cos x sin x
W = = = cos2 x + sin2 x = 1.
y′ y′ − sin x cos x

u(x) = −
Z
1

y2 R(x)
2

dx
ee rin
g.n
W
Z Z
sin x × x sin xdx
=− = − x sin2 xdx
1
=−

=−
Z
x(1 − cos 2x)

1  x2

Z
2

xd
sin 2x 
1
2
Z
dx = − [ xdx −
Z
x cos 2xdx]
et
2 2 2
1  x2  x sin 2x
Z
sin 2x 
=− − − dx
2 2 2 2
1  x2 x sin 2x cos 2x 
=− − − .
Z2 2 2 Z 4
y1 R(x)
v(x) = dx = cos x × x sin xdx
W

Download From : www.EasyEngineering.net


Download From : www.EasyEngineering.net

370 Engineering Mathematics - I

Z Z
sin 2x 1 cos 2x 
= x dx = xd −
2 2 2
Z
1  x cos 2x cos 2x  1  x cos 2x sin 2x 
=− − dx = − − .
2 2 2 2 2 4
Hence, y p = u(x)y1 + v(x)y2
1  x2 x sin 2x cos 2x  1  x cos 2x sin 2x 
= − cos x − − − sin x − .
2 2 2 4 2 2 4
Solution is y = yc + y p .
x2 x sin 2x cos 2x
! !
1 1 x cos 2x sin 2x

ww y = c1 cos x + c2 sin x − cos x


2
Example 5.88. Solve by the method of
2

2

4
− sin x
2 2

4
variation of
.

parameters

w.E
d2 y
dx2
+ y = cosecx cot x.
Solution. A.E is m2 + 1 = 0 ⇒ m2 = ± − 1 ⇒ m = ±i.
[Dec 2007]

asy
yc = c1 cos x + c2 sin x.


En
y1 = cos x, y2 = sin x.

gin
y1 y2 cos x sin x
W = = = cos2 x + sin2 x = 1.
y′ y′ − sin x cos x
1 2

Z
y2 R(x)
ee
y p = u(x)y1 + v(x)y2 , R(x) = cosecx cot x.

Z
rin
g.n
u(x) = − dx = − sin xcosecx cot xdx = − log(sin x).
W
Z Z
y1 R(x)
v(x) = dx = cos xcosecx cot xdx

=
Z

Z
W
cos2 x
2
sin x
2
dx =
Z
cot2 xdx
Z
2
Z
et
= (cosec x − 1)dx = cosec xdx − dx = − cot x − x.

y p = u(x)y1 + v(x)y2 = −(log sin x) cos x + (− cot x − x) sin x.

Solution is, y = yc + y p .

y = c1 cos x + c2 sin x − (log sin x) cos x + (− cot x − x) sin x.

Download From : www.EasyEngineering.net


Download From : www.EasyEngineering.net

Differential Equations 371

Example 5.89. Solve by the method of variation of parameters (D2 + a2 )y = tan ax.
[Dec 2014, Jun 2009]
Solution. A.E is m2 + a2 = 0 ⇒ m = ±ai.

yc = c1 cos ax + c2 sin ax.

y1 = cos ax, y2 = sin ax.

R(x) = tan ax.



y1 y2 cos ax

ww
sin ax
W = = = a.
y′ y′ −a sin ax a cos ax
1 2

w.E y p = u(x)y1 + v(x)y2 .

u(x) = −
Z
y2 R(x)
W
dx = −
Z
sin ax tan ax
a
dx

=−
1
a
Z
asy
sin2 ax
cos ax
dx = −
1
a
Z
1 − cos2 ax
cos ax
dx

En
Z Z
1 1
=− sec axdx + cos axdx
a a
=−
Za gin
1 log(sec ax + tan ax)
a Z
1
+ 2 sin ax.
a
v(x) =

=
1
a
Z
y1 R(x)
W
dx =
ee
1
cos ax tan ax

sin axdx = − 2 cos ax.


a
a
dx

rin
y p = u(x)y1 + v(x)y2
g.n
et
1 cos ax sin ax
=− 2
(log(sec ax + tan ax) − sin ax) cos ax − .
a a2

Solution is y = yc + y p .

1 cos ax sin ax
y = c1 cos ax + c2 sin ax − 2
(log(sec ax + tan ax) − sin ax) cos ax − .
a a2

d2 y
Example 5.90. Solve by the method of variation of parameters 2 + 8y = tan 2x.
dx2
[Dec 2013]

Download From : www.EasyEngineering.net


Download From : www.EasyEngineering.net

372 Engineering Mathematics - I

Solution. The given equation can be written as

d2 y
!
2 + 4y = tan 2x
dx2
d2 y tan 2x
+ 4y = .
dx2 2
The A.E. is m2 + 4 = 0

m2 = −4

ww m = ±2i.

w.E
yc = c1 cos 2x + c2 sin 2x.

∴ y1 = cos 2x, y2 = sin 2x.

asy
tan 2x
R(x) = .
2
y1 y2 cos 2x sin 2x
W =
En
=
y′ y′ −2 sin 2x 2 cos 2x
1 2

gin
= 2 cos2 2x + 2 sin2 2x

= 2(cos2 2x + sin2 2x)

= 2.

y p = u(x)y1 + v(x)y2 .
ee rin
u(x) = −
Z
y2 R(x)
W
dx g.n
=−

=−
Z

1
Z
sin 2x · tan 2x
2×2
sin 2x ·
dx.
sin 2x
dx
et
4 cos 2x
sin2 2x
Z
1
=− dx
4 cos 2x
1 − cos2 2x
Z
1
=− dx
4 cos 2x
Z
1
=− (sec 2x − cos 2x)dx
4

Download From : www.EasyEngineering.net


Download From : www.EasyEngineering.net

Differential Equations 373

" #
1 log(sec 2x + tan 2x) sin 2x
=− −
4 2 2
−1  
= log(sec 2x + tan 2x) − sin 2x
Z8 Z
y1 R(x) tan 2x
v(x) = dx = cos 2x · dx.
W 2×2
Z
1 sin 2x
= cos 2x · dx
4 cos 2x
Z
1
= sin 2xdx.
4
!

ww
1 cos 2x cos 2x
= − =−
4 2 8
∴ y p = u(x)y1 + v(x)y2

w.E 1
= − [log(sec 2x + tan 2x) − sin 2x] cos 2x −
8
cos 2x sin 2x
8
Solution is y = yc + y p
asy 1
y = c1 cos 2x + c2 sin 2x − [log(sec 2x + tan 2x) − sin 2x] cos 2x −
sin 4x
.

En 8 16

gin
Example 5.91. Solve the differential equation
d2 y
dx2
+ 2
dy
dx
+ y =
e−x
x2
by the method
of variation of parameters.
Solution. The A.E. is ee rin
[Jun 2013]

2
m + 2m + 1 = 0
g.n
(m + 1)2 = 0

m = −1, −1.

yc = e−x (c1 + c2 x)
et
y1 = e−x , y2 = xe−x .
e−x
R(x) = 2
.
x
y1 y2 e−x xe−x
W= =
y′ y′ −e−x −xe−x + e−x
1 2

Download From : www.EasyEngineering.net


Download From : www.EasyEngineering.net

374 Engineering Mathematics - I


e−x xe−x
=
−e−x e−x (1 − x)

1 x
= e−x × e−x
−1 1 − x

= e−2x (1 − x + x) = e−2x .

y p = u(x)y1 + v(x)y2 .
Z
y2 R(x)
u(x) = − dx

ww =−
Z

Z x e
W
xe−x · e−x
2 −2x
dx.

w.E =−
Z
1
x
dx = − log x.
y1 R(x)
Z −x −x
e e
v(x) =
Z
asy
1
W
dx =
Z
−2
x2 e−2x
x−1
dx

En
= 2
dx = x dx = =− .
x −1 x
Now y p = u(x)y1 + v(x)y2

gin 1
= − log x(e−x ) − xe−x
x

Solution is y = yc + y p
ee
= −e−x (log x + 1)

rin
y = e−x (c1 + c2 x) − e−x (log x + 1).
g.n
Example 5.92. Solve by the method of variation of parameters the differential
d2 y
equation 2 + y = cosecx.
dx
Solution. The auxiliary equation is
[Dec 2012, May 2011] et
m2 + 1 = 0

m2 = −1

m = ±i.

yc = c1 cos x + c2 sin x.

Download From : www.EasyEngineering.net


Download From : www.EasyEngineering.net

Differential Equations 375

y1 = cos x, y2 = sin x.

R(x) = cosecx.

y1 y2 cos x sin x
W = =
y′ y′ − sin x cos x
1 2

= cos2 x + sin2 x = 1.

y p = u(x)y1 + v(x)y2 .
Z
y2 R(x)
Now, u(x) = − dx

ww Z
W
= − sin x · cosecxdx.

w.E Z
= − dx = −x.
Z
y1 R(x)
v(x) =
asy
Z
W
dx

cos x · cosecxdx

En
=
Z

gin
= cot xdx = log sin x.

∴ y p = u(x)y1 + v(x)y2

Solution is y = yc + y p
ee
= −x cos x + log(sin x) · sin x.

rin
y = c1 cos x + c2 sin x − x cos x + sin x log sin x.
g.n
5.5 Simultaneous linear differential equations with constant
coefficients
et
We have seen so far, the method of solving a single differential equation involving
one independent variable x and one dependent variable y. Quite often we come
across linear differential equations in which there will be two or more dependent
variables and a single independent variable. Such equations are known as

Download From : www.EasyEngineering.net


Download From : www.EasyEngineering.net

376 Engineering Mathematics - I

simultaneous linear equations. In this section, we consider linear differential


equations with one independent variable t and two dependent variables x and y.
We need two differential equations to solve for x and y. Hence, we will be given a
system of two linear differential equations which need not be of the same order.
We shall consider here only first order linear differential equations with constant
coefficients and we consider three types of equations.

5.5.1 Type I

wwWe consider simultaneous equations of the form

w.E a1
dx
dt
+ b1 y = f (t), a2
dy
dt
+ b2 x = g(t).

asy
First we eliminate one of the dependent variables from the two equations which
results in a second order linear differential equation with constant coefficients in

En
the other dependent variable and the independent variable t.
✎ ☞

gin ✍
Worked Examples

dy dx

Example 5.93. Solve for x and y if


Solution. The given equations are
dy
=x
ee dt
= x,
dt
= y.

rin
[May 2004]

(1)

g.n
dt
dx
= y. (2)
dt
Differentiating (2) w. r. t. x we get

d2 x dy
dt2
=
dt
= x[from (1)]
et
d2 x
−x=0
dt2
d
(D2 − 1)x = 0where D = .
dt
A.E is m2 − 1 = 0

m2 = 1

Download From : www.EasyEngineering.net


Download From : www.EasyEngineering.net

Differential Equations 377

m = ±1

∴ x = c1 et + c2 e−t .

dx
Now from (2) we have y = = c1 et − c2 e−t .
dt
dx dy
Example 5.94. Eliminate y from the system + 2y = − sin t and − 2x = cos t.
dt dt
Solution. Differentiating the first equation w.r.t. t we get

d2 x dy

ww 2
+ 2 = − cos t.
dt dt
d2 x
+ 2[2x + cos t] = − cos t.
dt2

w.E d2 x
dt2
+ 4x + 2 cos t + cos t = 0

asy
d2 x
dt2
+ 4x = −3 cos t,

En
which is the required equation.
dx

dy
+ x = cos t. gin
Example 5.95. Eliminate y from the simultaneous equations
dt
+ y = sin t and

dt

d2 x dy
+ = cos t.
ee
Solution. Differentiating the first equation w.r.t. t we get

rin
d2 x
dt2 dt
+ cos t − x = cos t. g.n
dt2
d2 x
dt2
− x = 0, et
which is the required equation.
dx dy
Example 5.96. Solve − y = t, + x = t2 . [May 2011, Jun 2006]
dt dt
Solution. The equations are
dx
− y = t. (1)
dt
and

Download From : www.EasyEngineering.net


Download From : www.EasyEngineering.net

378 Engineering Mathematics - I

dy
+ x = t2 . (2)
dt
Differentiating (1) w. r. t. t we get
d2 x dy
− =1
dt2 dt
2
d x
− [−x + t2 ] = 1 [ from (2)]
dt2
d2 x
2
+ x = 1 + t2 .
dt
This is a second order differential equation.
Now, A.E is m2 + 1 = 0 ⇒ m = ±i

ww xc = CF = c1 cos t + c2 sin t.

w.E
1
xp = 1 + t2 = (1 + D2 )−1 (1 + t2 )
1 + D2
= (1 − D2 + D4 − · · · )(1 + t2 ) = 1 + t2 − 2 = t2 − 1.

asy
Solution is x = xc + x p = c1 cos t + c2 sin t + t2 − 1.
From (1) we get
y=
dx
En
− t = −c1 sin t + c2 cos t + 2t − t
dt
gin
y = c2 cos t − c1 sin t + t.

ee
The required solution is x = c1 cos t + c2 sin t + t2 − 1.

y = c2 cos t − c1 sin t + t.
rin
Example 5.97. Solve
dx
dt
+ y = sin t,
dy
dt
g.n
+ x = cos t given that x = 2, y = 0 when t = 0.
[Dec 2009]
Solution. The given equations are
dx
dt
+ y = sin t.
et (1)
dy
+ x = cos t. (2)
dt
Differentiating (1) w.r.t t we get
d2 x dy
+ = cos t
dt2 dt
d2 x
− x + cos t = cos t [from (2)]
dt2

Download From : www.EasyEngineering.net


Download From : www.EasyEngineering.net

Differential Equations 379

d2 x
− x = 0.
dt2
This is a second order linear differential equation with constant coefficients.
A.E is m2 − 1 = 0 ⇒ m2 = 1 ⇒ m = ±1.
The solution is x = c1 et + c2 e−t .
dx
From (1) we obtain y = − + sin t
dt
i.e., y = c1 et − c2 e−t + sin t.

ww Given, x = 2 when t = 0

w.E =⇒ c1 + c2 = 2.

y = 0 when t = 0.
(3)

asy
=⇒ c1 − c2 = 0. (4)

En
(3) + (4) =⇒ 2c1 = 2 ⇒ c1 = 1 ⇒ c2 = 1.

gin
The solution is x = et + e−t

y = et − e−t + sin t.

Example 5.98. Solve


dx
dt
+ y = et , x −
Solution. The equations are
dy
dt ee
= t.

rin
[Dec 2012]

dx
dt
+ y = et .
g.n (1)

x−
dy
dt
= t.

Differentiating(1) w.r.t. t we get


et (2)

d2 x dy
+ = et
dt2 dt
d2 x
+ x − t = et [from (2)]
dt2
d2 x
+ x = t + et .
dt2

Download From : www.EasyEngineering.net


Download From : www.EasyEngineering.net

380 Engineering Mathematics - I

The auxiliary equation is

m2 + 1 = 0 ⇒ m2 = −1

m = ±i.

xc = c1 cos t + c2 sin t
1
xp = (t + et )
D2 +1
1 1
= 2 t+ 2 et
D +1 D +1

ww = (1 + D ) t + et
2 −1 1
2
= (1 − D2 + D4 · · · )t +
et

w.E =t+
et
2
.
2

asy
The solution for x is

En
x = xc + x p
et
.
x = c1 cos t + c2 sin t + t +
2
Substituting in (1) we get
dxgin
y = et −

t
"dt
ee
= e − −c1 sin t + c2 cos t + 1 +
et
2
#

rin
= et + c1 sin t − c2 cos t − 1 −
et
e
2
t

g.n
y = + c1 sin t − c2 cos t − 1.
2

Example 5.99. Solve the simultaneous equations


dx
dt
+ 2y = sin 2t,
dy
dt
et
− 2x = cos 2t.
[Jun 2012]
Solution. The given equations are
dx
+ 2y = sin 2t (1)
dt
dy
− 2x = cos 2t. (2)
dt

Download From : www.EasyEngineering.net


Download From : www.EasyEngineering.net

Differential Equations 381

Differentiating (1) w.r.t.t we get


d2 x dy
2
+ 2 = 2 cos 2t.
dt dt
d2 x
+ 2[2x + cos 2t] = 2 cos 2t. [ f rom(2)]
dt2
d2 x
+ 4x + 2 cos 2t = 2 cos 2t.
dt2
d2 x
+ 4x = 0.
dt2
The auxiliary equation is

ww m2 + 4 = 0 ⇒ m2 = −4

w.E m = ±2i.

xc = c1 cos 2t + c2 sin 2t.

x p = 0.
asy
The solution for x is
En
x = xc + x p = c1 cos 2t + c2 sin 2t.
Substituting in (1) we get
gin
ee
dx
2y = sin 2t −
dt
= sin 2t − [−2c1 sin 2t + 2c2 cos 2t]
rin
= sin 2t + 2c1 sin 2t − 2c2 cos 2t.
sin 2t
y= + c1 sin 2t − c2 cos 2t. g.n
2

Example 5.100. Solve


dx
dt
− y = t and
dy
dt
et
+ x = t2 given that x(0) = y(0) = 2.
[Dec 2011]
Solution. Refer Example 2.104.
The solution is

x = c1 cos t + c2 sin t + t2 − 1

y = c2 cos t − c1 sin t + t.

Download From : www.EasyEngineering.net


Download From : www.EasyEngineering.net

382 Engineering Mathematics - I

Given: when t = 0, x = 2
∴ c1 − 1 = 2 ⇒ c1 = 3.
Also, when t = 0, y = 2.
∴ c2 = 2.
The solutions are

x = 3 cos t + 2 sin t + t2 − 1

y = 2 cos t − 3 sin t + t.

wwExample 5.101. Solve


dx
dt
+ 2y = − sin t,
dy
dt
− 2x = cos t given, x = 1 and y = 0 at

w.E
t = 0.
[Dec 2010]

asy
Solution. The given equations are

dx

En
+ 2y = − sin t (1)
dt
dy
− 2x = cos t. (2)
dt
gin
Differentiating (1) w.r.t.t we get

d2 x
d2 x
dt2
dy
+ 2 = − cos t
dt
ee rin
dt2
d2 x
+ 2[2x + cos t] = − cos t [from (2)].
g.n
dt2
+ 4x + 2 cos t = − cos t.
d2 x
dt2
+ 4x = −3 cos t.
et
The auxiliary equation is

m2 + 4 = 0 ⇒ m2 = −4

m = ±2i.

xc = c1 cos 2t + c2 sin 2t.

Download From : www.EasyEngineering.net


Download From : www.EasyEngineering.net

Differential Equations 383

1
xp = (−3 cos t)
D2 +4
cos t
= −3 ·
−1 + 4
cos t
= −3 = − cos t.
3
∴ The solution for x is
x = xc + x p
x = c1 cos 2t + c2 sin 2t − cos t.
Substituting in (1) we get

ww 2y = − sin t −
dx
dt

w.E = − sin t − [−c1 2 sin 2t + 2c2 cos 2t + sin t]

= − sin t + 2c1 sin 2t − 2c2 cos 2t − sin t

asy
= 2c1 sin 2t − 2c2 cos 2t − 2 sin t.

En
y = c1 sin 2t − c2 cos 2t − sin t.

Given: when t = 0, x = 1.
∴ c1 − 1 = 1 ⇒ c1 = 2.
gin
When t = 0, y = 0.
−c2 = 0 ⇒ c2 = 0.
∴ The solutions are
ee rin
x = 2 cos 2t − cos t
g.n
y = 2 sin 2t − sin t.

5.5.2 Type II
et
We consider a system of first order linear differential equations of the form
dx dx
+ b1 x + c1 y = f (t) and a2
a1 + b2 x + c2 y = g(t).
dt dt
d
Procedure. Replace by D and rewrite the given equations involving D.
dt
Eliminating either x or y from the two equations we obtain a second order

Download From : www.EasyEngineering.net


Download From : www.EasyEngineering.net

384 Engineering Mathematics - I

differential equation in the other variable for which the solution can be obtained
by earlier methods. Substituting this value in any one of the given equations, we
obtain the solution for the other variable.
✎ ☞
Worked Examples
✍ ✌
dx dy
Example 5.102. Solve = 3x + 8y, = −x − 3y, x(0) = 6, y(0) = −2. [May 2007]
dt dt
d
Solution. Let = D. The given equations are reduced to
dt
dx

ww dt
= 3x + 8y

(D − 3)x − 8y = 0. (1)

w.E dy
dt
= −x − 3y

asy
(D + 3)y + x = 0.

Operating (1) by D + 3 we obtain


(2)

En
(D + 3)(D − 3)x − 8(D + 3)y = 0

gin
(D2 − 9)x + 8x = 0

ee
(D2 − 9 + 8)x = 0

(D2 − 1)x = 0
rin
m2 − 1 = 0.

m = ±1. g.n
From (1) we have 8y =
dx
x =c1 et + c2 e−t .

− 3x =⇒ y =
"
1 dx
− 3x
# et
dt 8 dt
1n o 1 −1
y= [c1 et − c2 e−t ] − 3[c1 et + c2 e−t ] = [−2c1 et − 4c2 e−t ] = [c1 et + 2c2 e−t ].
8 8 4
When t = 0, x = 6.

=⇒ c1 + c2 = 6. (3)

Download From : www.EasyEngineering.net


Download From : www.EasyEngineering.net

Differential Equations 385

When t = 0, y = −2.
c1 + 2c2
⇒ = −2
−4
c1 + 2c2 = 8. (4)
(4) − (3) ⇒ c2 = 2 ⇒ c1 = 4.
Solution is x = 4et + 2e−t
1
y = − [4et + 4e−t ] = −[et + e−t ].
4
dx dy
Example 5.103. Solve + 2x − 3y = t, − 3x + 2y = e2t . [Dec 2014, Jun 2006]
dt dt

wwSolution. Let
d
dt
= D. The given equations are reduced to

w.E (D + 2)x − 3y = t.

−3x + (D + 2)y = e2t .


(1)

(2)

asy
(1) × 3 =⇒ 3(D + 2)x − 9y = 3t (3)

En
Operating (2) by D + 2 we obtain

gin
−3(D + 2)x + (D + 2)2 y = (D + 2)e2t = 2e2t + 2e2t = 4e2t . (4)

ee
(3) + (4) =⇒ ((D + 2)2 − 9)y = 4e2t + 3t.

(D2 + 4D + 4 − 9)y = 4e2t + 3t

(D2 + 4D − 5)y = 4e2t + 3t. rin


This is a second order linear differential equation. g.n
A.E is, m2 + 4m − 5 = 0 ⇒ (m + 5)(m − 1) = 0 ⇒ m = −5, m = 1.

C.F. = yc = c1 e−5t + c2 et .
et
1 1
yp = 4e2t +  3t
D2+ 4D − 5 2
−5 1 − D +4D
5
e2t 3  D2 + 4D −1
=4 − 1− t
7 5 5
4 3 D2 4 
= e2t − 1 + + D + ··· t
7 5 5 5

Download From : www.EasyEngineering.net


Download From : www.EasyEngineering.net

386 Engineering Mathematics - I

4 3 4
PI = e2t − t +
7 5 5
4 3 4
y = c1 e−5t + c2 et + e2t − t + .
7 5 5

From (2) we have


dy
3x = + 2y − e2t
dt " #
4 3 4 3 4
= −5c1 e−5t + c2 et + 2e2t − + 2 c1 e−5t + c2 et + e2t − (t + ) − e2t
7 5 7 5 5
9 6 39
3x = −3c1 e−5t + 3c2 et + e2t − t − .
7 5 25

ww −5t t 3 2t 2
x = −c1 e + c2 e + e − t − .
7 5
9
5

w.E
Example 5.104. Solve (D + 2)x + 3y = 0, 3x + (D + 2)y = 2e2t .
Solution. The given equations are
[May 2003]

asy
(D + 2)x + 3y = 0. (1)

En
3x + (D + 2)y = 2e2t . (2)

gin
Operate (1) by (D + 2) we get

ee
(D + 2)2 x + 3(D + 2)y = 0.

(2) × 3 ⇒ 9x + 3(D + 2)y = 6e2t .


rin
(3)

(4)

(3) − (4) ⇒ (D + 2)2 − 9 x = −6e2t


g.n


D2 + 4D + 4 − 9 x = −6e2t


D2 + 4D − 5 x = −6e2t .


A.E is, m2 + 4m − 5 = 0 ⇒ (m + 5)(m − 1) = 0.


et
xc = c1 e−5t + c2 et .
e2t
x p = −6
D2 + 4D − 5
e2t −6 2t
= −6 = e .
7 7

Download From : www.EasyEngineering.net


Download From : www.EasyEngineering.net

Differential Equations 387

The solution for x is

x = xc + x p .
6
x = c1 e−5t + c2 et − e2t .
7

From (1) we have

3y = −(D + 2)x = −Dx − 2x


12 2t 12
= 5c1 e−5t − c2 et + e − 2c1 e−5t − 2c2 et + e2t

ww 7
24
3y = 3c1 e−5t − 3c2 et + e2t
7
7

w.E 1
3
−5t t 24 
y = 3c1 e − 3c2 e + e2t = c1 e−5t − c2 et + e2t .
7
8
7

Example 5.105. Solve


dx
dt asy
+ 4x + 3y = t;
dy
dt
+ 2x + 5y = e2t . [Dec 2013]
Solution. The given equations are

En
(D + 4)x + 3y = t.

2x + (D + 5)y = e2t . gin (1)

(2)

Operate (1) by D + 5, we get ee rin


(D + 4)(D + 5)x + 3(D + 5)y = (D + 5)t = 1 + 5t.
g.n
(2) × 3 ⇒
6x + 3(D + 5)y = 3e2t .
et
Subtracting, we get

(D + 4)(D + 5)x − 6x = 1 + 5t − 3e2t

(D2 + 9D + 20 − 6)x = 1 + 5t − 3e2t

(D2 + 9D + 14)x = 1 + 5t − 3e2t .

Download From : www.EasyEngineering.net


Download From : www.EasyEngineering.net

388 Engineering Mathematics - I

The auxiliary equation is

m2 + 9m + 14 = 0

(m + 2)(m + 7) = 0

m = −2, − 7.

xc = c1 e−2t + c2 e−7t .
1
xp = (1 + 5t − 3e2t )
D2 + 9D + 14

ww = 2
1
D + 9D + 14
1
(1 + 5t) − 2
3
D + 9D + 14
3e2t
e2t

w.E = 
14 1 + 9D+D
1
14
2


9D + D2
(1 + 5t)

!−1

4 + 18 + 14

3e2t

asy =
14
1
· 1+
14
9D + D 2
!
(1 + 5t) −
36
e2t

En
= 1− · · · (1 + 5t) −
14 14 12
2t
" #
1 9 e

gin
=
14
1
"
1 + 5t −
14
45
#
·5 −

e2t
12

=
ee
14

14
"
1 + 5t −

1 14 + 70t − 45
14
14

#
12


e2t
12
rin
=
70t − 31 e
196

2t

12
.
g.n
The solution for x is
x = xc + x p
70t − 31 e2t
et
x = c1 e−2t + c2 e−7t + − .
196 12
Substituting in (1) we get

3y = t − (D + 4)x.
70t − 31 e2t
!
−2t −7t
= t − (D + 4) c1 e + c2 e + −
196 12

Download From : www.EasyEngineering.net


Download From : www.EasyEngineering.net

Differential Equations 389

e2t 70t − 31 e2t


!
−2t −7t 70
= t − −2c1 e − 7c2 e + − + 4c1 e−2t + 4c2 e−7t + −
196 6 49 3
70 e2t 70t − 31 e 2t
= t − 2c1 e−2t + 3c2 e−7t − + − + .
196 6 49 3
70t 31 70 e2t
=t− + − − 2c1 e−2t + 3c2 e−7t + .
49 49 196 2
21t 54 e2t
3y = − + − 2c1 e−2t + 3c2 e−7t + .
49 196 2
e2t
!
1 21t 54 −2t −7t
y= − + − 2c1 e + 3c2 e + .
3 49 196 2

ww5.5.3 Type III

w.E
Here, we consider a system of linear first order differential equations of the form

a1
dx dy
+ b1 + c1 x = f (t)

asy a2
dt
dx
dt
dy
+ b2 + c2 y = g(t).

En
dt dt
d
As in Type II, replace by D and eliminate any one variable and solve for the

gin
dt
other variable by obtaining a second order linear differential equation and finally
on substitution of this value in any one of the equations, we obtain the solution for
the other variable.

Example 5.106. Solve


dx dy

dt dt
ee
+ 2y = cos 2t,
dx dy
+
dt dt
− 2x = sin 2t.
rin [Dec 2008]

Solution. Let
d
dt
= D. The given equations can be written as
g.n
Dx − (D − 2)y = cos 2t

(D − 2)x + Dy = sin 2t
et (1)

(2)

Operating (1) by D we get

D2 x − D(D − 2)y = −2 sin 2t (3)

Operating (2) by (D − 2) we obtain

(D − 2)2 x + D(D − 2)y = (D − 2) sin 2t = 2 cos 2t − 2 sin 2t (4)

Download From : www.EasyEngineering.net


Download From : www.EasyEngineering.net

390 Engineering Mathematics - I

(3)+(4)=⇒

(D2 + (D − 2)2 )x = 2 cos 2t − 4 sin 2t

(D2 + D2 − 4D + 4)x = 2 cos 2t − 4 sin 2t

(2D2 − 4D + 4)x = 2 cos 2t − 4 sin 2t

(D2 − 2D + 2)x = cos 2t − 2 sin 2t.

A.E is, m2 − 2m + 2 = 0

ww (m − 1)2 + 2 − 1 = 0

(m − 1)2 = −1

w.E m − 1 = ±i ⇒ m = 1 ± i.

asy
xc = et (c1 cos t + c2 sin t).
1

En
xp = (cos 2t − 2 sin 2t)
D2 − 2D + 2
1 1
cos 2t − 2 2

gin
= 2 sin 2t
D − 2D + 2 D − 2D + 2
1 1
= cos 2t − 2 sin 2t

=
−4 − 2D + 2
1
−2 − 2D
−1 1
ee
cos 2t − 2

cos 2t +
−4 − 2D + 2
1
−2 − 2D
1
sin 2t

sin 2t rin
=
2 D+1
−1 (D − 1)
2 D2 − 1
D+1
cos 2t + 2
(D − 1)
D! − 1
sin 2t g.n
=

=
−1 −2 sin 2t − cos 2t
2 −5
2 sin 2t + cos 2t
!
+
+
2 cos 2t − sin 2t
−5
2 cos 2t − sin 2t
et
−10 −5
−1 −1
= (2 sin 2t + cos 2t + 4 cos 2t − 2 sin 2t) = (5 cos 2t).
10 10
−1
xp = (cos 2t)
2
1
The solution is x = xc + x p = et (c1 cos t + c2 sin t) − (cos 2t).
2

Download From : www.EasyEngineering.net


Download From : www.EasyEngineering.net

Differential Equations 391

dx
(1) + (2) ⇒ 2 + 2y − 2x = cos 2t + sin 2t
dt
dx
2y = 2x − 2 + cos 2t + sin 2t.
" dt #
t cos 2t
= 2 e (c1 cos t + c2 sin t) −
2
" !#
t t 1 − sin 2t
− 2 e (−c1 sin t + c2 cos t) + e (c1 cos t + c2 sin t) − + cos 2t + sin 2t
2 2
sin 2t
= 2et ((c1 sin t − c2 cos t) +
2

ww t
y = e ((c1 sin t − c2 cos t) +
sin 2t
4
.

w.E
asy
En
gin
ee rin
g.n
et

Download From : www.EasyEngineering.net


Download From : www.EasyEngineering.net

ww
w.E
a syE
ngi
nee
rin
g.n
et

**Note: Other Websites/Blogs Owners Please do not Copy (or) Republish


this Materials, Students & Graduates if You Find the Same Materials with
EasyEngineering.net Watermarks or Logo, Kindly report us to
easyengineeringnet@gmail.com Download From : www.EasyEngineering.net

Das könnte Ihnen auch gefallen